You are on page 1of 318

AUDITION/VESTIBULAR FUNCTION

1. Which of these is true of sound waves?


A. actually take the shape of a wave
B. can travel through air and through a vacuum
C. maximum frequency heard by humans is 200,000 Hz
D. pressure in zone of compression is greater than that in zone of rarefaction
2. Ella complained of deafness in her right ear. Jayremae performed the tuning fork tests
on her. In the Weber test Ella heard the sound louder in her right ear, and in the Rinne
test she did not hear the sound after bone conduction was over. Based on these test
results, Ella may have a dysfunction of any of these structures.
A. External ear, tympanic membrane, ear ossicles, and cochlear nerve
B. External ear, tympanic membrane, and ear ossicles only
C. External ear and tympanic membrane only
D. External ear only
3. During a tutorial session, Danica asked Jay F to give the reason why there is acoustic
impedance mismatch between the outer and inner ears. If you were Jay F, what
would be your answer?
A. the ear ossicles are poorly movable
B. air pressure is greater in outer ear than in inner ear
C. air and water have different sound conducting properties
D. area of oval window is smaller than that of tympanic membrane
4. Which of these is true of the basilar membrane?
A. covers the organ of Corti
B. serves as roof of the cochlear duct
C. when pushed down, hair cells of organ of Corti are stimulated
D. basal portions vibrate maximally with sounds of higher frequencies
5. Sound masking is greatest when the masking tone frequency is;
A. increasing
B. less than that of the masked tone
C. identical to that of the masked tone
D. close but not identical to that of the masked tone
6. Dr Brian decided to use the elevator to go up the 5 th floor of the hospital. As the
elevator started he turned his head to the left to see who was behind him. Which
vestibular organs were stimulated?
A. saccule and left lateral semicircular duct
B. saccule and right lateral semicircular duct
C. utricle and right lateral semicircular duct
D. utricle and left lateral semicircular duct
7. Which of the following is true of the hair cells of the otolithic organs?
A. located in the crista ampullaris
B. kinocilia are located close to the cochlea
C. sensitive to linear acceleration of the head
D. movement of fluid waves bends the stereocilia and kinocilia
8. Polie let Rose sit on a swivel chair and told her to tilt her head 30 degrees forward.
She  then  rotated  the  chair  to  Rose’s  left.  After  12  turns  Polie  suddenly  stopped  the  
rotation. Polie noticed that Rose had nystagmus which lasted for 25 seconds. This
means that the hair cells of Which semicircular duct was stimulated?
A. right lateral semicircular duct C. right anterior semicircular duct
B. left lateral semicircular duct D. left anterior semicircular duct
ESSAY

Trace the sequence of events that lead to hair cell release of neurotransmitters from the
time the basilar membrane vibrates. Correction ends at the point where you commit a
mistake. 2 pts

Answer:
Basilar membrane is pushed up  stereocilia are bent away from the spiral limbus 
opening of K+ channels  K+ influx  depolarization of cell  opening of Ca++
channels  Ca++ influx  vesicles bind to basal membrane  exocytosis of
neurotransmitters

BBS FINAL EXAMINATION


MODULE 1 – CELL
1. Which of the following is not true of the mitochondria?
A. accumulate in parts of the cell where energy use is higher
B. made up of an inner & outer membranes
C. cristae are folds in the outer mitochondrial membrane
D. low energy requiring cells have few mitochondria and short cristae
2. Which of the following is the major role of the cytoskeleton?
A. Stores secretory products
B. Maintains shape & for organelle movement
C. Excretes waste products
D. Supports metabolic processes
3. Transitory components of the cell which store colored substances
A. zymogen granules C. lipid droplets
B. pigments D. glycogen
4. The following are true of chromatin, EXCEPT
A. nuclear component
B. made up of coiled strands of DNA
C. in interphase, it maintains its coiled state
D. euchromatin is less coiled than heterochromatin
5. Which is not true of anaphase?
A. nucleolus disappears
B. migration of chromosomes to poles
C. beginning of cell division
D. appearance of cleavage furrow
6. Which phase in interphase is characterized by DNA synthesis and the start of
centrosome duplication?
A. G1 B. S C. G2 D. G0
7. As a result of meiosis 1 the amount of DNA is
A. Doubled C. Tripled
B. Remains the same D. Halved

MODULE 2 – TISSUES
1. Which of the following is not a basic structural component of a simple epithelial tissue?
A. lining epithelium C. muscularis mucosae
B. lamina propria D. basement membrane
2. The prostate gland is an example of which type of compound gland
A. Tubular C. Saccular
B. Acinar D. Coiled
3. An epithelial cell specialization found on the lateral surface whose role is for
communication
A. zonulae adherents C. gap junctions
B. hemidesmosomes D. tight junctions
4. The following connective tissue cells play a role in immunologic defense, EXCEPT
A. Plasma C. Mast cell
B. Osteoblast D. Basophil
5. A connective tissue type characterized by abundant ground substance, very few fibers,
fibroblasts predominate
A. Mucous C. Dense
B. Adipose D. Reticular
6. A cell member of the mononuclear phagocyte system found in the skin
A. Monocyte C. Langerhans
B. Melanocyte D. Dendritic
7. Which of the following is not true of the dermis?
A. Made up of loose and dense connective tissues
B. The reticular layer projects to the epidermis
C. Contains the arrector pili muscles
D. Where the sweat & sebaceous glands are found

8. The layer of the epidermis whose role is for protection and as a permeability barrier
A. stratum lucidum C. stratum corneum
B. stratum spinosum D. stratum basale

MODULE 3 – NERVOUS TISSUE


1. Which of the following is not true of the histologic structure of the neuron?
A. The perikaryon contains highly developed RER
B. Golgi complex is seen up to the axon
C. Dendrites are usually short and branched
D. May or may not have an axon
2. This neuroglial cell is found in the CNS and mainly functions for support, repair and
metabolism
A. Astrocyte C. Oligodendrocyte
B. Ependymal D. Microglia
3. Which of the following differentiates the spinal cord from the other CNS organs?
A. its white matter do not contain neuronal cell bodies
B. glial cells and dendrites are seen in the gray matter
C. arrangement of the gray and white matter
D. fiber tracts are found in the white matter
4. The smallest neurons in the body are found in this layer of the cerebrum
A. Molecular C. Granular
B. Purkinje D. White matter
5. Which of the following is true of the Schwann cell?
A. Serves several axons at one time
B. Also found in the cerebrum
C. Binds neurons to capillaries and the meninges
D. Non-branching & forms myelin around one axon
6. Which of the following is not true of the ganglia?
A. encapsulated by myelin sheath
B. contains nerve cell bodies and glial cells
C. has a connective tissue support
D. associated with either cranial or spinal nerves

MODULE 5 – UPPER EXTREMITY


1. The microscopic appearance of osteoblasts during matrix deposition
A. reduced number of RER
B. spherical with acidophilc cytoplasm
C. columnar in shape with basophilic cytoplasm
D. large and becomes branched
2. A cross section of this bone type on gross observation shows dense areas with
absence of cavities
A. Developing C. Compact
B. Cancellous D. None of the above
3. Which of the following is true of cartilages?
A. all types are surrounded by perichondrium
B. fibrocartilage is the most common type
C. collagen type II is the principal fiber in all forms
D. variations in the composition of the matrix produce the 3 types
4. Which of the following is not true of endochondral ossification?
A. occurs in the presence of mesenchymal tissue
B. responsible for the formation of short and long bones
C. requires a intervening cartilage step
D. formation of primary and secondary ossification centers

MODULE 6 – BACK
1. Which of the following is not rue of skeletal muscle tissue?
A. made up of long, cylindrical multinucleated cells
B. each muscle fiber is ensheathed by connective tissue
C. muscle fibers show cross striations of alternating light and dark bands
D. the sarcoplasm is filled with intra and extrafusal fibers
2. Which of the following membrane junctions is not found in the intercalated discs?
A. fasciae adherents C. tight junctions
B. maculae adherents D. gap junctions

MODULE 9 – BLOOD CELLS


1. The primary component of the buffy coat
A. Platelets C. Red blood cells
B. Leukocytes D. Clootign factors
2. Which of the following leucocytes is not a terminal cell?
A. Monocyte C. Basophil
B. Eosinophil D. Neutrophil
3. Biconcave, anucleated cells which never leave the circulatory system in normal
conditions
A. Platelets C. Erythrocytes
B. Reticulocytes D. Plasma cells
4. These agranular cells are spherical with similar morphologic characteristics, but
classified according to their distinct surface markers
A. Monocytes C. Lymphocytes
B. Macrophage D. Lymphoblast

MODULE 10 – IMMUNE SYSTEM


1. A lymphoepithelial organ found in the mediastinum and is of dual embryonic origin
A. Tonsils C. Bone marrow
B. Spleen D. Thymus
2. Which of the following is not divided into cortex and medulla?
A. Thymus C. Lymph nodes
B. Spleen D. None of the above
3. Which of the following is not true of MALT?
A. encapsulated aggregates of lymphoid tissues
B. found in the GIT, GUT and respiratory tracts
C. occur in the mucosa and submucosa
D. major cell component is the lymphocyte

BBS – Finals

Module 4
1. Which of the following helps explain why stimulation of the sympathetic division gives
rise to a widespread response? B
A. The sympathetic division innervates practically all organ-systems of the body.
B. The sympathetic chain consists of 22 interconnected ganglia.
C. The transmitter of the sympathetic division is inactivated slowly.
D. All preganglionic fibers of the sympathetics arise from the spinal cord.
2. Which of the following statements as regards the sympathetic division is most correct?
C
A. White rami communicantes are formed by preganglionic fibers and spinal nerves.
B. All spinal nerves receive white rami communicantes.
C. Gray rami communicantes are formed by postganglionic fibers.
D. There are approximately 31 white rami communicantes.
3. Which of the following is most likely to occur upon stimulation of postganglionic fibers
that exit from the ciliary ganglion? A
A. pupilloconstriction C. lacrimation
B. salivary secretion D. bradycardia
4. Which of the following is most likely to occur upon stimulation of the vagus? D
A. secretion of scanty, mucoid saliva C. bronchodilation
B. penile erection D. increased gastric tone and motility
5. Which of the following cranial nerves contains autonomic efferent fibers only? A
A. oculomotor B. facial C. glossopharyngeal D. vagus
6. Which of the following responses to sympathetic stimulation can be attributed to
stimulation of 1 receptors only? C
A. liver glycogenolysis C. piloerection
B. decreased intestinal motility D. lipolysis

Module 9

1. Which of the following formed elements of blood possess phagocytic function, but die
soon after phagocytosis? D
A. eosinophils B. basophils C. monocytes D. neutrophils
2. Which of the following conditions is most likely to give rise to neutropenia? B
A. allergy B. viral infection C. stress D. chronic uremia
3. The principal hemopoietic organ at birth is the: C
A. liver C. marrow of bones of the axial skeleton
B. spleen D. marrow of the proximal ends of long bones only
4. Which of the following multilineage growth factors enhances synthesis of other growth
factors? A
A. IL 1 B. IL 3 C. IL 6 D. IL 11
5. Which of the following lineage-specific growth factors is chemotactic for neutrophils?
D
A. IL 2 B. IL 5 C. IL 7 D. IL 8

Module 10

1. Which of the following is basically capable of stimulating an immune response only


when bound to protein? C
A. antigen B. antibody C. hapten D. cytokine
2. Which of the following cells are mainly responsible for a faster immune response upon
re-exposure to a foreign antigen? B
A. effector T and B cells C. natural killer cells
B. memory T and B cells D. plasma cells
3. Which of the following serves as an indicator that the B cell is fully mature? D
A. CD 25 B. CD 22 C. surface Ig M D. surface Ig D
4. Which of the following antibodies is most likely to be present in the serum of an
exclusively bottle-fed newborn? A
A. Ig G B. Ig M C. Ig A D. Ig D
5. Which of the following is mainly responsible for electrical repulsion between red cells?
C
A. pH of blood C. ionic cloud
B. temperature of blood D. zeta potential

Module 11

1. Which of the following genes is mainly responsible for production of the basic
precursor in the Rh system? B
A. gene H B. gene Z C. gene C D. gene Se
2. Which of the following antibodies is most likely to be detected only in the anti-globulin
phase of the cross-match? D
a
A. anti-Le B. anti-P C. anti-E D. anti-Fya
3. Which of the following donor blood types will most likely give rise to a compatible
major cross-match but an incompatible minor cross-match if the recipient is type A?
C
A. type A B. type B C. type O D. type AB
4. While performing the tube method of blood typing, a student reported agglutination,
when in reality it was just rouleaux formation. Which of the following procedures is the
easiest way to confirm rouleaux formation? C
A. dispersion upon dilution of the serum with normal saline
B. confirmation by microscopic examination
C. dispersion upon gentle agitation
D. performing the procedure at 37o Celsius

Module 15

1. Which of the following pressures, when increased, will enhance air flow into the
alveoli? C
A. intrathoracic pressure C. transthoracic pressure
B. intrapulmonic pressure D. transpulmonary pressure
2. Which of the following muscles of respiration is mobilized only during forced
expiration? B
A. diaphragm C. sternocleidomastoid
B. internal intercostals D. serratus posterior superior
3. A subject exhales quietly. He then exhales maximally. The air that is expelled is called:
D
A. tidal volume C. functional residual capacity
B. residual volume D. expiratory reserve volume
4. Which of the following methods of measuring lung volumes and capacities measures
trapped gas? D
A. helium dilution method C. wet spirometry
B. nitrogen washout method D. body plethysmography
5. Which of the following measures of ventilatory capacity is used to assess
neuromuscular coordination particularly of the muscles of respiration? A
A. maximal breathing capacity C. forced expiratory vital capacity
B. timed vital capacity D. forced expiratory volume

Module 16

1. Which of the following serves as the site of integration of afferent information as


regards termination of inspiration? D
A. limbic system and hypothalamus C. cerebral cortex
B. pontine respiratory group D. apneustic center
2. Which of the following serve as the principal initiators of phrenic nerve activity?
Neurons of the: C
A. preBotzinger complex C. dorsal respiratory group
B. Botzinger complex D. ventral respiratory group

3. Which of the following, when increased, will reduce the rate of diffusion of oxygen?
B
A. membrane surface area C. chemical reactions with hemoglobin
B. diffusion distance D. gas temperature
4. Which of the following gas laws considers the amount of gas movement in both
gaseous and liquid phases? D
A.  Avogadro’s  law C.  Graham’s  law
B.  Dalton’s  law D.  Henry’s  law

Module 17

1. The clearance of a substance is lower than that of inulin. Which of the following is the
most likely reason for this result? B
A. the substance is freely filtered across the glomeruli
B. the substance is reabsorbed by the tubules
C. the substance is secreted by the fubules
D. the substance is synthesized by the tubules
2. Majority of the absorbable substances filtered across the glomeruli are reabsorbed at
the: A
A. proximal tubule C. loop of Henle
B. distal tubule D. collecting duct
3. Which of the following best explains why fluid that enters the distal tubule is generally
hypo-osmolar? B
A. at the loop of Henle, about 25% of water is reabsorbed, while 15% of salt is
reabsorbed
B. at the loop of Henle, about 15% of water is reabsorbed, while 25% of salt is
reabsorbed
C. at the distal tubule water reabsorption is greater than salt reabsorption
D. at the distal tubule salt reabsorption is greater than water reabsorption
4. The bladder volume of an individual is 200 ml. At this volume, which of the following is
most likely to take place? C
A. voluntary control of micturition is lost
B. sensation of bladder filling is experienced
C. first desire to void is experienced
D. rise in intravesical pressure
BB BM 1-09

1. Which of these components is primarily responsible for the fluidity of the plasma
membrane?
A. Extrinsic proteins C. lipids
B. intrinsic proteins D. carbohydrates
2. Protein components of plasma membrane may act as:
A. carriers, channels, pores, and receptors
B. carriers, channels and pores
C. carriers and channels
D. carriers
3. This membrane junction allows an action potential in one cell to be rapidly conducted
to other cells of the same organ. U
A. zonula adherens C. desmosomes
B. zonula occludens D. nexus
4. Which of these is true of facilitated transport across plasma membrane?
A. requires ATP to function
B. requires carrier protein
C. transports substances through membrane pores
D. transports substances against electrical gradient
5. Which of the following is a characteristic of secondary active transport but not of
primary active transport:
A. requires ATP C. requires an ion concentration gradient
B. requires carrier D. can be saturated and inhibited
6. This is true of the Na+/K+ pump in a resting membrane.
A. An extrinsic membrane protein
B. Transports K+ into the cell against electrical gradient
C. Transports 2 Na+ out of the cell and 3 K+ into the cell
D. maintains the intracellular concentrations of Na+ and K+
7. Which of these describes exocytosis?
A. passive process
B. requires carrier protein
C. removes a portion of the plasma membrane
D. transports substances through membrane channels
8. In a physiology experiment, the students were asked to expose the red cells to the
following solutions. In which of these will crenation occur? P
A. 0.05% NaCl C. 0.9 % NaCl
B. 0.85% NaCl D. 1.9% NaCl
9. Two compartments are filled with 0.75% NaCl solution. Compartment (Cpt) A contains
90 ml while Cpt B contains 20 ml. If a semipermeable membrane is placed between
the two compartments, the net movement of water across the membrane would be: P
A. From Cpt A to Cpt B C. unpredictable
B. From Cpt B to Cpt A D. zero
10. The plasma membranes of most cells in the body exhibit this characteristic. U
A. can produce action potentials spontaneously
B. only small and soluble substances can pass through it
C. threshold stimulation in one area depolarizes the entire membrane
D. cannot be stimulated when the Na+/K+ pump is actively working
11. If a stimulus causes the membrane potential to increase in negativity, which of these
would most likely occur? U
A. action potential C. equilibrium potential
B. graded potential D. diffusion potential
12. Which of these is true of the upstroke phase of the AP in neurons? U
A. membrane polarity remains negative
B. begins at the start of stimulation
C. due to rapid and large K+ influx
D. membrane is resistant to stimulation
13. Which of the following would bring back the membrane to its resting potential from the
undershoot phase of the action potential?
A. closure of the n gates C. closure of the m gates
B. closure of the h gates D. closure of the h and n gates
14. The cell is absolutely refractory to stimulation during these phases of the action
potential:
A. depolarization, overshoot, early repolarization, undershoot
B. depolarization, overshoot, early repolarization
C. depolarization, overshoot
D. depolarization
15. During the relative refractory period:
A. K+ conductance is low
B. All Na+ channels are still inactivated
C. A larger than normal stimulus may fire an action potential (AP)
D. occurs during the early part of the depolarization phase of the AP
16. The sodium channels involved in the upstroke phase of an action potential are:
A. leak channels C. ligand-gated channels
B. voltage-gated channels D. mechanically gated channels
17. If the membrane potential is at the equilibrium potential of K+, which of these would
occur? U
A. membrane potential is less negative than its resting potential
B. there is no net movement of K+ across the membrane
C. membrane is permeable to K+ but not to Na+
D. K+ efflux is greater than Na+ efflux
18. When the plasma membrane is at its resting potential:
A. Membrane is inexcitable
B. membrane is more permeable to Na+ than to K+
C. membrane is negative outside relative to the inside
D. there is no net movement of Na+ and K+
19. This is a characteristic of an action potential but not of a graded potential.
A. can be summated
B. conducted by local current flow
C. amplitude is dependent on the stimulus
D. can occur only when membrane is depolarized to threshold
20. This is true of the neuromuscular junction.
A. junctional cleft is filled with myoplasm
B. axon terminals of the motor neuron are myelinated
C. Postjunctional plasma membrane cannot fire action potentials
D. Acetylcholine esterases are located on the prejunctional membrane
21. If acetylcholinesterase is made unavailale at the neuromuscular junction, which of the
following would most likely occur? P
A. Blocked acetylcholine receptors
B. Rapid, repeated activation of acetylcholine receptors
C. Marked increase in acetylcholine level in synaptic cleft
D. Increased release of acetylcholine from presynaptic neuron
22. Which of these is a characteristic of the resting sarcomere?
A. actin filaments from two opposite Z lines overlap at the M line
B. both actin and myosin filaments are present in the A band
C. myosin heads are bound to myosin binding sites in actin filaments
D. ryanodine receptors and dihydropyridine receptors are mechanically coupled
23. This structure maintains its length as myofibril contracts. U
A. Sarcomere C. I band
B. A band D. H band
24. This structure contains myosin but not actin filaments.
A. A band C. H band
B. I band D. A band and H band

25. Actin filaments are attached to the Z-line by these proteins.


A. α-Actinin and capZ C. titin and capZ
B. α-Actinin and nebulin D. titin and nebulin
26. Binding of calcium to troponin-C is immediately followed by: U
A. Binding of ATP to myosin heads
B. binding of myosin heads to binding sites in actin
C. myosin heads move perpendicular to actin filaments
D. movement of tropomyosin away from myosin binding sites in actin
27. The  “power  stroke”  of  muscle  contraction  is  immediately  preceded  by:  U
A. binding of myosin heads with myosin binding sites in actin
B. binding of ATP with myosin head
C. hydrolysis of ATP to ADP and Pi
D. release of ADP and Pi from ATP
28. When L-type calcium channels open, which of the following immediately follows?
A. Depolarization of calcium release channels
B. Release of Ca++ from the sarcoplasmic reticulum
C. conformational change in the L-type calcium channels
D. coupling between L-type calcium channel and calcium release channel
29. A contracted muscle fiber remains contracted when: P
A. Action potential cannot reach the T tubule
B. Calcium level in cytoplasm is low
C. ATP is no longer made available
D. All myosin heads contain ATP
30. Which of the following immediately precedes the binding of Ca++ to troponin-C?
A. depolarization of the T tubule
B. opening of the L-type calcium channels
C. increased Ca++ level in the cytoplasm
D. pumping of Ca++ into the sarcoplasmic reticulum
31. Which of these forms of vit D is synthesized in the skin?
A. Vit D2
B. Cholecalciferol
C. 25-hydroxycholecalciferol
D. 1,25-dihydroxycholecalciferol
32. Which of these steps in Vit D biosynthesis is enhanced by the parathyroid hormone?
A. formation of cholecalciferol from 7-dehydrocholesterol
B. formation of 25-hydroxycholecalciferol from cholecalciferol
C. formation of 1,25-dihydroxycholecalciferol from 25-hydroxycholecalciferol
D. formation of both 25-hydroxycholecalciferol and 1,25 dihydroxycholecalciferol
33. The final step in the synthesis of 1,25-dihydroxyvitamin D (calcitriol) from vitamin D2 or
Vitamin D3 occurs in the:
A. liver C. kidneys
B. spleen D. bone
34. Which of these is true of plasma calcium?
A. low plasma calcium level stimulates parathyroid hormone (PTH) synthesis
B. Only 10% of plasma calcium is in the ionized and active form
C. 70% of plasma calcium is bound to albumin
D. Alkalosis increases the plasma ionized calcim
35. Which of the following is true of calcium in bone?
A. only 10% of bone calcium is freely exchangeable with ECF calcium
B. bone calcium constitutes 50% of total body calcium
C. PTH accelerates removal of calcium from bone
D. Calcitriol inhibits calcium resorption from bone

BB BM 1-10

1. Which of the following is a characteristic of secondary active transport but not of


primary active transport:
A. requires ATP C. requires an ion concentration gradient
B. requires carrier D. can be saturated and inhibited
2. This is true of the Na+/K+ pump in a resting membrane.
A. An extrinsic membrane protein
B. Inactive when the cell is at its RMP
C. Transports 2 Na+ out of the cell and 3 K+ into the cell
D. maintains the intracellular concentrations of Na+ and K+
3. Which of these describes endocytosis?
A. passive process
B. requires carrier protein
C. removes a portion of the plasma membrane
D. transports substances through membrane channels
4. In a physiology experiment, the students were asked to expose the red cells to the
following solutions. In which of these will crenation occur? P
A. 0.05% NaCl C. 0.9 % NaCl
B. 0.85% NaCl D. 1.9% NaCl
5. The plasma membranes of most cells in the body exhibit this characteristic. U
A. can produce action potentials spontaneously
B. only small and soluble substances can pass through it
C. threshold stimulation in one area depolarizes the entire membrane
D. cannot be stimulated when the Na+/K+ pump is actively working
6. In an experiment, the Na+ and K+ channels of a motor neuron were opened at the
same time. Which of these most likely developed? U
A. an action potential C. a graded potential
B. a threshold potential D. an end plate potential
7. Which of these is true of the upstroke phase of the AP in neurons? U
A. membrane polarity remains negative
B. begins at the start of stimulation
C. due to rapid and large K+ influx
D. membrane is resistant to stimulation
8. Which of the following would bring back the membrane to its resting potential from the
undershoot phase of the action potential?
A. closure of the n gates C. closure of the m gates
B. closure of the h gates D. closure of the h and n gates
9. The cell is absolutely refractory to stimulation during these phases of the action
potential:
A. depolarization, overshoot, early repolarization, late repolarization
B. depolarization, overshoot, early repolarization
C. depolarization, overshoot
D. depolarization
10. The sodium channels involved in the upstroke phase of an action potential are:
A. leak channels C. ligand-gated channels
B. voltage-gated channels D. mechanically gated channels
11. When the plasma membrane is at its resting potential:
A. Membrane is inexcitable
B. membrane is more permeable to Na+ than to K+
C. membrane is negative outside relative to the inside
D. there is no net movement of Na+ and K+
12. This is true of the neuromuscular junction.
A. junctional cleft is filled with myoplasm
B. axon terminals of the motor neuron are myelinated
C. Postjunctional plasma membrane cannot fire action potentials
D. Acetylcholine esterases are located on the prejunctional membrane
13. This structure maintains its length as myofibril contracts. U
A. Sarcomere C. I band
B. A band D. H band
14. Actin filaments are attached to the Z-line by these proteins.
A. α-Actinin and capZ C. titin and capZ
B. α-Actinin and nebulin D. titin and nebulin
15. The  “power  stroke”  of  muscle  contraction  is  immediately  preceded  by:  U
A. binding of myosin heads with myosin binding sites in actin
B. binding of ATP with myosin head
C. hydrolysis of ATP to ADP and Pi
D. release of ADP and Pi from ATP
16. When L-type calcium channels open, which of the following immediately follows?
A. Depolarization of calcium release channels
B. Release of Ca++ from the sarcoplasmic reticulum
C. conformational change in the L-type calcium channels
D. coupling between L-type calcium channel and calcium release channel
17. A contracted muscle fiber remains contracted when: P
A. Action potential cannot reach the T tubule
B. Calcium level in cytoplasm is low
C. ATP is no longer made available
D. All myosin heads contain ATP
18. Which of the following immediately precedes the binding of Ca++ to troponin-C?
A. depolarization of the T tubule
B. opening of the L-type calcium channels
C. increased Ca++ level in the cytoplasm
D. pumping of Ca++ into the sarcoplasmic reticulum
19. When an AP reaches the T-tubule, which of the following immediately follows?
A. stimulation of the calcium pump in the sarcoplamic reticulum (SR)
B. opening of calcium channels in the SR
C. binding of calcium to troponin-C
D. Calcium entry into the cell
20. In the relaxed state of a skeletal muscle fiber, which of the following is true?
A. binding sites in actin are not covered by tropomyosin
B. myosin heads are attached to actin
C. cytosolic calcium level is low
D. calcium is bound to troponin-C

Xavier University – Dr Jose P Rizal College of Medicine


Basic Biomedical Sciences – Second Bimonthly Exam
PHYSIOLOGY

NAME ________________________ Oct. 19, 2006 SY 2009 – 10

MULTIPLE CHOICE: Write the letter of the BEST answer in CAPITAL letter in blue or black
ink
on the answer sheet. NO ERASURES/SUPERIMPOSITIONS ALLOWED!

51. Which of the following events occur to effect primary hemostasis?


A. injured blood vessel dilate
B. platelets become flattened discs
C. von Willebrand factor (VWF) binds platelets to fibrinogen
D. fibrinogen forms a bridge between two opposing platelets
52. When this factor is deficient, both primary and secondary hemostasis are also
deficient:
A. fibrinogen C. thrombogen
B. prothrombin D. thrombospondin
53. Aside from calcium and phospholipid, the conversion of prothrombin to thrombin
requires the presence of these factors:
A. Xa and XIa C. VIIIa and IXa
B. Va and Xa D. Xa and VIIa
54. When thrombin binds to thrombomodulin, thrombin:
A. converts fibinogen to fibrin
B. stabilizes the loose fibrin clot
C. is activated to destroy factors Va and VIIIa
D. activates Protein C, which together with cofactor inhibits coagulation
55. If Sam has factor VII deficiency, which of the following tests would be prolonged?
A. Bleeding time (BT)
B. Activated Partial Thromboplastin time (APTT)
C. Prothrombin time (PT)
D. Thrombin time(TT)
56. This substance is synthesized by endothelial cells and prevents activated platelets
from adhering to uninjured endothelium.
A. prostacyclin C. adenosine diphosphatase
B. antithrombin III D. thromboxane A2
57. Aleth’s  laboratory  test  results  are  as  follows:  BT  =  normal;;  APTT  =  prolonged;;
PT = normal; TT = normal. Based on these results, she could be deficient in any one
or a combination of these groups of factors:
A. V, VII, X C. VIII, IX, XI
B. II, X, V D. V, VIII, IX
58. Which of the following associations between phases of the fast response action
potential (FRAP) and their primary causes is correct, EXCEPT:
A. phase 0 – rapid and large K+ influx
B. phase 1 – closure of K+ channels
C. phase 2 – low outward K+ current
D. phase 3 – rapid outward K+ current

59. The volume of blood present in the ventricle at the end of atrial systolic phase of the
cardiac cycle is the:
A. end systolic volume C. systolic reserve volume
B. end diastolic volume D. diastolic reserve volume
60. The chest x-ray of Mr Hart showed left ventricular enlargement. If an ECG is done,
which of the following is expected to be prolonged?
A. ST segment C. QRS interval
B. PR interval D. R-R interval
61. Compared to the first heart sound (S1), the second heart sound (S2) is:
A. louder in intensity C. longer in duration
B. dull in quality D. higher in pitch
62. Amer wanted to listen to the heart sounds of Rene. He placed the bell of his
stethoscope over the 5th intercostal space along the left midclavicular line. Which
heart sounds will he most likely hear?
A. S1, S2, S3, S4 C. S1, S2
B. S1, S2, S3 D. S1
63. Applying  Starling’s  law  of  the  heart,  the  force of myocardial contraction is increased
when this is increased:
A. CVP C. preload stress
B. PVP D. afterload stress
64. The following events occur during the isovolumetric contraction phase of the cardiac
cycle.
A. opening of semilunar valves C. decrease in ventricular volume
B. rapid rise in ventricular pressure D. second heart sound is heard
65. If impulse conduction to the right branch of the bundle of His is blocked such that no
action potential can pass through the block, which of the following would occur? C
A. impulse is not conducted to the right ventricle
B. delayed closure of the aortic valve
C. splitting of the first and second heart sounds
D. first heart sound is due only to closure of the mitral valve
66. Ms Mitra is suspected of having diseased papillary muscles of the left ventricle. If this
is true, which of the following would result?
A. systolic murmur C. decreased end diastolic volume of left ventricle
B. aortic regurgitation D. increased stroke volume of left ventricle
67. The following are the laboratory results of Ms Ace Demi who was suspected of having
diabetic acidosis. Which of these results will show that her acidosis is not
compensated?
A. pH = 7.32 C. PCO2 = 42 mmHg
B. [HCO3-] = 16.1 D. [BE] = (-) 8.1
68. As a result of pure buffer response to a primary gain in HCO3-, which of the following
parameters will be decreased at the new equilibrium?
A. pH C. [HBuf]
B. S.PCO2 D. [BB]
69. Body responses to the acid-base disturbance secondary to renal failure will result in:
A. decreased respiratory rate
B. acidification of urine
C. renal formation of new bicarbonate
D. buffering by the BBS and NBBS
70. R N, a first year medical student was shown this laboratory result of a friend who was
suspected of having an acid-base disturbance: pH = 7.474; [HCO3-] = 23.4 mmol; [BE]
= (-) 1.5 mmol/L; PCO2 = 28.67 mmHg. Based on these lab results, R N can say that
his friend is likely to have:
A. uncompensated respiratory alkalosis
B. partially compensated respiratory alkalosis
C. uncompensated metabolic alkalosis
D. partially compensated metabolic alkalosis
71. Which of the following events occur to effect swallowing?
A. tongue is pressed against the soft palate
B. respiration is inhibited during the oral phase
C. palatopharyngeal folds move inward toward one another
D. larynx is moved upward to prevent reflux of food into nasopharynx
72. As regards gastric filling and mixing, which of these is true?
A. receptive relaxation occurs at the antrum and pylorus
B. mixing activity primarily occurs in the stomach fundus
C. carbohydrates form a thin layer on top of all the other gastric contents
D. gastric contents may remain unmixed for about one hour after eating
73. This is true of gastric contractions.
A. strong gastric contractions are preceded by slow waves
B. gastric contractions begin when food enters the stomach
C. gastric peristalsis is generated by a pacemaker located near the stomach body
D. gastric contractions begin at the fundus and travel towards the pylorus
74. Which of the following is true of the migrating myoelectric complex (MMC)?
A. starts soon after feeding
B. occurs every 30 minutes
C. propagated from the stomach down to the colon
D. characterized by a period of intense contractions followed by period of
quiescence
75. The increased secretory and motor activities of the stomach increase the motility of the
terminal ileum. This is known as the:
A. ileogastric reflex C. gastrocolic refex
B. gastroileal reflex D. intestinointestinal reflex
76. Which of these is true of peristalsis in the small intestine?
A. It is the most frequent type of contraction of the small intestine
B. The contractions divide the small intestine into small adjacent segments
C. It is the progressive contraction of successive sections of the small intestine
D. more effective than segmentation in mixing chyme with digestive secretions
77. Which of these is true of the motor activity of the small intestine (SI)?
A. peristalsis in the terminal ileum closes the ileoceacal sphincter
B. contractions of the duodenal bulb mix chyme with pancreatic & biliary
secretions
C. overdistention of one segment of the SI causes the rest of the intestine to
contract
D. the rate of segmental contractions is higher in the ileum than in the duodenum
78. This is true of defecation
A. it is a reflex without voluntary action
B. it is initiated when rectum is filled with feces
C. the pudendal nerves are the principal motor pathway of the reflex
D. the integrating center of the reflex is in the lumbar spinal cord

79. As regards motor activity of the colon, which of these is true?


A. localized peristaltic contractions divide the colon into haustra
B. antipropulsive contractions predominate in the distal colon
C. distension of one part of the colon causes contraction of the other parts
D. entry of food into the stomach reflexly increases motility of proximal & distal
colon
80. Which of these events occur to effect vomiting?
A. pyloric sphincter and stomach relax
B. forced inspiration against an open glottis
C. lower esophageal sphincter reflexly constricts
D. reverse peristalsis from terminal ileum to the duodenum
81. Control of salivary gland secretion is primarily by the:
A. Sympathetic nervous system C. hormones
B. Parasympathetic nervous system D. enteric nervous system
82. Secretion of which of these salivary glands contain mucin?
A. parotid and submandibular glands
B. parotid and sublingual glands
C. submandibular and sublingual glands
D. parotid gland only
83. Some of the components of gastric juice are:
A. HCl, pepsin, bicarbonate, mucus
B. HCl, pepsin and bicarbonate
C. HCl and pepsin
D. HCl and bicarbonatae
84. HCl secretion by parietal cell involves which of these events?
A. Cl- enters cell across basolateral membrane via Cl-, Na+ countertransport
B. Cl- enters canaliculus by an electrogenic ion channel
C. H+ is pumped into canaliculus along concentration gradient
D. H+ is pumped into canaliculus via the H+, Na+ - ATPase
85. These substances stimulate parietal cells to secrete HCl.
A. histamine, acetylcholine, gastrin, and prostaglandin
B. histamine, acetylcholine and gastrin
C. histamine and acetylcholine
D. histamine
86. Gastric acid secretion is inhibited by which of the following substances?
A. gastric inhibitory peptide (GIP), secretin, bulbogastrone, & cholecystokinin
(CCK)
B. GIP, secretin, and bulbogastrone only
C. GIP and secretin only
D. GIP only
87. Which of these is true regarding pancreatic exocrine secretion?
A. Sympathetic fibers stimulate exocrine secretion
B. The aqueous component is secreted primarily by pancreatic duct cells
C. The hormone secretin stimulates the enzyme component
D. The hormone CCK stimulates the aqueous component
88. Which of these is a characteristic of the intestinal phase of pancreatic secretion?
A. acid in chyme increases volume of pancreatic juice with low enzyme
concentration
B. decreased volume of duodenal contents stimulate pancreatic secretion
C. fatty acids and monoglycerides in the duodenum inhibits pancreatic secretion
D. peptides in duodenum inhibits pancreatic secretion
89. Which of the following is true of bile acids?
A. they are normally conjugated with glycine and taurine
B. substances that stimulate bile acid secretion are called cholecystagogues
C. they are secreted into the secretory canaliculus by primary active transport
D. conjugated and unconjugated bile acids are absorbed in the terminal jejunum
90. Bile is synthesized by hepatocytes and its components include which of the following?
A. bile acids, bile pigments, cholesterol and phospholipids
B. bile acids, bile pigments and cholesterol only
C. bile acids and bile pigments only
D. bile acids only
91. Which of these is true regarding carbohydrate digestion?
A. it begins in the stomach where they are digested into oligosaccharides
B. Oligosaccharidases present inside epithelial cells further digest
oligosasccharides
C. Activity of the oligosaccharidases is highest in the duodenum and upper
jejunum
D. Lactase splits lactose into glucose and fructose
92. Glucose absorption in the duodenum and upper jejunum involves this step.
A. glucose together with galactose bind with the transporter SGLT1
B. SGLT1 transports one glucose, one galactose and two Na+ ions
C. Na+ gradient provides the energy for the transport protein SGLT1
D. Glucose & galactose bind with GLUT2 to leave cell at basolateral membrane
93. This transport protein transports fructose across the brush border of epithelial cells
A. SGLT1 C. GLUT326
B. GLUT2 D. GLUT5
94. As regards protein digestion, which of these is true?
A. it begins in the lumen of the duodenum and jejunum
B. pancreatic proteases reduce proteins to oligopeptides & amino acids
C. brush border peptidases reduce peptides to oligopeptides and amino acids
D. all di- and tripeptides are reduced to amino acids before entering epithelial cell
95. Which of these is true regarding protein absorption?
A. significantly absorbed as intact proteins, as small peptides and as amino acids
B. amino acid transporters across brush border are all Na+ - dependent
C. Some amino acids may be transported across brush border by simple diffusion
D. Amino acids but not di-and tripeptides are then absorbed into the blood
96. Which of these steps is involved in triglyceride digestion?
A. Stomach digestion of triglycerides is primarily by lingual lipase
B. Triglycerides are emulsified in the stomach by bile acids with the aid of lecithin
C. Products of triglyceride digestion form mixed micelles with bile acids
D. Concentration of triglycerides must  reach  the  “critical  micelle  concentration”
97. Which of these is true of lipid absorption?
A. micelles cross the brush border plasma membrane by simple diffusion
B. fatty acids cross brush border plasma membrane only by facilitated diffusion
C. resynthesized lipids are processed into chylomicrons in the golgi apparatus
D. chylmicrons are released by exocytosis and they enter into blood capillaries
98. Net absorption of water is greatest in the:
A. Duodenum C. ileum
B. Jejunum D. colon

99. These substances stimulate absorption of electrolytes and water in the small intestine
and / or large intestine.
A. aldosterone, glucocorticoids, epinephrine, and somatostatin
B. aldosterone, glucocorticoids, and epinephrine
C. aldosterone and glucocorticoids
D. alsosterone only
100. Which of the following statements regarding these ions is true?
E. Calcium is absorbed mainly in the proximal small intestine
F. Nonheme iron is absorbed mainly in the jejunum
G. Magnesium is absorbed mainly in the duodenum
H. Phosphate is absorbed mainly in the ileum

XAVIER UNIVERSITY – JOSE P RIZAL COLLEGE OF MEDICINE


Basic Biomedical Sciences
Fourth Bimonthly Period
QUIZ # 1 (Spinal Cord)

NAME ____________________ 28 January 2008 SCORE ____________


MULTIPLE CHOICE: write the letter of the BEST answer on the answer sheet. Use capital
letters in blue/black ink. NO EASURES/SUPERIMPOSITIONS ALLOWED!

1. Which of the following may not be present in the simplest reflex arc?
A. afferent neuron C. effector organ
B. interneuron D. receptor organ
2. Spinal shock syndrome is followed by decerebrate rigidity, which if turn is followed by
paraplegia in extension. This last condition is due to severance of all descending
tracts to the spinal cord but with persistence of this tract.
A. vestibulospinal tract C. reticulospinal tract
B. corticospinal tract D. tectospinal tract
3. The center for the knee jerk is located at:
A. C1, C2
B. C.
4. The pyramidal tract refers to the:
A. vestibulospinal tract C. reticulospinal tract
B. corticospinal tract D. tectospinal tract
5. Lesions restricted to the pyramidal tracts result in the presence of:
A. clasp-knife reaction C. Babinski sign
B. hypertonicity D. exaggerated deep muscle reflexes
6. Fibers of the corticospinal tract arise as axons of pyramidal cells in the cerebral cortex.
The descending fibers then converge in the:
A. Pyramids C. internal capsule
B. corona radiate D. superior cerebellar peduncle
7. Fibers of the lateral corticospinal tract terminate in the anterior gray column of:
A. the spinal cord segments in the cervical and upper thoracic regions
B. the spinal cord segments in the upper thoracic region only
C. the spinal cord segments in the cervical region only
D. all the spinal cord segments
8. Fibers of the anterior corticospinal tract terminate in the anterior gray column of:
A. the spinal cord segments in the cervical and upper thoracic regions
B. the spinal cord segments in the upper thoracic region only
C. the spinal cord segments in the cervical region only
D. all the spinal cord segments
9. The afferent fast acting pain fibers stimulate the second-order neurons of the:
A. lateral spinothalamic tract C. posterior spinocerebellar tract
B. anterior spinothalaamic tract D. internal arcuate fibers
10. Axons of the second-order neurons of the light touch and pressure pathway form the:
A. lateral spinothalamic tract C. posterior spinocerebellar tract
B. anterior spinothalamic tract D. internal arcuate fibers

Xavier University – Dr Jose P Rizal College of Medicine


Basic Biomedical Sciences – First Bimonthly Exam
Set B: Modules 7 – 10

NAME ________________________ August 9, 2006 SY 2006 – 07

MULTIPLE CHOICE: Write the letter of the BEST answer in CAPITAL letter in ink on
the answer sheet. NO ERASURES/SUPERIMPOSITIONS ALLOWED!

1. A substance capable of eliciting an immune response is called: C


A. antigen B. antibody C. immunogen D. immunoglobulin
2. Which of the following cytokines is primarily involved in inflammatory and cytotoxic
reactions? D
A. interferon C. colony stimulating factor
B. interleukin D. transforming growth factor
3. Which of the following refers to a set of interacting enzymes that can produce cell lysis
when activated? B
A. interleukin C. tumor necrosis factor
B. complement D. transforming growth factor
4. A substance which, by itself, cannot initiate an immune response but may be able to
do so upon combination with a macromolecule is called: C
A. antigen C. hapten
B. immunogen D. cytokine
5. Innate immunity brought about by activity of phagocytes is the result of: D
A. genetic factors C. biochemical factors
B. mechanical barriers D. cellular mechanisms
6. A grade school boy was exposed to chickenpox but did not develop the disease. He
then found out that he already had chickenpox when he was 2 years old. This type of
immunity is called: D
A. innate immunity C. passive adaptive immunity
B. genetic immunity D. active adaptive immunity
7. Which of the following lymphocytes is cytotoxic for virus-coated cells, can release
gamma interferon and is capable of intercellular communication via leukocyte function
antigen? D
A. T helper cells C. B lymphocytes
B. T cytotoxic cells D. natural killer cells
8. Which of the following cells is considered as an auxiliary cell of the immune system
because it can release mediators of inflammation? A
A. basophil B. macrophage C. neutrophil D. eosinophil
9. Which of the following cluster of differentiation (CD) markers is present in all T cells as
they migrate to the thymus? C
A. CD 1 B. CD 3 C. CD 7 D. CD 4
10. Which of the following CD markers is essential for migration of mature T cells to the
medulla of the thymus and eventually to peripheral lymphoid tissue? D
A. CD 71 B. CD 38 C. CD 19 D. CD 44
11. When the B lymphocyte acquires CD 22, it is called: B
A. pro-B cell C. immature B cell
B. pre-B cell D. mature B cell
12. When the B lymphocyte acquires surface Ig D molecules, it is called: D
A. pro-B cell C. immature B cell
B. pre-B cell D. mature B cell
13. Which of the following cells possesses CD 56 and CD 16? D
A. helper T cell C. mature B cell
B. cytotoxic T cell D. natural killer cell
14. Which of the following cells of the immune system is destroyed once it phagocytizes a
foreign antigen? C
A. macrophage B. eosinophil C. neutrophil D. basophil
15. Which of the following immediately precedes secretion of interleukin 1 by the antigen
presenting cell (APC)? B
A. processing of epitope on the cell surface of APC
B. epitope is presented to T helper cell
C. clonal proliferation of T helper cell
D. phagocytosis of foreign antigen by APC
16. Cell-mediated immune response involves activity of the following cells, EXCEPT:
A. T lymphocytes C. B lymphocytes
B. macrophages D. natural killer cells
17. The following are characteristics of primary humoral immune response, EXCEPT:
A. occurs upon initial exposure to a foreign antigen
B. involves production of Ig G and Ig M antibodies
C. synthesis of Ig M precedes synthesis of Ig G
D. occurs 2-3 days after exposure to a foreign antigen
18. Which of the following is directly involved in the release of cytokines?
A. effector T cells C. memory T cells
B. effector B cells D. memory B cells
19. Which of the following best explains why Ig G titer is much higher than Ig M titer during
the secondary humoral immune response? B
A. memory B cells can rapidly transform into plasma cells which produce antibodies
B. there are more memory B cells producing Ig G than memory B cells producing Ig M
C. memory B cells producing Ig G have higher affinity for antigen receptors
D. memory B cells producing Ig M have higher affinity for antigen receptors
20. Which of the following portions of the immunoglobulin molecule confers specificity to
the molecule? A
A. variable region of the Fab fragment
B. constant region of the Fab fragment
C. the disulfide bridge linking the light and heavy chains
D. the lower portion of the tail or the Fc fragment
21. Which of the following is the most likely antibody present in a bottle-fed newborn? B
A. Ig M B. Ig G C. Ig A D. Ig D
22. Which of the following is considered as the only complete antibody? A
A. Ig M B. Ig G C. Ig A D. Ig D
23. Which of the following is believed to play a part in immunity to helminths? C
A. Ig A B. Ig D C. Ig E D. Ig G
24. The binding force between antigen and antibody that depends on ionic clouds is
called:
A. electrostatic force C. hydrophobic bond
B. hydrogen bond D. van der Waals bond D
25. Antigen-antibody reaction involving red cells and typing sera is optimal if the following
are met, EXCEPT: B
A. addition of antiglobulin serum C. pH of 7
B. room temperature of 4 C D. reaction time of 2 minutes
26. Antigen-antibody reaction is unlikely to occur if there is an excess of antigen. This is
called: A
A. postzone phenomenon C. high zone tolerance
B. prozone phenomenon D. low zone tolerance
27. Failure of the immune response due to excess of antigen is called: C
A. postzone phenomenon C. high zone tolerance
B. prozone phenomenon D. low zone tolerance
28. Functional elimination of forbidden clones brought about by down-regulation of the
immune response is called: D
A. clonal deletion C. clonal suppression
B. clonal abortion D. clonal anergy
29. Which of the following multi-lineage growth factors interacts synergistically with
lineage-specific growth factors? B
A. IL 1 B. IL 3 C. IL 6 D. IL 11
30. Which of the following multi-lineage growth factors specifically stimulates colony
formation in the early stages of hemopoiesis? D
A. IL 1 B. IL 3 C. GM-CSF D. stem cell factor
31. Which of the following lineage-specific growth factorsis chemotactic for neutrophils?
C
A. IL 2 B. IL 4 C. IL 8 D. IL 9
32. Iron destined to be used for synthesis of hemoglobin enters the red cell at which of the
following stages of maturation? A
A. prorubricyte B. rubricyte C. metarubricyte D. reticulocyte
33. The amino acid sequence in the alpha chain of globin is dictated by a gene present in:
C
A. chromosome 11 C. chromosome 16
B. chromosome 19 D. chromosome 21
34. Heme synthesis takes place at the: D
A. ribosomes of rubriblasts C. mitochondria of rubriblasts
B. ribosomes of rubricytes D. mitochondria of rubricytes
35. Which of the following is necessary for condensation of succinyl Co-A and glycine?
C
A. ALA dehydrase C. pyridoxal phosphate
B. heme synthase D. ferrochelatase
36. One hemoglobin molecule has how many atoms of iron? D
A. one B. two C. three D. four
37. Which of the following laboratory tests is most useful in the morphologic classification
of anemia? B
A. red cell count C. examination of a stained peripheral blood smear
B. red cell indices D. examination of a stained bone marrow smear
38. Which of the following laboratory test results is highly suggestive of an allergic
response or parasitism? A
A. neutrophils = 60%; lymphocytes = 25%; monocytes = 5%; eosinophils = 10%
B. neutrophils = 70%; lymphocytes = 20%; monocytes = 6%; eosinophils = 4%
C. neutrophils = 80%; lymphocytes = 15%; monocytes = 3%; eosinophils = 2%
D. neutrophils = 50%; lymphocytes = 35%; monocytes = 12%; eosinophils = 3%
39. Which of the following laboratory tests is most useful in the study of blood cell
morphology in general? C
A. hemoglobin concentration C. examination of a stained peripheral smear
B. hematocrit determination D. examination of a stained bone marrow smear
40. Which of the following is most useful in evaluation of bleeding disorders? B
A. red cell count C. examination of a stained peripheral smear
B. platelet count D. examination of a stained bone marrow smear
41. Which of the following is atypical cervical vertebra?
A.C2 C. C5
B. C4 D. C6
42. The following are characteristics of the lumbar vertebra EXCEPT:
A. large, massive bodies C.spinous process is short ,quadrangular
B.costal facets D.triangular vertebal foramina
43. These are parts of the sacral vertebra EXCEPT:
A. promontory C.sacral foramina
B. symphysis pubis D.base

44. The joint between two vertebral bodies is formed by:


A.intervertebral disc C.membrana tectoria
B. ligamentum flavum D. anterior longitudinal ligament
45. Numerous arteries and veins are found in which layer of the scalp?
A.skin C.dense connective tissue layer
B.pericranium D. loose connective tissue layer
46. The cranial point where great wing of sphenoid meets with parietal bone is :
A.nasion C.bregma
B.lambda D.pterion
47. The zygomatic arch is formed by :
A.zygomatic process, temporal bone C.temporal process, zygomatic bone
B.frontal process , zygomatic bone D.both A & C
48. The following are openings of the greater wing of sphenoid EXCEPT:
A..optic canal C.foramen rotundum
B.foramen ovale D.foramen spinosum
49. Which is not a branch of the facial nerve?
A.temporal C. maxillary
B. zygomatic D.buccal
50. Lymphatic vessels of the face drain into which of the following?
A.submental lymph nodes C. submandibular lymph nodes
B. buccal lymph nodes D. all of the above
51. These are smaller subdivisions of the anterior triangle of the neck EXCEPT:
A.carotid C. muscular
B. occipital D.digastric
52. What is considered as the dangerous area of the neck?
A. investing layer, cervical fascia C.pretracheal layer, cervical fascia
B.prevertebral , cervical fascia D.between A & C
53. What is the normal location of the spleen?
A.right hypochondriac region C. left lumbar region
B.left hypochondriac region D. epigastric region
54. What is the origin of the splenic aretry?
A.coeliac C.left gastric
B.hepatic D.superior mesenteric
55. Lymphoid tissues found at the lateral walls of the oropharynx are called:
A. pharyngeal tonsils C.lingual tonsils
B.palatine tonsils D.tubal tonsils
56. The thoracic duct empties into the :
A.left brachiocephalic vein C. right brachiocephalic vein
B.left internal jugular vein D. right subclavian vein
57. What is the location of the thymus gland?
A.superior mediastinum C. middle mediastinum
B.posterior mediastinum D. none of the above
58. The mucosa – associated lymphoid tissues include:
A.tonsils C.Paeyers patches
B. vermiform appendix D. all of the above
59. The following are parts of the femur EXCEPT:
A. Adductor tubercle C. linea aspira
B. Gluteal tuberosity D. surgical neck
60. The following are true of the hipbone EXCEPT:
A. Consists of ilium, ischium and pubis
B. Articulates with the sacrum
C. The ischium forms the upper flattened part of the bone
D. The three parts meet at the acetabulum
61. The following are parts of the tibia EXCEPT:
A. Soleal line C. Medial malleolus
B. Malleolar fossa D.Tuberosity

62. The following are descriptive of the hip joint EXCEPT:


A. Enclosed by a capsule
B. Type synovial ball- and- socket joint
C. Most complicated joint in the body
D. Capable of circumduction movement
63. Dorsiflexion movement at the ankle joint is performed by the following
muscles Except:
A. Soleus C. Extensor hallucis longus
B. Tibialis anterior D. Peroneus tertius
64. The following are type synovial joints of plane variety EXCEPT:
A. Subtalar C. Cuneonavicular
B. Tarsometatarsal D. Intercuneiform
65. Which of the following muscles extends and laterally rotates the hip
joint?
A. Gluteus maximus C. Gluteus minimus
B. Gluteus medius D. Quadratus femoris
66. The following muscles are supplied by the superior gluteal nerve
EXCEPT:
A. Gluteus maximus C. Gluteus minimus
B. Gluteus minimus D. tensor fasciae latae
67. Which of the following muscles extends the leg at knee joint?
A. Iliacus C. Sartorius
B. Psoas D. Quadriceps femoris
68. The following muscles are inserted to the greater trochanter of the
femur EXCEPT:
A. Obturator internus C. piriformis
B. Psoas D. Gluteus medius
69. The common nerve supply to the medial fascial compartment of the thigh:
thigh:
A. Femoral C. Sciatic
B. Obturator D. Sacral plexus
70. The following are branches of the lumbar plexus EXCEPT:
A. Genitofemoral
B. Ilioinguinal
C. Obturator
D. Posterior cutaneous nerve of the thigh
71.The sacral plexus is formed from the anterior rami of the following
nerves EXCEPT:
A. 2nd and 3rd lumbar nerves
B. 4th and 5th lumbar nerves
C. 1st and 2nd sacral nerves
D. 3rd and 4th sacral nerves
72. The following are true of the small saphenous vein EXCEPT:
A. Arises from the lateral part of the dorsal venous arch of foot
B. In its course passes behind lateral malleolus
C. In company of the saphenous nerve in the leg
D. A tributary of the popliteal vein
73. The following are true of the vertical group of superficial inguinal
lymph nodes EXCEPT:
A. Lies along the terminal part of the small saphenous vein
B. Receives most of the superficial lymph vessels of the lower limb
C. Lies in the superficial fascia below the inguinal ligament
D. Its efferent vessels pass through the saphenous opening
74. The intermediate compartment of the femoral sheath is occupied by the:
A. Femoral nerve C. Femoral artery
B. Femoral vein D. Lymph vessels

75. The following are descriptive of the anterior tibial artery EXCEPT:
A. Accompanied by the deep peroneal nerve
B. In front of the ankle joint, becomes the dorsalis pedis artery
C. Terminates by dividing into medial and lateral plantar arteries
D. A branch of the popliteal artery
76.  The  following  are  contents  of  the  Adductor’s  canal EXCEPT:
A. Femoral artery C. Femoral vein
B. Femoral nerve D. Saphenous nerve
77. Joins the lateral plantar artery in the sole of the foot:
A. Medial plantar C. Dorsalis pedis
B. anterior tibial D. Peroneal
78. The popliteal fossa contains the following EXCEPT:
A. Common peroneal nerve
B. Saphenous nerve
C. Tibial nerve
D. Small saphenous vein
79.The following are functions of blood, EXCEPT
A. Transport of carbon dioxide & oxygen
B. Regulation of body temperature
C. Synthesis of vitamins
D. Defense against infection
80. The most abundant component of blood plasma
A. Proteins C. Inorganic substance
B. Water D. Organic substance
81. All of the following are true of erythrocytes, EXCEPT
A. Anucleated and biconcave discs
B. Surface antigens on its plasma membrane
C. Number more than the leukocytes
D. Abundant cytoplasmic granules
82. Which formed elements help initiate an immune response and defend the body
against pathogens?
A. Segmenters C. Monocytes
B. Platelets D. Eosinophils
83. Leukocytes are divided into two classes based on the presence or absence of:
A. Mitochondria C. Specific granules
B. Polymorphic nuclei D. Ribosomes
84. Which of the following is NOT a characteristic of neutrophils?
A. most numerous leukocyte in peripheral blood
B. kill bacteria by secreting lysozyme
C. leave the blood to phagocytize pathogens in connective tissue
D. typically remain in blood, circulating from weeks to months
85. Which granulocyte phagocytizes allergens and chemically attacks parasitic
worms?
A. Basophil C. Neutrophil
B. Eosinophil D. Platelets
86. Which of the following does not describe platelets?
A. irregular, membrane-enclosed cellular fragments
B. less than ¼ the size of an erythrocyte
C. continually formed in the thymus by progenitor cells
D. normally about 150,000-400, 000/ul of adult blood
87. Which two types of cells release histamine and heparin?
A. neutrophils and platelets
B. basophils and lymphocytes
C. monocytes and eosinophils
D. mast cells and basophils

88. The cells that produce the smallest formed elements in peripheral blood
A. erythroblasts C. promyelocyte
B. megakaryocytes D. lymphoblast
89. Immature erythrocytes that still have nuclei and are actively synthesizing
hemoglobin
A. Normoblasts C. Pronormoblasts
B. Reticulocytes D. Progenitor cells
90. All blood cells arise from these stem cells
A. Precursor C. Pluripotential
B. Progenitor D. Blast
91. The intermediate cell stage just prior to a mature neutrophil
A. Promyelocyte C. Myelocyte
B. Metamyelocyte D. Band
92. Erythropoiesis occurs in
A. Lymphatic tissues C. Yellow marrow
B. Red marrow D. Kidneys
93.The hemopoietic organ of embryonic life which produces only erythrocytes
A. yolk sac C. liver
B. spleen D. red marrow
94. The major site of hemopoiesis at birth and in adulthood
A. Liver C. Spleen
B. Bone marrow D. Kidneys
95. Which of the following is NOT true of the spleen?
A. covered by dense connective tissue capsule
B. trabeculae divides it into cortex and medulla
C. splenic cords & sinusoids are found in the red pulp
D. white pulp contains central arteries surrounded by lymphocytes
96. The following are descriptive of the thymus, EXCEPT
A. has a connective tissue capsule
B. ts lobules are divided into cortex & medulla
C. abundant cortical thymic corpuscles
D. epithelial reticular cells are seen in the medulla
97. Which is NOT correct regarding the red bone marrow?
A. reticular connective tissue stroma
B. collagen fibers and proteoglycans in its matrix
C. parenchymal hematopoietic cords
D. areolar tissue support sinusoidal capillaries
98. The following are true of the lymph nodes, EXCEPT
A. divided into outer and inner cortex and medulla
B. reticular fibers and cells are found in the outer cortex
C. lymphatic nodules are more abundant in the inner cortex
D. branching cords containing B cells are seen in the medulla
99. Oval clusters of lymphatic cells with some extracellular matrix but no
connective tissue capsule describe
A. lymph nodes C. lymphatic sinuses
B. ymphatic nodules D. lymphatic cords
100. The following are mucosa-associated lymphoid tissues, EXCEPT
A. inguinal lymph nodes C. appendix
B. tonsils D.    Peyer’s  patches

Xavier University – Dr Jose P Rizal College of Medicine


Basic Biomedical Sciences – Second Bimonthly Exam
Set A: Modules 11 – 16

NAME ________________________ Oct. 9, 2006 SY 2006 – 07

MULTIPLE CHOICE: Write the letter of the BEST answer in CAPITAL letter in ink on
the answer sheet. NO ERASURES/SUPERIMPOSITIONS ALLOWED!

100. The following events occur to effect primary hemostasis, EXCEPT: D


A. injured blood vessel constricts
B. platelets are activated
C. von Willebrand factor (VWF) binds platelets to collagen
D. thrombospondin binds platelets to other platelets
101. When this factor is deficient, both primary and secondary hemostasis are also
deficient: A
A. fibrinogen C. thrombogen
B. prothrombin D. thrombospondin
102. Platelets are directly/indirectly involved in which of the following events? A
A. primary hemostasis, secondary hemostasis, clot retraction, vasoconstriction
B. primary hemostasis, secondary hemostasis, clot retraction
C. primary hemostasis, secondary hemostasis
D. primary hemostasis
103. Aside from calcium and phospholipid, the conversion of prothrombin to thrombin
requires the presence of these factors: B
A. Xa and XIa C. VIIIa and IXa
B. Va and Xa D. Xa and VIIa
104. When thrombin binds to thrombomodulin, thrombin: D
A. converts fibinogen to fibrin
B. stabilizes the loose fibrin clot
C. is activated to destroy factors Va and VIIIa
D. activates Protein C, which together with cofactor inhibits coagulation
105. Dissolution of clots after blood vessel damage has been repaired is primarily
the function of: C
A. platelets C. plasmin
B. thrombin D. antithrombin III
106. If Glena has factor VII deficiency, which of the following tests would be
prolonged?C
A. Bleeding time (BT)
B. Activated Partial Thromboplastin time (APTT)
C. Prothrombin time (PT)
D. Thrombin time(TT)
107. Ms  Philia’s  laboratory  test  results  are  as  follows:  BT  =  normal;;  APTT  =  
prolonged;
PT = normal; TT = normal. Based on these results, she could be deficient in any one
of these factors: C
A. V, VII, X C. VIII, IX, XI
B. I, II, X, V D. V, VIII, IX
108. Impulse conduction is slowest in the:
A. atrial muscles C. Bundle of His
B. AV node D. right bundle branch
109. The following associations between phases of the fast response action potential
(FRAP) and their primary causes are correct, EXCEPT: C
A. phase 0 – rapid and large Na+ influx
B. phase 1 – closure of Na+ channels
C. phase 2 – low outward K+ current
D. phase 3 – rapid outward K+ current

110. The volume of blood present in the ventricle at the end of atrial systole phase of
the cardiac cycle is the: B
A. end systolic volume C. systolic reserve volume
B. end diastolic volume D. diastolic reserve volume
111. Which of these represents ventricular repolarization? D
A. P wave C. QRS complex
B. PR interval D. T wave
112. The chest x-ray of Mr Hart showed left ventricular enlargement. If an ECG is
done, which of the following is expected to be prolonged?
A. ST segment C. QRS interval C
B. PR interval D. R-R interval
113. Compared to the first heart sound (S1), the second heart sound (S2) is: D
A. louder in intensity C. longer in duration
B. dull in quality D. higher in pitch
114. A medical intern wanted to listen to the heart sounds of his healthy 35 year old
brother. He placed the bell of his stethoscope over the 5 th intercostal space along the
left midclavicular line. Which heart sounds will he be able to hear? C
A. S1, S2, S3, S4 C. S1, S2
B. S1, S2, S3 D. S1
115. The following contribute to the generation of the pacemaker potential: C
A. Ca++ entry through T channels
B. Ca++ entry through T and L Ca++ channels
C. time dependent fall in K+ efflux and Ca++ entry via T Ca++ channels
D. time dependent fall in K+ efflux and Ca++ entry via L Ca++ channels
116. Applying  Starling’s  law  of  the  heart,  the  force  of  myocardial  contraction  is  
increased when these are increased, EXCEPT: D
A. end diastolic volume C. preload stress
B. venous return D. afterload stress
117. The following events occur during the isovolumetric phase of the cardiac cycle,
EXCEPT: C
A. onset of ventricular contraction C. decrease in ventricular volume
B. rapid rise in ventricular pressure D. first heart sound is heard
118. If impulse conduction to the right branch of the bundle of His is blocked such
that no action potential is conducted there, which of the following would occur? C
A. impulse is not conducted to the right ventricle
B. delayed closure of the aortic valve
C. splitting of the first and second heart sounds
D. first heart sound is due only to closure of the mitral valve
119. Both hyper- and hypokalemia may show the following effects, EXCEPT: D
A. decreased resting membrane potential
B. slow conduction of action potential
C. heart may stop in diastole
D. shortened repolarization
120. Ms Mitra is suspected of having diseased papillary muscles of the left ventricle.
If this is true, the following are the expected findings, EXCEPT: D
A. systolic murmur C. increased end diastolic volume of left ventricle
B. mitral regurgitation D. increased stroke volume of left ventricle
121. When the force of right ventricular contraction is markedly diminished, which of
the following would increase? C
A. Central venous pressure (CVP) and peripheral venous pressure (PVP)
B. CVP, PVP, capillary hydrostatic pressure (Pc)and venous return (VR),
C. CVP, PVP and Pc
D. CVP
122. Which of the following, when increased, would decrease venous return? B
A. blood volume C. activity of the skeletal muscle pump
B. CVP D. activity of the respiratory pump

123. Which of the following tend to oppose capillary filtration? A


A. decreased mean arterial pressure (MAP) and decreased blood volume
B. decreased plasma protein and decreased plasma osmotic pressure
C. decreased plasma protein and decreased blood volume
D. arteriolar dilation and venous obstruction
124. When the MAP is increased, which of the following will reflexly increase to bring
down the MAP towards normal? D
A. angiotensin II C. aldosterone
B. antidiuretic hormone D. atrial natriuretic factor
125. Increasing the following tends to increase MAP: C
A. mean systemic filling pressure (Psf), heart rate (HR), stroke volume (SV)
B. Psf, HR, SV, VR, hematocrit, arteriolar diameter
C. Psf, HR, SV, VR, hematocrit
D. Psf, HR, SV, VR
126. Which of the following is involved in the short-term regulation of MAP:A
A. carotico-aortic sinus reflex C. renin-angiotensin-aldosterone system
B. vasopressin system D. renal mechanism
127. Which of the following genes of the ABO blood group system is necessary for
the expression of A and B genes on red cells? B
A. Z gene B. H gene C. Se gene D. X gene
128. Which of the following antigens of the Rh blood group system has the greatest
immunogenicity property? C
A. antigen c B. antigen C C. antigen D D. antigen e
129. During routine blood typing, agglutination occurred at room temperature, at 4 o
Celsius but not at 37o Celsius. Based on these results alone, the following conditions
may be suspected, EXCEPT: D
A. viral pneumonia C. paroxysmal cold hemoglobinuria
B.  Raynaud’s  phenomenon D. acquired hemolytic anemia
130. During cross-matching, agglutination was observed at the thermo phase. Which
of the following antibodies may have been responsible for this effect? A
a
A. anti-c B. anti-M C. anti-Le D. anti-K
131. During cross-matching, agglutination was observed in the minor but not in the
major cross-match. Which of the following should be done next? C
A. perform the direct antiglobulin test
B. cross-matching should be performed again
C. donor blood may be transfused but at a slow rate
D. dilute the serum with normal saline in a 1:2 or 1:3 ratio
132. The parents of a baby are Ror’   and   R1R2, respectively. The following are
possible genotypes of their offspring, EXCEPT: C
A. cDe/Cde B. cDe/cDE C. Cde/cde D. Cde/Cde
133. A D negative mother is pregnant for the first time. Her husband is D positive.
Hemolytic disease of the newborn due to Rh incompatibility may occur if: D
A. fetal red cells enter the maternal circulation at the 26 th week of gestation
B. the baby is D positive and the mother has a cross-reacting antibody (anti-C)
C. the mother has received D positive blood 3 years prior to the pregnancy
D. the baby is D positive and the mother has immune anti-D in her blood
134. Blood typing showed negative results with anti-A and anti-B on forward typing.
Reverse typing showed negative results with A cells, B cells and O cells. Assuming
that blood typing was performed properly, which of the following is the most logical
conclusion? C
A. type AB, newborn C. type O, newborn
B. type AB, elderly D. type Oh
135. Which of the following is mainly responsible for capillary hematocrit being lower
than venous hematocrit? D
A. Fahreus-Lindqvist phenomenon C. tank treading
B. shear thinning D. plasma skimming

136. Blood may behave as a Bingham body in which of the following blood vessels?
B
A. large arteries C. large veins
B. capillaries D. medium-sized arteries
137. Which of the following is the main reason why the entire output of the right
ventricle goes to the pulmonary circulation? C
A. Poiseuille-Hagen formula C. continuity principle
B. law of Laplace D. Bernouilli principle
138. The following factors can give rise to turbulent flow of blood, EXCEPT: A
A. increase in blood viscosity C. Reynolds number over 1000
B. decrease in vessel radius D. increase in blood temperature
139. Excess afterflow following removal of an occluding force in a blood vessel is
called:
A. myogenic mechanism C. active hyperemia
B. metabolic mechanism D. reactive hyperemia
140. Which of the following substances released by vascular endothelium is currently
believed to be responsible for penile erection? C
A. prostacyclin C. nitric oxide
B. thromboxane A2 D. endothelin 1
141. Which of the following is mainly responsible for phagocytic defense at the
interstitial spaces of the lung parenchyma? A
A. macrophage B. mast cell C. histiocyte D. plasma cell
142. Which of the following nerve fibers is/are involved in the cough reflex? C
A. trigeminal B. olfactory C. vagus D. all of these
143. Which of the following lung pressures becomes positive only during forced
expiration? B
A. intrapulmonic pressure C. transpulmonary pressure
B. intrathoracic pressure D. transthoracic pressure
144. Which of the following lung pressures is mainly responsible for expansion of the
alveoli?
A. intrapulmonic pressure C. transpulmonary pressure C
B. intrathoracic pressure D. transthoracic pressure
145. An increase in both the supero-inferior and antero-posterior diameters of the
thorax is directly attributed to activity of the: D
A. external intercostals C. scalene muscles
B. rectus abdominis D. diaphragm
146. A normal subject breathes quietly in and out of the mouthpiece connected to a
spirometer. After a quiet exhalation, he exhales forcibly into the mouthpiece. The
amount of air that remains in his lungs is the: B
A. functional residual capacity C. vital capacity
B. residual volume D. expiratory reserve volume
147. Which of the following methods of measurement of lung volumes makes use of
helium?
A. Benedict-Roth spirometry C. closed circuit method C
B. plethysmography D. open circuit method
148. Physiologic dead air space is increased by the following, EXCEPT: B
A. extension of the neck C. regional vasoconstriction
B. bronchoconstrictors D. snorkel breathing
149. Which of the following is directly responsible for allowing pressures in different-
sized alveoli to remain the same? C
A. law of Laplace C. surfactant
B. elastance D. surface tension
150. Which of the following measures of ventilatory capacity measures the largest
volume of gas that can be moved into and out of the lungs in one minute by voluntary
effort? A
A. maximal breathing capacity C. timed vital capacity
B. respiratory minute volume D. forced expiratory volume

151. Which   of   the   following   serves   to   “fine-tune”   the   breathing   pattern   of   a   normal
individual?
A. pontine respiratory group C. dorsal respiratory group A
B. preBotzinger complex D. ventral respiratory group
152. J receptors are involved in the following reflexes, EXCEPT: B
A. Hering-Breuer deflation reflex C. pulmonary embolism reflex
B. pharyngeal dilator reflex D. pulmonary chemoreflex
153. Tachypnea is the expected response of which of the following reflexes? D
A. Hering-Breuer inflation reflex C. arterial chemoreceptor reflex
B. Hering-Breuer deflation reflex D. pulmonary vascular congestion reflex
154. A lesion at the brain stem extending from the dorsomedial medulla caudally to
the obex will most likely result in which of the following types of respiration? C
A. Kussmaul respiration C.  Biot’s  respiration
B. hyperventilation D. Cheyne-Stokes respiration
155. Diffusion of oxygen from the alveoli to the capillary is in accordance with which
of the following gas laws? D
A.  Avogadro’s  law C.  Charles’  law
B.  Graham’s  law D.  Henry’s  law
156. The oxygen carrying capacity of blood of a normal male with 18 gm% of
hemoglobin is:
A. 24.48 ml/100 ml of blood C. 25.04 ml/100 ml of blood A
B. 24.48 ml/1000 ml of blood D. 25.04 ml/1000 ml of blood
157. The following, when increased, will increase the rate of gas diffusion, EXCEPT:
C
A. gas temperature C. molecular weight of the gas
B. gas solubility coefficient D. membrane surface area
158. Which of the following is the main reason for the upper lung having greater
ventilation than the lower lung? B
A. descent of the diaphragm C. expansion of the ribs
B. transpulmonary pressure differences D. assumption of the erect position
60. The heart and the pericardium is located in the ___ mediastinum:
A. anterior C.middle
B. superior D.posterior
61. The following are openings of the right atrium EXCEPT:
A.superior vena cava C.left atrioventricular
B. inferior vena cava D.coronary sinus
62. What guards the right atrioventricular opening?
A.pulmonary valve C.mitral valve
B. aortic valve D.tricuspid valve
63. Which is false regarding the right ventricle of the heart?
A. forms the apex of the heart
B. its wall is thinner than those of the left ventricle
C. moderator band crosses ventricular cavity
D. where pulmonary trunk arises
64. Oxygenated blood from the lungs will next be carried to the left atrium
thru :
A.pulmonary artery C. pulmonary trunk
B. pulmonary veins D. none of the above
65. The following are branches of the right coronary artery EXCEPT:
A.anterior descending C.anterior ventricular ( marginal)
B. right conus D. atrial branches
66. Blood from the anterior part of the apex of the heart is drained by which
cardiac vein?
A.middle cardiac C.small cardiac
B.great cardiac D. anterior cardiac
67. The following are branches of the coeliac trunk EXCEPT:
A.right gastric C.common hepatic
B. left gastric D. splenic
68. These are branches of the thoracic aorta EXCEPT:
A. bronchial C. esophageal
B. phrenic D. lumbar
69. What is the origin of the brachial artery?
A. radial C.axillary
B. ulnar D.subclavian
70. These are paired branches of the abdominal aorta EXCEPT:
A. common iliac C.renal
B. superior mesenteric D.gonadal
71. The long saphenous vein empties into which of the following veins?
A. femoral C.popliteal
B. external iliac D. posterior tibial
72. What is the superficial vein which is found in front of the elbow?
A.brachial C.median cubital
B. radial D. basilic
73. The nasal septum is formed by the ethmoid and which of the following?
A. palatine C,maxilla
B. sphenoid D.vomer
74. What opening is found in the inferior nasal meatus ?
A.nasolacrimal duct C. frontal sinus
B. ethmoid sinus D.maxillary
75. Which cartilage of the larynx covers laryngeal inlet during swallowing?
A.thyroid C.epiglottis
B.cricoid D. arytenoid
76. The following intrinsic muscles of the larynx are supplied by the
recurrent laryngeal nerve EXCEPT:
A.oblique arytenoid C.transverse arytenoid
B. cricothyroid D.thyroarytenoid
77. These are anterior relations of the trachea EXCEPT:
A.esophagus C.thymus
B. arch of aorta D. left brachiocephalic veins
78.The following describe the left bronchus EXCEPT:
A.longer C.more vertically placed
B.narrower D. divides into superior and inferior lobar bronchus
79. Which of the following supplies the visceral pleura?
A. pulmonary plexus C.intercostal nerves
B. phrenic nerve D.all of the above
80. Which is false regarding the left lung?
A.has two fissures
B. long and narrow
C.has the cardiac notch
D. divided into the superior and inferior lobes
81.The following are true of cardiac muscle cells, EXCEPT C
A. its striations are similar to that of skeletal muscle
B. the large diameter of T-tubules slows down its contraction
C. ventricular cells are more granulated than atrial muscle fibers
D. terminals of sarcoplasmic reticulum form a diad with T-tubules
82.The following are descriptive of the endocardium, EXCEPT D
A. Has a single layer of flat cells
B. Has a layer of collagen and elastic fibers
C. Contains Purkinje cells
D. Supported by reticular connective tissue
83.Which of these is NOT true of the epicardium? D
A. lined by mesothelium
B. contains loose connective and adipose tissue
C. where blood and nervous supply to the heart are found
D. also called the parietal pericardium
84.Which of the following cardiac cells is specialized for contraction? A
A. Ordinary C. Transitional
B. P D. Purkinje
85.The following are true of transitional cells, EXCEPT B
A. its inner structure is intermediate between P and ordinary cells
B. make connections with Purkinje cells only
C. form contacts with other transitional cells
D. has membrane to membrane contact with P cells
86. All of the following are found in the intercalated disc, EXCEPT D
A. macula adherens C. gap junctions
B. fascia adherens D. tight junctions
87. Which of the following provide direct cell-to-cell communication and channels for
passage of small ions in cardiac muscle cell? C
A. macula adherens C. gap junctions
B. fascia adherens D. tight junctions
88. Which of the following is characteristic of an arteriole? C
A. thick adventitia, large lumen
B. large amounts of elastic fibers in intima
C. muscular media, absent external lamina
D. thick subendothelial layer
89.The following are descriptive of medium-sized arteries, EXCEPT D
A. subendothelial fibroblasts
B. adipose tissue in adventitia
C. prominent elastic laminae
D. abundant connective tissue in media
90.The following are generally true of veins, EXCEPT B
A. the adventitia is the thickest layer in venules
B. prominent subendothelial layer
C. smooth muscle cells in the media
D. collagenous adventitial layer
91.Which of the following would be easiest for invading cancer cells to enter? A
A. lymph capillary C. fenestrated capillary
B. venule D. sinusoids
92. Which capillary type has a continuous basal lamina and a diaphragm covering large
fenestrae? B
A. Somatic C. Sinusoidal
B. Visceral D. Lymphatic
93. The following are structural differences between veins and lymphatic vessels,
EXCEPT C
A. large lymphatic vessels have more internal valves
B. large veins have morel-developed adventitia
C. numerous fenestrae in large lymphatic vessels
D. large veins have well-developed intima
94.Which of the following cell types is not likely to be found as part of the alveolar septa
of the lung? D
A. Type II C. Macrophages
B. Type I D. Ciliated columnar
95. The true vocal cord can be distinguished from the false vocal cord because the true
vocal cord A
A. is lined by stratified squamous epithelium
B. contains only serous glands
C. contains a core of hyaline cartilage
D. made up of a core of elastic connective tissue
96.Which of the following is not a part of the respiratory membrane barrier? B
A. Pneumonocyte I C. Basal lamina of endothelial cell
B. Pulmonary macrophage D. Endothelial cell

97.In which of the following would you find the highest concentration of Clara cells? C
A. extrapulmonary bronchus C. terminal bronchiole
B. intrapulmonary bronchus D. interalveolar septum
98.Which of these is the most important structural difference between a terminal
bronchiole and the respiratory bronchiole? D
A. presence of hyaline cartilage
B. arrangement of smooth muscle cells
C. ciliated epithelium
D. absence/presence of alveoli
99.The following are descriptive of the extrapulmonary bronchus, EXCEPT B
A. hyaline cartilage in adventitia
B. simple columnar ciliated epithelium with goblet cells
C. smooth muscles cells in lamina propria
D. distinct basement membrane
100.Which of these is NOT true of the pharynx? D
A. the nasopharynx has a well-developed submucosa
B. dense connective tissue in the lamina propria
C. mixed glands in areas with ciliated epithelium
D. three-layers thick muscular wall

Xavier University – Dr Jose P Rizal College of Medicine


Basic Biomedical Sciences – Second Bimonthly Exam
Set B: Modules 17 – 20

NAME ________________________ Oct. 11, 2006 SY 2006 – 07

MULTIPLE CHOICE: Write the letter of the BEST answer in CAPITAL letter in ink on
the answer sheet. NO ERASURES/SUPERIMPOSITIONS ALLOWED!
1. Which of the following substances is synthesized and secreted by the kidneys, and
plays an important part in blood coagulation?
A. erythropoietin B. calcitriol C. kallikrein D. renin
2. An average male has 5.2 mg/dl of uric acid in his serum. Assuming that glomerular
filtration rate is normal, which of the following will most likely happen?
A. uric acid will be freely filtered and completely reabsorbed
B. uric acid will be freely filtered and partly reabsorbed
C. uric acid will be partly filtered, partly reabsorbed and partly secreted
D. uric acid will be freely filtered but not reabsorbed
3. An average male has a hematocrit of 55 vol%. Assuming that glomerular filtration rate
is normal, the amount of plasma that will be filtered across the glomeruli is: C
A. 495 ml/min C. 99 ml/min
B. 605 ml/min D. 121 ml/min
4. Which of the following substances is secreted by the tubules via diffusion trapping?
A. hydrogen B. organic acid C. potassium D. aspirin
5. Most of the substances filtered at glomeruli are reabsorbed at the: A
A. proximal tubule C. loop of Henle
B. distal tubule D. collecting duct
6. Which of the following is mainly responsible for concentration of fluid in the collecting
duct? C
A. countercurrent multiplier system C. anti-diuretic hormone
B. countercurrent exchange D. aldosterone
7. Composition of urine is final once it enters the: B
A. collecting duct C. major calyces
B. papillary ducts of Bellini D. minor calyces
8. Bladder plasticity is mainly attributed to: D
A. inhibition of phrenic nerve activity by pontine centers
B. voluntary control of micturition by the cerebral cortex
C. contraction of the external vesical sphincter
D. unfolding of mucosal folds
9. Which of the following will suggest an inhibitory influence on micturition? C
A. spinal cord transection C. supracollicular transection
B. intercollicular transection D. transhypothalamic transection
10. Gower syndrome is mainly due to: D
A. sudden loss of temperature and BP upon micturition
B. stimulation of nerves at the tip of the penis
C. activity of descending pathways from the pons
D. activation of a vasovagal reflex
11. Which of the following is expected to take place in a normal male whenever his
extracellular fluid volume decreases? B
A. increase in filtration fraction
B. increase in renal vascular resistance
C. stimulation of the thirst center
D. release of anti-diuretic hormone
12. A 70 kg female has a hematocrit of 40%. Her extracellular fluid volume would be
approximately: D
A. 42 liters C. 21 liters
B. 35 liters D. 14 liters
13. Which of the following is most likely to take place in a person with extensive burns?
A. decrease in ECF volume, normal ECF osmolarity
B. decrease in ECF volume, decrease in ECF osmolarity
C. decrease in ECF volume, increase in ECF osmolarity
D. increase in ECF volume, decrease in ECF
14. Pure buffer response to a primary gain in HCO3- results in the following directional
changes when the new equilibrium is reached, EXCEPT: A
A. PCO2 = increased C. [BB] = increased
B. [HBuf] = decreased D. pH = increased
15.Renal formation of new HCO3- is a compensatory response in: C
A. metabolic acidosis C. respiratory acidosis
B. metabolic alkalosis D. respiratory alkalosis
16.Ten year old Tom was admitted to the hospital because of severe diarrhea. Aside
from dehydration, he also had an acid-base disturbance. Body responses to such a
disturbance would include following, EXCEPT: D
A. hyperventilation
B. renal addition of new HCO3-
C. renal excretion of acid
D. buffering by the bicarbonate and non-bicarbonate systems
17. Marie was admitted to the hospital during an asthmatic attack. Initial laboratory
tests showed mild acid-base disturbance. Three days later a repeat lab tests were
done and the results were the following: pH = 7.39; [HCO3-] = 25.89 mmls/L;
PCO2 = 44.97 mmHg; [BE] = (+) 1.9 mmols/L. Based on these second lab results,
you can say that Marie has a: D
A. completely compensated respiratory acidosis
B. partially compensated respiratory acidosis
C. completely compensated metabolic alkalosis
D. normal acid-base status
18. PCO2 is increased in the following conditions, EXCEPT: A
A. uncompensated respiratory acidosis
B. partially compensated respiratory alkalosis
C. partially compensated metabolic alkalosis
D. completely compensated metabolic alkalosis
19. The following are true of the  Henle’s  loop,  EXCEPT  C
A. Descending limb is lined by simple squamous cells
B. Sparse and short microvilli
C. Ascending limb lined by ciliated cells
D. Numerous mitochondria in cells of ascending limb
20.The following are descriptive of the ureter, EXCEPT D
A. transitional epithelium
B. collagen and elastic fibers in lamina propria
C. ILOC arrangement of tunica muscularis, upper portion
D. Elastic outer coat
21.The following comprise the juxatglomerular apparatus, EXCEPT C
A. macula densa C. brush cells
B. extraglomerular mesangial cells D. juxtaglomerular cells
22.Which is NOT true of the urinary bladder? A
A. ciliated lining epithelium
B. elastic and reticular tissue in lamina propria
C. 3 layers of smooth muscle fibers
D. lined by serous membrane in non-peritoneal part
23.The following are descriptive of the female urethra, EXCEPT D
A. fibroelastic tissue in adventitia
B. largely lined by stratified sqhamous
C. mucosa arranged in elongated folds
D. abundant adipose tissue in tunica adventitia

24.Which of the following cells is the site of action of ADH? C


A. Mesangial C. Principal
B. Intercalated D. Macula densa
25. The following are descriptive of the kidney Except:
A. An intraperitoneal organ
B. Covered by fibrous capsule
C. The right lies slightly lower than the left
D. Lies on the posterior abdominal wall
26. The following are true of the ureter Except:
A. Its upper expanded end is formed by the renal pelvis
B. Courses down the lateral wall of the pelvis
A. Measures about 10 inches long
B. Terminates at the superior surface of the urinary bladder
27.The following are descriptive of the urinary bladder Except:
A. It has a base, apex,, fundus, and body
B. Partially covered by peritoneum
C. Its base rests on the prostate gland in male
D. Lies within the pelvis
28. The following are constrictions of the ureter Except:
A. Uteropelvic junction C. vesicouterine junction
B. Where it crosses pelvic brim D. where it crosses psoas muscle
29.The following are true of the male urethra Except:
A. Acts as a route for urine and semen
B. About 20 cm long
C. Divided into prostatic, membranous, and penile urethra
D. Extends from the base of the bladder to the external meatus
30. The following coverings support the kidneys and hold them in position on the
posterior abdominal wall:
A. Fibrous capsule C. renal fascia
B. Perirenal fat D. pararenal fascia
31.The rectus sheath is formed by the aponeuroses of the following muscles Except:
A. Rectus abdominis C. internal oblique
B. External oblique D. transversus abdominis
32. The rectus sheath contains the following structures Except:
A. The anterior rami of the lower six thoracic nerves
B. Superior epigastric vessels
C. Iliohypogastric nerve
D. Inferior epigastric vessels
33. The following are posterior abdominal wall muscles Except:
A. Iliacus C. psoas
B. Piriformis D. quadratus lumborum
34.The region of the abdomen found between the lumbar regions is the:
A. Umbilical C. epigastric
B. Hypogastric D. iliac
35. The following extrinsic muscles of the tongue are supplied by the hypoglossal
nerve, Except:
A. Hyoglossus C. palatoglossus
B. Genioglossus D. styloglossus
36. The following are descriptive of the esophagus Except:
A. Commences at the level of 6th cervical vertebra
B. Lies anterior to the trachea
C. 25 cm long
D. Runs through the posterior mediastinum in the thorax
37.The following structures form the stomach bed Except:
A. Liver C. left suprarenal gland
B. Pancreas D. transverse colon

38. The part of the stomach between the cardiac notch and the incisura angularis is
the:
A. Body C. pyloric canal
B. Antrum D. fundus
39. The following structures pass to the porta hepatis within lesser omentum Except:
A. Hepatic vein C. hepatic artery
B. Portal vein D. common bile duct
40. The following are true of the pancreas Except:
A. Consists of fundus, body, and neck
B. Its ducts open into the second part of duodenum
C. Both an exocrine and endocrine glands
D. Supplied by the splenic artery
41. The small intestine differ from the large intestine by the presence of the following,
EXCEPT:
A. Plicae circulares C.    Peyer’s  patches
B. Appendices epiploicae D. interstinal villi
42. The following are sites of portal-systemic anastomoses Except:
A. Esophagus C. rectum
B. Stomach D. umbilicus
43. Which of the following is not a tributary of the portal vein?
A. Superior mesenteric vein C. inferior mesenteric
B. Hepatic vein D. splenic vein
44. Which of the following arteries supplies the GIT from the lower 3rd of the
esophagus to the 2nd part of the duodenum?
A. Gastroduodenal C. celiac
B. Superior mesenteric D. inferior mesenteric
45. Which of the following do NOT relate to the tongue?
A. Serous membrane at bottom of circumvallate papillae
B. Oral part has many filiform papillae
C. Tip is innervated by CN V nerve for general sensation
D. Skeletal muscle at the center with muscle spindles
46. The bulk of the tooth is formed by this cell:
A. Fibroblast C. Odontoblast
B. Cementocyte D. Ameloblast
47. The following are true regarding the large intestine, EXCEPT:
A. Simple columnar epithelium, thin striate border, many goblet cells
B. Submucosa with loose connective tissue, blood vessels, nerve cells, without glands
C. Network of neurons in the muscularis externa for intestinal contractions
D. Both layers of the muscularis externa surround the organ completely
48. Which of these features is NOT correct regarding the esophagus?
A. Stratified squamous epithelium
B. Mucous glands is submucosa
C. Muscularis externa with smooth muscle entirely
D. Outermost wall could be adventitia or serosa
49. The pyloric glands contain these cells, EXCEPT:
A. Chief cells C. Gastrin cells
B. Mucous cells D. Parietal cells
50.  Which  of  these  ‘cell  to  role’  association  is  NOT  correct?
A. D cell > inhibition of other enteroendocrine cells
B. Oxyntic cells > intrinsic factor secretion for Vit B 12 absorption
C. G cells > secretes gastric acid directly
D.  Paneth’s  cell  >antibacterial  lysozyme  secretion
51. The stomach mucosa is protected from the effects of gastric acid by the secretion of
this cell:
A. Columnar epithelial cell C. Mucous neck cell
B.  Goblet  cell                                                                                  D.  Brunner’s  cells

52. These cells are common to both small and large intestine, EXCEPT:
A. Enterocytes C. Goblet cells
B. Enteroendocrine cells D.  Paneth’  cells
53. A microscopic section shows plicae circulares, having intestinal villi, glands of
Lieberkuhn with no mucous glands. This organ is likely the:
A. Ileum C. Duodenum
B. Jejunum D. Appendix
54. A microscopic section displays a parenchyma with demilunes. Which of the following is
NOT likely?
A. Sublingual gland C. Submaxillary gland
B. Parotid gland D. Laryngeal glands
55. The precursors of digestive enzymes are secreted by the:
A. Alpha cell C. Acinar cells
B. Beta cell D. Delta cells
56. Which organelle in the hepatocytes is most involved in liver detoxification?
A. Smooth endoplasmic reticulum C. Rough endoplasmic reticulum
B. Golgi complex D. Lysosome
57. The hepatic sinusoids is a pathway for the following, EXCEPT:
A. Arterial blood C. Bile
B. Venous blood D. No exception mentioned
58. The following are true of mastication, EXCEPT:
A. it is purely under reflex control
B. it is not an absolutely essential step in digestion
C. it breaks food to smaller pieces to facilitate swallowing
D. it increases surface area of food and expose them to digestive enzymes
59. The following occur during deglutition, EXCEPT:
A. tongue moves a bolus of food into the pharynx
B. soft palate moves up to prevent reflux of food into the nasopharynx
C. respiration is inhibited, larynx raised, glottis closed so food does not enter trachea
D. respiration resumes just before the food enters the esophagus
60.These are true of the basic electrical rhythm (BER), EXCEPT:
A. these originate from interstitial cells of Cajal
B. these are spontaneous rhythmic fluctuations in membrane potential
C. present in stomach, small and large intestines but not in esophagus
D. each BER wave is usually followed by contraction
61.Peristalsis occurs in these parts of the GIT:
A. esophagus
B. esophagus, stomach
C. esophagus, stomach, small intestine
D. esophagus, stomach, small intestine, large intestine, rectum
59. During gastric emptying, there is increased contractions of the ileum. This is known as
the:
A. ileogastric reflex C. gastrocolic reflex
B. gastroileal reflex D. intestinointestinal reflex
60. The following associations between gastrointestinal hormones and their actions are
correct, EXCEPT:
A. gastrin – stimulate gastric acid secretion
B. histamin – inhibit gastric acid secretion
C. cholecystokinin – cause contraction of the gallbladder
D. Secretin – secretion of watery, alkaline pancreatic juice
61. These are true of saliva, EXCEPT:
A. facilitates swallowing
B. contains two digestive enzymes, lipase and amylase
C. food in the mouth causes reflex secretion of saliva
D. sympathetic stimulation causes large amounts of watery saliva

62. The following are true of bile, EXCEPT:


A. produced by hepatocytes
B. released into the duodenum via common bile duct
C. during meals bile flows into the gallbladder for storage
D. composition of hepatic bile differs from that of gallbladder bile
63. The following are true of fat digestion/absorption, EXCEPT:
A. digestion occurs in the small intestine
B. bile salts are needed to emulsify fats to enhance digestion
C. colipase splits triglyceride to monoglyceride and fatty acids
D. absorbed lipids enter into lacteals as chylomicrons
64. As regards carbohydrate digestion/absorption, which of these is NOT true?
A. digestion may begin in the mouth by salivary amylase
B. digestion also occurs at the brush border of small intestinal epithelium
C. absorbed as monosaccharides glucose, fructose, & galactose
D. glucose absorption is enhanced by potssium
65. The following are true of protein digestion/absorption, EXCEPT:
A. digestion begins in the stomach by the action of pepsin
B. luminal enzymes my digest proteins to peptides
C. transported into epithelial cells as oligopeptides or amino acids
D. final absorption of amino acids occur in the small and large intestines
66. The following are true of defecation, EXCEPT:
A. defecation center is located in L5 – S4 of spinal cord
B. the urge to defecate first occurs when rectal pressure reaches 18 mmHg
C. can be voluntarily inhibited by contracting the external sphincter
D. voluntary control of defecation is lost when rectal pressure reaches 55 mmHg
67. These structures contract to effect vomiting:
A. diaphragm, abdominal muscles, pylorus, fundus, cardia
B. diaphragm, abdominal muscles, pylorus, fundus
C. diaphragm, abdominal muscles, pylorus
D. diaphragm, abdominal muscles
68. An individual is considered overweight when actual body weight is how many percent
above the desirable body weight?
A. 20-25% B. 5-10% C. 30% D. 11-20%
72.  Using  the  Thanhausser’s  formula,  a  Filipino  laborer  who  is  5’5  tall  should  ideal  weigh:
A. 70 kg B. 63 kg C. 60 kg D. 65 kg
73. Supplemental amount of this vitamin is recommended immediately after birth: A.
A B. D C. K D. E
74. All of the following are rich sources of Vitamin C, EXCEPT:
A. water melon B. ripe papaya C. guava D. lacatan
75. Serves as coenzyme in a large number of oxidation-reduction reactions:
A. Thiamine B. Niacin C. Riboflavin D. Pyridoxine
76. Scurvy is most likely to occur in the following group of persons, EXCEPT: A.
those subsisting on monotonous diet C. infant without vitamin supplement
B. psychoneurotic individual D. none of the above
77. The following may result in the presence of vitamin D deficiency, EXCEPT:
A. separation of the epiphysis C. anterior curving of the thigh
B. softening of the cranial bones D. enlarged costochondral junction
78. The neurologic disorder associated with deficiency of which of these vitamins may
be secondary to a relative deficiency of histidine: A.
B1 B. B2 C. B6 D. B12
79. Deficiency of this vitamin can potentiate deficiency of Niacin:
A. Folic acid B. Pyridoxin C. Ascorbic acid D. Pantothenic acid
80. Milk with highest fat content:
A.  cow’s B.  Carabao’s C. human D.  goat’s
81. Special component of human milk that helps in iron absorption:
A. Lactoferron B. Transferrin C. Curd D. Ferritin

82. At the beginning of a single nursing, the milk secreted is high in :


A. water B. fats C. protein D. minerals
83. Specific protein that transport retinal within the cell:
A. aporetinol C. retinal binding protein
B. aporetinal D. cellular retinal binding protein
84. The following are true of colustrum, EXCEPT:
A. yellowish in color C. has laxative effect
B. coagulates on boiling D. lower iodine number in fats
85. Human milk is the best food for infants because of the following reasons, EXCEPT:
A. excellent source of vitamin A & niacin
B. provides immunity against some diseases
C. milk curds are easier to digest
D. reduces the incidence of GI upsets
86. The RDA is intended :
A. for daily individual use
B. to determine excellent food sources
C. to evaluate nutrient content of a diet
D. as basis for estimating national food needs
87. The RDA for calories for a reference woman in calories/day is:
A. 1,500 B. 1,900 C. 2,300 D. 2,500
88. The recommended nutritional requirements may be used by an individual if they fit
within the context of:
A. basal metabolism C. reference man or woman
B. ideal body weight D. safe practical allowance
89. The specific dynamic action of food is computed based on:
A. 10% of IBW B. 10% of BMR C. 10% of PA D. 10% of TER
90. This vitamin occurs bound to an active Coenzyme A group and is a transient carrier
of acyl group:
A. Pyridoxine B. Riboflavin C. Nicotinamide D. Pantothenic acid
91. A short method of calculating energy for physical activity of a male medical student is to
allow how many percent of his basic needs?
A. 50% B. 30% C. 25% D. 75%
92. The clinical triad of dermatitis, diarrhea and dementia is characteristic of:
A. scurvy B. xerophthalmia C. pellagra D. Beriberi
93. 170 calories would be equivalent to which of the following meal exchange list?
A. 1 milk exchange C. 4 fat exchange
B. 1 meat exchange D. 1 rice exchange
94. The daily requirement of this vitamin depends on caloric intake:
A. Thiamin B. cyanocobalamin C. folic acid D. Biotin
95. One exchange of group 1-B vegetable is equal to:
A. 1/ cup red squash, cooked C. ½ cup monggo bean sprout
B. ½ cup alugbati D. 1 sliced avocado
96. Biochemical function of folacin:
A. CO2 transfer reaction C. Methylation reaction
B. one-carbon transfer reaction D. thymidylate synthesis
97. This trace element is closely associated with vitamin E action:
A. Copper B. Lead C. Selenium D. Cobalt
98. The percentage edible portion(EP%) for 1 piece yellow squash weighing 200grams
and refuse of 40grams is:
A. 160 B. 80 C. 240 D. 60
99. Which of the following food can be eaten as desired:
A. fruits B. rice C. milk D. vegetables
100. Efren has eaten 1 slice of papaya 200(grams) EP for breakfast. How much vitamin
C does he get if 100 grams of papaya yield CHO=29grams, Vit. A-600IU,
Vit. C =25 mg?
A. 75 grams B. 175mg C. 50grams D. 50mgs
Module 12 – Hemostasis

159. Which of the following events occur to effect primary hemostasis?


A. injured blood vessel dilate
B. platelets become flattened discs
C. von Willebrand factor (VWF) binds platelets to fibrinogen
D. fibrinogen forms a bridge between two opposing platelets
160. When this factor is deficient, both primary and secondary hemostasis are also
deficient:
A. fibrinogen C. thrombogen
B. prothrombin D. thrombospondin
161. Platelets are directly/indirectly involved in which of the following events?
A. primary hemostasis, secondary hemostasis, clot retraction,
vasoconstriction
B. primary hemostasis, secondary hemostasis, clot retraction
C. primary hemostasis, secondary hemostasis
D. primary hemostasis
162. Aside from calcium and phospholipid, the conversion of prothrombin to thrombin
requires the presence of these factors:
A. Xa and XIa C. VIIIa and IXa
B. Va and Xa D. Xa and VIIa
163. When thrombin binds to thrombomodulin, thrombin:
A. converts fibinogen to fibrin
B. stabilizes the loose fibrin clot
C. is activated to destroy factors Va and VIIIa
D. activates Protein C, which together with cofactor inhibits coagulation
164. The end point of the first stage of coagulation in the extrinsic pathway is;
A. formation of the complex of factor VII, tissue thromboplastin, and Ca++
B. formation of the prothrombin converting complex
C. conversion of prothrombin to thrombin
D. conversion of fibrinogen to fibrin
165. Aside from dissolving blood clots, plasmin also destroys these coagulation
factors:
A. V and VIII C. VIII and IX
B. V and IX D. IX and XI
166. If Sam has factor VII deficiency, which of the following tests would be
prolonged?
A. Bleeding time (BT)
B. Activated Partial Thromboplastin time (APTT)
C. Prothrombin time (PT)
D. Thrombin time(TT)
167. This substance is synthesized by endothelial cells and prevents activated
platelets from adhering to uninjured endothelium.
A. prostacyclin C. adenosine diphosphatase
B. antithrombin III D. thromboxane A2
168. Aleth’s  laboratory  test  results  are  as  follows:  BT  =  normal;;  APTT  =  prolonged;;
PT = normal; TT = normal. Based on these results, she could be deficient in any one
or a combinations of these groups of factors:
A. V, VII, X C. VIII, IX, XI
B. II, X, V D. V, VIII, IX

Module 13 – CVS 1
169. Impulse conduction is slowest in the:
A. atrial muscles C. Bundle of His
B. AV node D. right bundle branch
170. Which of the following associations between phases of the fast response action
potential (FRAP) and their primary causes is correct, EXCEPT:
A. phase 0 – rapid and large K+ influx
B. phase 1 – closure of K+ channels
C. phase 2 – low outward K+ current
D. phase 3 – rapid outward K+ current

171. The volume of blood present in the ventricle at the end of atrial systolic phase of
the cardiac cycle is the:
A. end systolic volume C. systolic reserve volume
B. end diastolic volume D. diastolic reserve volume
172. Which of these represents ventricular repolarization?
A. P wave C. QRS complex
B. PR interval D. T wave
173. The chest x-ray of Mr Hart showed left ventricular enlargement. If an ECG is
done, which of the following is expected to be prolonged?
A. ST segment C. QRS interval
B. PR interval D. R-R interval
174. Compared to the first heart sound (S1), the second heart sound (S2) is:
A. louder in intensity C. longer in duration
B. dull in quality D. higher in pitch
175. Dido wanted to listen to the heart sounds of Jojie. She placed the bell of her
stethoscope over the 5th intercostal space along the left midclavicular line. Which
heart sounds will she most likely hear?
A. S1, S2, S3, S4 C. S1, S2
B. S1, S2, S3 D. S1
176. According to Guyton, automaticity of the sinus nodal cells is due to:
A. opening of more Ca++ channels at end of an action potential
B. decrease in k+ efflux towards the end of an action potential
C. inherent leakiness of membrane to Na+ ions
D. spontaneous slow inward Ca++ current
177. Applying  Starling’s  law  of  the  heart, the force of myocardial contraction is
increased when this is increased:
A. CVP C. preload stress
B. PVP D. afterload stress
178. The following events occur during the isovolumetric contraction phase of the
cardiac cycle.
A. opening of semilunar valves C. decrease in ventricular volume
B. rapid rise in ventricular pressure D. second heart sound is heard
179. If impulse conduction to the right branch of the bundle of His is blocked such
that no action potential is conducted there, which of the following would occur? C
A. impulse is not conducted to the right ventricle
B. delayed closure of the aortic valve
C. splitting of the first and second heart sounds
D. first heart sound is due only to closure of the mitral valve
180. Both hyper- and hypokalemia may show which of the following effects?
A. more negative resting membrane potential
B. fast conduction of action potential
C. heart may stop in diastole
D. shortened repolarization
181. Ms Mitra is suspected of having diseased papillary muscles of the left ventricle.
If this is true, which of the following would result?
A. systolic murmur C. decreaqsed end diastolic volume of left ventricle
B. aortic regurgitation D. increased stroke volume of left ventricle

Module 14 – CVS 2
182. When the force of right ventricular contraction is markedly diminished, which of
the following would most likely increase?
A. Central venous pressure (CVP) C. stroke volume
B. mean arterial pressure (MAP) D. venous return

183. Which of the following, when increased, would decrease venous return?
A. blood volume C. activity of the skeletal muscle pump
B. CVP D. activity of the respiratory pump
184. Which of the following tend to oppose capillary filtration?
A. decreased mean arterial pressure (MAP) and decreased blood volume
B. decreased plasma protein and decreased plasma osmotic pressure
C. decreased plasma protein and decreased blood volume
D. arteriolar dilation and venous obstruction
185. When the MAP is increased, which of the following will reflexly increase to bring
down the MAP towards normal?
A. angiotensin II C. aldosterone
B. antidiuretic hormone D. atrial natriuretic factor
186. Mr Red has a hematocrit of 60 vol %. His MAP is likely to increase because his
_____ is also likely to increase as a result of his hematocrit level.
A. blood volume C. mean systemic filling pressure
B. venous return D. total peripheral resistance
187. Which of the following is involved in the short-term regulation of MAP:
A. stress-relaxation mechanism
B. renal-blood volume control system
C. carotic-aortic sinus reflex
D. renin-angiotensin-aldosterone system

Xavier University – Dr Jose P Rizal College of Medicine


Basic Biomedical Sciences – Third Bimonthly Exam
Set A: Modules 21 – 25

NAME ________________________ 8 Jan 2007 SY 2006 – 07

MULTIPLE CHOICE: Write the letter of the BEST answer in CAPITAL letter in ink on
the answer sheet. NO ERASURES/SUPERIMPOSITIONS ALLOWED!

1. The following are true of bilirubin transport, EXCEPT:


A. bilirubin bound to albumin is transported from peripheral tissues to the liver
B. binding with albumin makes bilirubin more soluble in plasma
C. in 100 ml of plasma, about 75 mg of bilirubin can be tightly bound to
albumin
D. antibiotics may compete with bilirubin for the binding sites in albumin
2. Which of the following is NOT true of conjugated bilirubin?
A. it is water soluble
B. secreted into bile by facilitated diffusion
C. most bilirubin secreted into bile are the form of bilirubin diglucuronide
D. B-glucuronidases in terminal ileum remove the diglucuronide from bilirubin
3. The following associations between conditions with jaundice and type of bilirubin
primarily increased are correct, EXCEPT:
A. hemolytic anemia – conjugated bilirubin
B. biliary tree obstruction – conjugated bilirubin
C. neonatal physiologic jaundice – unconjugated bilirubin
D. hepatitis – unconjugated bilirubin
4. Bilirubin is expected to be present in these conditions:
A. obstructive jaundice, hepatitis, hemolytic anemia, normal person
B. obstructive jaundice, hepatitis, hemolytic anemia
C. obstructive jaundice, hepatitis
D. obstructive jaundice
5. The rate limiting step in the metabolism of bilirubin is the:
A. uptake of bilirubin by the liver parenchymal cells
B. conjugation of bilirubin in the SER
C. secretion of conjugated bilirubin into the bile
D. conversion of glucuronides into urobilinogens in the intestine
6. The following are true of very low density lipoprotein (VLDL), EXCEPT:
A. most plasma VLDL is formed by liver cells
B. it is the precursor of intermediate-density lipoprotein (IDL)
C. its density is greater than that of chylomicrons (<0.05; VLDL, 0.95-1.006)
D. transports primarily cholesterol from liver to extrahepatic tissues
(triacylglycerol)
7. Which of the following is NOT true of high density lipoprotein (HDL)?
A. synthesized and secreted from liver and intestine
B. contains phospholipids and cholesterol
C. responsible for reverse cholesterol transport
D. its main apolipoprotein is apolipoprotein B
8. The main apolipoprotein of LDL is:
A. apolipoprotein A C. apolipoprotein C
B. apolipoprotein B D. apolipoprotein E
9. Glycogenolysis in muscle leads to formation of lactate instead of glucose due to the
absence of:
A. glucose-6-phosphatase C. debranching enzyme
B. phosphoglucomutase D. phosphorylase
10. The reaction that is common to glycolysis and glycogenesis is:
A. phosphorylation of glucose to glucose 6-phosphate
B. conversion of glucose to glucose 1-phosphate
C. glucose 1-phosphate reacts with uridine triphosphate
D. hydrolysis of pyrophosphate to inorganic phosphate
11. This liver enzyme is responsible for the inactivation of phosphorylase a to
phosphorylase b:
A. phosphorylase kinase C. protein phosphatase-1
B. cAMP-dependent protein kinase D. glycogen synthase
12. This disease is characterized by a deficiency of glucose-6-phosphatase:
A. Her’s  disease C.    Pompe’s  disease
B. Tarui’s  disease D. Von  Gierke’s  disease
13. In liver glycogenesis, glucose is phophorylated to glucose-6-phosphate. This reaction
is catalyzed by: FINALS
A. hexokinase C. glycogen phosphorylase
B. glucokinase D. glycogen synthase
14. Which of the following integrates the regulation of glycogenolysis and glycogenesis by
simultaneous activation of phosphorylase and inhibition of glycogen synthase?
A. Ca++ C. cAMP
B. calmodulin D. glucokinase
15. The phosphorylitic cleavage of the 1--> 4 linkages of glycogen to yield glucose 1-
phosphate is catalyzed by:
A. glucokinase C. phosphoglucomutase
B. glycogen synthase D. glycogen phosphorylase
16. These enzymes are involved in liver glycogenolysis, EXCEPT:
A. glycogen synthase C. glucan transferase
B. glycogen phosphorylase D. debranching enzyme
17. In the liver phosphorylase b is rephosphorylated to active phosphorylase a. This
reaction is catylyzed by:
A. phosphorylase kinase C. glucose-6-phosphatase
B. protein phosphatase – 1 D. glucomutase
18. Phosphorylase in muscle differs from that in liver because in muscle:
A. phosphorylase exists in two forms
B. phosphorylase a is active
C. phosphorylase b is dephosphorylated
D. phosphorylase is activated by cAMP
19. Which of these inactivates phosphorylase a? FINALS
A. Ca++ C. inhibitor-1
B. cAMP D. protein phosphatase-1
20. Which enzyme in glycogenolysis exposes the 1 --> 6 branch point?
A. transglucosidase C. branching enzyme
B. glucan transferase D. glucose-6-phosphatase
21. The final digestion of proteins to amino acids occur in these locations, EXCEPT:FINAL
A. intestinal lulmen C. cytoplasm of mucosal cells
B. brush border D. interstitial fluid bathing mucosal cells
22. These are true of pepsins, EXCEPT:
A. precursors are called pepsinogens
B. precursors are activated by gastric HCl
C. immediate products of peptic digestion are amino acids (polypeptides of
different sizes)
D. Alkaline pancreatic juice in duodenum & ileum terminate action of pepsin
23. The following associations between amino acids and their catabolic end products are
correct, EXCEPT:
A. proline – a-ketoglutarate C. threonine – pyruvate
B. arginine – oxaloacetate (aKG) D. phenylalanine – fumarate
24. Which of these is/are ketogenic amino acids? FINALS
A. leucine, lysine, cysteine, glycine
B. leucine, lysine, cysteine
C. leucine, lysine
D. leucine
25. In replication, which of these steps/stages follows elongation of the daughter strand?
A. termination of replication
B. rewinding of the DNA molecule
C. primase creates temporary complementary RNA primase
D. DNA binding protein binds tightly to separated strands
26. In translation, which of these steps/stages follows elongation?
A. activation of amino acid C. folding and processing
B. termination and release D. initiation of the polypeptide chain
27. Which of the following best describes Okazaki fragments?
A. fragments of DNA attached to an initiator component
B. fragments of continuous DNA which are eventually joined to form DNA
C. fragments of DNA that are copied in direction away from replication fork
D. synthesized  from  5’  to  3’  and proceeds in same direction as replication fork
28. The enzyme responsible for unwinding of duplex DNA segments is:
A. helicase C. DNA polymerase
B. ligase D. DNA topoisomerase
29. As regards transcription, the following are true, EXCEPT:
A. builds  RNA  in  a  5’  to  3’ direction
B. follows the Watson-Crick base pairing
C. phosphodiester bonds link adjacent nucleotides
D. builds RNA in the direction parallel to that of the DNA template
30. The following describe both replication and transcription, EXCEPT:
A. phases/stages are similar in terms of initiation and elongation
B. direction of synthesis is the same
C. direction of elongation is the same
D. both make use of a primer
31. Which of the following enzymes in replication functions as proof-reading device?
A. 5’exonuclease C. 3’exonuclease
B. polymerase enzyme D. all of the above
32. This amino acid represents the start of translation?
A. tyrosine C. methionine
B. glycine D. serine
33. Which of these directs proper initiation of transcription?
A. promoter C. repressor
B. regulator D. operator
34.

Basic Biomedical Sciences – Third Bimonthly Exam


Set A: Modules 21 – 25

NAME ________________________ 8 Jan 2007 SY 2006 – 07

MULTIPLE CHOICE: Write the letter of the BEST answer in CAPITAL letter in ink on
the answer sheet. NO ERASURES/SUPERIMPOSITIONS ALLOWED!

1. The following are true of heme biosynthesis, EXCEPT


A. begins with succinyl CoA & glycine
B. glycine is activated by pyridoxal phosphate
C. the rate-limiting step is catalyzed by a dehydratase
D. does not occur in red blood cells
2. Which is not true of erythropoietic protoporhyria?
A. enzyme involved is a uroporphyrinogen III cosynthase
B. photosensitivity is a major complaint
C. presence of fecal and red cell protoporphyrin
D. happens when conversion of protoporphyrin to heme is blocked
3. The following are true of bilirubin transport, EXCEPT:
A. bilirubin bound to albumin is transported from peripheral tissues to the liver
B. binding with albumin makes bilirubin more soluble in plasma
C. in 100 ml of plasma, about 75 mg of bilirubin can be tightly bound to albumin
D. antibiotics may compete with bilirubin for the binding sites in albumin
4. Which of the following is NOT true of conjugated bilirubin?
A. it is water soluble
B. secreted into bile by facilitated diffusion
C. most bilirubin secreted into bile are the form of bilirubin diglucuronide
D. B-glucuronidases in terminal ileum remove the diglucuronide from bilirubin
5. The rate limiting step in the metabolism of bilirubin is the:
A. uptake of bilirubin by the liver parenchymal cells
B. conjugation of bilirubin in the SER
C. secretion of conjugated bilirubin into the bile
D. conversion of glucuronides into urobilinogens in the intestine
6. The following statements are true regarding the properties of lipids EXCEPT:
A. Lipids are insoluble in water.
B. Lipids are soluble in inorganic solvents like dilute acids and dilute alkali.
C. The melting point of even-numbered-carbon fatty acids increases with chain
length.
D. Triglycerol containing all saturated fatty acids of 12 C or more is solid at body
temperature.
7. Gastric lipase acts on triglycerides to produce:
A. diglyceride and fatty acids B. lysophospholipid and fatty acids
B. cholesterol and fatty acids D. monoglycerides and fatty acids
8. The following phospholipases are involved in the degradation of phospholipids:
A. A B. B C. C D. D
9. The glycerol backbone in phospholipids is derived from:
A. glycerol phosphate C. dihydroxyacetone phosphate
B. phosphatidic acid D. diacyl glycerol
10. The following are true of very low density lipoprotein (VLDL), EXCEPT:
A. most plasma VLDL is formed by liver cells
B. it is the precursor of intermediate-density lipoprotein (IDL)
C. its density is greater than that of chylomicrons
D. transports primarily cholesterol from liver to extrahepatic tissues

11. The main apolipoprotein of LDL is:


A. apolipoprotein A C. apolipoprotein C
B. apolipoprotein B D. apolipoprotein E
12. The following are descriptive of Acetyl-CoA carboxylase, EXCEPT
A. rate limiting enzyme in fatty acid synthesis
B. it is inhibited by citrate
C. palmitoyl CoA prevents its polymerization
D. regulated by phosphorylation
13. Which of the following paired enzymes catalyze the first two reactions in ketogenesis?
A. carnitine acyltransferase and acetyl CoA carboxylase
B. cholesterol acyltransferase and HMG CoA reductase
C. thiolase and HMG CoA synthase
D. ferrochelatase and UDPG transferase
14. The following are descriptive of mechanisms of homeostatic glucose regulation,
EXCEPT
A. Almost all of the extrahepatic tissue cells are relatively impermeable to glucose
B. Hexokinase phosphorylates glucose that may enter extrahepatic tissues
C. (-) feedback is exerted by inhibition of hexokinase by glucose 6-PO4
D. Insulin causes glycogenolysis by activating phosphorylase
15. The following enzymes catalyze irreversible reactions which prevent reversal of
glycolysis, EXCEPT
A. pyruvate carboxylase C. fructose 1,6-biphosphatase
B. PEP carboxylase D. glucose 6-phosphatase
16. If a patient is glycosuric his blood glucose is
A. >9.5-10 mmol/L C. <10 mmolL
B. 8-10 mmol/L D. 6-10 mmol/L
17. A mixture is said to be racemic when:
A. It is naturally occurring
B. the OH group is located to the right of the C atom
C. there are equal amounts of the D and L forms and there is no optical activity
D. all of the above
18. The luminal phase of carbohydrate digestion starts in the mouth and involves this
step:
A. cleavage of the OH groups from the C atoms
B. cleavage of the C to C double bonds
C. hydrolysis of alpha 1, 4 glycosidic bonds
D. hydrolysis of alpha 1, 6 glycosidic bonds
19. The most common form of dissacharidase deficiency is:
A. galactase deficiency C. fructose deficiency
B. lactase deficiency D. biphosphatase deficiency
20. The uronic acid pathway includes conversion of glucose to the following, EXCEPT:
A. glucuronic acid C. uric acid
B. ascorbic acid D. pentoses
21. The reaction that allows glycolysis to proceed in the absence of oxygen is catalyzed
by:
A. Lactate dehydrogenase C. enzyme that forms NADH
B. Oxidation of FAH D. none of the above
22. Which of these enzymes facilitate the transfer of TCA intermediate oxaloacetate into
the main gluconeogenetic pathway?
A. pyruvate carboxykinase C. fructose 1,6 biphosphatase
B. pyruvate carboxylase D. phosphoenolpyruvate carboxykinase
23. Glycolysis results in the net yield of ____ ATPs per mole of glucose:
A. one B. two C. three D. four
24. Which of these enzymes catalyzes the rate limiting reaction in the glycolytic pathway?
A. Phosphofuctokinase C. hexokinase
B. pyruvate kinase D. glucokinase

25. This disease is characterized by a deficiency of glucose-6-phosphatase:


A. Her’s  disease C.    Pompe’s  disease
B. Tarui’s  disease D.    Von  Gierke’s  disease
26. The reaction that is common to glycolysis and glycogenesis is:
A. phosphorylation of glucose to glucose 6-phosphate
B. conversion of glucose to glucose 1-phosphate
C. glucose 1-phosphate reacts with uridine triphosphate
D. hydrolysis of pyrophosphate to inorganic phosphate
27. This liver enzyme is responsible for the inactivation of phosphorylase a to
phosphorylase b:
A. phosphorylase kinase C. protein phosphatase-1
B. cAMP-dependent protein kinase D. glycogen synthase
28. Which of the following integrates the regulation of glycogenolysis and glycogenesis by
simultaneous activation of phosphorylase and inhibition of glycogen synthase?
A. Ca++ C. cAMP
B. calmodulin D. glucokinase
29. The phosphorylitic cleavage of the 1--> 4 linkages of glycogen to yield glucose 1-
phosphate is catalyzed by:
A. glucokinase C. phosphoglucomutase
B. glycogen synthase D. glycogen phosphorylase
30. In the liver, phosphorylase b is rephosphorylated to active phosphorylase a. This
reaction is catylyzed by:
A. phosphorylase kinase C. glucose-6-phosphatase
B. protein phosphatase – 1 D. glucomutase
31. Phosphorylase in muscle differs from that in liver because in muscle:
A. phosphorylase exists in two forms
B. phosphorylase a is active
C. phosphorylase b is dephosphorylated
D. phosphorylase is activated by cAMP
32. Which enzyme in glycogenolysis exposes the 1 --> 6 branch point?
A. transglucosidase C. branching enzyme
B. glucan transferase D. glucose-6-phosphatase
33. In hemolytic anemia, which enzyme is commonly deficient?
A. Phosphofructokinase C. glukokinase
B. Hexokinase D. phosphofructokinase
34. This is defined as the polypeptide backbone made up of a unique sequence of amino
acids joined by peptide bonds in a protein:
A. primary backbone C. polypeptide structure
B. primary structure D. none of the above
35. These enzymes catalyze the cleavage of bonds by the addition of water:
A. isomerases C. transferases
B. hydrolases D. oxidoreductates
36. The following are the general properties of enzymes, EXCEPT:
A. active sites interact with substrate C. accelerate rate of reaction
B. most if not all are proteins D. consumed in overall process
37. These are true of pepsins, EXCEPT:
A. precursors are called pepsinogens
B. precursors are activated by gastric HCl
C. immediate products of peptic digestion are amino acids
D. Alkaline pancreatic juice in duodenum & ileum terminate action of pepsin
38. The following associations between amino acids and their catabolic end products are
correct, EXCEPT:
A. proline – a-ketoglutarate C. threonine – pyruvate
B. arginine – oxaloacetate D. phenylalanine – fumarate
39. The following are descriptive of protein degradation, EXCEPT
A. proteins with short half lives follow the PEST sequence
B. all of nitrogen form urea
C. excess proteins are degraded not stored
D. ubiquitin is needed for degradation of short-lived proteins
40. Which of the following happens first in amino acid catabolism?
A. Transamination C. Translation
B. Deamination D. Hydration
41. Which  of  the  following  enzymes  catalyze  the  formation  of  5’phosphoribosylamine  in  
purine synthesis?
A. glutamine PRPP amidotransferase
B. ribonucleotide reductase
C. thioredoxin reductase
D. carbamy phosphate synthetase
42. Which of the following enzymes catalyzes the reaction of adding a methyl group to
dUMP to form dTMP?
A. Adenylate cyclase C. Thymidylate cyclase
B. Nucelotidase D. Kinase
43. Nucleosides are different from nucleotides because of the absence of this:
A. carboxyl group C. purine or pyrimidine base
B. pentose sugar D. phosphate group
44. In replication, which of these steps/stages follows elongation of the daughter strand?
A. termination of replication
B. rewinding of the DNA molecule
C. primase creates temporary complementary RNA primase
D. DNA binding protein binds tightly to separated strands
45. In translation, which of these steps/stages follows elongation?
A. activation of amino acid C. folding and processing
B. termination and release D. initiation of the polypeptide chain
46. Which of the following best describes Okazaki fragments?
A. fragments of DNA attached to an initiator component
B. fragments of continuous DNA which are eventually joined to form DNA
C. fragments of DNA that are copied in direction away from replication fork
D. synthesized from  5’  to  3’  and  proceeds  in  same  direction  as  replication  fork
47. The enzyme responsible for unwinding of duplex DNA segments is:
A. helicase C. DNA polymerase
B. ligase D. DNA topoisomerase
48. As regards transcription, the following are true, EXCEPT:
A. builds RNA  in  a  5’  to  3’  direction
B. follows the Watson-Crick base pairing
C. phosphodiester bonds link adjacent nucleotides
D. builds RNA in the direction parallel to that of the DNA template
49. The following are functions of nonsense codon, EXCEPT:
A. they base pair with the anticodon
B. they code for non-essential amino acids
C. they are responsible for degeneracy of the genetic code
D. they may be used in the cell as termination signals
50. Which  posttranscriptional  processing  prevents  3’exonuclease  on  polypeptide?
A. poly A tail C. addition of extra COOH group
B. splicing reaction D. methylguanosine triphosphate cap

Xavier University – Dr Jose P Rizal College of Medicine


Basic Biomedical Sciences – Third Bimonthly Exam
Set B: Modules 26 – 29
NAME ________________________ 10 Jan 2007 SY 2006 – 07

MULTIPLE CHOICE: Write the letter of the BEST answer in CAPITAL letter in ink on
the answer sheet. NO ERASURES/SUPERIMPOSITIONS ALLOWED!

1. The following are true of the thyroid gland, EXCEPT:


A. a vascular organ
B. consists of right and left lobes
C. lobes are related medially to the larynx
D. supplied by veins that drain into the external jugular vein
2. Which of these is NOT true of hormones in general?
A. different hormones can have antagonistic effects
B. regulation may occur through feedback control
C. regulation can involve hierarchic levels of control
D. a hormone may change the product of a target cell
3. The first step in the action of any intercellular chemical messenger involves:
A. conversion of the messenger to a cytosolic protein
B. conversion of the messenger to a protein-binding site
C. binding of the messenger to a ligand-gated channel
D. binding of the messenger to its specific target proteins
4. Which of the following is not true of the functions of insulin?
A. decreases hepatic ketogenesis
B. increases glucose entry to adipose tissue
C. increases release of glucose by liver cells
D. increases glycogen synthesis in muscle tissue
5. These following glands would respond to stress, EXCEPT:
A. pituitary gland C. thyroid
B. parathyroid D. adrenal
6. One of the following is a primary sex organ of the male:
A. Penis C. prostate
B. Testis D. seminal vesicles
7. The membranous layer of the superficial fascia of the scrotum is the:
A. Campers C. cremasteric
B. Scarpas D. Colles
8. The following describe the penis, EXCEPT:
A. contains three cylindrical masses
B. ventrally located corpus spongiosum
C. contains serous-secreting Littre glands
D. mostly pseudostratified columnar epithelium lines the penile urethra
9. The following describe the prostate gland, EXCEPT:
A. a fibromuscular gland
B. divided into five lobes
C. surrounds membranous urethra
D. lies between the urinary bladder and the urogenital diaphragm
10. The Wolffian duct differentiates into the following, EXCEPT:
A. Epididymis C. ductus deferens
B. Prostate D. seminal vesicles
11. The following are the microscopic features of the testes, EXCEPT:
A. fibrous septa from tunica albuginea divides testes into lobules
B. seminiferous tubules are enmeshed in loose connective tissue
C. a complex stratified epithelium lines the tubules
D. spermatogenic cells are arranged in layers

12. The following are descriptive of the prostate, EXCEPT:


A. has a rich fibromuscular stroma
B. covered by a fibroelastic capsule
C. branched tubuloalveolar glands
D. its central zone occupies 70% of the gland
13. One of the following is true of spermatogenesis:
A. the process requires approximately 70 – 80 days
B. it is a sertoli-supported process regulated by locally produced hormones
C. maturation of spermatids occurs in the apical folds of Leydig cells
D. involves proliferation of spermatogonia by meiosis to produce type A
spematogonia
14. The effects of testosterone include all of the following, EXCEPT:
A. formation of fetal penis C. descent of testis into scrotum
B. initiation of ejaculation D. increased skin thickness
15. Penile erection is caused primarily by:
A. contraction of the bulbo-carvernosus muscle
B. reflex sympathetic contraction of arterioles
C. reflex parasympathetic contraction of venules
D. parasympathetically induced dilatation of arterioles
16. The first step in the biosynthesis of testosterone is the conversion of cholesterol to
pregnenolone which is catalyzed by this enzyme:
A. 3B-Hydroxysteroid dehydrogenase C. 11B-hydroxylase
B. side-chain cleavage enzyme D. 17a-Hydroxylase
17. Before puberty testosterone fails to produce its effects because the:
A. hypothalamus is set to a very low level
B. level of plasma testosterone is too low to exert effects
C. negative feedback mechanism is highly active
D. gonadostat is very sensitive to the negative feedback regulation
18. The sex of an individual determined at the time of fertilization is known as:
A. gonadal sex C. genetic sex
B. genital sex D. psychological sex
19. Failure of the testes to descend into the scrotum is called:
A. Sterility C. eunochism
B. Impotency D. cryptorchidism
20. Which of the following stages of spermatogenesis is characterized by release of
spermatozoa from their attachment to Sertoli cells?
A. Meiosis C. spermiogenesis
B. Spermeation D. spermatocytogenesis
21. Common symptoms of benign prostatic hypertrophy include the following, EXCEPT:
A. Fever C. weak urinary stream
B. Nocturia D. increased frequency of urination
22. Gonadal differentiation involves the following, EXCEPT:
A. gonads appear initially as a proliferation product of epithelium
B. sex of the embryo is determined genetically at the time of fertilization
C. germ cells appear in the genital ridges on the 4th week of development
D. gonads acquire male or female morphology during the 7 th week of development
23. Which of the following is true of embryos with an XX chromosome:
A. secondary cortical cords fail to develop
B. characterized by regression of the gonadal medullary cords
C. primitive germ cell clusters occypy cortical part of the future ovary
D. cortical cords split into isolated cell clusters that later develop into oogonia
24. The following are true of the ovary, EXCEPT:
A. a primary female sex organ
B. the uterine tube arches over it
C. found behind the broad ligament
D. attached to the broad ligament by the mesosalpinx

25. Which of the following open into the vestibule of the vagina?
A. Urethral C. ducts of the vestibular glands
B. Vaginal D. all of the above
26. Which of the following is NOT true regarding the uterus?
A. its lower part is the cervix
B. its normal position is retroversion
C. its uterine artery is closely related to the ureter at its termination
D. located between the urinary bladder and the rectum
27. The following are descriptive of the ovary, EXCEPT:
A. divided into cortex and medulla
B. its cortex contains follicles of varying shapes
C. lined by a continuous sheet of squamous epithelium
D. loose connective tissue and blood vessels are found in the medulla
28. The following are true of the uterus, EXCEPT:
A. The myometrium is divided into four layers
B. Its outermost layer is mostly covered by peritoneum
C. Mucosal epithelium is made up of secretory and ciliated cells
D. The endometrial sublayers are sloughed off during menses
29. The following are true of the second stage of oogenesis, EXCEPT:
A. starts after menarche C. formation of antral follicle
B. formation of antrum D. oocyte is covered by theca cells
30. Menstrual abnormalities include the following, EXCEPT:
A. primary amenorrhea characterized by absence of menses by 14 years old in the
absence of secondary sexual characteristics
B. Menometrorrhagia characterized by irregularity in duration and interval of
bleeding
C. Dysfunctional uterine bleeding secondary to progesterone excess
D. Oligomenorrhea characterized by scanty menstruation
31. These are true of the menstrual cycle, EXCEPT:
A. includes the uterine and ovarian cycles
B. characterized by periodic vaginal bleeding
C. cycle length varies because length of secretory phase varies
D. period from first day of menstrual flow to first day of the next menstrual flow
32. The cyclic changes in the uterus during the menstrual cycle include following,
EXCEPT:
A. decidua functionalis is sloughed during menstrual flow
B. during proliferative phase all uterine layers increase markedly in thickness
C. during secretory phase the glands become coiled and secrete clear fluid
D. secretory phase is influenced by increasing plasma level of progesterone
33. The following statements are correct regarding ovarian follicles, EXCEPT:
A. The final stage of follicular development occurs only in the postpubertal
reproductive ovary
B. During the reproductive lifespan of the woman, only 400 to 500 oocytes
undergo ovulation
C. At puberty, only about 300,000 of the original 6 – 7 million oocytes are available
for ovulation
D. In humans, further oogonial formation occurs postnatally
34. Peak levels of these hormones occur just before ovulation:
A. FSH, LH, estrogen, progesterone
B. FSH, LH, estrogen
C. FSH, LH
D. FSH

35. The following are true of the female libido, EXCEPT:


A. a complex phenomenon
B. modulated by circulating sex steroids
C. decreases around the time of ovulation
D. consists of physical and psychological effects
36. At this stage of the female sex response, the woman experiences a feeling of personal
satisfaction, well being, and relaxation of sexual desire:
A. First C. third
B. Second D. fourth
37. Capacitation is characterized by the following, EXCEPT:
A. An intact zona for protection against mechanical damage
B. Physiologic process not specific to the reproductive system
C. Spermatozoa acquire the ability to penetrate the zona pelucida
D. Removal of a protective protein coat from the sperm cell membrane
38. The process of implantation is characterized by which of the following?
A. Zona pellucida remains intact
B. blastocyst implants at its abembryonic pole
C. occurrence at 6 – 7 days after fertilization
D. point of entry of blastocyst remains open
39. Prevention of polyspermy is achieved through:
A. species-specific recognition of germ cell
B. fusion of cortical granules with the oolema
C. inactivation of sperm receptors through endocytosis
D. release of proteolytic enzymes to dissolve the granulose cell matrix
40. The following events occur after the entry of the spermatozoon into the cytoplasm of
the ovum, EXCEPT:
A. acrosome reaction
B. extrusion of the 2nd polar body
C. swelling of male and female pronuclei
D. rotation of the sperm head by 180 degrees
41. The developmental stage of the zygote when it enters the uterine cavity:
A. 4-cell C. 16-cell
B. 8-cell D. 32-cell
42. The following major events happen during the 4 th to the 8th week of gestation,
EXCEPT:
A. formation of head and caudal eminence
B. the yolk sac dislodges from its stalk
C. all major organ systems form from three germ layers
D. expansion of the amnion envelops the yolk and allantois
43. The following are descriptive of placental development, EXCEPT:
A. the villous chorion is the larger fetal placental part
B. presence of a smooth chorion, the avascular bare area
C. the deciduas basalis forms the fetal part of the placenta
D. the placental and the fetal membranes originate from the zygote
44. The maternal adaptations to pregnancy in these organ systems include the following,
EXCEPT:
A. cardiovascular system – decreased BP during first half of pregnancy
B. respiratory system – increased tidal volume
C. musculoskeletal system – dorsolumbar scoliosis
D. renal system – increased glomerular filtration rate
45. The muscle cramps that some pregnant women experience is mainly due to:
A. Hypocalcemia C. hyponatremia
B. hypokalemia D. hypercalcemia

46. The following are true of human chorionic gonadotropin (hCG), EXCEPT:
A. secreted by the syncytiotrophoblast
B. exerts interstitial cell-stimulating effect on the testes
C. a glycoprotein that functions as a luteinizing and luteotropic hormone
D. it is first detected in the urine 30 days after conception
47. The first day of the last normal menstruation of Hilda was January 1, 2007. If she has
a 28-day menstrual cycle, when is her expected date of confinement?
A. September 8, 2007 C. November 8, 2007
B. October 8, 2007 D. December 8, 2007
48. Which of the following pregnancy tests offers the most conclusive evidence of
pregnancy?
A. positive cytochemical test C. positive frog test
B. high hCG on radioimmunoassay D. high urinary estriol
49. Human  milk  is  better  than  cow’s  milk  for  the  following  reasons,  EXCEPT:
A. it has lower protein content
B. it has 48% unsaturated fatty acids
C. it has higher total ash content
D. it has more vit. C and niacin
50. Which of the following is a probable evidence of pregnancy?
A. Mastodyna C. ballottment
B. Quickening D. amenorrhea

Xavier University – Dr Jose P Rizal College of Medicine


Basic Biomedical Sciences – Third Bimonthly Exam
Set B: Modules 26 – 29

NAME ________________________ 10 Jan 2007 SY 2006 – 07

MULTIPLE CHOICE: Write the letter of the BEST answer in CAPITAL letter in ink on
the answer sheet. NO ERASURES/SUPERIMPOSITIONS ALLOWED!

1. Which of these is NOT true of hormones in general?


A. different hormones can have antagonistic effects
B. regulation may occur through feedback control
C. regulation can involve hierarchic levels of control
D. a hormone may change the product of a target cell
2. The first step in the action of any intercellular chemical messenger involves:
A. conversion of the messenger to a cytosolic protein
B. binding of the messenger to a ligand-gated channel
C. binding of the messenger to specific target receptors
D. conversion of the messenger to a protein-binding site

Module 27-MRS

3. One of the following is true of spermatogenesis:


A. involves proliferation of spermatogonia by meiosis to produce type A
spermatogonia
B. a Sertoli-supported process regulated by locally produced hormones
C. maturation of spermatids takes place in the apical folds of Leydig cells
D. process requires approximately 70 – 80 days
4. Which of the following functions of Sertoli cells mediates negative feedback control of
FSH secretion?
A. excessive growth of semineferous tubules C. synthesis of testosterone
B. aromatization of testosterone D. synthesis of inhibin
5. Penile erection is caused primarily by:
A. reflex sympathetic constriction of arterioles
B. contraction of the bulbo-cavernosus muscle
C. reflex parasympathetic constriction of venules
D. parasympathetically induced dilatation of arterioles
6. The first step in the biosynthesis of testosterone is the conversion of cholesterol to
pregnenolone which is catalyzed by this enzyme:
A. 3B-Hydroxysteroid dehydrogenase C. 11B-hydroxylase
B. side-chain cleavage enzyme D. 17a-Hydroxylase
7. Before puberty testosterone fails to produce its effects because the:
A. hypothalamus is set to a very low level
B. level of plasma testosterone is too low to exert effects
C. negative feedback mechanism is highly active
D. gonadostat is very sensitive to the negative feedback regulation
8. Which of the following is NOT an action of testosterone?
A. exerts a protein-catabolic effect
B. exerts inhibitory feedback on pituitary LH secretion
C. develops and maintains male secondary sex characteristics
D. along with FSH, maintains spermatogenesis

MOdule 28-FRS 1
9. One of the following is true of sex differentiation:
A. a process that is exclusive for sex genes
B. a complex process that involves one gene
C. absence of the SRY protein dictates female development
D. key is the sex-determining region on the long arm of the Y chromosome
10. One of the following is true of male gonadal differentiation, EXCEPT:
A. once the gonad becomes an indifferent gonad, it is still predestined to be
neither a male or female gonad
B. the medullary cords are formed through the influence of the peptide product of
the SRY gene
C. Sertoli cells are derived from fibrous connective tissue while Leydig cells
are from mesenchymal tissues
D. Testosterone production of Leydig cells dictate sexual differentiation by the 8 th
week of gestation
11. Which of the following is true of embryos with an XX chromosome?
A. the secondary cortical cords fail to develop
B. characterized by regression of the gonadal medullary cords
C. primitive germ cell clusters occupy the cortical part of the future ovary
D. cortical cords split into isolated cell clusters that subsequently develop into
oogonia
12. These are true of the menstrual cycle, EXCEPT:
A. includes the uterine and ovarian cycles
B. characterized by periodic vaginal bleeding
C. cycle length varies because length of proliferative phase varies
D. period from last day of menstrual flow to last day of the next menstrual
flow
13. The cyclic changes in the uterus during the menstrual cycle include following,
EXCEPT:
A. decidua functionalis is sloughed during menstrual flow
B. during proliferative phase all uterine layers increase markedly in
thickness
C. during secretory phase the glands become coiled and secrete clear fluid
D. secretory phase is influenced by increasing plasma level of progesterone
14. Which of the following is NOT true of ovulation?
A. characterized by an increase in basal body temperature
B. characterized by very thick cervical mucus
C. triggered by a surge in LH secretion by pituitary
D. occurs about 14 days prior to the next menstrual flow
15. The following are true of human chorionic gonadotropin (hCG), EXCEPT:
A. secreted by the syncytiotrophoblast
B. exerts interstitial cell-stimulating effect on the testes
C. a glycoprotein that functions as a luteinizing and luteotropic hormone
D. it is first detected in the urine 30 days after conception

Module 29-Pregnancy
16. Capacitation is characterized by the following, EXCEPT:
A.
17. The following are produced by the corpus luteum of pregnancy:
A. progesterone, estrogen, relaxin, enkephalins
B. progesterone, estrogen, relaxin
C. progesterone, estrogen
D. progesterone
18. During pregnancy, estrogens cause the following, EXCEPT:
A. enlargement  of  mother’s  uterus
B. enlargement  of  mother’s  breasts
C. enlargement  of  mother’s  external  genitalia
D. contraction  of  mother’s  pelvic  ligaments
19. Relaxin helps maintain pregnancy by:
A. inhibiting myometrial contractions
B. causing the relaxation of the pelvic ligaments
C. preventing the opening of the cervix
D. relaxing the breasts

XAVIER UNIVERSITY – DR JOSE P RIZAL COLLEGE OF MEDICINE


Basic Biomedical Sciences
Third Bimonthly Period
QUIZ # 5 (ENDOCRINE)

NAME ____________________________ 11 December 2006 SCORE ____________


MULTIPLE CHOICE: write the letter of the BEST answer on the answer sheet. Use capital
letters in blue/black ink. NO EASURES/SUPERIMPOSITIONS ALLOWED!

1. The arterial supply to the pituitary gland comes from the:


A. external carotid C.middle cerebral
B. internal carotid D. anterior cerebral
2. What is the origin of the inferior thyroid artery?
A. external carotid C. brachiocephalic
B. internal carotid D. thyreocervical
3. Lymphatic vessels of the thyroid gland would drain into the:
A. deep cervical nodes C. superficial cervical nodes
B. paratracheal nodes D. both A & B
4. Which part of the pancreas is found in the splenicorenal ligament?
A.head C.neck
B.body D. tail
5. Which part of the pancreas is most likely involved in obstructive jaundice?
A.head C..neck
B.body D. tail
6. Which is false regarding the parathyroid glands?
A. usually four in number C. located at the anterior surface of thyroid gland
B. secretes parathormone D..found in the capsule of the thyroid gland
7. The following are arterial supplies to the adrenal glands EXCEPT:
A.inferior phrenic C.abdominal aorta
B.superior phrenic D.renal
8. Anterior relations of the left suprarenal gland include the following EXCEPT:
A.respiratory diaphragm C.stomach
B.pancreas D. lesser sac
9. The left ovarian / testicular vein drains into the:
A. spleenic vein C.renal vein, left
B .inferior vena cava D. inferior mesenteric vein
10. Lymphatic vessels of the testes drain into the following:
A .lumbar nodes C.paraaortic nodes
B .inguinal nodes D. A & C
11. Which of the following is not true of the neurohypohysis? C
A. made up of the pars nervosa and neural stalk
B. consists of unmyelinated axons
C. has abundant secretory cells
D. contains glial cells
12. Which of the following cell secrete prolactin?
a. Somatotropic C. Mammotropic
b. Gonadotropic D. Corticotropic
13. The following are histologic features of the thyroid gland, EXCEPT C
a. composed of colloid-containing follicular spheres
b. loose connective tissue septa in the parenchyma
c. purely cuboidal epithelium-lined follicles
d. its stroma contains reticular fibers
14. Which of the following is true of the parafolluicular cells? A
a. forms clusters between thyroid follicles
b. smaller in size compared to the follicular cells
c. abundant rough endoplasmic reticulum
d. contains large hormone-secreting granules
15. The following are descriptive of the chief cells of the parathyroid, EXCEPT B
a. small and polygonal C. has cytoplasmic granules
b. strongly basophilic cytoplasm D. vesicular nucleus
16. The following are true of the islets of Langerhans, EXCEPT C
a. embedded within exocrine pancreatic tissue
b. made up of four types of cells
c. enclosed by areolar connective tissue capsule
d. cord-like arrangement of islet cells
17. Which of the following layers occupy the least of the total volume of the adrenal gland?
C
a. zona glomerulosa C. zona reticularis
b. zona fasciculata D. medulla
18. The following are descriptive of the zona fasciculata, EXCEPT D
a. Contains polyhedral-shaped cells
b. Lipid droplets in the cytoplasm of its cells
c. Its cells become vacuolated in tissue preparation
d. Cord-like arrangement of cells
19. Which of the following associations between endocrine gland, hormone secreted and
hormone function is NOT true? C
a. hypothalamus – dopamine – inhibits prolactin secretion
b. anterior pituitary – TSH – regulates T3 & T4 synthesis
c. parathyroids – PTH – activates calcitriol to form calcidiol
d. pancreas – somatostatin – inhibit secretion of glucagons
20. The following organ/cells and hormone associations are TRUE, EXCEPT A
a. GIT – Somatomedin C. Heart - ANF
b. Kidney – erythropoietin D. Skin - hydroxycholecalciferol
21. One basic principle in homeostatic control systems states that the systems cannot
maintain complete constancy of any given feature of the internal environment. This
principle is shown in which of the following conditions?
A. Normal pH is maintained at 7.35 – 7.45
B. Na+ balance is maintained when Na+ intake equals Na+ excretion
C. blood pressure is set at a level higher than normal
D. compensatory increase in plasma [HCO3-] in respiratory acidosis
22. Gino and his medical classmates climbed a very high mountain. They noticed that
their heart rates (HR) and respiratory rates (RR) immediately increased as they were
climbing. Gino explained that the increase was the response of their bodies to the
hypoxemia they were experiencing. This response is called:
A. Adjustment C. habituation
B. adaptation D. accommodation
23. White blood cells are programmed to die within 24 hours. This is called:
A. Aging C. necrosis
B. Apoptosis D. programmed cell death
24. Which of the following is NOT true of endocrine hormones?
A. circulate in the blood stream
B. may act as first or second messengers
C. produce their effects by binding with specific receptors
D. release may be stimulated by changes in the internal environment
25. Which of these hormones (H) is/are considered prohormone?
A. Prolactin, growth H, thyroid stimulating H (TSH), follicle stimulating H (FSH),
B. Prolactin, growth H, TSH
C. Prolactin, growth H
D. prolactin
26. High levels of TSH inhibit the secretion of thyrotropin releasing hormone. This is due
to:
A. simple feedback loop C. long-loop negative feedback
B. short-loop negative feedback D. positive feedback
27. Which of the following is/are true of the hormonal control systems?
A. hormones can have antagonistic actions
B. may involve interaction with the CNS
C. regulation occurs through feedback control
D. all of the above
28. The following associations between hormones and their types are correct, EXCEPT:
A. cortisol – mineralocorticoid
B. epinephrine – amino acid derivative
C. thyroxine – amino acid derivative
D. thyrotropin releasing hormone – hypophysiotropic hormone
29. Hormone receptors may be located in the:
A. cell membrane, cytoplasm, nucleus, mitochondria
B. cell membrane, cytoplasm, nucleus
C. cell membrane, cytoplasm
D. cell membrane
30. The concentration of hormones in the blood is determined by these factors:
A. site of synthesis, site of action, site of excretion, source of precursor
B. site of synthesis, site of action, site of excretion
C. site of synthesis, site of action
D. site of synthesis
31. The following systems have been identified to be associated with the stress
response, EXCEPT:
A. nervous system C. digestive system
B. endocrine system D. immune system
32. The following are categorized as occupational stressors, EXCEPT:
A. financial difficulties C. problems with supervisors
B. fear of inadequacy D. poor communication skills
33. A 17 year old coed was walking in a dark alley on her way home when suddenly she
was confronted by the neighborhood bully. Her heart began to beat faster, and she
started to perspire. She is most probably at which stage of the general adaptation
syndrome?
A. stage of resistance C. stage of exhaustion
B. shock phase D. countershock phase
34. During a fire, a male student managed to carry his television set to a safe place 10
meters away. Later on, when he wanted to return the television set back to his
boarding house, he found out that he could not do it without resting several times.
Which of the following hormones may have been responsible for his ability to carry
the television set to a far distance without resting?
A. cortisol B. T3 C. epinephrine D. vasopressin
35. The following are intermediate effects of the stress response, EXCEPT:
A. muscle glycolysis C. increased RBC count
B. increased muscle strength D. increased protein metabolism
36. A first time diver was reprimanded by his diving instructor when they surfaced
because the diver refused to pay attention to his instructor while they were
underwater. The diver then stated that he did not know what happened to him
because he was so stressed out and just did what he thought was right. Which of the
following  is  the  most  probable  mechanism  to  explain  the  diver’s  behavior?
A. exhaustion of the mechanism to cope with the stressor
B. activity of the ACTH axis and the vasopressin axis
C. temporary shutdown of the neocortex
D. release of cytokines that modify stress responses
37. The  following  may  have  contributed  to  the  diver’s  stress,  EXCEPT:
A. poorly maintained equipment C. poor visibility underwater
B. lack of physical fitness D. familiarity with equipment
38. As part of the stress response, discrete autacoids are released by the following,
EXCEPT:
A. hypothalamus C. pituitary gland
B. thyroid gland D. adrenal gland
39. A medical drug representative was admitted to the hospital with signs and symptoms
suggestive of influenza, except for the absence of fever. It turned out that the signs
and symptoms were mostly psychosomatic and were the result of stress in the
workplace. These manifestations are suggestive of which of the following phases of
the general adaptation syndrome?
A. shock phase C. stage of resistance
B. countershock phase D. stage of exhaustion
40. According to Richmond, the following maxims should be considered in order to help
minimize stress in the workplace, EXCEPT:
A. some activities are more stressful than others
B. different relaxation methods have different ways of relieving stress
C. some activities are more efficient in relieving stress than others
D. stress is perceived individually and must be recognized by the individual as such
41. One of the following is a general characteristic of a receptor:
A. acts as a signal
B. may be a protein or a lipoprotein
C. non-specifically binds to a ligand
D. is almost always an ion channel
ANS: B

42. Receptors linked to G proteins are characterized by:


A. They are the largest family of ligands
B. They couple cell-surface receptors to downstream effectors
C. Have intracellular N terminus
D. Hydrophilic domain at the N terminus
ANS: B (extracellular N terminus & hydrophilic C terminus)

43. Paracrine hormones are:


A. Released by cells and act on the same cells that released them
B. Released by cells and act on neighboring cells
C. Released by organs involved in excitation of distant organs
D. Released by nerves
ANS: B

44. For paracrine hormonal signals to be delivered to their target organs, their diffusion
must be limited. This is achieved through any of the following EXCEPT:
A. Rapid exocytosis of the chemical signal
B. Destruction by cellular enzymes
C. Immobilization by the extracellular matrix
D. Exemplified by events at the neuromuscular junction
ANS: A (ENDOCYTOSIS)

45. Recognition of a signal by its receptor involves the same noncovalent interactions
that characterize substrate-enzyme interactions. They include:
A. Ionic bonds between groups of the same charge
B. Electrostatic interaction like the van der waals interaction
C. Hydrophobic interactions between polar groups
D. Disulfide interactions
ANS: B (ionic bonds bet opposite charges; hydrophobic bonds bet nonpolar
groups; no disulfide interactions)

46. With regards to second messengers, the following are true EXCEPT:
A. Rapidly produced by the cell
B. Broken down by apoptosis
C. Provides opportunities to amplify a signal
D. Exemplified by cyclic AMP
ANS: B

47. Ligand-gated ion channels are examples of plasma membrane receptors that
A. bind with lipid-soluble messengers
B. are integral membrane proteins
C. opens & closes as controlled by the receptor itself
D. act via metabolic pathways, hence, are called ionotropic receptors
ANS: B

48. General characteristics of hormones include the following EXCEPT:


A. They are chemicals secreted from endocrine tissues
B. Located on the surface, in the cytosol or in the nucleus of specific cells
C. Carried through the blood
D. Circulate at very high concentrations
ANS: D (circulate at very low concentrations 10(-9) to 10(-12) M)

49. Physiologic regulation of receptors include the following EXCEPT:


A. decrease the affinity of the receptors to their messengers
B. down regulation
C. exocytosis of receptors
D. endocytosis of ligands
ANS: D

XAVIER UNIVERSITY – DR JOSE P RIZAL COLLEGE OF MEDICINE


Basic Biomedical Sciences
Third Bimonthly Period
QUIZ # 4 (GENETICS)

NAME ____________________________ 4 December 2006 SCORE ____________


MULTIPLE CHOICE: write the letter of the BEST answer on the answer sheet. Use capital
letters in blue/black ink. NO EASURES/SUPERIMPOSITIONS ALLOWED!

Questions for Module 25 – Genetics (IOs # 2.1-2.5, 10 questions)

1. The following are descriptive of nucleotides, EXCEPT


A. dimeric precursors of DNA
B. source of ATP
C. metabolic regulators
D. part of the structure of coenzymes
2. Which of the following is not true of the structure of nucleosides?
A. heterocyclic compounds
B. linked by 6 C sugar
C. glycosidic linkage between base and sugar
D. contains a non carbon atom
3. Which of the following naturally occurring nucleotide derivatives is the precursor of
glycogen?
A. GDP C. UDPGlc
B. cAMP D. CDP
4. A type of synthetic nucleotide derivative which is used in the treatment of arthritis
A. Azathioprine C. 6-Mercaptopurine
B. 5-Fluorouracil D. Allopurinol
5. The following are descriptive of the structure of DNA, EXCEPT
A. made up of 4 repeating monomeric units
B. its  units  are  held  by  a  5’3’  phosphodiester  bridge
C. anti-parallel double helix strands
D. contains minor and major grooves for protein interaction
6. Which of the following is responsible for the specific pairings of the bases in DNA?
A. nitrogen bonds C. arrangement of carbon atoms
B. phosphodiester bonds D. hydrogen bonds
7. Which of the following is not true of RNA?
A. equal amounts of purine and pyrimidine nucleotides
B. its sugar moiety is ribose
C. can acquire a double stranded appearance by folding upon itself
D. can be hydrolyzed to cyclic diesters by alkali
8. The following describe mRNA, EXCEPT
A. conveys the information for protein synthesis
B. most heterogenous and stable of all RNA classes
C. 7-methylguanosine  caps  its  3’  terminus
D. its  3’  end  contains  a  poly  A  tail  preventing  attack  of  exonucleases
9. Which class of RNA is made up of 75 nucleotides and has 4 main arms?
A. rRNA B. tRNA C. mRNA D. none of the above
10. Which part of the cell does rRNA carry out its function of synthesizing proteins?
A. Nucleolus C. Nucleus
B. Cytoplasm D. Endoplasmic reticulum
11. Both purines and pyrimidines require ___________ as the donor of ribose phosphate
group of the nucleotide:
A. carbon C. phosphoribosylpyrrophosphate
B. respiratory CO2 D. methenyl tetrahydrofolate

12. The following statement is NOT true regarding the synthesis of purine nucleotides:
A.  the  formation  of  5”  phosphoribosylamine  is  catalyzed  by  glutamine  PRPP
amidotransferase
B. the amide group from glutamine is transferred to C-10’  of  PRPP,  where  it
replaces the pyrophosphate
C. the rate limiting step in purine nucleotide synthesis is the formation of 5-
phosphoribosylamine
D. changes in PRPP concentration results in proportional increase in the rate of
phosphoribosylamine synthesis

13. The following statement is NOT true regarding the synthesis of purine nucleotides:
A.  the  formation  of  5”  phosphoribosylamine  is  catalyzed  by  glutamine  PRPP
amidotransferase
E. the amide group from glutamine is transferred to C-10’  of  PRPP,  where  it
replaces the pyrophosphate
F. the rate limiting step in purine nucleotide synthesis is the formation of 5-
phosphoribosylamine
G. changes in PRPP concentration results in proportional increase in the rate of
phosphoribosylamine synthesis
14. In the synthesis of deoxyribonucleotides, this acts as the reducing agent.
A. thioredoxin C. thioredoxin tranferase
B. thioredoxin reductase D. deoxyribonucleotide diphosphate\
15. The salvage of a purine base and its conversion to nucleoside triphosphate requires
how many high-energy phosphate bonds?
A. 2 C. 7
B. 4 D. 9
16. The parent nucleotide in the de novo synthesis of pyrimidines is:
A. orotidine monophosphate C. carbamoyl phosphate
B. carbamoyl aspartate D. orotic acid
17. The following statements are true regarding the synthesis of deoxythymidine
monophosphate:
A. the intermediate precursor of dTMP is deoxyuridine momophosphate (dUMP)
B. addition of a methyl group to C-5 in the ring of dUMP, results in dTMP
C. the above step is catalyzed by the enzyme thymidylate synthase
D. the catalytic cycle is completed by the oxidation of DHF to THF
18. The terminal step in purine nucleotide degeneration is:
A. removal of the amino group from adenosine and guanosine
B. release of ribose-1-phosphate
C. sequential oxidation of hypoxanthine and xanthine to uric acid
D. addition of phosphate across the bond linking ribose to the purine ring
19. In purine catabolism the phosphate groups are removed by:
A. nucleotidases C. phosphorylases
B. oxidases D. all of the above
20. Degradation of pyrimidine nucleotides starts by removal from the ring of the following
EXCEPT:
A. amino groups C. ribose
B. phosphate D. carbon
21. This inherited disorder is characterized by hyperurecemia and the deposition of
monosodium urate crystals in the tissues, particularly around joints:
A. xanthinuria C. Lesch-Nyhan syndrome
B. gout D. renal lithiasis
22. The last step in purine catabolism is the:
A. reduction of hypoxanthine and xanthine
B. oxidation of hypoxanthine and xanthine
C. removal of phosphate group
D. phosphate is added across the bond linking ribose to purine ring

23. The end products of pyrimidine catabolism include the following:


A. NH4+, CO2, B-alanine, B-aminoisobutyrate
B. NH4+, CO2, B-alanine
C. NH4+, CO2
D. NH4+
24. Hypourecemia results when this is deficient:
A. adenosine deaminase
B. xanthine oxidase
C. ornithine transcarbomylase
D. hypoxanthine guanine phosphoribosyl transferase
25. The enzyme/s that may be deficient in the presence of immunodeficiency
characterized by T cell and B cell dysfunction:
A. adenosine deaminase
B. adenosine deaminase and xanthine oxidase
C. adenosine deaminase and purine nucleoside phosphorylase
D. xanthine oxidase and purine nucleoside phosphorylase
26. The following are true of RNA synthesis, EXCEPT:
A. makes use of ribonucleotides C. a segment of DNA is copied
B. requires presence of primase D. there is no proofreading
27. The enzymes involved in replication include the following:
A. RNA polymerase, DNA polymerase, ligase, helicase
B. RNA polymerase, DNA polymerase, ligase
C. RNA polymerase, DNA polymerase
D. RNA polymerase
28. The initial step in DNA replication is the:
A. unwinding of dsDNA to provide an ssDNA template
B. recognition of the origin of replication
C. formation of the replication fork
D. formation of the RNA primer
29. In the process of replication, which of these steps follows after the RNA primer has
been formed?
A. unwinding of DNA strands
B. elongation of the daughter strand
C. DNA binding protein binds tightly to separated strands
D. DNA polymerase at replication fork starts replication
30. Which of these initiates the synthesis of RNA molecule that is essential for priming
DNA synthesis?
A. DNA helicase C. DNA polymerase
B. DNA primase D. single-strand-binding protein (SSBs)
31. Which of the following prevents the premature reannealing of DNA strands?
A. DNA helicase C. DNA ligase
B. DNA primase D. SSBs
32. Regarding Okazaki fragments, which of the following is NOT true?
A. consists of short lengths of DNA C. located in the lagging strand
B. formed during transcription D. requires primase
33. The following are non-sense codons:
A. UAA, UAG, UGA, AUG C. UAA, UAG
B. UAA, UAG, UGA D. UAA
34. Which of the following descriptions is/are common to DNA synthesis, RNA synthesis
and protein synthesis?
A. makes use of amino acids
B. follows  5’  to  3’  polarity
C. sequential steps of initiation, elongation and termination
D. all of the above
35. Which of the following associations between nuclear DNA-dependent RNA
polymerase and products are correct?
A. polymerase I – rRNA C. polymerase III – mRNA
B. polymerase II – tRNA D. all of the above
36. These are true of transcription, EXCEPT:
A. entire DNA strand is copied
B. TATA box is involved in the initiation of transcription
C. transcription unit extends from promoter to the terminator
D. DNA-dependent RNA polymerase is the main transcription enzyme
37. This mutation is due to deletion of a single nucleotide from a coding strand of DNA:
A. silent mutation C. suppressor mutation
B. frameshift mutation D. acceptable missense mutation
38. These are involved in translation:
A. mRNA, tRNA, rRNA, ATP, GTP
B. mRNA, tRNA, rRNA, ATP
C. mRNA, tRNA, rRMA
D. mRNA, tRNA
39. The following contain the code for amino acid sequence:
A. codon, anticodon, introns, TATA box
B. codon, anticodon, introns
C. codon, anticodon
D. codon
40. True of TATA box
A. binds anticodon C. binds RNA polymerase
B. encodes repressor protein D. located in the 1st structural gene
40. This enzyme is involved in proofreading:
A. primase C. DNA polymerase
B. helicase D. aminoacyl synthetase
41. This is an example of transition mutation:
A. A to G B. A to C C. G to T D. C to G
42. The following enzymes are involved in translation function, EXCEPT
A. releasing factors C. holoenzyme
B. elongation factors D. aminoacyl-AMP
43. The molecular basis of Xeroderma pigmentosum is the:
A. lack endonuclease C. lack exonuclease
B. defective ligase closure D. defective DNA polymerase
44. What is the mechanism of regulation of gene expression
A. negative feedback C. repression
B. end product inhibition D. repression-derepression
45. A promoter site on DNA functions in which of the following?
A. transcribes repressor C. regulates termination
B. initiates transcription D. codes for RNA polymerase
46. Nucleoside modification (splicing), capping are associated with
A. translation C. replication
B. transcription D. none of these
47. In the process of protein synthesis, all polypeptides are begun with
A. lysine C. serine
B. threonine D. methionine
48. DNA region that extends from the promoter and the terminator is defined as
A. gene C. extron
B. intron D. transcription unit
45. Which of these is  required  for  activation  of  amino  acids  to  their  respective  tRNA’s
A. ATP B. GTP C. CTP D. cAMP
46.. Sigma factor is best described as
A. subunit of 50s ribosome that catalyzes peptide bond synthesis
B. subunit of DNA polymerase for synthesis in both 3’5’  and  5’3’  directions
C. factor that forms the bridge between 30S and 50S particles of the 70S ribosome
D. subunit of RNA polymerase for specificity of initiation of transcription of RNA
47. True of DNA-directed RNA polymerase
A. RNA polymerase I product is rRNA
B. RNA                      “                  II          “                “    tRNA
C. RNA                      “                  III        “                “    mRNA
D. All are true
48. In semiconservative replication, this is true, EXCEPT
A. it is an accepted manner of replication
B. each daughter DNA is identical to the parent DNA
C. succeeding  generation  of  DNA’s  will  yield  light  DNA’s
D. “                                    “                    “            “                “            “        2  light  DNA’s  and  2  hybrid  DNA’s  
49. A single code designates 1 amino acid. This refers to which characteristic of the genetic
code?
A. degeneracy C. unambiguous
B. overlapping D. no punctuation
50. Which posttranslational modification is needed to protect native conformation of protein
from denaturation?
A. methylation C. formation of S-S cross-link
B. carboxylation D. addition of prosthetic group

XAVIER UNIVERSITY – DR JOSE P RIZAL COLLEGE OF MEDICINE


Basic Biomedical Sciences
Third Bimonthly Period
QUIZ # 2

NAME ____________________________ 20 Nov 2006 SCORE ____________


MULTIPLE CHOICE: write the letter of the BEST answer on the answer sheet. Use capital
letters in blue/black ink. NO EASURES/SUPERIMPOSITIONS ALLOWED!

11. Lipoproteins transport the following in the circulation:


A. triglycerides (TG), cholesterol esters, phospholipids, free fatty acids
B. TG, cholesterol esters, phospholipids
C. TG, cholesterol esters
D. TG
12. These are true of chylomicrons, EXCEPT:
A. transport triglycerides (TG) & other lipids from the intestine to the blood
B. are delivered to the blood via the lymphatic system
C. chylomicron remnants are free of any lipids
D. chylomicron remnants enter the liver by receptor mediated endocytosis
13. The following are characteristics of low density lipoproteins (LDL), EXCEPT:
A. formed from IDL after this has lost more TG and proteins
B. are taken up by the liver by facilitated transport
C. transports cholesterol to extra hepatic tissues
D. LDL receptors are present in all nucleated cells
14.

XAVIER UNIVERSITY – DR JOSE P RIZAL COLLEGE OF MEDICINE


Basic Biomedical Sciences
Third Bimonthly Period
QUIZ # 2

NAME ____________________________ 20 Nov 2006 SCORE ____________


MULTIPLE CHOICE: write the letter of the BEST answer on the answer sheet. Use capital
letters in blue/black ink. NO EASURES/SUPERIMPOSITIONS ALLOWED!

11. Lipoproteins transport the following in the circulation:


A. triglycerides (TG), cholesterol esters, phospholipids, free fatty acids
B. TG, cholesterol esters, phospholipids
C. TG, cholesterol esters
D. TG
12. These are true of chylomicrons, EXCEPT:
A. transport triglycerides (TG) & other lipids from the intestine to the blood
B. are delivered to the blood via the lymphatic system
C. chylomicron remnants are free of any lipids
D. chylomicron remnants enter the liver by receptor mediated endocytosis
13. The following are characteristics of low density lipoproteins (LDL), EXCEPT:
A. formed from IDL after this has lost more TG and proteins
B. are taken up by the liver by facilitated transport
C. transports cholesterol to extra hepatic tissues
D. LDL receptors are present in all nucleated cells
14.
XAVIER UNIVERSITY – JOSE P RIZAL COLLEGE OF MEDICINE
Basic Biomedical Sciences
Third Bimonthly Period
QUIZ # 1

NAME ____________________________ 13 Nov 2006 SCORE ____________


MULTIPLE CHOICE: write the letter of the BEST answer on the answer sheet. Use capital
letters in blue/black ink. NO EASURES/SUPERIMPOSITIONS ALLOWED!

1. Porphyrins are cyclic compounds which emit red flourescence when the rings are
linked by:
A. methyl C. proprionate
B. methenyl D. acetate group
2. The porphyrins are classified on the basis of the:
A. 4 pyrrole rings C. side chain substituent for the 8 H atoms
B. affinity for the metal ions D. carbon bridges that link the pyrrole rings
3. Which of the following catalyzes the rate-limiting step in porhyrin biosynthesis in
mammalian liver?
A. ALA synthase C. porphobilinogen synthase
B. ALA dehydratase D. uroporphyrinogen syntase
4. The first precursor pyrrole ring formed in the pathway is:
A. ALA C. porphobilinogen
A. Uroporphyrin 1 D. uroporphyrinogen 1
5. Which of the following is true of heme biosynthesis?
A. Heme synthase appears to be the principal rate limiting step
B. Oxidation of the methyl side chain substituent forms protoporphyrin
C. the final step involves the incorporation of iron into protoporphyrin
D. all of the above
6. Which of the following precursors of heme is derived from TCA?
A. glycine C. succinyl-CoA
B. Coenzyme A D. pyridoxal phosphate
7. Which of the following is true of heme catabolism?
A. involves initial oxidation of iron in heme
B. catalyzed by heme oxygenase system
C. is carried out in the microsomal fraction of RE cells
D. all of the above
8. Carbon monoxide is formed/released during heme catabolism. The carbon atom is
derived from the:
A. reaction of CO2 and ferric ion
B. splitting of pyrrole rings I and IV
C. removal of alpha methynyl bridge
9. The bilirubin formed from RES are transported to the liver bound to plasma:
A. albumin C. lipoprotein
B. globulin D. glucoronide
10. Which of the following catalyzes the addition of a polar group to bilirubin in the
hepatocytes?
A. dismutase C. bilirubin reductase
B. UDPG transferase D. biliverdin reductase
11. Bacterial enzymes that catalyze the removal of glucoronides from bilirubin that
reaches the intestine are:
A. beta glucuronidases C. bilirubin reductases
B. UDPG transferases D. bilverdin reductases
12. The  Erlich’s  test  for  bilirubin  utilizes:
A. urine sample C. methanol as medium of reaction
B. diazotized sulfanilic acid D. all of the above
13. Bilirubin is secreted into bile by:
A. osmosis C. facilitated diffusion
B. simple diffusion D. active transport

XAVIER UNIVERSITY – DR JOSE P RIZAL COLLEGE OF MEDICINE


Basic Biomedical Sciences
Third Bimonthly Period
QUIZ # 3 (Proteins)

NAME ____________________________ 27 Nov 2006 SCORE ____________


MULTIPLE CHOICE: write the letter of the BEST answer on the answer sheet. Use capital
letters in blue/black ink. NO EASURES/SUPERIMPOSITIONS ALLOWED!

1. All amino acids share the absolute configuration of:


A. L-glycine C. L-glyceraldehyde
B. L-glutamine D. L-glycine
2. One of the following is a nutritionally essential amino acid:
A. Alanine C. asparagine
B. Arginine D. aspartate
3. Alpha helix features include the following EXCEPT:
A. Secondary structure
B. 4-50 residues
C. distance per turn is 5.4 nm
D. carboxyl groups directed outward from the helix axis
4. Amino acids do not absorb visible light. However, three (3) amino acids absorb UV
light. One of them is:
A. Threonine C. glycine
B. Tryptophan D. glutamine
5. The following are hydrophobic amino acids. Which one has an aromatic R group?
A. Tyrosine C. proline
B. Threonine D. hydroxyproline
6. Which of the following is TRUE of fibrous proteins?
A. Asymmetrical in shape
B. Have axial ratio greater than 10
C. Usually serve as transport proteins
D. Examples include collagen & hemoglobin
7. Enzymatic digestion of proteins initially occurs in the:
A. mouth C. duodenum
B. stomach D. ileum
8. The proteins that are digested in the GIT come from:
A. ingested food C. desquamated mucosal cells
B. digestive juices D. all of the above
9. The following enzymes needed for protein digestion are located in the brush border of
the intestinal mucosal cells:
A. aminopeptidases, carboxypeptidases, endopeptidases, dipeptidases
B. aminopeptidases, carboxypeptidases, endopeptidases
C. aminopeptidases, carboxypeptidases
D. aminopeptidases
10. Which of the following is/are TRUE of amino acid absorption:
A. absorption is slowest in the duodenum and jejunum
B. some transport mechanisms across luminal membrane are Na+
dependent
C. transport across the basolateral membrane are not Na+ dependent
D. all of the above
11. Enzymes with central positions in amino acid biosynthesis are the following EXCEPT:
A. Glutamate Dehydrogenase C. transaminase
B. Glutamate synthase D. glutamate hydrolase
12. These nutritionally nonessential amino acids are formed from nutritionally essential
amino acids, EXCEPT:
A. cysteine C. hydroxylysine
B. tyrosine D. glycine

13. Which of the following statements regarding phenylalanine and tyrosin is FALSE?
A. Provided that the diet contains adequate nutritionally essential Phenylalanine,
tyrosine is nutritionally non-essential.
B. Dietary Tyrosine can replace Phenylalanine.
C. Tyrosine is a nutritionally non-essential amino acid.
D. Phenylalanine is a nutritionally essential amino acid.
14. The following associations between precursors and amino acid products are correct,
EXCEPT:
A. pyruvate ---> alanine C. a-ketoglutarate ---> glutamine
B. oxaloacetate ---> aspartate D. glutamate ---> proline
15. IO5 – Enzymes with central positions in amino acid biosynthesis are the following
EXCEPT:
A. Glutamate Dehydrogenase C. transaminase
B. Glutamate synthase D. glutamate hydrolase
15. IO5 – True to conversion of Aspartate to Asparagine EXCEPT:
A. Catalyzed by asparaginesynthase
B. Resembles glutamine synthesis
C. Glutamine provides the Nitrogen
D. ATP is produced in the process
16. IO5 – The following amino acids are needed in large amounts for cysteine & tyrosine
production:
A. Methionine & phenylalanine
B. Methionine & proline
C. Phenylalanine & proline
D. Proline & tryptophan
17. Tissue aminotransferases reversibly interconvert the following amino acids and their
corresponding a-keto acids, EXCEPT:
A. valine C. leucine
B. isoleucine D. lysine
18. The carbon skeletons of the following amino acids are degraded to glutamate and
subsequently to a-ketoglutarate, EXCEPT:
A. proline C. arginine
B. histidine D. asparagine
19. The carbon skeletons of the following amino acids are converted to pyruvate and then
to acetyl-CoA:
A. alanine, cysteine, glycine, serine, threonine, hydroxyproline
B. alanine, cysteine, glycine, serine, threonine
C. alanine, cysteine, glycine
D. alanine, cysteine
20. The carbon skeletons of the following amino acids are degraded to succinyl-CoA,
EXCEPT:
A. Tyrosine C. methionine
B. Isoleusine D. valine
21. Which of the following amino acids can be converted to glycine?
A. serine, threonine, cystine, leucine
B. serine, threonine, cystine
C. serine, threonine
D. serine
22. These are true of protein degradation, EXCEPT:
A. Excess proteins are either stored or degraded
B. daily turnover is 1-2% of total body protein
C. intracellular proteases hydrolyze internal peptide bonds
D. of the liberated amino acids 70-80% are reutilized for new protein synthesis

23. Catabolism of branched chain amino acids cannot occur in the liver because the liver
does not contain:
A. branched chain amino transferase C. glutaminase
B. alanine transaminase D. all of the above
24. The rate-limiting enzyme of the urea cycle is:
A. gutamine synthase C. argininosuccinase
B. carbamoyl phosphate synthase I D. ornithine transcarbamoylase
25. Which of these steps in urea biosynthesis occurs in the mitochondria?
A. formation of carbamoyl phosphate
B. formation of argininosuccinate
C. formation of ornithine
D. release of urea from arginine
26. The end product of nitrogen catabolism in man is mainly:
A. glutamate C. uric acid
B. ammonia D. urea
27. IO6 - The formation of ammonia from A-amino groups occurs mainly via the A-amino
Nitrogen of:
A. Glutamine C. glutamate
B. Glycine D. tyrosine
28. IO6 – The following amino acids does not participate in transamination:
A. Threonine C. hydroxylysine
B. Tyrosine D. leucine
29. IO6 – All the amino Nitrogen from amino acids that undergo transamination can be
concentrated in:
A. Glutamate C. alanine
B. Tyrosine D. pyruvate
30. IO7 – All amino acids except_______ are at least partly glucogenic:
A. Lysine C. glycine
B. Isoleucine D. cysteine
31. IO 7 – These are the fates of the carbon skeletons in the catabolism of amino acids
EXCEPT:
A. Oxidized to form CO2 & H20
B. Degraded to Pyruvate
C. Degraded to acetyl-CoA
D. Oxidized to ammonia
32. The following are disorders associated with defects in the urea cycle, EXCEPT:
A. hyperammonemia C. citrullinemia
B. histidinemia D. argininosuccinicacidemia
33. What is the defect in hyperargininemia?
A. low level of erythrocyte arginase
B. absence of arginino succinase
C. deficiency of ornithine transcarboxylase
D. all of the above
34. Darkening of the urine upon exposure to air is seen in:
A. Hartnup disease C. phenylketonuria
B. cystinuria D. alkaptonuria
35. IO9 – Which one of the following statements concerning a 3-week old male infant with
CLASSICAL PKU is INCORRECT?
A. Tyrosine becomes an essential amino acid.
B. High levels of Phenylalanine are found in his blood
C. The diagnosis of PKU is performed after the infant has received protein-
containing nourishment.
D. A diet devoid of Phenylalanine should be initiated immediately.
36. IO 9 – Hyperphenylalaninemias arise from anomalies in the following, EXCEPT:
A. Phenylalanine hydroxylase
B. Dihydrobiopterin reductase
C. Dihydrobiopterin biosynthesis
D. Tetrahydrobiopterin biosynthesis
ANS: D (tetrahydrobiopterine is not synthesized in the process, it is catabolized)

37. IO1- One of the following is a nutritionally essential amino acid:


A. Alanine
B. Arginine
C. Asparagine
D. Aspartate
ANS: B

38. IO2- All amino acids share the absolute configuration of: BM
A. L-glycine
B. L-glutamine
C. L-glyceraldehyde
D. L-glycine
ANS: C

39. IO2- Alpha helix features include the following EXCEPT:


A. Secondary structure
B. 4-50 residues
C. carboxyl groups directed outward from the helix axis
D. distance per turn: 5.4 nm
ANS: C (amino acyl groups directed outward)

40. IO2-Peptide bond formation involves:


A. Removal of ATP & condensation w/ A-carboxyl group of a 2nd amino acid ???
B. Activation of carboxyl group of one amino acid & condensation with one
molecule of water
C. Removal of one molecule of ATP & subsequent production of amino acyl
adenylate
D. Activation of carboxyl group of one amino acid & removal of one molecule of
water
ANS: D (activation of carboxyl group involves condensation with ATP & the bond
formed by the 2 amino acids involves removal of one molecule of water)

41. IO2- When proteins lose their biologic activity, they have undergone the following
EXCEPT:???
A. Protein analysis
B. Denaturation
C. Disruption of peptide bonds
D. Non-disruption of sulfide bonds
ANS: A (denatured proteins need not always undergo analysis)

42. IO2- Amino acids do not absorb visible light. However, 3 amino acids absorb UV light.
One of them is:
A. Threonine
B. Tryptophan
C. Glycine
D. Glutamine
ANS: B (they are tryptophan, phenylalanine & tyrosine)

43. IO2- The following are hydrophobic amino acids. Which one has an aromatic R group?
A. Tyrosine
B. Threonine
C. Proline
D. Hydroxyproline
ANS: A

44. IO2 – One of the following amino acids induces bends in A-helices:
A. Proline
B. Lysine
C. Leucine
D. Hydroxylysine
ANS: A (proline & glycine do)

45. IO2- Which of the following does not contain sulfur?


A. Methionine
B. Cystathione
C. Cysteine
D. Valine
ANS: D

46. IO2- Fibrous proteins:


A. Asymmetrical in shape
B. Have axial ratio greater than 10
C. Usually serve as transport proteins
D. Examples include collagen & hemoglobin
ANS: B (they are symmetrical, with axial ratio greater than 10, serve as structural
proteins & hemoglobin is not one of them)

LECTURE QUESTIONS
47. IO2- The only amino acid whose a-carbon is NOT chiral:
A. Glutamate
B. Glycine
C. Lysine
D. A-ketoglutarate
ANS:  B    (all  aa’s  have  chiral  a-carbons except glycine)

48. IO6 - The formation of ammonia from A-amino groups occurs mainly via the A-amino
Nitrogen of:
A. Glutamine
B. Glycine
C. Glutamate
D. Tyrosine
ANS: C

49. IO5 – Enzymes with central positions in amino acid biosynthesis are the following
EXCEPT:
A. Glutamate Dehydrogenase
B. Glutamate synthase
C. Transaminase
D. Glutamate hydrolase
ANS: D

50. IO5 – Which of the following statements is false:


A. Provided that the diet contains adequate nutritionally essential Phenylalanine,
tyrosine is nutritionally non-essential.
B. Dietary Tyrosine can replace Phenylalanine.
C. Tyrosine is a nutritionally non-essential amino acid.
D. Phenylalanine is a nutritionally essential amino acid.
ANS: B (the conversion of phenylalanine to tyrosine is irreversible).

51. IO5 – Which of the following statements is true:


A. Dietary hydroxylysine is incorporated into protein by its specific tRNA.
B. Dietary hydroxyproline is NOT completely degraded.
C. For every mole of proline or lysine hydroxylated , one molecule of A-
ketoglutarate is decarboxylated.
D. Impaired hydroxylation in the presence of Vit.C leads to scurvy.
ANS:  C  (  there  is  no  tRNA  for  either  hydroxylated  aa’s.  Hence,  neither  is  
incorporated to proteins. Both are completely degraded. ABSENCE of Vit.C causes
impaired hydroxylation leading to scurvy).

52. IO4 – One of the following statements is false:


A. Human diet must contain adequate quantities of the ten essential amino acids.
B. Neither humans nor animals can synthesize the ten essential amino acids.
C. Kwashiorkor results when a child is weaned onto a starchy diet poor in protein.
D. Marasmus results when there is deficient caloric intake.
ANS: D ( marasmus is deficient in both carbohydrates & protein)

53. IO5 – True to conversion of Aspartate to Asparagine EXCEPT:


A. Catalyzed by asparaginesynthase
B. Resembles glutamine synthesis
C. Glutamine provides the Nitrogen
D. ATP is produced in the process
ANS: D (ATP is used up in the process)

54. IO6 – The following amino acids does not participate in transamination:
A. Threonine
B. Tyrosine
C. Hydroxylysine
D. Leucine
ANS:  A  (the  four  aa’s  are:  lysine,  threonine,  proline  &  hydroxyproline)

55. IO6 – All the amino Nitrogen from amino acids that undergo transamination can be
concentrated in:
A. Glutamate
B. Tyrosine
C. Alanine
D. Pyruvate
ANS: A
56. IO7 – All amino acids except_______ are at least partly glucogenic:
A. Lysine
B. Isoleucine
C. Glycine
D. Cysteine
ANS: A (lysine & leucine are ketogenic)
57. IO 7 – The fate of carbon skeletons in the catabolism of amino acids EXCEPT:
A. Oxidized to form CO2 & H20
B. Degraded to Pyruvate
C. Degraded to acetyl-CoA
D. Oxidized to ammonia
ANS: D (carbon skeletons do not participate in the Urea cycle)

58. IO5 – The following amino acids are needed in large amounts for cysteine & tyrosine
production:
A. Methionine & phenylalanine
B. Methionine & proline
C. Phenylalanine & proline
D. Proline & tryptophan
ANS: A (methionie-cysteine; phenylalanine-tyrosine)

59. IO 7 – The fate of carbon skeletons in the catabolism of amino acids EXCEPT:
A. Oxidized to form CO2 & H20
B. Degraded to Pyruvate
C. Degraded to acetyl-CoA
D. Oxidized to ammonia
ANS: D (carbon skeletons do not participate in the Urea cycle)

60. IO 9 – Hyperphenylalaninemias arise from anomalies in the following, EXCEPT:


A. Phenylalanine hydroxylase
B. Dihydrobiopterin reductase
C. Dihydrobiopterin biosynthesis
D. Tetrahydrobiopterin biosynthesis
ANS: D (tetrahydrobiopterine is not synthesized in the process, it is catabolized)

61. IO5 – Which one of the following is synthesized from an essential amino acid:
A. Aspartate
B. Tyrosine
C. Alanine
D. histidine
ANS: B (tyrosine is from phenylalanine which is essential)

62. IO6 – Which one of the following statements is correct?


A. An increase in gluconeogenesis from amino acids results in a decrease in urea
formation
B. Ornithine & citrulline are found in tissue proteins
C. Cysteine is an essential amino acid in individuals consuming a diet devoid of
methionine
D. In the presence of adequate dietary sources of tyrosine , phenylalanine is NOT
an essential amino acid.
ANS: C (inc gluconeogenesis  inc aa catabolism  inc urea; tyr can not produce
PA bec the reaction is irreversible, so PA is still essential; sulfur for cysteine comes
from methionine. So in the absence of meth., cysteine need be supplied).

63. IO9 – Which one of the following statements concerning a 3-week old male infant with
CLASSICAL PKU is INCORRECT:
A. Tyrosine becomes an essential amino acid.
B. High levels of Phenylalanine are found in his blood
C. The diagnosis of PKU is performed after the infant has received protein-
containing nourishment.
D. A diet devoid of Phenylalanine should be initiated immediately.
ANS: D (a sufficient phenylalanine is needed for growth & development)

64. IO 6 – Which one of the following statements is correct?


A. In humans, the major route of Nitrogen metabolism from amino acids to urea is
catalyzed by the combined actions of transaminase & glutamate
dehydrogenase
B. Ammonia is quantitatively the most important Nitrogen containing compound in
human urine.
C. Amino acids in excess of biosynthetic needs are stored as proteins.
D. An aminotransferase reaction using alanine & A-ketoglutarate as substrates
yields glutamate & oxaloacetate as products.
ANS:  A  (urea  in  urine;;  excess  aa’s  are  broken  down;;  alanine  &  A-KG – Pyruvate&
glutamate)

65. IO 6 – Which one of the following statements about the urea cycle is correct?
A. The two N atoms that are incorporated into urea enter the cycle as ammonia &
alanine.
B. Urea is produced directly by the hydrolysis of ornithine.
C. ATP is required for the reaction in which arginosuccinate is cleaved to form
arginine.
D. Urinary urea is increased by a diet rich in proteins.
ANS: D (N atoms enter as ammonia & N-acetylglutamate; urea from the hydrolysis
of arginine; ATP required for CAP production; excess protein is broken down to
urea)

66. IO5 - Tissue aminotransferases reversibly interconvert the following amino acids &
their corresponding a-keto acids, EXCEPT:
A. Valine
B. Isoleucine
C. Leucine
D. Lysine
ANS: D (leucine, isoleucine & valine can be reversibly interconverted by
aminotrasferases)

67. IO5 – True of glycine synthesis:


A. Has two possible routes of production
B. The main pathway is through glycine transaminase
C. One possible pathway is through lysine
D. Folic acid is needed
ANS: D (has 3 possible routes; main path is through serine
hydroxymethyltransferase; lysine is not among the 3; folic acid is needed as
tetrahydrofolate).

CEREBRUM – 4TH BM

1. Which of these components of the circle of Willis is a branch of the basilar artery?
A. anterior communicating artery C. anterior cerebral artery
B. posterior communicating artery D. posterior cerebral artery
2. Which of the following associations between layers of cerebral cortex and predominant
cells found in these layers is correct?
A. external granular layer – cells of Cajal
B. external pyramidal layer – stellate cells
C. internal granular layer – cells of Martinotti
D. ganglionic layer – pyramidal cells
3. Which of these statements is true of the cerebral ventricular system?
A. The lateral ventricles communicate with the 3rd through aqueduct of Syvius.
B. The 3rd ventricle communicates with the 4th through foramina of Monro.
C. The 4th ventricle is continuous with the narrow central canal of spinal cord
D. Choroid plexus is found in lateral, 3rd, & 4th ventricles and in aqueduct of Sylvius
4. As regards the basal ganglia or nuclei, which of these is true?
A. are made up of the corpus striatum, claustrum and substancia nigra
B. are involved in the control of posture and voluntary movement
C. make direct connections to the cerebellum and to the spinal cord
D. outflow from basal nuclei is through the caudate nucleus and putamen
5. Which of the following associations  between  Brodmann’s  (B)  areas  and  functions  is  
correct?
A. B 4 – somatosensory C. B 17 – vision
B. B 43 – audition D. B 41, 43 – fine motor skills
6. Which of the following hypothalamic nuclei to function associations is true?
A. Paraventricular nucleus – synthesizes antidiuretic hormone
B. Medial hypothalamic nuclei – initiate eating and increase food intake
C. anterior hypothalamic nuclei – regulate temperature (response to heat)
D. suprachiasmatic nucleus – controls parasympathetic system
7. Mr Nick, a journalist, consulted a neurologist because he noticed that lately he had
been having much difficulty in finding the correct words to express his thoughts.
Neurologic examination revealed the following: faster than normal spoken language,
normal grammatic construction but was unable to find the correct words to express his
thoughts so he substituted one word for another. There was no associated hemiplegia.
Based on these data, Mr Nick is likely to have a lesion in the:
A.    Broca’s  area C.    Wernicke’s  area
B. Paralimbic area D. Prefrontal area
8. A lesion in the hippocampus results in:
A. selective amnesia C. retrograde amnesia
B. anterograde amnesia D. all of the above

Fourth Bimonthly

Module 34

1. Which of the following confers specificity to receptors? C


A.  Mueller’s  doctrine C. Law of appropriate stimulus
B. Weber-Fechner law D. Pattern theory
2. After eating sour pickles, an individual noticed that the water he drank tastes sweet.
This is most probably due to: B
A. simultaneous contrast C. positive after image
B. successive contrast D. negative after image
3. In an experiment, a student placed a red disc on a sheet of white paper. While staring
at the red disc, he noticed a ring of green surrounding the red disc. This is most
probably due to:A
A. simultaneous contrast C. positive after image
B. successive contrast D. negative after image
4. Which of the following is a rapidly adapting receptor? B
A. pressoreceptor C. muscle spindle
B. Pacinian corpuscle D. A delta fiber
5. The most common site of pain modulation is at the: D
A. rostral ventromedial medulla C. pontine raphe nucleus
B. locus ceruleus D. ventrolateral periacqueductal gray
6. Which of the following pain modulators inhibits C fibers? C
A. endorphin B. enkephalin C. serotonin D. dynorphin

Module 35
1. Which of the following statements is characteristic of cones? A
A. Cones are sensitive to chromatic stimuli.
B. Cones are distributed at all parts of the retina.
C. Cones have low visual acuity as compared to rods.
D. Cones are more sensitive to light than rods.
2. Which of the following statements is characteristic of color vision? C
A. A color sensation, upon constant stimulation, is modified rapidly.
B. The color sensation of a central stimulus is the same as that of a peripheral
stimulus.
C. Every color sensation is unalyzable into stimulus components.
D. Every color sensation is evoked by a finite number of color stimuli.
3. Which of the following color disturbances is presumably present equally in males and
females? B
A. monochromat B. tritanope C. deuteranope D. protanope
4. When light passes through two apertures located close to each other, light waves that
emanate from the apertures tend to overlap at certain points producing: D
A dispersion B. refraction C. reflection D. diffraction
5. Which of the following errors of refraction is commonly associated with the aging
process?
A. myopia B. hyperopia C. presbyopia D. astigmatism C

Aging

1. The theory of aging which states that aging is programmed at birth by a certain
number of genes is the: D
A. genetic theory C. specie theory
B. apoptosis theory D. pacemaker theory
2. The theory of aging which attributes aging to accumulation of free radicals is the: A
A. oxidative theory C. wear and tear theory
B. error catastrophe theory D. cross-linking theory

3. Which of the following is most likely to happen with a decrease in sensitivity of


chemoreceptors and baroreceptors in the elderly population? B
A. less efficient pumping of the heart C. tendency to develop pulmonary edema
B. postural hypotension D. tendency for the heart to increase in size
4. Which of the following is a physiological change associated with aging? C
A. decrease in bronchial secretions C. decrease in acid secretion
B. decreased resistance to gas flow D. decrease in cholesterol secretion
5. Which of the following is an anatomical change associated with aging? D

A. increase in liver mass and blood flow C. decrease in skin pigmentation


B. increase in size of the thoracic cage D. decrease in lean body mass
6. Which of the following taste sensations is the first to diminish with aging? C
A. sweet B sour C. salty D. umami
7. Which of the following types of memory improves with aging? A
A. semantic memory C. episodic memory
B. procedural memory D. sensory memory
8. Which of the following is increased as part of the aging process? B
A. retronasal olfaction C. functional speech hearing
B. opacity of the lens D. proprioception
9. Which of the following foods is believed to delay aging by its action on free radicals?
D
A. broccoli B. apple C. ginger D. tea
10. Which of the following foods are believed to contain saponins, which lower
cholesterol?
A. carrots B. flaxseed C. peas D. onions
Neurobiologic Rhythms

1. The best way to synchronize the circadian rhythm to the exact 24 hours imposed on
man by the sun is to make use of: B
A. entrainment B. zeitgebers C. drugs D. muscle activity
2. The  principal  component  of  the  “internal  clock  “  is  the: A
A. suprachiasmatic nucleus C. retina
B. intergeniculate leaflet D. pineal gland
3. Fluctuations in body temperature is an example of: C
A. circadian rhythm C. ultradian rhythm
B. infradian rhythm D. circaseptan rhythm
4. Which of the following neurotransmitters/neuroreceptor ligands can produce arousal?
A. melatonin B. glutamate C. muscimol D. serotonin B
5. Which of the following is most useful in the prevention of jet lag among tourists? C
A. adjusting sleep schedule several days before departure
B. embark on a physical training program before departure
C. stay as close as possible to the day-night schedule of his/her residence
D. take regular meals at appropriate times, e.g., cooked meal at midnight
6. Majority of the tonic and phasic activities that occur during REM sleep are due to
which of the following? D
A. high activity of ascending reticulo-activating system
B. low activity of ascending reticulo-activating system
C. high activity of serotoninergic nuclei of pontine raphe
D. activity of giganto-cellular tegmental field
7. Which of the following makes up the greater bulk of the sleep time of a young adult?
B
A. stage 1 B. stage 2 C. delta sleep D. delta sleep
8. A 45 year old obese male consulted a physician because of excessive daytime
sleepiness. In the sleep clinic, he fell asleep within 5 seconds after the lights were
turned off, but awaked 35 seconds later gasping for air. The cycle repeated itself for
the  next  ten  hours  of  “sleep”.  This  individual  is  most  probably  suffering  from: C
A. narcolepsy C. sleep apnea
B. primary hypersomnia D. neutral state syndrome

9. A 25 year old male clad only in shoes and shorts is in a room with a temperature of 21 o
C and a humidity of 20%. As he stands quietly, heat elimination will most probably be:
B
A. 65% by insensible perspiration C. 40% by evaporation
B. 65 % by radiation D. 40% by convection
10. If the above-mentioned male engages in a strenuous tennis match, most of his body
heat would be lost by: D
A. radiation B. conduction C. convection D. evaporation

Physiology in Unusual Environments

1. Which of the following is most likely to take place upon ascent to an altitude of 10,000
feet?
A. mild tremors C. impaired judgment C
B. reflex irritability D. personality changes
2. Which of the following would require permanent descent to sea level?
A. acute mountain sickness C. high altitude pulmonary edema
B. chronic mountain sickness D. High altitude cerebral edema
3. Which of the following is expected to occur within 72 hours of ascent to high altitudes?
A
A. increase in arterial pH C. increase in 2,3 DPG
B. increase in hemoglobin D. decrease in arterial bicarbonate
4. Which of the following is a characteristic of aviation environment? D
A. Most aircrafts are pressurized at sea level.
B. Sudden loss of cabin pressure can lead to hypoxia with slow decompression.
C. Sudden loss of cabin pressure causes increased oxygen saturation of blood.
D. Travel in pressurized aircrafts over long distances can cause dehydration.
5. One of the most serious hazards of long term missions in aerospace is: B
A. space adaptation syndrome C. muscle atrophy
B. decalcification of bone D. decrease in RBC by 5-15%
6. Prevention of muscle atrophy in aerospace is made possible by using a: C
A. Gamow bag C. penguin suit
B. decompression suit D. lower body negative pressure suit
7. Which of the following is expected to decrease upon immersion in water up to the
neck?
A. inspiratory reserve volume C. pulmonary blood volume B
B. functional residual capacity D. pulmonary diffusing capacity
8. Which of the following is expected to decrease upon immersion in water up to the
neck?
A. venous return C. stroke volume B
B. ADH secretion D. mean pulmonary artery pressure
9. Nitrogen narcosis at greater depths can produce:
A. euphoria C. disorientation
B. clumsiness D. loss of memory
10. Which of the following is associated with decreased lipid volume of nerves? D
A. decompression sickness C. oxygen toxicity
B. nitrogen narcosis D. high pressure nervous syndrome

Physiology of Exercise

1. The ability to resist fatigue is called: B


A. fitness B. endurance C. conditioning D. adaptation
2. Which of the following forms of sports makes use of anaerobic glycolysis? C
A. weight lifting B. 100 meter dash C. 100 meter swim D. diving
3. The target heart rate range of a 20 year old individual with a resting heart rate of
90/min., who embarks on an exercise program should be: A
A.156 – 176/min C. 176 – 198/min
B.150 – 170/min D. 110 – 200/min
4. Which of the following hormones only act in a permissive manner to support activity of
the other hormones increased during exercise? D
A. glucagon B. aldosterone C. epinephrine D. cortisol

5. An oxygen consumption of 1-2 liters/min is typical of which of the following? C


A. mild work C. hard work
B. moderate work D. maximal work
6. Which of the following measures heat production during exercise? B
A. open circuit spirometer C. cycle ergometer
B. calorimeter D. treadmill
7. A 21 year old 50 kg ballet dancer would have an energy expenditure of ___ at each
ballet session. A
A. 5.25 kcal/min B. 52.5 kcal/min C. 4.38 kcal/min D. 7 kcal/min
8. In order to be of value, carbohydrate-laden drinks should be taken: D
A. 1 hour before exercise C. 1 hour after exercise
B. 30 minutes before exercise D. at intervals during exercise
9. Which of the following can mask the presence of prohibited drugs in the urine of
athletes?
A. caffeine B. insulin C. dextran D. erythropoietin C
10. Which of the following serves as part of warm-up exercise? B
A. toe touch – 10 repetitions C. flutter kick – 30 repetitions
B. half knee bend – 10 repetitions D. push-up – 4 repetitions

Xavier University – Dr Jose P Rizal College of Medicine


Basic Biomedical Sciences – Fourth Bimonthly Exam
Set B: Modules 34 – ST3

NAME ________________________ 7 March 2007 SY 2006 – 07

MULTIPLE CHOICE: Write the letter of the BEST answer in CAPITAL letter in ink on
the answer sheet. NO ERASURES/SUPERIMPOSITIONS ALLOWED!

Module 34 –sensory pathways (5)

1. In a patient with loss of pain & temperature sensation on the right side of the body
L1 dermatome and below, the lesion is most likely in the: B
A. left spinothalamic tract, level of lower medulla
B. left spinothalamic tract, level of T11 spinal segment
C. right spinothalamic tract, level of L1 spinal segment
D. right spinothalamic tract, level of upper medulla
2. A lesion on this structure will show ipsilateral neurologic deficit:
A. ventral trigeminal lemniscus C. medial lemniscus
B. lateral spinothalamic tract D. fasciculus cuneatus
3. One of these structures is NOT part of the pain & temperature pathway from the face:
A. spinal tract of V C. posteromedial ventral nucleus
B. ventral trigeminal lemniscus D. lateral spinothalamic tract
4. This sinus passes through the jugular foramen to join the superior bulb of the internal
juguglar vein:
A. cavernous C. straight
B. transverse D. sigmoid

5. Which of the following confers specificity to receptors?


A.  Mueller’s  doctrine C. Law of appropriate stimulus
B. Weber-Fechner law D. Pattern theory
Module 35 – sense organs (15)
6. Which of the following is least likely a risk factor for a patient with transient monocular
blindness is
A. Diabetes C. Smoking
B. Hypertension D. Alcohol intake
7. Which of the following diagrams represent the visual field defect in an injury to the
right visual cortex? (left homonymous hemianopsia with macular sparing)
8. This structure in the visual somatic reflex pathway is the center for movements of the
head, eyes and body in response to visual stimuli
A. inferior collliculus C. lateral geniculate body
B. superior colliculus D. medial geniculate body
9. The response of lid closure in the menace reflex is brought about by
A. Rapidly approaching object C. Foreign object in the eye
B. Sudden bright light D. Splashing of water into the eye
10. A CT scan revealed a golf ball size tumor in the sella turcica. The patient complained
of bilateral blindness. What is the possible visual field defect?
A. Heteronymous hemianopsia C. Upper quadrantanopsia
B. Bitemporal hemianopsia D. Complete blindness
11. Which of the following statements is characteristic of cones?
A. Cones are sensitive to chromatic stimuli.
B. Cones are distributed at all parts of the retina.
C. Cones have low visual acuity as compared to rods.
D. Cones are more sensitive to light than rods.
12. Which of the following statements is characteristic of color vision?
A. A color sensation, upon constant stimulation, is modified rapidly.
B. The color sensation of a central stimulus is the same as that of a peripheral
stimulus.
C. Every color sensation is unalyzable into stimulus components.
D. Every color sensation is evoked by a finite number of color stimuli.
13. Vestibular nystagmus is characterized by which of the following?
A.may be initiated by visual impulses
B.occurs at the onset and at the end of head rotation
C.the slow component is of same direction as the head movement
D.direction of nystagmus is determined by the direction of the slow component
14. The organ responsible for matching sound impedance in the middle and inner ears:
A. tympanic membrane C. ear ossicles
B. oval window D. inner ear fluid
12. Salty substances (e.g. sodium) stimulate taste receptor cells by this mechanism:
A. blockage of ionic channels
B. opening of ionic channels
C. activation of second-messenger systems
D. direct passage of sodium through open ion channels
13. The arterial supplies to the ear are branches of the following :
A.external carotid C.maxillary
B. superficial temporal D. all of the above
14.Feeling of nausea associated with infection of the external auditory canal
attributed to which cranial nerve?
A.CN VIII C. CN V
B. CN X D. CN VII
15. The thalamic nucleus that is part of the auditory pathway:
A. posteromedial ventral nucleus C. medial geniculate body
B. inferior colliculus nucleus D. posterolateral ventral nucleus
16. One of these structures is NOT part of the auditory pathway:
A. trapezoid body C. spiral ganglion
C. lateral lemniscus D. Area 43
17.One of these cranial nerve nuclei does NOT receive vestibular fibers:
A. oculomotor C. trigeminal
B. trochlear D. abducens
18.The nucleus in the brain stem that mediates taste sensation:
A. main sensory nucleus of V C. nucleus ambiguus
B. nucleus solitarius D. facial nucleus

19. Belongs to the vascular pigmented coat of the eyeball:


A. Sclera C. cornea
B. Choroid D. retina
C. Cornea
D. Retina
20. Which of the following is not true of the retina:
A. Lies beneath the choroid
B. Forms the nervous coat of the eyeball
C. It has a blind spot called fovea centralis
D. Pierced by the central retinal artery
Module 36 – domestic violence (9)

21. Which of the following is most likely to happen with a decrease in sensitivity of
chemoreceptors and baroreceptors in the elderly population?
A. less efficient pumping of the heart C. tendency to develop pulmonary edema
B. postural hypotension D. tendency for the heart to increase in size
22.Which of the following is an anatomical change associated with aging?
A. increase in liver mass and blood flow C. decrease in skin pigmentation
B. increase in size of the thoracic cage D. decrease in lean body mass
23. Which of the following types of memory improves with aging?
A. semantic memory C. episodic memory
B. procedural memory D. sensory memory
24.The wife who is not given enough money for the household and personal
needs is a form of what violence?
A. physical C. social
B. psychological D. economic
25. Controlling what a person does, who she sees and talks to, where she
goes is :
A. isolation C. threat
B. emotional abuse D. intimidation
26. Violent verbal behavior of elderly persons will most likely lead to:
A. Physical and mental impairment
B. Transgenerational violence
C. Psychological deficiency
D. Caregiver stress
27.Which of the following signs and symptoms strongly suggest elderly abuse?
A. Evidence of fresh injuries
B. Abrupt seeking of treatment
C. Signs of withdrawal or depression
D. Injuries consistent with the history
28.One of the components of aging is the presence of damaged proteins that stick around
and eventually cause problems. One of the following is true in relation to this:
A .proteins develop cross-links to each other that increase its mobility & conductance
B. our bodies have the capacity to repair DNA but not all are complete &
accurate
C. the cross-linking of damaged proteins in blood vessel walls softens the arteries
leading to leakage of plasma in old age
D. the presence of cross-linkages activate proteases
29. Some theories for preventing aging include one of the following:
A. giving of carnosine to prevent DNA damage
B. giving of telomerase to enhance telomere shortening
C. protein & calorie restriction to speed up to a rapid metabolism
D. giving of carnosine to prevent cross-linking of fibroblasts
ANS: D (telomerase prevents DNA damage & repairs telomere shortening)
ST1 – NEUROBIO (7)
30.  The  principal  component  of  the  “internal  clock  “  is  the: A
A. suprachiasmatic nucleus C. retina
B. intergeniculate leaflet D. pineal gland
31. Which of the following neurotransmitters/neuroreceptor ligands can produce arousal?
A. melatonin B. glutamate C. muscimol D. serotonin B
32.Which of the following is most useful in the prevention of jet lag among tourists? C
A. adjusting sleep schedule several days before departure
B. embark on a physical training program before departure
C. stay as close as possible to the day-night schedule of his/her residence
D. take regular meals at appropriate times, e.g., cooked meal at midnight
33. Majority of the tonic and phasic activities that occur during REM sleep are due to which
of the following?
A. high activity of ascending reticulo-activating system
B. low activity of ascending reticulo-activating system
C. high activity of serotoninergic nuclei of pontine raphe
D. activity of giganto-cellular tegmental field
34. A 45 year old obese male consulted a physician because of excessive daytime
sleepiness. In the sleep clinic, he fell asleep within 5 seconds after the lights were
turned off, but awaked 35 seconds later gasping for air. The cycle repeated itself for
the  next  ten  hours  of  “sleep”.  This  individual  is  most  probably  suffering  from:
A. narcolepsy C. sleep apnea
B. primary hypersomnia D. neutral state syndrome

35. A 25 year old male clad only in shoes and shorts is in a room with a temperature of
21o C and a humidity of 20%. As he stands quietly, heat elimination will most probably
be: B
A. 65% by insensible perspiration C. 40% by evaporation
B. 65 % by radiation D. 40% by convection
36. If the above-mentioned male engages in a strenuous tennis match, most of his body
heat would be lost by:
A. radiation B. conduction C. convection D. evaporation
ST2 – UNUSUAL ENVIRON (7)
37.Which of the following would require permanent descent to sea level?
A. acute mountain sickness C. high altitude pulmonary edema
B. chronic mountain sickness D. High altitude cerebral edema
38.Which of the following is expected to occur within 72 hours of ascent to high altitudes?
A. increase in arterial pH C. increase in 2,3 DPG
B. increase in hemoglobin D. decrease in arterial bicarbonate
39.Which of the following is a characteristic of aviation environment?
A. Most aircrafts are pressurized at sea level.
B. Sudden loss of cabin pressure can lead to hypoxia with slow decompression.
C. Sudden loss of cabin pressure causes increased oxygen saturation of blood.
D. Travel in pressurized aircrafts over long distances can cause dehydration.
40.Prevention of muscle atrophy in aerospace is made possible by using a:
A. Gamow bag C. penguin suit
B. decompression suit D. lower body negative pressure suit
41.Which of the following is expected to decrease upon immersion in water up to the
neck?
A. venous return C. stroke volume
B. ADH secretion D. mean pulmonary artery pressure
42.Nitrogen narcosis at greater depths can produce:
A. euphoria C. disorientation
B. clumsiness D. loss of memory
43.Which of the following is associated with decreased lipid volume of nerves?
A. decompression sickness C. oxygen toxicity
B. nitrogen narcosis D. high pressure nervous syndrome

ST3 – EXERCISE (7)


44.Which of the following forms of sports makes use of anaerobic glycolysis?
A. weight lifting B. 100 meter dash C. 100 meter swim D. diving
45.The target heart rate range of a 20 year old individual with a resting heart rate of
90/min., who embarks on an exercise program should be:
A.156 – 176/min C. 176 – 198/min
B.150 – 170/min D. 110 – 200/min
46.Which of the following hormones only act in a permissive manner to support activity of
the other hormones increased during exercise?
A. glucagon B. aldosterone C. epinephrine D. cortisol
47.Which of the following measures heat production during exercise?
A. open circuit spirometer C. cycle ergometer
B. calorimeter D. treadmill
48.A 21 year old 50 kg ballet dancer would have an energy expenditure of ___ at each
ballet session.
A. 5.25 kcal/min B. 52.5 kcal/min C. 4.38 kcal/min D. 7 kcal/min
49. Which of the following can mask the presence of prohibited drugs in the urine of
athletes?
A. caffeine B. insulin C. dextran D. erythropoietin
50.Which of the following serves as part of warm-up exercise?
A. toe touch – 10 repetitions C. flutter kick – 30 repetitions
B. half knee bend – 10 repetitions D. push-up – 4 repetitions

Xavier University – Dr Jose P Rizal College of Medicine


Basic Biomedical Sciences – Fourth Bimonthly Exam
Set B: Modules 34 – ST3

NAME ________________________ 7 March 2007 SY 2006 – 07

MULTIPLE CHOICE: Write the letter of the BEST answer in CAPITAL letter in ink on
the answer sheet. NO ERASURES/SUPERIMPOSITIONS ALLOWED!

1. In a patient with loss of pain & temperature sensation on the right side of the body
L1 dermatome and below, the lesion is most likely in the: B
E. left spinothalamic tract, level of lower medulla
F. left spinothalamic tract, level of T11 spinal segment
G. right spinothalamic tract, level of L1 spinal segment
H. right spinothalamic tract, level of upper medulla
2. A lesion on this structure will show ipsilateral neurologic deficit:
A. ventral trigeminal lemniscus C. medial lemniscus
B. lateral spinothalamic tract D. fasciculus cuneatus
3. One of these structures is NOT part of the pain & temperature pathway from the face:
A. spinal tract of V C. posteromedial ventral nucleus
B. ventral trigeminal lemniscus D. lateral spinothalamic tract
15. This sinus passes through the jugular foramen to join the superior bulb of the internal
juguglar vein:
A. cavernous C. straight
B. transverse D. sigmoid
16. Which of the following confers specificity to receptors?
A.  Mueller’s  doctrine C. Law of appropriate stimulus
B. Weber-Fechner law D. Pattern theory
17. Which of the following is least likely a risk factor for a patient with transient monocular
blindness is
A. Diabetes C. Smoking
B. Hypertension D. Alcohol intake
18. Which of the following diagrams represent the visual field defect in an injury to the
right visual cortex? (left homonymous hemianopsia with macular sparing)

19. This structure in the visual somatic reflex pathway is the center for movements of the
head, eyes and body in response to visual stimuli
A. inferior collliculus C. lateral geniculate body
B. superior colliculus D. medial geniculate body
20. The response of lid closure in the menace reflex is brought about by
A. Rapidly approaching object C. Foreign object in the eye
B. Sudden bright light D. Splashing of water into the eye
21. A CT scan revealed a golf ball size tumor in the sella turcica. The patient complained
of bilateral blindness. What is the possible visual field defect?
A. Heteronymous hemianopsia C. Upper quadrantanopsia
B. Bitemporal hemianopsia D. Complete blindness
22. Which of the following statements is characteristic of cones?
A. Cones are sensitive to chromatic stimuli.
B. Cones are distributed at all parts of the retina.
C. Cones have low visual acuity as compared to rods.
D. Cones are more sensitive to light than rods.
23. Which of the following statements is characteristic of color vision?
A. A color sensation, upon constant stimulation, is modified rapidly.
B. The color sensation of a central stimulus is the same as that of a peripheral
stimulus.
D. Every color sensation is unalyzable into stimulus components.
E. Every color sensation is evoked by a finite number of color stimuli.
24. Vestibular nystagmus is characterized by which of the following?
A.may be initiated by visual impulses
B.occurs at the onset and at the end of head rotation
C.the slow component is of same direction as the head movement
D.direction of nystagmus is determined by the direction of the slow component
25. The organ responsible for matching sound impedance in the middle and inner ears:
A. tympanic membrane C. ear ossicles
B. oval window D. inner ear fluid
12. Salty substances (e.g. sodium) stimulate taste receptor cells by this mechanism:
A. blockage of ionic channels
B. opening of ionic channels
C. activation of second-messenger systems
D. direct passage of sodium through open ion channels
13. The arterial supplies to the ear are branches of the following :
A.external carotid C.maxillary
B. superficial temporal D. all of the above
14.Feeling of nausea associated with infection of the external auditory canal
attributed to which cranial nerve?
A.CN VIII C. CN V
B. CN X D. CN VII
15. The thalamic nucleus that is part of the auditory pathway:
A. posteromedial ventral nucleus C. medial geniculate body
B. inferior colliculus nucleus D. posterolateral ventral nucleus
16. One of these structures is NOT part of the auditory pathway:
A. trapezoid body C. spiral ganglion
C. lateral lemniscus D. Area 43
17.One of these cranial nerve nuclei does NOT receive vestibular fibers:
A. oculomotor C. trigeminal
B. trochlear D. abducens
18.The nucleus in the brain stem that mediates taste sensation:
A. main sensory nucleus of V C. nucleus ambiguus
B. nucleus solitarius D. facial nucleus
19. Belongs to the vascular pigmented coat of the eyeball:
E. Sclera C. cornea
F. Choroid D. retina
20. Which of the following is not true of the retina:
E. Lies beneath the choroid
F. Forms the nervous coat of the eyeball
G. It has a blind spot called fovea centralis
H. Pierced by the central retinal artery
21. Which of the following is most likely to happen with a decrease in sensitivity of
chemoreceptors and baroreceptors in the elderly population?
A. less efficient pumping of the heart C. tendency to develop pulmonary edema
B. postural hypotension D. tendency for the heart to increase in size
22.Which of the following is an anatomical change associated with aging?
A. increase in liver mass and blood flow C. decrease in skin pigmentation
B. increase in size of the thoracic cage D. decrease in lean body mass
23. Which of the following types of memory improves with aging?
A. semantic memory C. episodic memory
B. procedural memory D. sensory memory
24.The wife who is not given enough money for the household and personal
needs is a form of what violence?
A. physical C. social
B. psychological D. economic

25. Controlling what a person does, who she sees and talks to, where she
goes is :
A. isolation C. threat
B. emotional abuse D. intimidation
26. Violent verbal behavior of elderly persons will most likely lead to:
E. Physical and mental impairment C. psychological deficiency
F. Transgenerational violence D. caregiver stress
27.Which of the following signs and symptoms strongly suggest elderly abuse?
E. Evidence of fresh injuries
F. Abrupt seeking of treatment
G. Signs of withdrawal or depression
H. Injuries consistent with the history
28.One of the components of aging is the presence of damaged proteins that stick around
and eventually cause problems. One of the following is true in relation to this:
A .proteins develop cross-links to each other that increase its mobility & conductance
E. our bodies have the capacity to repair DNA but not all are complete &
accurate
F. the cross-linking of damaged proteins in blood vessel walls softens the arteries
leading to leakage of plasma in old age
G. the presence of cross-linkages activate proteases
29. Some theories for preventing aging include one of the following:
A. giving of carnosine to prevent DNA damage
E. giving of telomerase to enhance telomere shortening
F. protein & calorie restriction to speed up to a rapid metabolism
G. giving of carnosine to prevent cross-linking of fibroblasts
30.  The  principal  component  of  the  “internal  clock  “  is  the:
A. suprachiasmatic nucleus C. retina
B. intergeniculate leaflet D. pineal gland
31. Which of the following neurotransmitters/neuroreceptor ligands can produce arousal?
A. melatonin B. glutamate C. muscimol D. serotonin
32.Which of the following is most useful in the prevention of jet lag among tourists? C
A. adjusting sleep schedule several days before departure
B. embark on a physical training program before departure
C. stay as close as possible to the day-night schedule of his/her residence
D. take regular meals at appropriate times, e.g., cooked meal at midnight
33. Majority of the tonic and phasic activities that occur during REM sleep are due to which
of the following?
A. high activity of ascending reticulo-activating system
B. low activity of ascending reticulo-activating system
C. high activity of serotoninergic nuclei of pontine raphe
D. activity of giganto-cellular tegmental field
34. A 45 year old obese male consulted a physician because of excessive daytime
sleepiness. In the sleep clinic, he fell asleep within 5 seconds after the lights were
turned off, but awaked 35 seconds later gasping for air. The cycle repeated itself for
the  next  ten  hours  of  “sleep”.  This  individual  is  most  probably suffering from:
A. narcolepsy C. sleep apnea
B. primary hypersomnia D. neutral state syndrome
35. A 25 year old male clad only in shoes and shorts is in a room with a temperature of
21o C and a humidity of 20%. As he stands quietly, heat elimination will most probably
be: B
A. 65% by insensible perspiration C. 40% by evaporation
B. 65 % by radiation D. 40% by convection
36. If the above-mentioned male engages in a strenuous tennis match, most of his body
heat would be lost by:
A. radiation B. conduction C. convection D. evaporation

37.Which of the following would require permanent descent to sea level?


A. acute mountain sickness C. high altitude pulmonary edema
B. chronic mountain sickness D. High altitude cerebral edema
38.Which of the following is expected to occur within 72 hours of ascent to high altitudes?
A. increase in arterial pH C. increase in 2,3 DPG
B. increase in hemoglobin D. decrease in arterial bicarbonate
39.Which of the following is a characteristic of aviation environment?
A. Most aircrafts are pressurized at sea level.
B. Sudden loss of cabin pressure can lead to hypoxia with slow decompression.
C. Sudden loss of cabin pressure causes increased oxygen saturation of blood.
D. Travel in pressurized aircrafts over long distances can cause dehydration.
40.Prevention of muscle atrophy in aerospace is made possible by using a:
A. Gamow bag C. penguin suit
B. decompression suit D. lower body negative pressure suit
41.Which of the following is expected to decrease upon immersion in water up to the
neck?
A. venous return C. stroke volume
B. ADH secretion D. mean pulmonary artery pressure
42.Nitrogen narcosis at greater depths can produce:
A. euphoria C. disorientation
B. clumsiness D. loss of memory
43.Which of the following is associated with decreased lipid volume of nerves?
A. decompression sickness C. oxygen toxicity
B. nitrogen narcosis D. high pressure nervous syndrome
44.Which of the following forms of sports makes use of anaerobic glycolysis?
A. weight lifting B. 100 meter dash C. 100 meter swim D. diving
45.The target heart rate range of a 20 year old individual with a resting heart rate of
90/min., who embarks on an exercise program should be:
A.156 – 176/min C. 176 – 198/min
B.150 – 170/min D. 110 – 200/min
46.Which of the following hormones only act in a permissive manner to support activity of
the other hormones increased during exercise?
A. glucagon B. aldosterone C. epinephrine D. cortisol
47.Which of the following measures heat production during exercise?
A. open circuit spirometer C. cycle ergometer
B. calorimeter D. treadmill
48.A 21 year old 50 kg ballet dancer would have an energy expenditure of ___ at each
ballet session.
A. 5.25 kcal/min B. 52.5 kcal/min C. 4.38 kcal/min D. 7 kcal/min
49. Which of the following can mask the presence of prohibited drugs in the urine of
athletes?
A. caffeine B. insulin C. dextran D. erythropoietin
50.Which of the following serves as part of warm-up exercise?
A. toe touch – 10 repetitions C. flutter kick – 30 repetitions
B. half knee bend – 10 repetitions D. push-up – 4 repetitions

XAVIER UNIVERSITY – DR JOSE P RIZAL COLLEGE OF MEDICINE


BASIC MEDICAL SCIENCES

Fourth Bimonthly Exam: Set B (Modules 33-36 & Special Topics) March 1, 2006
Write the letter of the BEST answer on the answer sheet. Use CAPITAL letters in ink.
NO ERASURES/SUPERIMPOSITIONS ALLOWED!

1. The prosencephalon gives rise to the following, EXCEPT:


A. cerebral hemisphere C. epithalamus
B. hypothalamus D. cerebellum
2. The circle of Willis is formed by these arteries, EXCEPT:
A. anterior, posterior cerebral arteries C. anterior, posterior communicating arteries
B. internal carotid arteries D. vertebral arteries
3. The frontal lobe is involved in these functions, EXCEPT:
A. making decisions C. listening an audiotape
B. speaking in class D. eye movement
4. Which layer of the cerebral cortex receives most of its afferent input?
A. internal granular C. external granular
B. internal pyramidal D. external pyramidal
5. Which of these fibers is a commissural fiber?
A. arcuate fasciculus C. cingulum
B. corpus callosum D. frontopontine
6. The major components of the basal ganglia are the following, EXCEPT:
A. amygdala C. putamen
B. caudate nucleus D. globus pallidus
7. Cerebrospinal fluid is formed by the choroid plexus in these areas, EXCEPT:
A. roof of lateral ventricle C. roof of fourth ventricle
B. roof of third ventricle D. floor of lateral ventricle
8. An obstruction of the CSF flow at the level of the foramen of Magendie and Luschka will
produce dilatation of the following, EXCEPT:
A. lateral ventricle C. cisterna magna
B. third ventricle D. fourth ventricle
9. Myelin formation in the brain is the role of this cell:
A. astrocyte C. microglia
B. oligodendroglia D.  Schwann’s  cell
10. The sensory language areas lie in these cortical areas, EXCEPT:
A. Area 45 C. Area 39
B. Area 40 D. Area 22
11. Which of these associations is NOT right?
A. Area 6 > premotor area C. Area 24 > gyrus cinguli
B. Area 9,10 11,12 > anterior frontal gyri D. Area 18,19 > temporal gyri
12.  Broca’s  aphasia  is  characterized by the following, EXCEPT:
A. poorly produced sounds C. incomplete word comprehension
B. slow speech D. telegraphic speech
13.  Gertmann’s  syndrome  is characterized by the following, EXCEPT:
A. dressing apraxia C. acalculia
B. finger agnotia D. agraphia
14. Which of these structures provide a pathway for olfactory and emotional states to influence
visceral responses?
A. habenula C. subthalamic nucleus
B. pineal body D. ventromedial nucleus
15. This thalamic nucleus is involved in integration of somatic, visceral, olfactory information and
relation to emotional feelings and subjective states:
A. anterior C. midline reticular
B. dorsomedial D. intralaminar
16. The Papez circuit is involved in the following roles, EXCEPT:
A. motivated behavior C. motor programming
B. memory D. reproduction

17. Which of these structure to role associations is NOT right?


A. anterior hypothalamic area > body temperature regulation
B. hippocampus > long term memory formation
C. amygdala > aggression and fear
D. centromedian nucleus > pleasure center
18. The best way to let an emotion sub side is to allow impulses to reach the: D
A. mammillary body C. gyrus cinguli
B. hippocampus D. frontal lobe
19. The following are absent in decerebrate rigidity, EXCEPT: B
A. conditioned reflexes C. initiative and memory
B. spinal reflexes D. righting reflexes
20. Appearance of involuntary movements, with difficulty in initiating a movement is likely a sign
of affection of the:
A. cerebellum C. cerebral cortex
B. corpus striatum D. any of the above
21. Which of the following parts of the diencephalon acts as both a relay area and a gateway for
information transfer to the cerebral cortex?
A. thalamus C. hypothalamus
B. epithalamus D. subthalamus
22. Which of these signs is NOT related to temporal crebral cortical dysfunction?
A. anosmia C. amnesia
B. alexia D. none of the above
23. The second-order neuron of the proprioceptive pathway from the body occurs at the level of:
A. spinal cord C. pons
B. medulla D. diencephalon
24. Which pathway from the body makes synapse at its point of entry in the CNS?
A. pain & temperature C. light touch & pressure
B. proprioception D. none of the above
25. The lateral cervical pathway mediates the sensation of:
A. touch C. pain
B. proprioception D. audition
26. The tract in the brain stem which mediates pain & temperature from the face:
A. lateral spinothalamic tract C. ventral trigeminal tract
B. tractus solitarius D. lateral lemniscus
27. The spinal lemniscal fibers ends at:
A. posterolateral ventral nucleus C. posterior horn cells
B. posterior column nuclei D. substancia gelatinosa
28. Which of the following modulators of pain acts by blocking delta receptors?
A. endorphin C. dynorphin
B. enkephalin D. serotonin
29.The following receptors respond to continuing stimuli by increasing in intensity as the
stimulus persists, EXCEPT:
A. olfactory receptors C. pressoreceptors
B. nociceptors D. muscle spindles
30.The following are characteristics of peripheral neuropathy, EXCEPT: C
A. pain, usually at night
B. onset is usually symmetrical
C. motor weakness involving flexors more than extensors
D. muscle atrophy and pansensory loss as it worsens
31. The following structures pass through the superior orbital fissure, EXCEPT:
A. oculomotor C. abducens
B. optic D. frontal
32. The nasolacrimal duct opens into which meatus?
A. superior C. inferior
B. middle D. none of the above
33. The tympanic membrane serves as the ____ wall of the middle ear.
A. anterior C. lateral
B. posterior D. roof

34. Which of the following supports the organ of Corti?


A. tectorial membrane C. secondary tympanic membrane
B. basilar membrane D.    Reissner’s  membrane
35. The following are true of the ciliary body, EXCEPT:
A. made up of smooth muscles
B. its pigmented epithelium secrete aqueous humor
C. its blood vessels are embedded in connective tissue
D. has 2 layers of cuboidal epithelium
36. Which of these associations, is NOT right?
A. cornea > anterior outer eye wall without blood vessels
B. choroid > contain smooth muscle, melanocytes, blood vessels
C. Meibonian glands > secretion prevents rapid evaporation of tear film
D. conjunctiva > stratified columnar epithelium with goblet cells
37. In which layer of the retina has the cell bodies of the photoreceptors?
A. photoreceptor layer C. inner nuclear layer
B. outer nuclear layer D. outer plexiform layer
38. The following are histologic features of the tympanic membrane, EXCEPT:
A. has an external stratified squamous epithelial lining
B. internally lined by simple cuboidal epithelium
C. contains connective tissue
D. collagen fibers are absent in the pars flaccida
39. The following associations are correct, EXCEPT:
A. pinna > elastic cartilage support
B. organ of Corti > auditory receptors & supporting cells
C. Maculae > lie at scala media
D. stria vascularis > secrete endolymph
Questions 40-41
Lionel a 15 y.o. student fell from his bike while attempting a somersault during an extreme
game competition in their town. He hit his head on the pavement and lost consciousness. When he
woke up he complained of left eye blindness.
40. Which of these tests may be used to check his visual acuity?
A. finger counting C. light perception
B.    Snellen’s  chart D. all of the above
41. During the finger movement test, Lionel is unable to see halfway beyond the midline in his left eye,
laterally. Which of these visual pathway components is least likely affected?
A. optic chiasma C lateral geniculate nucleus
B. optic tract D. cuneus
42. If the injury Lionel sustained resulted in damage of the right posterior cerebral artery, which
of these visual field defects is possible?
A. Left homonymous hemianopsia C. right homonymous hemianopsia
C. Left lower quadrantanopsia D. nasal hemianopsia
43. If pupillary light reflex test revealed (+) consensual response but (-) direct response to the
affected eye, which part of the pathway is involved?
A. CN II C. pretectal area
B. CN III D. Edinger-Westphal nucleus
44. A post-stroke patient complains of blindness on the upper part of his right eye, what part of the
visual pathway is affected?
A. Left optic tract C. cuneus, left
B. Optic radiation, left D. lingual gyrus, left
45. If the entire right visual cortex is affected the visual defect would be:
A. right homonymous hemianopsia
B. left homonymous hemianopsia
C. bitemporal hemianopsia
D. upper quadrantanopsia left
46. The following statements are true as regards color vision, EXCEPT: D
A. single cones do not encode the wave length of a light stimulus
B. light of a single wave length stimulates all three types of cones
C. the fovea has limited color discrimination because of the absence of blue cones
D. the peripheral retina has greatest color discrimination because all 3 cones are
present

47. The following take place in accommodation for near vision, EXCEPT: A
A. the lens becomes more concave C. the pupils constrict
B. the two eyes converge D. the medial rectus muscle contracts
48. A school teacher is prescribed lenses of +3 diopters. This means that: B
A. she has myopia and the lenses correct for this
B. the lenses have a focal distance of 33.3
C. her lens has a principal focus of 33.3
D. her cornea possesses several meridians of curvature
49.  A  subject  stands  20  feet  away  from  the  Snellen’s  chart.  At  this  distance,  he is able
to read only up to line 4 of the chart. His visual acuity is: B
A. 20/70 C. 20/40
B. 20/50 D. 20/30
50. The following motor cranial nerve nuclei are involved in olfactovisceral reflex, EXCEPT:
A. VII C. X
B. IX D. XII
51. Which structure is NOT part of the taste pathway?
A. Area 43 C. tractus solitarius
B. spiral ganglion D. vagus nerve
52. The receptors of the vestibular pathway are found in what stucture of the utricle?
A. crista ampullaris C. macula
B. organ of Corti D. cupula
53. The auditory pathway decussates at what level of the CNS?
A. medulla C. midbrain
B. pons D. diencephalon
54. Hair cells of the organ of Corti are hyperpolarized when: D
A. stereocilia are bent away from the limbus
B. stereocilia are bent toward the spiral ligament
C. organ of corti is deflected upward
D. basilar membrane is deflected toward the scala tympani
55. When the sound stimulus from same source is more intense in right than in left ear of a
subject, source of sound is perceived to be: A
A. at the right of the subject C. above the subject
B. at the left of the subject D. in front of the subject
56. Hair cells in the vestibular apparatus are stimulated when: A
A. stereocilia are bent toward the kinocilium
B. stereocilia are bent away from the kinocilium
C. kinocilium is bent toward the stereocilia
C. stereocilia and kinocilium are bent in the same direction
57. Riding in a train stimulates primarily the hair cells in the: C
A. horizontal semicircular ducts C. utricle
B. vertical semicircular ducts D. saccule
58. Mr. Pedro Cruz often insults his wife in front of his friends. This act is a form of: B
A. physical abuse B. psychological abuse
B. social abuse D. economic abuse
59. To exercise power and control  over  one’s  partner,  a  husband/wife  may  use  which  of  these
methods: D
A. Making him/her feel bad about himself/herself
B. Prevents him/her from getting a job
C. Threatens to commit suicide if the partner does do what he/she wants
D. All of the above
60. The following actions/beliefs promote patriarchy, EXCEPT: B
A. men are naturally superior to women
B. women are allowed to work outside the home even if the family is financially stable
C. family violence is a family matter and thus no one should interfere
D. some women really deserve the abuse they get from thier husbands
61. In the cycle of violence, which of these immediately follows the violent outburst phase? A
A. remorse phase C. pursuit phase
B. honeymoon phase D. standover phase

62. Which of these statements regarding elderly abuse is NOT true? D


A. it is abuse inflicted on people aged 60 and above
B. Failure to provide for the basic needs of an elderly constitute  “neglect”  of  the  elderly
C. An elderly is capable of abusing himself/herself
D. Abandoning an elderly who has a bad temper does not constitute elderly abuse
63. Which of the following may trigger/cause elderly abuse by their relatives/caregivers? D
A. caring for the elderly is stressful to the caregiver
B. caregiver was abused by the elderly when the former was yet a child
C. caregiver is an alcoholic
D. all of the above
64. Financial abuse is committed against the elderly in these conditions, EXCEPT: C
A.    stealing  his  money                                  C.    using  elderly’s  money  for  salary  of  caregiver
B.    changing  his  will                                          D.    selling  elderly’s  clothes  without  his  consent  
65. Which of the following theories of aging serves as the basis of the popularity of anti-
oxidants as possible substances that can delay aging? C
A. cross-linking theory C. free radical theory
B. rate of living theory D. wear and tear theory
66. Postural hypotension is common in the elderly because of: A
A. decreased sensitivity of baroreceptors
B. less efficient pumping of the heart
C. deposition of fat and protein in the heart
D. increased lumen of cardiac chambers
67. The following are changes in the respiratory system as part of the aging process,
EXCEPT: B
A. decrease in the size of the thoracic cage
B. decrease in resistance to gas flow
C. increase in bronchial secretions
D. increase in elastin content of alveoli
68. The following are changes in the digestive system as part of the aging process,
EXCEPT: D
A. dentures become loose C. constipation due to inadequate bulk
B. diarrhea due to milk intolerance D. increase in acid secretion
69. Alteration of the nigrostriatal-dopaminergic system in the elderly produces the
following effects, EXCEPT: C
A. standing with flexed hips and knees C. peripheral neuropathy
B. shuffling gait D. postural sway
70. The taste sensation that is affected earliest by aging is: B
A. sweet C. sour
B. salty D. bitter
71. The following are effects of aging on the eye, EXCEPT: D
A. presbyopia C. cataract
B. poor convergence D. mydriasis
72. Which of the following types of memory improves with age? C
A. procedural memory C. semantic memory
B. episodic memory D. eidetic memory
73. The following foods are believed to contain anti-oxidants, EXCEPT: A
A. broccoli C. spinach
B. carrots D. winter squash
74. According to the Harvard Study on Adult Development, the following factors can
delay aging, EXCEPT: B
A. wearing of sunscreens C. reduction of anxiety
B. sedentary lifestyle D. develop coping skills
75. A group of friends decided to go mountain climbing. The following are expected,
EXCEPT: A
A. the higher they climb, the higher the barometric pressure
B. the higher they climb, the lesser the partial pressure of oxygen
C. at 4,000 feet, they may experience increased heart rate
D. at 8,000 feet, they will have increased respiratory rate

76. A Gamow bag would be most useful in cases of: C


A. acute mountain sickness C. high altitude pulmonary edema
B. high altitude cerebral edema D. chronic mountain sickness
77. Which of the following occurs only after 3 weeks of living at high altitudes? B
A. increase in erythropoietin C. increase in minute ventilation
B. increase in 2,3 DPG D. decrease in oxygen partial pressure
78. The following are principles in aviation environment, EXCEPT: C
A. stresses include barotrauma and motion sickness
B. air in pressurized aircraft cabins is extremely dry
C. aircrafts are pressurized to sea level
D. popping of the ears may occur during ascent or descent
79. Hypoxia with slow decompression is characterized by the following, EXCEPT: D
A. euphoria C. poor judgement
B. incoordination D. nausea and vomiting
80. The following may occur in prolonged missions in aerospace, EXCEPT: B
A. decalcification of bone C. negative potassium balance
B. decrease in central venous pressure D. breakdown of muscle tissue
81.  When  a  person’s  body  is  immersed  in  water  up  to  the  neck,  the  following are
expected to occur, EXCEPT: C
A. increase in pulmonary diffusing capacity
B. increase in central blood volume
C. increase in functional residual capacity
D. decrease in ADH secretion
82. The following are major physiologic stresses in underwater physiology, EXCEPT: A
A. the lower the dive, the lesser the ambient pressure
B. the lower the dive, the lesser the effects of gravity
C. alteration in respirations
D. hypothermia
83. When pressure decreases in body cavities and contained gas expands, a person is
said to experience: D
A. decompression sickness C. nitrogen narcosis
B. high pressure nervous syndrome D. squeeze
84. The ability to work for long periods of time and resist fatigue is called: C
A. fitness C. endurance
B. conditioning D. adaptation
85. A 23 year old medical student with a resting heart rate of 87/min, will have a target
heart rate range of about: B
A. 60 – 80 beats/min C. 247 – 267 beats/min
B. 147 – 167 beats/min D. 160 – 180 beats/min
86. The following hormones act by enhancing the effects of other hormones, EXCEPT:
A. cortisol C. growth hormone
B. thyroxine D. aldosterone *
87. Work associated with an oxygen consumption of 0.8 L/min is considered as: B
A. mild work C. hard work
B. moderate work D. maximal work
88. Which of the following tends to measure heat production? C
A. cycle ergometer C. direct calorimetry
B. treadmill D. indirect calorimetry
89. A 19 year old, 50 kg student enrolls in a ballet class. If the METS of ballet is 6, the
student is expected to have an energy expenditure of ___ each time she goes to
ballet class. A
A. 5.25 kcal/min C. 1.99 kcal/min
B. 9.97 kcal/min D. 4.37 kcal/min
90. In order to benefit from a carbohydrate-rich drink, an athlete should take it: C
A. immediately before exercise C. 30 – 60 min before exercise
B. immediately after exercise D. 30 – 60 min after exercise

91. The following are beneficial effects of warm-up before exercise, EXCEPT: B
A. increases oxyhemoglobin dissociation
B. increases muscle viscosity
C. increases speed of nerve impulses
D. decreases tendon-related injuries
92. The following are conditioning exercises, EXCEPT: A
A. knee lift C. push-up
B. sprinter D. toe touch
93. Which of the following phases of learning requires a mental set? B
A. motivation phase C. acquisition phase
B. apprehension phase D. retention phase
94. Which of the following phases of learning involves transfer of learning? C
A. retention phase C. generalization phase
B. recall phase D. performance phase
95. Which of the following phases of learning produces an independent learner? B
A. apprehension phase C. retention phase
B. recall phase D. performance phase
96. Which of the following learning outcomes proposed by Gagne produces a self-
learner? D
A. verbal information C. attitudes
B. intellectual skills D. cognitive strategies
97. Which of the following is facilitated by review? B
A. sensory memory C. long-term memory
B. short-term memory D. reflexive memory
98. Brief, detailed retention of visual stimuli is known as: C
A. sensory memory C. iconic memory
B. eidetic memory D. hypermnesia
99. The only substance recommended to improve memory is:
A. caffeine C. physostigmine
B. nicotine D. ADH
100. Loss of memory following a significant life event is called: D
A. patchy amnesia C. retrograde amnesia
B. confabulation D. anterograde amnesia

4th BM – SY 2006-2007

Module 34 – Sensory Pathways

1. The mesencephalic nucleus of the V in the brain stem receives fibers mediating what
sensation? FINALS
A. touch & pressure C. proprioception
B. pain & temperature D. taste
2. In a patient with loss of pain & temperature sensation on the right side of the body
L1 dermatome and below, the lesion is most likely in the: B
A. left spinothalamic tract, level of lower medulla
B. left spinothalamic tract, level of T11 spinal segment
C. right spinothalamic tract, level of L1 spinal segment
D. right spinothalamic tract, level of upper medulla
3. First order neurons of sensory pathways from the body are in: FINALS
A. sensory ganglia of spinal nerves C. receptor
B. autonomic ganglia D. dorsal root ganglia
4. The following characterize the medial lemniscus: FINALS
A. arises from the posterior column nuclei
B. travels through the spinal cord and brain stem
C. ends in the posterolateral ventral nucleus of the thalamus
D. conveys sensations carried by the fasciculus gracilis only
5. Sensory pathway that decussate at the level of the medulla: FINALS
A. pain & temperature from face C. proprioception from face
B. touch from body D. pain & temperature from body
6. A lesion on this structure will show ipsilateral neurologic deficit: D
A. ventral trigeminal lemniscus C. medial lemniscus
B. lateral spinothalamic tract D. fasciculus cuneatus
7. One of these structures is NOT part of the pain & temperature pathway from the face: D
A. spinal tract of V C. posteromedial ventral nucleus
B. ventral trigeminal lemniscus D. lateral spinothalamic tract
8. The lateral cervical system forms part of the pathway for: FINALS
A. proprioception C. touch
B. pain D. audition
9. The second order neurons of the proprioceptive pathway from the body occurs at the
level of:
A. diencephalon C. medulla
B. pons D. spinal cord
10. One of these structures is NOT part of the pathway for touch & pressure from the body:
A.  Brodmann’s  area  3,1,2 C. posterolateral ventral nucleus D
B. ventral spinothalamic tract D. naked nerve endings
11. Which sensory pathway/s from the body makes synapse at its point of entry in the CNS?
A. light touch & pressure C. proprioception FINALS
B. pain & temperature D. all of these

Module 35
Auditory & Vestibular Pathways

1. Bilateral deafness may result in a lesion of: FINALS


A. spiral ganglion C. cochlear nerve
B. trapezoid body D. cochlear nuclei
2. The thalamic nucleus that is part of the auditory pathway: C
A. posteromedial ventral nucleus C. medial geniculate body
B. inferior colliculus nucleus D. posterolateral ventral nucleus
3. One of these structures is NOT part of the auditory pathway: D
A. trapezoid body C. spiral ganglion
C. lateral lemniscus D. Area 43
4. A sensory pathway that is associated with the temporal lobe: FINALS
A. vision C. audition
B. taste D. touch
5. One of these cranial nerve nuclei does NOT receive vestibular fibers: C:
A. oculomotor C. trigeminal
B. trochlear D. abducens
6. One of these structures is NOT associated with the vestibular pathway: FINALS
A. semicircular ducts C. utricle
B. spiral organ D. saccule
7. Second order neurons of the vestibular pathway are found in: FINALS
A. vestibular ganglion C. motor cranial nerve nuclei
B. vestibular nuclei D. flocculo-nodular lobe, cerebellum
8. The following tract/s carry vestibular fibers:
A. reticulospinal tract C. vestibulospinal tract
B. medial longitudinal fasciculus D. all of these

Olfactory & Taste Pathways

1. The medial olfactory stria ends at: FINALS


A. anterior perforated substance C. septal area
B. uncus D. hippocampus
2. The tract that ends in the motor cranial nerve nuclei responsible for olfacto-somatic
correlation: C
A. habenulotegmental tract C. habenulotectal tract
B. habenulopeduncular tract D. stria medullaris thalami
3. Fibers from taste buds in the epiglottis is associated with which ganglion? FINALS
A. geniculate C. nodose
B. Gasserian D. petrosal
4. The nucleus in the brain stem that mediates taste sensation:
A. main sensory nucleus of V C. nucleus ambiguus
B. nucleus solitarius D. facial nucleus

A 1. Regarding the blood supply to the spinal cord, which is false?


A. there are two anterior spinal arteries
B. the posterior spinal arteries originate from the vertebral artery
C. there are reinforcing arteries called radicular arteries
D. its veins drain into the internal vertebral venous plexuses
C 2. What is the ideal site for doing the lumbar tap? FINALS
A.between LI and L2 C.between L3 and L4
B. between L2 and L3 D.between L4 and L5
B 3. Bundle of anterior and posterior roots of lumbar, sacral and coccygeal
spinal nerves and filum terminale is called :
A. conus medullaris C.filum terminale
B. cauda equine D. ligamentum denticulatum
D 4. The spinal cord in the adult ends inferiorly at level: FINALS
A. L3 vertebra C. T12 vertebra
B. L5 vertebra D. LI vertebra
C 5. Where do you find the cerebrospinal fluid in the spinal cord? FINALS
A. epidural space C. subarachnoid space
B. subdural space D.all of the above
A 6. The following are TRUE regarding the meninges of the spinal cord
EXCEPT:
A. dura mater ends inferiorly at lumbar one level
B. arachnoid mater is delicate impermeable membrane
C. Pia mater is attached to dura mater by ligamentum denticulatum
D. Pia mater is vascular membrane
A 7. Edinger Westphal nucleus is found in the:
A. midbrain C. upper medulla
B. pons D. medulla, caudal part
C 8. Which of the cranial nerve nuclei is found in the medulla? FINALS
A, CN III C. CN XII
B. CN IV D. CN VI
B 9. CN VIII takes its exit from the skull through what opening? FINALS
A. jugular foramen C. foramen magnum
B.internal auditory meatus D. hypoglossal
D 10. The following nuclei give origin to general somatic efferent fibers
EXCEPT: FINALS
A. facial C. trochlear
B. oculomotor D. hypoglossal
C 11. The cerebellum is associated with the following EXCEPT:
A. regulation of muscle tone
B. coordination of somatic motor activity
C. programming of rapid skilled voluntary movements
D. integration of proprioception with reflex activity
A 12. Which part of the cerebellum receives projection from vestibular nuclei?
A.flocculonodular lobe C.posterior lobe
B. anterior lobe D. vermis
B 13. Which part of the cerebellum receives projections from the cerebral
hemispheres? FINALS
A. anterior lobe C. flocculonodular lobe
B. posterior lobe D. vermis
A 14. A fissure that divides frontal and parietal from temporal lobe is: FINALS
A.Sylvian fissure C. sulcus of Rolando
B. longitudinal fissure D. parieto-occipital sulcus
D 15. Which lobe is immediately posterior to central sulcus of Rolando?
A.occipital C.temporal
B. frontal D.parietal
C 16. Which of the following is a primary somatosensory area? FINALS
A. precentral gyrus C. postcentral gyrus
B. inferior frontal gyrus D. superior temporal gyrus
D 17. The following are found in the temporal lobe of the cerebrum EXCEPT:
A. primary auditory area C. primary olfactory
B. Wernickes area D. primary visual area
A 18.The tympanic membrane serves as which boundary of the middle ear?
A.lateral C.medial FINALS
B. anterior D. posterior
D 19. The arterial supplies to the ear are branches of the following :
A.external carotid C.maxillary
B. superficial temporal D. all of the above
B 20. Feeling of nausea associated with infection of the external auditory canal
attributed to which cranial nerve?
A.CN VIII C. CN V
B. CN X D. CN VII
A 21. The possibility of pharyngitis to spread to the middle ear can be explained
anatomically due to the presence of: FINALS
A. Eustachian tube C.external auditory meautus
B. internal auditory meatus D. none of the above
B 22. The victim of violence feels a building of tension and at this point she
becomes compliant , anticipating his every whim. This phase of violence is
called:: FINALS
A. violent phase C. remorse phase
B. build up phase D. honeymoon phase
D 23. The wife who is not given enough money for the household and personal
needs is a form of what violence?
A. physical C. social
B. psychological D. economic
A 24. Controlling what a person does, who she sees and talks to, where she
goes is :
A. isolation C. threat
B. emotional abuse D. intimidation

FOURTH BIMONTHLY AND FINAL EXAMINATIONS

MODULE 30 – SPINAL CORD


1. Which of the following does not develop from the rostral end of the neural
tube?FINALS
A. cerebral hemispheres C. diencephalon
B. cerebellum D. spinal cord
2. The mesencephalon develops into which of the following
A. cerebral hemispheres C. midbrain
B. cerebellum D. pons
3. A 36 year old skydiver sustained a back injury during a skydiving exhibition. On PE,
tenderness was elicited at the middle thoracic area. MRI revealed a fracture of the 5 th
thoracic vertebra. Which spinal cord segment is most likely injured?
A. T3 C. T4
B. T7 D. T6
4. Which of the following is not true of the central nervous system?FINALS
A. its components are covered by meninges
B. of its cellular content, neurons number more than the neuroglia
C. made up of the brain and spinal cord
D. its parts are suspended in cerebrospinal fluid
5. Most of the neurons of the brain are of this type
A. Unipolar C. multipolar
B. Bipolar D. pseudounipolar
6. This neuroglial cell is inactive in a normal central nervous system FINALS
A. fibrous astrocyte C. oligodendroglia
B. microglia D. ependymocytes
7. Which of the following is true of myelination in the PNS?
A. each Schwann cell serves many nerve numbers
B. formation of the mesaxon
C. absence of Schmidt –Lanterman incisures
D. nodes of Ranvier may not be present
8. A mechanoreceptor in hairless skin mediated by A fibers and with slow adaptability
FINALS
A. Meissner’s  corpuscle C. Merkel’s  discs
B. Ruffini’s  corpuscle D. Free nerve endings

MODULE 31 – BRAIN STEM


1. The following are general functions of the brain stem, EXCEPT FINALS
A. regulation of heart rate and respiration
B. control of sleep and wakefulness
C. channel for ascending and descending tracts
D. contains all the cranial nerve nuclei
2. A 25 year old pump attendant of a gasoline station was brought to the hospital
because of difficulty in swallowing and speaking, vertigo and nystagmus. Neurologic
examination revealed: ipsilateral analgesia on the face. CT scan revealed a
thrombosis of the posterior inferior cerebellar artery. This artery supplies the lateral
part of the medulla oblongata. Which of the following group of structures is likely
affected based on the presenting symptoms? FINALS
A. nucleus ambiguus, nucleus and spinal tract of CN V, vestibular nuclei
B. cerebellum, spinothalamic tract, nucleus ambiguus
C. pons, cerebellum, midbrain
D. nucleus ambiguus, vestibular nuclei, spinal tract of CN V
3. A retired teacher sought consultation for an eye problem. On examination, the doctor
noted persistent drooping of the right eyelid with an unmoving eyeball, a dilated pupil
nonresponsive to light, vision was normal. The opposite eye was also normal. Patient
sustained a blow to his head, 2 days PTC, when he fell from his motorcycle. He
claimed that he did not seek consultation because there were no visible injuries. His
problem started a few hours PTC. Based on the presenting signs which part of the
brainstem is most likely involved?
A. Pons C. Midbrain
B. Medulla D. Entire brainstem
4. Which cranial nerves are most likely injured in the problem above (#3)?
A. II & III B. III, IV C. I, III D. II, IV

MODULE 32
1. Which of the following structures is not found in the molecular layer of the cerebellum?
A. stellate cells C. Golgi type I cells
B. basket cells D. dendrites of Purkinje cells
2. Which of the following is not true of the granular layer? FINALS
A. contains small, multipolar cells
B. dendrites of granule cells synapse with climbing fibers
C. axons of granule cells proceed to the molecular layer
D. neuroglial cells are found throughout this layer
3. The vermal cortex of the cerebellum influences movements of the following, EXCEPT
A. Feet C. Neck
B. Shoulder D. Hips
4. The efferent fibers of this cerebellar nucleus form a large part of the superior
cerebellar peduncle
A. Emboliform C. Fastigial
B. Globose D. Dentate
5. This fiber type forms the greater part of the cerebellar white matter FINALS
A. Intrinsic C. Afferent
B. Efferent D. Extrinsic

MODULE 33
1. Which of the following is true of the motor speech area of Broca?
A. found in the inferior frontal gyrus
B. corresponds to areas 44 & 45
C. injury to this area in the non-dominant sphere produces aphasia
D. it  is  concerned  with  regulation  of  the  depth  of  a  person’s  feelings
2. The function of this cortical area is to relate the visual information received by the
primary visual area to past visual experiences
A. 6, 8 9 B. 17 C. 22 D. 18, 19
3. The origin of the projection fibers to the auditory area arise from which structure
FINALS
A. medial geniculate body C. mammillary bodies
B. lateral geniculate body D. superior colliculi
4. A cortical area which is important for planning or coordinating the articulatory
movements needed for speech FINALS
A. Uncus C. Straight gyrus
B. Insula D. Postcentral gyrus
5. A form of apraxia wherein a person knows what he or she wants to do but is unable to
do it
A. Gait C. Ideomotor
B. Kinetic D. Ideational
6. Which of the following manifestations is not referable to a basal ganglia dysfunction?
cortex? FINALS
A. Hemiballismus C. Chorea
B. Athetosis D. Agnosia
7. The structural pathology in a patient with conduction aphasia is FINALS
A. disconnection  of  Wernicke’s  area  from  Broca’s  area
B. vascular lesions in the internal capsule
C. tumor in the postcentral gyrus
D. all of the above
8. A patient presents with the following neurologic signs and symptoms: faster than
normal spoken language, normal grammatic construction but is unable to find the
correct words to express his thoughts so he substitutes one word for another. There is
no associated hemiplegia. The lesion in this dysfunction is most likely in the
A. Broca’s  area C.    Wernicke’s  area
B. Paralimbic area D. Prefrontal area

MODULE 35
1. Which of the following is least likely a risk factor for a patient with transient monocular
blindness is
A. Diabetes C. Smoking
B. Hypertension D. Alcohol intake
2. Which of the following structures is not common to the visual, pupillary and
accommodation pathways?
A. Retina C. Optic tract
B. Pretectal nucleus D. Optic nerve
3. Which of the following is not a test for vision? FINALS
A. Finger counting C. Flashlight test
B. Confrontation method D.    Snellen’s  chart
4. An injury to the right lower part of the geniculocalcarine tract will result to FINALS
A. Left homonymous hemianopsia
B. Left homonymous superior quadrantanopsia
C. Left homonymous inferior quadrantanopsia
D. Left heteronymous hemianopsia
5. Which of the following diagrams represent the visual field defect in an injury to the
right visual cortex? (left homonymous hemianopsia with macular sparing)
6. The efferent path of the pupillary light reflex begins with which of the following
structures
A. Edinger-Westphal nucleus C. Ciliary ganglion FINALS
B. Pretectal area D. Oculomotor nerve
7. This structure in the visual somatic reflex pathway is the center for movements of the
head, eyes and body in response to visual stimuli
A. inferior collliculus C. lateral geniculate body
B. superior colliculus D. medial geniculate body
8. An optical reflex test which signifies the sensitivity of the eye to touch FINALS
A. Ishihara C. Corneal reflex test
B. Reading cards D. Flashlight test
9. The response of lid closure in the menace reflex is brought about by
A. Rapidly approaching object C. Foreign object in the eye
B. Sudden bright light D. Splashing of water into the eye
10. Right nasal hemianopsia can result from an injury to which of the following FINALS
A. optic chiasma C. right optic nerve
B. right optic tract D. right angle of the chiasma
11. A CT scan revealed a golf ball size tumor in the sella turcica. The patient complained
of bilateral blindness. What is the possible visual field defect?
A. Heteronymous hemianopsia C. Upper quadrantanopsia
B. Bitemporal hemianopsia D. Complete blindness

Xavier University – Dr Jose P Rizal College of Medicine


Basic Biomedical Sciences – Fourth Bimonthly Exam
Set A: Modules 30-33

NAME ________________________ 5 March 2007 SY 2006 – 07

MULTIPLE CHOICE: Write the letter of the BEST answer in CAPITAL letter in ink on
the answer sheet. NO ERASURES/SUPERIMPOSITIONS ALLOWED!

1. Which of the following does not develop from the rostral end of the neural tube?
A. cerebral hemispheres C. diencephalon
B. cerebellum D. spinal cord
2. A 36 year old skydiver sustained a back injury during a skydiving exhibition. On PE,
tenderness was elicited at the middle thoracic area. MRI revealed a fracture of the 5 th
thoracic vertebra. Which spinal cord segment is most likely injured?
A. T 3 B. T 7 C. T 4 D. T 6
3. Most of the neurons of the brain are of this type
A. Unipolar C. multipolar
B. Bipolar D. pseudounipolar
4. Which of the following is true of myelination in the PNS?
A. each Schwann cell serves many nerve numbers
B. formation of the mesaxon
C. absence of Schmidt –Lanterman incisures
D. nodes of Ranvier may not be present
5. Which of the following components of the reflex arc possesses transducer function?
A. afferent fiber C. receptor
B. efferent fiber D. effector
6. Decerebrate rigidity is due to overactivity of the:
A. gamma efferents to muscle spindles C. corticospinal tract
B. alpha motor neurons to skeletal muscles D. vestibulo spinal tract
7. Upper motor neuron lesions are characterized by the following, EXCEPT:
A. prominent muscle atrophy C. spasticity
B. increased tendon reflexes D. positive Babinski sign
8. The corticospinal tracts are pathways primarily concerned with:
A. hypothalamic control of sympathetic outflow
B. facilitation of the activity of extensor muscles and inhibition of the activity of
flexor muscles in the maintenance of balance
C. voluntary, discrete, skilled movements
D. control of sympathetic and parasympathetic systems
9. One of the following is TRUE in lower motor neuron lesions:
A. spastic paralysis of muscle C. chorea
B. hyperreflexia D. atrophy
10. Regarding the blood supply to the spinal cord, which is false?
A. there are two anterior spinal arteries
B. the posterior spinal arteries originate from the vertebral artery
C. there are reinforcing arteries called radicular arteries
D. its veins drain into the internal vertebral venous plexuses
11. A bundle of anterior and posterior roots of lumbar, sacral and coccygeal spinal nerves
plus the filum terminale is called :
A. conus medullaris C.filum terminale
B. cauda equine D. ligamentum denticulatum
12. The following are TRUE regarding the meninges of the spinal cord, EXCEPT:
A. dura mater ends inferiorly at lumbar one level
B. arachnoid mater is delicate impermeable membrane
C. Pia mater is attached to dura mater by ligamentum denticulatum
D. Pia mater is vascular membrane

13. This nerve cell group in the posterior gray column of the spinal cord receives afferent
fibers concerned with pain, temperature and touch from the posterior root:
A. Nucleus proprius C. nucleus dorsalis
B. Substancia gelatinosa D. visceral afferent nucleus
14. Which of the following is not a main somatosensory pathways to consciousness?
A. Spinoreticular tract C. fasciculus gracilis and cuneatus
B. Lateral spinothalamic tract D. anterior spinothalamic tract
15. A 25 year old pump attendant of a gasoline station was brought to the hospital
because of difficulty in swallowing and speaking, vertigo and nystagmus. Neurologic
examination revealed: ipsilateral analgesia on the face. CT scan revealed a
thrombosis of the posterior inferior cerebellar artery. This artery supplies the lateral
part of the medulla oblongata. Which of the following group of structures is likely
affected based on the presenting symptoms?
A. nucleus ambiguus, nucleus and spinal tract of CN V, vestibular nuclei
B. cerebellum, spinothalamic tract, nucleus ambiguus
C. pons, cerebellum, midbrain
D. nucleus ambiguus, vestibular nuclei, spinal tract of CN V
16. A retired teacher sought consultation for an eye problem. On examination, the doctor
noted persistent drooping of the right eyelid with an unmoving eyeball, a dilated pupil
nonresponsive to light, vision was normal. The opposite eye was also normal. Patient
sustained a blow to his head, 2 days PTC, when he fell from his motorcycle. He
claimed that he did not seek consultation because there were no visible injuries. His
problem started a few hours PTC. Based on the presenting signs which part of the
brainstem is most likely involved?
A. Pons C. Midbrain
B. Medulla D. Entire brainstem
17. Which cranial nerves are most likely injured in the problem above (#3)?
A. II & III B. III, IV C. I, III D. II, IV
18. Which of these nuclei is found at both levels of the midbrain, the inferior colliculi and
superior colliculi?
A. trochlear nucleus C. Edinger-Westphal nucleus
B. substancia nigra D. oculomotor nucleus
19. Which of the following associations between structure and role is NOT correct?
A. substancia nigra – muscle tone
B. superior colliculus – audition
C. medial lemniscus – discriminative sense pathway
D. spinal nucleus of V – pain and temperature sensation
20. The Edinger Westphal nucleus is found in the:
A. midbrain C. upper medulla
B. pons D. medulla, caudal part
21.Which of the following nerves does not emerge in the groove between the pons and
the medulla oblongata?
A. Abducent C. vestibular
B. Facial D. vagus
22. Part of the midrain which are centers for visual reflexes and lower auditory centers:
A. Crus cerebri C. corpora quadrigemina
B. Lateral geniculate body D. medial geniculate body
23.A 45yo male was brought in for paralysis of the L lower face. On examination, the L
pupil is dilated and the L eye is fixed. Which of the following statements is NOT
true?
A. if  this  was  a  case  of  Weber’s  syndrome,  there  should  also  be  ophtalmoplegia  on
the Right
B. if  this  was  a  case  of  Benedikt’s  syndrome,  there  should  also  be  involuntary  
movements of the Right
C. if this was a midbrain lesion, there should also be paralysis of the contralateral
arm/leg
D. this cannot be a case of Pontine hemorrhage

24.A vehicular accident caused one patient to be poikilothermic upon admission to the ER.
Which of the following statements is NOT true?
A. the accident disconnects the heat-regulating centers in the basal ganglia
B. this manifestation could also be found in severe pontine hemorrhage
C. if this was a case of severe pontine hemorrhage, there is also involvement of the
ocular sympathetic nerves
D. pupils could manifest as pinpoint
25. Which of the following structures is not found in the molecular layer of the cerebellum?
A. stellate cells C. Golgi type I cells
B. basket cells D. dendrites of Purkinje cells
26. The vermal cortex of the cerebellum influences movements of the following, EXCEPT
A. Feet B. shoulder C. Neck D. hips
27. The efferent fibers of this cerebellar nucleus form a large part of the superior
cerebellar peduncle
A. Emboliform C. Fastigial
B. Globose D. Dentate
28. The following signs are suggestive of cerebellar disease, EXCEPT:
A. intention tremor C. decreased muscle tone
B. wide base gait D. ipsilateral muscle fasciculations
29. Cerebellar efferent fibers influence motor activity on the same side of the body through
this tract:
A. corticopontocerebellar C. fastigial vestibular
B. globose-emboliform-rubral D. cerebro-olivo-cerebellar
30. The fibers of this pathway end up by synapsing with cells in the ventrolateral nucleus
of the thalamus.
A. dentothalamic C. fastigial reticular
B. globothalamic D. cerebroreticulocerebellar
31.The cerebellum is associated with the following EXCEPT:
A. regulation of muscle tone
B. coordination of somatic motor activity
C. programming of rapid skilled voluntary movements
D. integration of proprioception with reflex activity
32.Which part of the cerebellum receives projection from vestibular nuclei?
A.flocculonodular lobe C.posterior lobe
B. anterior lobe D. vermis
33.The following are descriptive of the cerebellar peduncles except:
A. The superior connects the cerebellum to the midbrain
B. The middle consists almost entirely of afferent fibers
C. The inferior consists chiefly of afferent fibers
D. Found at the sides of the third ventricle
34. Which of the following pathways /fiber tracts enters the cerebellum through the
superior cerebellar peduncle?
A. Anterior spinocerebellar tract
B. Cuneocerebellar tract
C. Posterior spinocerebellar tract
D. Cerebellar afferent fibers from vestibular nerve
35. Which of the following is true of the motor speech area of Broca?
C. found in the inferior frontal gyrus
D. corresponds to areas 44 & 45
E. injury to this area in the non-dominant sphere produces aphasia
F. it is concerned with  regulation  of  the  depth  of  a  person’s  feelings
36. The function of this cortical area is to relate the visual information received by the
primary visual area to past visual experiences
A..6, 8, 9 B. 17 C. 22 D. 18, 19
37. A form of apraxia wherein a person knows what he or she wants to do but is unable to
do it
A Gait B. kinetic C. Ideomotor D. ideational

38. A patient presents with the following neurologic signs and symptoms: faster than
normal spoken language, normal grammatic construction but is unable to find the
correct words to express his thoughts so he substitutes one word for another. There is
no associated hemiplegia. The lesion in this dysfunction is most likely in the
A.    Broca’s  area C.    Wernicke’s  area
B. Paralimbic area D. Prefrontal area
39. Cell bodies of the descending motor pathways lie in this layer of the cerebral cortex:
A. internal granular C. internal pyramidal
B. external granular D. external pyramidal
40. Which of these structures is mainly responsible for the blood-brain barrier?
A. foot processes of the astrocytes
B. endothelial cells in the walls of capillaries
C. basement membrane surrounding the capillary
D. tight junctions between the endothelial cells
41. Which of the following is the correct CSF outflow from the lateral ventricle to the
central canal?
A. foramen of Monroe > 3rd ventricle > foramen of Magendie > 4th ventricle >
central canal
B. foramen of Monroe > 3rd ventricle > foramen of Luschka > 4th ventricle > central
canal
C. foramen of Luschka > 3rd ventricle > foramen of Monroe > 4th ventricle > central
canal
D. foramen of Monroe > 3rd ventricle > cerebral aqueduct > 4th ventricle >
central canal
42. Which of these thalamic nuclei influences activity of the motor cortex?
A. anterior nuclei and dorsomedial nuclei
B. intralaminar nuclei and reticular nuclei
C. ventral anterior nuclei and ventral lateral nuclei
D. ventral posteromedial nuclei and ventral posterolateral nuclei
43. The body of the fornix is connected to the undersurface of the corpus callosum by this
structure:
A. septum pellucidum C. commisure of the fornix
B. mammillary bodies D. stria terminalis
44. The following are afferent connections of the hippocampus EXCEPT:
A. fibers from the cingulated gyrus C. fibers from the indusium griseum
B. fibers from the septal nuclei D. fibers from anterior nuclei of thalamus
45. A lesion in the hippocampus results in:
A. selective amnesia C. retrograde amnesia
B. anterograde amnesia D. all of the above
46. Which lobe is immediately posterior to central sulcus of Rolando?
A.occipital B. frontal C.temporal D. parietal
47. The following are found in the temporal lobe of the cerebrum EXCEPT:
A. primary auditory area C. primary olfactory
B. Wernickes area D. primary visual area
48, Which of the following efferent cerebellar pathways influences ipsilateral extensor
muscle tone?
A. Fastigial reticular C. dentothalamic
B. Globose-emboliform rubral D. fastigial vestibular
49. Which of the following afferent cerebellar pathways takes origin from the sensorimotor
areas?
A. Posterior spinocerebellar C. corticopontocerebellar
B. Cerebroreticulocerebellar D. anterior spinocerebellar
50. Which of the following venous sinuses is not a content of falx cerebri?
A. Superior sagittal C. inferior sagittal
B. Occipital D. straight

XAVIER UNIVERSITY – DR JOSE P RIZAL COLLEGE OF MEDICINE


BASIC BIOMEDICAL SCIENCES

Final Exam: BLOCK 1 (Modules 1 – 10) Oct. 3, 2005

XAVIER UNIVERSITY – DR JOSE P RIZAL COLLEGE OF MEDICINE


BASIC BIOMEDICAL SCIENCES

Final Exam: BLOCK 2 (Modules 11 – 20) Oct. 3, 2005

1. The temporary hemostatic plug is primarily due to: c


A. red cells B. white cells C. platelets D. fibrin
2. Bladdy was suspected of having a hemorrhagic disorder. Laboratory tests were performed
and these are the results: APTT = abnormal; Prothrombin time = normal; thrombin time =
abnormal. Based on these limited data, Bladdy is most probably deficient in this factor: c
A. factor XII B. factor VII C. factor VIII D. factor I
3.

14-20
14. When the SA node is diseased, which of the following is most likely to take its place as
the cardiac pacemaker? b
A. atrial muscles C. bundle of His
B. AV node D. terminal Purkinje network
15. When the right atrium is enlarged, which of these ECG configurations will be prolonged?c
A. PR interval and QRS interval C. P wave duration and PR interval
B. P wave duration and ST segment D. QRS interval and QT interval
16. If the P vector is located at (-) 10 degrees, the P wave is tallest in this ECG lead: a
A. lead I B. lead II C. lead aVL D. lead aVR
17. Peak aortic (systolic) pressure occurs at the end of this phase of the cardiac cycle:d
A. isovolumetric contraction C. protodiastole
B. isovolumetric relaxation D. rapid ejection
18. The following events occur during cardiac diastole, EXCEPT: d
A. ventricular filling C. atrial contraction
B. ventricular relaxation D. opening of semilunar valves
19. If the chordae tendinae in the left ventricle are cut, the following would occur, EXCEPT: c
A. blood regurgitates into the left atrium during systole
B. mitral valve would be everted during systole
C. diastolic murmur would be heard
D. stroke volume would be decreased
20. The heart may stop in systole in which of these conditions? c
A. hypercalcemia C. hyperkalemia
B. hypocalcemia D. hypokalemia
21. Stimulation of the vasopressin system reflexly results in the following, EXCEPT: d
A. stimulation of the thirst center C. increased renal reabsorption of water
B. increased release of ADH D. dilute urine
22. When right ventricular contractions are weak, which of the following would increase? a
A. central venous pressure (CVP) and peripheral venous pressure (PVP)
B. right ventricular stroke volume (SV) and venous return (VR)
C. CVP, PVP, VR and SV
D. CVP, PVP, and VR
23. Capillary filtration tends to increase in these conditions, EXCEPT: b
A. hypoproteinemia C. venous obstruction
B. arteriolar constriction D. increased PVP
72. Jane has passed 8 watery stools for the past 6 hours. She feels thirsty and tired. Based
on this short history we can say that the following are the likely changes occurring in her
body fluids: c
A. decreased ECF volume and ICF volume
B. decreased ECF volume ECF osmolarity
C. decreased ECF volume, no change in ECF osmolarity, no change in ICF volume
D. decreased ECF volume and osmolarity and no change in ICF volume and osmolarity

73. Mia was admitted to the hospital because of severe asthmatic attack. On admission, the
results of laboratory exams also showed an acid base disturbance. After three days,
laboratory exams were again done and these were the results: pH = 7.352; [HCO3-] =
25.9 mmols/liter; PCO2 = 49.98 mmHg; [BE] = 1.89 mmols/liter. Based on these results,
we can say that Mia has a: a
A. normal acid-base status C. fully compensated metabolic alkalosis
B. fully compensated respiratory acidosis D. fully compensated respiratory alkalosis
74. Body responses to metabolic alkalosis secondary to ingestion of a bottle of bicarbonate
tablets include the following, EXCEPT: a
A. buffering by the bicarbonate buffer system (BBS) C. alkalinization of the urine
B. buffering by the non-BBS D. decreased alveolar ventilation
75. Given the following laboratory results of Gino: pH = 7.438; [HCO3-] = 20.2 mmols/liter;
PCO2 =30.8 mmHG; [BE] = (-) 2 mmols/liter. Based on these results,he is likely to have:
A. fully compensated metabolic alkalosis C. uncompensated metabolic alkalosis
B. fully compensated respiratory alkalosis D. normal acid-base status

XAVIER UNIVERSITY – DR JOSE P RIZAL COLLEGE OF MEDICINE


BASIC BIOMEDICAL SCIENCES

Final Exam: BLOCK 3 (Modules 21 – 29) Oct. 3, 2005

XAVIER UNIVERSITY – DR JOSE P RIZAL COLLEGE OF MEDICINE


BASIC BIOMEDICAL SCIENCES

Final Exam: BLOCK 4 (Modules 30 – 36 & Special Topics) Oct. 3, 2005

80. The organ responsible for matching sound impedance in the middle and inner ears:
A. tympanic membrane C. ear ossicles
B. oval window D. inner ear fluid
81. Which of the following is NOT true of nystagmus?
A. initiated by visual impulses
B. slow component is opposite in direction to head rotation
C. quick component of same direction as head rotation
D. direction of nystagmus is determined by direction of quick component
82. The vestibulo-ocular reflex in response to head turning to the left in the upright posture involves
these structures:
A. horizontal semicircular ducts C. utricles
B. vertical semicircular ducts D. saccules
83. Juan refuses to talk to his wife until she follows his wishes. This action constitute:
A. physical abuse C. social abuse
B. sexual abuse D. psychological abuse
84. Domestic violence is also known as:
A. wife battering C. woman abuse
B. marital violence D. all of the above
85. A married couple was having an argument. She felt his anger rising so she stopped arguing
because she felt that at any time now he would begin to hit her with his fist. He has done this
times in the past. In the cycle of violence this is called the:
A. build-up phase C. violent phase
B. pursuit phase D. remorse phase
86. Juan often shouts at his elderly mother every time she refuses to take her medicines. He tells her
to obey or else he would leave her. This behavior of Juan constitutes:
A. physical abuse C. psychological abuse
B. financial abuse D. sexual abuse

Xavier University – Dr Jose P Rizal College of Medicine


Basic Biomedical Sciences – Fourth Bimonthly Exam
Set A: Modules 30-33

NAME ________________________ 5 March 2007 SY 2006 – 07


MULTIPLE CHOICE: Write the letter of the BEST answer in CAPITAL letter in ink on
the answer sheet. NO ERASURES/SUPERIMPOSITIONS ALLOWED!

Module 30 – sp. Cord (14)


10. Which of the following does not develop from the rostral end of the neural tube?
A. cerebral hemispheres C. diencephalon
B. cerebellum D. spinal cord
11. A 36 year old skydiver sustained a back injury during a skydiving exhibition. On PE,
tenderness was elicited at the middle thoracic area. MRI revealed a fracture of the 5 th
thoracic vertebra. Which spinal cord segment is most likely injured?
A. T 3 B. T 7 C. T 4 D. T 6
12. Most of the neurons of the brain are of this type
A. Unipolar C. multipolar
B. Bipolar D. pseudounipolar
13. Which of the following is true of myelination in the PNS?
A. each Schwann cell serves many nerve numbers
B. formation of the mesaxon
C. absence of Schmidt –Lanterman incisures
D. nodes of Ranvier may not be present
14. Which of the following components of the reflex arc possesses transducer function?
A. afferent fiber C. receptor
B. efferent fiber D. effector
15. Decerebrate rigidity is due to overactivity of the:
A. gamma efferents to muscle spindles C. corticospinal tract
B. alpha motor neurons to skeletal muscles D. vestibulo spinal tract
16. Upper motor neuron lesions are characterized by the following, EXCEPT:
A. prominent muscle atrophy C. spasticity
B. increased tendon reflexes D. positive Babinski sign
17. The corticospinal tracts are pathways primarily concerned with:
A. hypothalamic control of sympathetic outflow
B. facilitation of the activity of extensor muscles and inhibition of the activity of
flexor muscles in the maintenance of balance
C. voluntary, discrete, skilled movements
D. control of sympathetic and parasympathetic systems
18. One of the following is TRUE in lower motor neuron lesions:
A. spastic paralysis of muscle C. chorea
B. hyperreflexia D. atrophy
10. Regarding the blood supply to the spinal cord, which is false?
A. there are two anterior spinal arteries
B. the posterior spinal arteries originate from the vertebral artery
C. there are reinforcing arteries called radicular arteries
D. its veins drain into the internal vertebral venous plexuses
11. Bundle of anterior and posterior roots of lumbar, sacral and coccygeal
spinal nerves and filum terminale is called :
A. conus medullaris C.filum terminale
B. cauda equine D. ligamentum denticulatum
12. The following are TRUE regarding the meninges of the spinal cord
EXCEPT:
E. dura mater ends inferiorly at lumbar one level
F. arachnoid mater is delicate impermeable membrane
G. Pia mater is attached to dura mater by ligamentum denticulatum
H. Pia mater is vascular membrane

13. This nerve cell groups in the posterior gray column of the spinal cord receives afferent
fibers concerned with pain, temperature and touch from the posterior root:
A. Nucleus proprius C. nucleus dorsalis
B. Substancia gelatinosa D. visceral afferent nucleus
14. Which of the following is not a main somatosensory pathways to consciousness?
A. Spinoreticular tract C. fasciculus gracilis and cuneatus
B. Lateral spinothalamic tract D. anterior spinothalamic tract
Module 31 – Brain stem (10)
20. A 25 year old pump attendant of a gasoline station was brought to the hospital
because of difficulty in swallowing and speaking, vertigo and nystagmus. Neurologic
examination revealed: ipsilateral analgesia on the face. CT scan revealed a
thrombosis of the posterior inferior cerebellar artery. This artery supplies the lateral
part of the medulla oblongata. Which of the following group of structures is likely
affected based on the presenting symptoms?
A. nucleus ambiguus, nucleus and spinal tract of CN V, vestibular nuclei
B. cerebellum, spinothalamic tract, nucleus ambiguus
C. pons, cerebellum, midbrain
D. nucleus ambiguus, vestibular nuclei, spinal tract of CN V
21. A retired teacher sought consultation for an eye problem. On examination, the doctor
noted persistent drooping of the right eyelid with an unmoving eyeball, a dilated pupil
nonresponsive to light, vision was normal. The opposite eye was also normal. Patient
sustained a blow to his head, 2 days PTC, when he fell from his motorcycle. He
claimed that he did not seek consultation because there were no visible injuries. His
problem started a few hours PTC. Based on the presenting signs which part of the
brainstem is most likely involved?
A. Pons C. Midbrain
B. Medulla D. Entire brainstem
22. Which cranial nerves are most likely injured in the problem above (#3)?
A. II & III B. III, IV C. I, III D. II, IV
23. Which of these nuclei is found at these two levels of the midbrain, the inferior colliculi
and superior colliculi?
A. trochlear nucleus C. Edinger-Westphal nucleus
B. substancia nigra D. oculomotor nucleus
24. Which of the following associations between structure and role is NOT correct?
A. substancia nigra – muscle tone
B. superior colliculus – audition [visual reflexes]
C. medial lemniscus – discriminative sense pathway
D. spinal nucleus of V – pain and temperature sensation
20. Edinger Westphal nucleus is found in the:
A. midbrain C. upper medulla
B. pons D. medulla, caudal part
21.Which of the following nerves does not emerge in the groove between the pons and
the medulla oblongata?
A. Abducent C. vestibular
B. Facial D. vagus
22. Part of the midrain which are centers for visual reflexes and lower auditory centers:
A. Crus cerebri C. corpora quadrigemina
B. Lateral geniculate body D. medial geniculate body
23.A 45yo male was brought in for paralysis of the L lower face. On examination, there are
dilated pupils fixed to light & accommodation. Which of the following statements is NOT
true?
E. if  this  was  a  case  of  Weber’s  syndrome,  there  should  also  be  ophtalmoplegia on
the Right
F. if  this  was  a  case  of  Benedikt’s  syndrome,  there  should  also  be  involuntary  
movements of the Right
G. if this was a midbrain lesion, there should also be paralysis of the contralateral
arm/leg
H. this cannot be a case of Pontine hemorrhage
24.A vehicular accident caused one patient to be poikilothermic upon admission to the ER.
Which of the following statements is NOT true?
A. the accident disconnects the heat-regulating centers in the basal ganglia
B. this manifestation could also be found in severe pontine hemorrhage
C. if this was a case of severe pontine hemorrhage, there is also involvement of the
ocular sympathetic nerves
D. pupils could manifest as pinpoint
ANS: A (heat-regulating centers in the hypothalamus are disconnected)
Module 32 – cerebellum (10)
31. Which of the following structures is not found in the molecular layer of the cerebellum?
A. stellate cells C. Golgi type I cells
B. basket cells D. dendrites of Purkinje cells
32. The vermal cortex of the cerebellum influences movements of the following, EXCEPT
A. Feet C. Neck
B. Shoulder D. Hips
33. The efferent fibers of this cerebellar nucleus form a large part of the superior
cerebellar peduncle
A. Emboliform C. Fastigial
B. Globose D. Dentate
34. The following signs are suggestive of cerebellar disease, EXCEPT:
A. intention tremor C. decreased muscle tone
B. wide base gait D. ipsilateral muscle fasciculations
35. Cerebellar efferent fibers influence motor activity on the same side of the body through
this tract:
A. corticopontocerebellar C. fastigial vestibular
B. globose-emboliform-rubral D. cerebro-olivo-cerebellar
36. The fibers of this pathway end up by synapsing with cells in the ventrolateral nucleus
of the thalamus.
A. dentothalamic C. fastigial reticular
B. globothalamic D. cerebroreticulocerebellar
31.The cerebellum is associated with the following EXCEPT:
A. regulation of muscle tone
B. coordination of somatic motor activity
C. programming of rapid skilled voluntary movements
D. integration of proprioception with reflex activity
32.Which part of the cerebellum receives projection from vestibular nuclei?
A.flocculonodular lobe C.posterior lobe
B. anterior lobe D. vermis
33.The following are descriptive of the cerebellar peduncles except:
A. The superior connects the cerebellum to the midbrain
B. The middle consists almost entirely of afferent fibers
C. The inferior consists chiefly of afferent fibers
D. Found at the sides of the third ventricle
34. Which of the following pathways /fiber tracts enters the cerebellum through the
superior cerebellar peduncle?
E. Anterior spinocerebellar tract
F. Cuneocerebellar tract
G. Posterior spinocerebellar tract
H. Cerebellar afferent fibers from vestibular nerve
Module 33 – cerebrum (16)
35. Which of the following is true of the motor speech area of Broca?
G. found in the inferior frontal gyrus
H. corresponds to areas 44 & 45
I. injury to this area in the non-dominant sphere produces aphasia
J. it  is  concerned  with  regulation  of  the  depth  of  a  person’s  feelings
45. The function of this cortical area is to relate the visual information received by the
primary visual area to past visual experiences
A..6, 8, 9 B. 17 C. 22 D. 18, 19
46. A form of apraxia wherein a person knows what he or she wants to do but is unable to
do it
A Gait B. kinetic C. Ideomotor D. ideational
47. A patient presents with the following neurologic signs and symptoms: faster than
normal spoken language, normal grammatic construction but is unable to find the
correct words to express his thoughts so he substitutes one word for another. There is
no associated hemiplegia. The lesion in this dysfunction is most likely in the
A.    Broca’s  area C. Wernicke’s  area
B. Paralimbic area D. Prefrontal area
48. Cell bodies of the descending motor pathways lie in this layer of the cerebral cortex:
A. internal granular C. internal pyramidal
B. external granular D. external pyramidal
49. Which of these structures is mainly responsible for the blood-brain barrier?
A. foot processes of the astrocytes
B. endothelial cells in the walls of capillaries
C. basement membrane surrounding the capillary
D. tight junctions between the endothelial cells
50. Which of the following is the correct CSF outflow from the lateral ventricle to the
central canal?
A. foramen of Monroe > 3rd ventricle > foramen of Magendie > 4th ventricle >
central canal
B. foramen of Monroe > 3rd ventricle > foramen of Luschka > 4th ventricle > central
canal
C. foramen of Luschka > 3rd ventricle > foramen of Monroe > 4th ventricle > central
canal
D. foramen of Monroe > 3rd ventricle > cerebral aqueduct > 4th ventricle >
central canal
51. Which of these thalamic nuclei influences activity of the motor cortex?
A. anterior nuclei and dorsomedial nuclei
B. intralaminar nuclei and reticular nuclei
C. ventral anterior nuclei and ventral lateral nuclei
D. ventral posteromedial nuclei and ventral posterolateral nuclei
52. The body of the fornix is connected to the undersurface of the corpus callosum by this
structure:
A. septum pellucidum C. commisure of the fornix
B. mammillary bodies D. stria terminalis
53. The following are afferent connections of the hippocampus EXCEPT:
A. fibers from the cingulated gyrus C. fibers from the indusium griseum
B. fibers from the septal nuclei D. fibers from anterior nuclei of thalamus
45. A lesion in the hippocampus results in:
A. selective amnesia C. retrograde amnesia
B. anterograde amnesia D. all of the above
46. Which lobe is immediately posterior to central sulcus of Rolando?
A.occipital B. frontal C.temporal D. parietal
47. The following are found in the temporal lobe of the cerebrum EXCEPT:
A. primary auditory area C. primary olfactory
B. Wernickes area D. primary visual area
48, Which of the following efferent cerebellar pathways influences ipsilateral extensor
muscle tone?
C. Fastigial reticular C. dentothalamic
D. Globose-emboliform rubral D. fastigial vestibular
49. Which of the following afferent cerebellar pathways takes origin from the sensorimotor
areas?
C. Posterior spinocerebellar C. corticopontocerebellar
D. Cerebroreticulocerebellar D. anterior spinocerebellar
50. Which of the following venous sinuses is not a content of falx cerebri?
C. Superior sagittal C. inferior sagittal
D. Occipital D. straight

_____1. This nerve cell groups in the posterior gray column of the spinal cord
receives afferent fibers concerned with pain, temperature and touch
from the posterior root:
A. Nucleus proprius
B. Substancia gelatinosa (B)
C. Nucleus dorsalis
D. Visceral afferent nucleus
_____2. Which of the following is not a main somatosensory pathways to
consciousness? FINALS
A. Spinoreticular tract
B. Lateral spinothalamic tract (A)
C. Fasciculus gracilis and cuneatus
D. Anterior spinothalamic tract
_____3. The elongated swelling found on each side of the posterior median
sulcus of the inferior half of the medulla is formed by the: FINALS
A. Olive
B. Gracile tubercle (B)
C. Pyramid
D. Inferior cerebral peduncle
_____4. Which of the following is not true of the pons as described? FINALS
A. Connects the medulla oblongata to the midbrain
B. The trigeminal nerve emerges on each side of the anterolateral surface
(D)
C. Connects the right and left cerebellar hemispheres
D. Its posterior surface is marked by a groove for the basilar artery
_____5. Which of the following nerves does not emerge in the groove between the
pons and the medulla oblongata?
A. Abducent
B. Facial
C. Vestibulocochlear (D)
D. Vagus
_____6. Part of the midrain which are centers for visual reflexes and lower
auditory centers:
A. Crus cerebri
B. Lateral geniculate body (C)
C. Corpora quadrigemina
D. Medial geniculate body
_____7. The following are descriptive of the cerebellar peduncles except:
A. The superior connects the cerebellum to the midbrain
B. The middle consists almost entirely of afferent fibers
C. The inferior consists chiefly of afferent fibers (D)
D. Found at the sides of the third ventricle
______8. The largest of the cerebellar nuclei: FINALS
A. Globose
B. Fastigial
C. Dentate (C)
D. Emboliform
______9. Which of the following cerebellar fibers constitute the output of the
cerebellum?
A. Efferent
B. Intrinsic (A)
C. Afferent
D. Extrinsic
______10. Which of the following afferent cerebellar pathways does not convey
information from muscles and joints? FINALS
A. Cerebroreticulocerebellar
B. Anterior spinocerebellar (C)
C. Cuneocerebellar
D. Posterior spinocerebellar

_____11. Which of the following pathways /fiber tracts enters the cerebellum through the
superior cerebellar peduncle? FINALS
I. Anterior spinocerebellar tract
J. Cuneocerebellar tract (A)
K. Posterior spinocerebellar tract
L. Cerebellar afferent fibers from vestibular nerve
_____12. Which of the following efferent cerebellar pathways influences ipsilateral
extensor muscle tone? FINALS
E. Fastigial reticular
F. Globose-emboliform rubral (D)
G. Dentothalamic
H. Fastigial vestibular
_____13. Which of the following afferent cerebellar pathways takes origin from the
sensorimotor areas?
E. Posterior spinocerebellar
F. Cerebroreticulocerebellar (B)
G. Corticopontocerebellar
H. Anterior spinocerebellar
_____14. Which of the following venous sinuses is not a content of falx cerebri?
E. Superior sagittal FINALS
F. Occipital
G. Inferior sagittal (B)
H. Straight
_____15. This sinus passes through the jugular foramen to join the superior bulb of
the internal jugular vein:
A. Cavernous
B. Transverse
C. Straight (D)
D. Sigmoid
_____16. The Circle of Willis is formed by the following arteries except: FINALS
A. Anterior cerebral
B. Middle cerebral
C. Posterior cerebral (B)
D. Basilar
_____17. The following are common causes of chronic pain in the elderly except:
A. Peripheral neuropathies FINALS
B. Degenerative joint disease
C. Appendicitis (C)
D. Osteoporosis
_____18. The following are consequences of chronic pain in the elderly except:
A. Sleep disturbance FINALS
B. Compromise overall quality of life
C. Depression (D)
D. Increased socialization
_____ 19. Failure of a caretaker to provide for the basic needs of an elderly person is
considered as: FINALS
A. Financial abuse
B. Neglect
C. Emotional abuse
D. Physical abuse (B)
_____ 20. Violent verbal behavior of elderly persons will most likely lead to:
G. Physical and mental impairment
H. Transgenerational violence
I. Psychological deficiency
J. Caregiver stress (D)
_____ 21. Which of the following signs and symptoms strongly suggest elderly abuse?
I. Evidence of fresh injuries
J. Abrupt seeking of treatment
K. Signs of withdrawal or depression
L. Injuries consistent with the history (C)

_____22. Nonpharmacologic management of chronic pain in the elderly include the


following except: FINALS
A. Therapeutic exercise program
B. Cognitive-behavioral therapy
C. Nonopioid analgesics (C)
D. Spiritual healing
_____23. Rotates the eyeball downwards and laterally: FINALS
A. Superior oblique
B. Inferior rectus
C. Superior rectus ( A)
D. Inferior oblique
_____24. Belongs to the vascular pigmented coat of the eyeball:
G. Sclera
H. Choroid
I. Cornea (B)
J. Retina
_____25. Which of the following is not true of the retina:
I. Lies beneath the choroid
J. Forms the nervous coat of the eyeball
K. It has a blind spot called fovea centralis ( C )
L. Pierced by the central retinal artery
QUIZ – CEREBRUM

1. The  taste  area  is  located  in  Brodmann’s  area:


A. 41 and 42 C. 17
B. 43 D. 18 and 19
2. Electrically stimulating this cortical area causes isolated movements on the opposite
side of the body, bilateral movements of the extraocular muscles, muscles of the upper
part of the face, the tongue, the mandible, the larynx and the pharynx.
A. Brodmann’s  area  4 C.    Brodmann’s  area  7
B. Brodmann’s  area  6 D.    Brodmann’s  area  8
3. Which  of  the  following  describes  Brodmann’s  area  3,  1,  and  2?
A. located in the precentral gyrus
B. the primary somesthetic area
C. area 3 contains several pyramidal cells
D. receive projection fibers primarily from the lateral geniculate body
4. Mario has a dominant left  cerebral  hemisphere.    A  lesion  in  his  Brodmann’s  area  44  
and 45 (left hemisphere) will result in:
A. paralysis of speech
B. loss of fine motor movements
C. inability to recognize and appreciate what is seen
D. loss of appreciation of the movements of the head in space
5. A  unilateral  lesion  of  which  of  these  Brodmann’s  areas  causes  partial  deafness  in  both  
ears, but with greater loss in the contralateral ear?
A. Areas 18 and 19 C. area 43
B. Area 22 D. areas 41 and 42
6. Which  of  the  following  is  true  of  Brodmann’s  area 17?
A. located in the parietal lobe
B. concerned with vision
C. receives afferent fibers from the medial geniculate body
D. stimulation causes conjugate deviation of the eyes to the opposite side
7. Ms dela Cruz brought his ten year old son to a doctor for consultation. She said that
her son has been having episodes of generalized convulsions for the past year.
Before the seizure starts, the son would complain of smelling rotten durian fruit. Based
on this limited data, which part of the brain is most likely involved in the epileptic
discharge?
A. frontal lobe C. temporal lobe
B. parietal lobe D. occipital lobe
8. Which of the following manifestations is likely to be present in a cerebral cortex
dysfunction rather than in a basal ganglia dysfunction?
A. alternating contraction of agonists and antagonists
B. equal rigidity of opposing muscle groups
C. inability to understand spoken words
D. postural disturbances
9. Which of the following manifestations will most likely lead you to exclude a diagnosis
of transient ischemic attack (TIA)?
A. confusion
B. trouble speaking clearly
C. severe headache that occurs suddenly
D. paralysis of one side of the body including extremities
10. Which of the following conditions will most likely increase the risk of developing TIA?
A. underweight
B. active lifestyle
C. uncontrolled hypertension
D. prolonged  use  of  Billing’s  method  of  contraception

BBS FINAL EXAMINATION


MODULE 13 – CVS 1
1. This layer of the heart wall is homologous to the intima of blood vessels
A. Pericardium C. Epicardium
B. Myocardium D. Endocardium
2. The smallest myocardial cell which is abundant in the SA node
A. Transitional C. Purkinje
B. Pacemaker D. Ordinary
3. The cardiac structure where the cardiac muscles have their origin and insertion
A. Valves C. cardiac skeleton
B. Epicardium D. chorda tendinae
4. Which of the following is not true of the histologic picture of the heart?
A. the epicardium is also the visceral pericardium
B. Purkinje fibers are found in the subepicardial layer
C. Parenchymal elements are found in the myocardium
D. Ordinary cells make up the bulk of the heart
5. The cardiac valves are made up of the following, EXCEPT
A. lined on one side by endothelium and mesothelium on the other
B. contains a central core of both collagen and elastic fibers
C. bases are attached to the annuli fibrosi of the cardiac skeleton
D. folds of endocardium or tunica intima

MODULE 14 – CVS 2
1. A series of efferent vessels which become smaller as they branch and whose role is
carry blood to the tissues
A. Veins C. Lymphatics
B. Capillaries D. Arteries
2. A type of capillary without fenestrae in its wall, found in all kinds of muscle tissue
A. Visceral C. Fenestrated without diapghram
B. Somatic D. Discontinuous
3. Which of the following is not a histologic feature of the tunica media
A. consists of layers of smooth muscle cells
B. contains elastic and reticular fibers
C. separated from the adventitia by an elastic laminae
D. where the vasa vasorum and nervorum are mainly found
4. Which of the following is least likely true of the veins
A. subendothelial layer may or may not be present
B. well-developed collagenous adventitia
C. all veins have well-developed tunica intima
D. valves are found in large veins
5. The following are histologic characteristics of the lymphatic capillaries, which is the
least descriptive?
A. small amount of intimal valves
B. contains a single layer of endothelial cells
C. absent fenestrations in endothelial cells
D. absent zonula occludens

MODULE 15 – RESPIRATORY 1
1. Which of the following associations of the layers of the laryngeal wall is not true?
A. Epithelium – stratified squamous lining the true cords
B. Lamina propria – rich in elastic fibers, with mixed glands
C. Tunica submucosa – with glands and fibers in the deeper layer
D. Tunica adventitia – with cartilages and smooth muscles
2. The  part  of  the  mucosa  of  the  secondary  bronchus  where  the  Reissenssen’s  muscles  
are found
A. Epithelium C. Basement membrane
B. Lamina propria D. Muscularis mucosae
3. Which of the following layers of the wall of the ordinary bronchiole are fused?
A. mucosa and submucosa C. submucosa and adventitia
B. epithelium and lamina propria D. basement membrane & submucosa
4. Which of the following structures is not part of the barrier between air in the alveoli and
capillary blood?
A. Interstitium of interalveolar septum
B. surface epithelium of the alveolar cells
C. cytoplasm of the alveolar and endothelial cells
D. fused basal laminae of alveolar and endothelial cells
5. An alveolar cell type which secrete pulmonary surfactant
A. Type I B. Type II C. Clara D. Squamous

MODULE 17 – URINARY
1. Which cell is not found in the glomerular capillaries?
A. Podocytes C. Endothelial
B. Mesangial D. Smooth muscle
2. The type of collecting tubules which deliver urine to the minor calyx
A. Cortical C. Papillary
B. Medullary D. Intercalated
3. The appearance of the mucosa when the bladder is filled with urine
A. retains its folded appearance
B. disappearance of the folds
C. stretched but retains its folded appearance
D. transitional cells become cuboidal in shape
4. Which of the following does not describe the penile urethra?
A. Serous secreting glands of Littre in the mucosa
B. loose fibroelastic tissue in the lamina propria
C. rich vascular supply
D. generally lined by psedustratified and columnar epithelium

MODULE 19 – GIT
1. The general histologic picture of the GIT is all of the following, EXCEPT
A. Generally lined by simple columnar epithelium
B. Glands, lymphatic tissue and nerves may be found in the submucosa
C. The serosa makes up the outermost layer in all GI organs
D. Layered smooth muscles are found in the tunica muscularis
2. The most numerous papillae found in the entire surface of the tongue
A. Filiform C. Circumvallate
B. Fungiform D. Foliate
3. This structure fixes the tooth to the alveolar socket
A. dentinal tubules C. cementum
B. periodontal ligament D. Gingiva
4. Which part of the esophageal wall are the cardiac glands found?
A. Mucosa C. Lamina propria
B. Submucosa D. Serosa
5. The following structures/substances provide protection from the effects of acid in the
stomach, which does not belong?
A. Mucus C. Pit cells
B. Tight junctions D. Paneth cells
6. Which of the following is not true of the microscopic appearance of the large intestine?
A. absence of mucosal folds except in the rectal portion
B. lymphoid cells and nodules in the lamian propria
C. abundant goblet cells
D. thick mucosal villi
Xavier University – Dr Jose P Rizal College of Medicine
Basic Biomedical Sciences

FINAL EXAM: Block 2 (Modules 11 – 20) 21 March 2007

INSTRUCTIONS: Write the letter of the BEST answer on the answer sheet. Use
CAPITAL letters in ink. NO ERASURES/SUPERIMPOSITIONS ALLOWED!
Write your name on this questionnaire and on the answer sheet.

NAME ____________________________

5. Which of the following genes is mainly responsible for production of the basic
precursor in the Rh system?
A. gene H B. gene Z C. gene C D. gene Se
6. Which of the following antibodies is most likely to be detected only in the anti-globulin
phase of the cross-match?
A. anti-Lea B. anti-P C. anti-E D. anti-Fya
7. Which of the following donor blood types will most likely give rise to a compatible
major cross-match but an incompatible minor cross-match if the recipient is type A?
A. type A B. type B C. type O D. type AB
8. While performing the tube method of blood typing, a student reported agglutination,
when in reality it was just rouleaux formation. Which of the following procedures is the
easiest way to confirm rouleaux formation?
A. dispersion upon dilution of the serum with normal saline
B. confirmation by microscopic examination
C. dispersion upon gentle agitation
D. performing the procedure at 37o Celsius
9. The following are involved in primary hemostasis, EXCEPT:
a. fibrinogen C. thrombin
b. platelets D. von Willibrand factor
10. The conversion of prothrombin to thrombin involves the following, EXCEPT:
a. phospholipid (PL), calcium, Xa, Va, VIIa C. PL, Calcium, Xa
b. PL, calcium, Xa, Va D. PL, Xa, Va
11. In the presence of platelet deficiency, which of these laboratory tests are expected to
be abnormal?
a. bleeding time (BT), activated partial thromboplastin time(APTT)
b. BT, tourniquet test (TT), APTT
c. BT, TT, clot retraction, APTT
d. BT, TT, clot retraction
12. Mr Ree accidentally cut his jaw while shaving. The bleeding continued for several
minutes. He became alarmed because he has a family history of hemophilia A (factor
VIII deficiency). He went to the hospital and these lab tests were done. Which of
these are expected to be prolonged?
a. APTT, PT, TT, BT C. APTT, PT
b. APTT, PT, TT D. APTT
13. This layer of the heart wall is homologous to the intima of blood vessels
a. Pericardium C. Epicardium
b. Myocardium D. Endocardium
14. The smallest myocardial cell which is abundant in the SA node
a. Transitional C. Purkinje
b. Pacemaker D. Ordinary
15. The cardiac structure where the cardiac muscles have their origin and insertion
a. Valves C. cardiac skeleton
b. Epicardium D. chorda tendinae
16. Which of the following is not true of the histologic picture of the heart?
a. the epicardium is also the visceral pericardium
b. Purkinje fibers are found in the subepicardial layer
c. functional elements are found in the myocardium
d. Ordinary cells make up the bulk of the heart
17. The cardiac valves are made up of the following, EXCEPT
a. lined on one side by endothelium and mesothelium on the other
b. contains a central core of both collagen and elastic fibers
c. bases are attached to the annuli fibrosi of the cardiac skeleton
d. folds of endocardium or tunica intima
18. When the SA node is diseased, which of the following is likely to take its place as the
pacemaker of the heart?
a. bundle of His C. ventricular muscles
b. terminal Purkinje network D. AV node
19. The plateau phase of the fast response action potential (FRAP) is primarily due to slow
and sustained:
a. Na+ influx via slow Na+ channels C. Ca++ influx via L channels
b. K+ efflux via slow K+ channels D. Ca influx via T channels
20. In the presence of right atrial hypertrophy, these ECG configurations are prolonged:
a. P wave, PR interval, QRS interval, QT interval
b. P wave, PR interval, QRS interval
c. P wave, PR interval
d. P wave
21. Clinically, cardiac systole is from:
a. S1 to S2 C. S1 to the next S1
b. S2 to the next S1 D. S2 to the next S2
22. The following characterize the rapid ejection phase of the cardiac cycle, EXCEPT:
a. venticular pressure greater than aortic pressure
b. rapid fall in ventricular volume
c. first heart sound is heard
d. atria act as blood reservoirs
23. Which of the following conditions may cause diastolic murmur?
a. aortic insufficiency and mitral stenosis
b. pulmonary stenosis and mitral stenosis
c. tricuspid stenosis and pulmonary stenosis
d. pulmonary insufficiency and tricuspid insufficiency
24. The following associations between cardiac reflexes and heart rate effects upon
stimulation are correct, EXCEPT:
a. Goltz reflex – decrease C. Bainbridge reflex - increase
b. Aschner Dagnini reflex – decrease D. McDowall reflex – decrease
21. Trauma to the sternocostal surface of the of the heart would most likely
damage the:
A. right atrium B. left atrium
B. right ventricle C. left ventricle
22. Which is false regarding the left ventricle of the heart?
A.has the thickest wall
B. fewer papillary muscles
C. its atrioventricular opening is guarded by mitral valve
D. presence of the moderator band
23. The coronary sinus receives the following cardiac veins EXCEPT::
A.middle cardiac vein C.great cardiac vein
B. anterior cardiac vein D. small cardiac vein
24. What is the anatomical location of the tricuspid valve?
A. right half of the sternum opposite 4th intercostal space
B. left half of the sternum opposite 4th costal cartilage
C. medial end of third left costal cartilage
D. left half of the sternum opposite third intercostal space
25. Which branch of the right coronary artery lies at the posterior
Interventricular sulcus?
A. right conus C. posterior descending
B. posterior ventricular branches D. atrial branches
26. A bullet accidentally hit the left fifth intercostal space.of the anterior
chest wall. Which part of the heart is most likely injured?
A. right ventricle C.right atrium
B. left ventricle D.left atrium
27. Which of the following is not a branch of the thoracic aorta?
A. inferior phrenic C.posterior intercostals ( 10 pairs)
B. bronchial D. esophageal
28. Which one of the following branches of the abdominal aorta is paired?
A. celiac C.renal
B. superior mesenteric D. inferior mesenteric
29. The following are direct tributaries of the inferior vena cava EXCEPT:
A. left gonadal vein C. renal.vein
B. hepatic vein D. lumbar vein
30. A medical technologist is assigned to extract sample of blood for
blood chemistry . What vein is the ideal site?
A. cephalic vein C.basilic vein
B. median cubital vein D. long saphenous vein
31. A series of efferent vessels which become smaller as they branch and whose role is
carry blood to the tissues
E. Veins B. capillaries C. Lymphatics D. arteries
32. A type of capillary without fenestrae in its wall, found in all kinds of muscle tissue
A. Visceral C. Fenestrated without diapghram
B. Somatic D. Discontinuous
33. Which of the following is not a histologic feature of the tunica media
A. consists of layers of smooth muscle cells
B. contains elastic and reticular fibers
C. separated from the adventitia by an elastic laminae
D. where the vasa vasorum and nervorum are mainly found
34. Which of the following is least likely true of the veins
A. subendothelial layer may or may not be present
B. well-developed collagenous adventitia
C. all veins have well-developed tunica intima
D. valves are found in large veins
35. The following are histologic characteristics of the lymphatic capillaries, which is the
least descriptive?
A. small amount of intimal valves
B. contains a single layer of endothelial cells
C. absent fenestrations in endothelial cells
D. absent zonula occludens
36. Mean arterial pressure (MAP) tends to increase when the following are increased:
A. stroke volume, blood volume, blood viscosity, arteriolar radius
B. stroke volume, blood volume, blood viscosity
C. stroke volume, blood volume
D. stroke volume
37. When contractile strength of the right ventricle (RV) is diminished, which of the
following may decrease:
A. stroke volume of right ventricle C. peripheral venous pressure (PVP)
B. central venous pressure (CVP) D. capillary hydrostatic pressure (Pc)
38. Capillary filtration tends to increase in these conditions:
A. increased PVP, venous obstruction, hypoproteinemia, arteriolar constriction
B. increased PVP, venous obstruction, hypoproteinemia
C. increased PVP, venous obstruction
D. increased PVP
39. Which of these conditions tend to decrease central venous pressure?
A. increased venous return
B. right ventricular failure
C. pooling of blood in peripheral veins
D. increased mean systemic filling pressure
40. Which of these factors, when increased, will increase blood viscosity?
A. hematocrit C. degree of anemia
B. rate of flow D. body temperature
41. Which of the following would most likely lead to a decrease in venous return?
A. right ventricular failure
B. decreased CVP
C. increased blood volume
D. increased mean systemic filling pressure
42. When the MAP increases, the following occurs to bring back the MAP towards normal,
EXCEPT:
A. stimulation of the carotico-aortic sinus reflex
B. decreased release of atrial natriuretic factor
C. decreased release of antidiuretic hormone
D. decreased release of aldosterone
43. What is the arterial supply to the lung substance?
A.bronchial artery C.pulmonary artery
B. pulmonary veins D. all of the above
44. The left sternal line of pleural reflection starts to deviate at
A. sixth costal cartilage C. fourth costal cartilage
B. second intercostal space D.:third intercostal space
45. The following describe the right lung EXCEPT:
A. two fissures C. three lobes
B. cardiac notch D. smaller volume
46.Which is false regarding the left bronchus?
A. long C. narrow in diameter
B. vertically placed D. less tendency for foreign body deposition
47. Which is unpaired among the cartilages of the larynx?
A. thyroid C.cuneiform
B. arytenoid D. corniculate
48. Which of the following associations of the layers of the laryngeal wall is not true?
B. Epithelium – stratified squamous lining the true cords
C. Lamina propria – rich in elastic fibers, with mixed glands
D. Tunica submucosa – with glands and fibers in the deeper layer
E. Tunica adventitia – with cartilages and smooth muscles
49. The part of  the  mucosa  of  the  secondary  bronchus  where  the  Reissenssen’s  muscles  
are found
A. Epithelium C. Basement membrane
B. Lamina propria D. Muscularis mucosae
50. Which of the following layers of the wall of the ordinary bronchiole are fused?
A. mucosa and submucosa C. submucosa and adventitia
B. epithelium and lamina propria D. basement membrane & submucosa
51. Which of the following structures is not part of the barrier between air in the alveoli and
capillary blood?
A. Interstitium of interalveolar septum
B. surface epithelium of the alveolar cells
C. cytoplasm of the alveolar and endothelial cells
D. fused basal laminae of alveolar and endothelial cells
52. An alveolar cell type which secrete pulmonary surfactant
A. Type I B. Type II C. Clara D. Squamous
53. Which of the following pressures, when increased, will enhance air flow into the
alveoli?
A. intrathoracic pressure C. transthoracic pressure
B. intrapulmonic pressure D. transpulmonary pressure

54. Which of the following muscles of respiration is mobilized only during forced
expiration?
A. diaphragm C. sternocleidomastoid
B. internal intercostals D. serratus posterior superior
55. A subject exhales quietly. He then exhales maximally. The air that is expelled is called:

A. tidal volume C. functional residual capacity


B. residual volume D. expiratory reserve volume
56. Which of the following methods of measuring lung volumes and capacities measures
trapped gas?
A. helium dilution method C. wet spirometry
B. nitrogen washout method D. body plethysmography
57. Which of the following measures of ventilatory capacity is used to assess
neuromuscular coordination particularly of the muscles of respiration?
A. maximal breathing capacity C. forced expiratory vital capacity
B. timed vital capacity D. forced expiratory volume
58. Which of the following pressures, when increased, will enhance air flow into the
alveoli?
A. intrathoracic pressure C. transthoracic pressure
B. intrapulmonic pressure D. transpulmonary pressure
59. Which of the following muscles of respiration is mobilized only during forced
expiration?
A. diaphragm C. sternocleidomastoid
B. internal intercostals D. serratus posterior superior
60. A subject exhales quietly. He then exhales maximally. The air that is expelled is called:

A. tidal volume C. functional residual capacity


B. residual volume D. expiratory reserve volume
61. Which of the following methods of measuring lung volumes and capacities measures
trapped gas?
A. helium dilution method C. wet spirometry
B. nitrogen washout method D. body plethysmography
62. Which of the following measures of ventilatory capacity is used to assess
neuromuscular coordination particularly of the muscles of respiration?
A. maximal breathing capacity C. forced expiratory vital capacity
B. timed vital capacity D. forced expiratory volume
63. Which is false regarding the right kidney?
A. higher in location C. retroperitoneal
B. located at the lumbar region D. related to the liver anteriorly
64. These are the anterior relations of the left ureter EXCEPT:
A. sigmoid colon B. duodenum
B. left colic vessels C. left testicular vessels
65. At renal hilus, which lies most anterior?
A.renal pelvis B.renal artery
B. renal vein C. renal columns
66. Which part of the urinary bladder touches the prostate gland?
A.apex C. neck
B. base D. superior surface
67. Which cell is not found in the glomerular capillaries?
E. Podocytes C. Endothelial
F. Mesangial D. Smooth muscle
68. The type of collecting tubules which deliver urine to the minor calyx
A. Cortical C. Papillary
B. Medullary D. Intercalated
69. The appearance of the mucosa when the bladder is filled with urine
A. retains its folded appearance
B. disappearance of the folds
C. stretched but retains its folded appearance
D. transitional cells become cuboidal in shape
70. Which of the following does not describe the penile urethra?
A. Serous secreting glands of Littre in the mucosa
B. loose fibroelastic tissue in the lamina propria
C. rich vascular supply
D. generally lined by psedustratified and columnar epithelium
71. The clearance of a substance is lower than that of inulin. Which of the following is the
most likely reason for this result? B
A. the substance is freely filtered across the glomeruli
B. the substance is reabsorbed by the tubules
C. the substance is secreted by the fubules
D. the substance is synthesized by the tubules
72. Majority of the absorbable substances filtered across the glomeruli are reabsorbed at
the:
A. proximal tubule C. loop of Henle
B. distal tubule D. collecting duct
73. Which of the following best explains why fluid that enters the distal tubule is generally
hypo-osmolar? B
A. at the loop of Henle, about 25% of water is reabsorbed, while 15% of salt is
reabsorbed
B. at the loop of Henle, about 15% of water is reabsorbed, while 25% of salt is
reabsorbed
C. at the distal tubule water reabsorption is greater than salt reabsorption
D. at the distal tubule salt reabsorption is greater than water reabsorption
74. The bladder volume of an individual is 200 ml. At this volume, which of the following is
most likely to take place? C
A. voluntary control of micturition is lost
B. sensation of bladder filling is experienced
C. first desire to void is experienced
D. rise in intravesical pressure
75. Excessive ingestion of pure water could result in an increase in the following:
A. ECF volume, ICF volume, ECF osmolarity, ICF osmolarity
B. ECF volume, ICF volume, ECF osmolarity
C. ECF volume, ICF volume
D. ECF volume
76. Both the bicarbonate- and the non-bicarbonate buffer systems are involved in buffering
these acid-base disturbances:
A. gain of strong acid, gain of strong base, gain of pCO2, gain of HCO3-
B. gain of strong acid, gain of strong base, gain of pCO2
C. gain of strong acid, gain of strong base
D. gain of strong acid
77. A patient was admitted to the hospital due to abdominal pain and diarrhea. Aside from
dehydration he also suffered from an acid-base disturbance. After three days of
treatment, acid-base parameters were again determined and the results were: pH =
7.358; [HCO3-] = 24 mM/L; pCO2 = 37 mmHg; [BE] = (-) 1.1 mM/L. At this time the
patient has a:
A. partially compensated metabolic acidosis
B. fully compensated metabolic acidosis
C. fully compensated metabolic alkalosis
D. normal acid-base status
78. Body responses to an acid-base disturbance secondary to respiratory acidosis include
the following to bring the pH towards normal:
A. Hypoventilation
B. renal formation of new HCO3-
C. buffering by the bicarbonate- and non-bicarbonate buffer systems
D. all of the above
79. A stab wound at the epigastric region will most likely injure which of the
following organs?
A. kidney C. caecum
B. spleen D.stomach
80. Lymphatics from the tip of the tongue drain into which of the following
Lymph nodes?
A. submandibular C. submental
B. superficial cervical D. deep cervical
81. The following describe the ileum EXCEPT:
A. longest part of the small intestines
B. supplied by the superior mesenteric artery
C. redder in color than the jejunum
D. its terminal part opens into the caecum
82. Which of the following is not found in the large intestines ?
A.Paeyers patches C. fatty tags
B. taenia coli D. haustrations
83. The following are found at porta hepatis EXCEPT:
A.portal vein C. hepatic artery
B. hepatic vein D. bile duct
84.Pain of the right iliac region is most suggestive of which of the
following as based on knowledge of location?
A.appendicitis C. hepatitis
B. cystitis D. gastritis
85. The main pancreatic duct is also called the:
A.duct of Santorini C. duct of Wirsung
B.Whartons duct D. none of the above
86. The following are anterior relations of the pancreas EXCEPT:
A. transverse colon C. portal vein
B.stomach D. common bile duct
87. The general histologic picture of the GIT is all of the following, EXCEPT
E. Generally lined by simple columnar epithelium
F. Glands, lymphatic tissue and nerves may be found in the submucosa
G. The serosa makes up the outermost layer in all GI organs
H. Layered smooth muscles are found in the tunica muscularis
88. The most numerous papillae found in the entire surface of the tongue
A. Filiform C. Circumvallate
B. Fungiform D. Foliate
89. This structure fixes the tooth to the alveolar socket
A. dentinal tubules C. cementum
B. periodontal ligament D. Gingiva
90. Which part of the esophageal wall are the cardiac glands found?
A. Mucosa C. serosa
B. Submucosa D. mucosa and submucosa
91. Secretion of which cells protects the stomach from its acidic effects?
A. Goblet C. mucous neck
B. Zymogen D. surface columnar
92. Which of the following is not true of the microscopic appearance of the large intestine?
A. absence of mucosal folds except in the rectal portion
B. lymphoid cells and nodules in the lamian propria
C. abundant goblet cells
D. thick mucosal villi
93. Which phase/s of deglutition is under voluntary control?
A. oral phase C. esophageal phase
B. pharyngeal phase D. all of the above
94. Which of these gastrointestinal events occur only during periods of fasting?
A. basic electrical rhythm (BER) C. gastric emptying
B. migrating motor complex (MMC) D. HCl secretion
95. The most frequent type of movement in the small intestine is:
A. segmentation C. BER
B. peristalsis D. MMC
96. The following directly/indirectly stimulate HCl secretion:
A. acetylcholine, histamine, gastrin, gastrin releasing peptide
B. acetylcholine, histamine, gastrin
C. acetylcholine, histamine
D. acetylcholine
97. Which of the following pancreatic enzymes are involved in the digestion/absorption of
proteins?
A. trypsin, chymotrypsin, elastase, amylase
B. trypsin, chymotrypsin, elastase
C. trypsin, chymotrypsin
D. trypsin
98. Your sister wants to reduce weight. You advise her to drink skimmed milk instead of
whole milk because in skimmed milk:
A. the sugar has been reduced by 50%
B. almost all the fat has been removed
C. casein has been removed by boiling
D. a preservative has been added
99. Jane asks you why her doctor advised her to breastfeed her baby soon after delivery.
Your best answer would be:
A. breast  milk  contains  more  vitamins  than  cow’s  milk
B. breast milk has a laxative effect because of its protein content
C. the  colustrum  has  higher  immune  globulin  content  than  cow’s  milk
D. the  colustrum  has  higher  carbohydrate  content  than  cow’s  milk
100. Supplemental amount of this vitamin is recommended immediately after birth to
prevent bleeding:
A. vit. A B. vit. D C. vit. E D. vit. K
101. Glucose absorption from the intestinal lumen is enhanced by this substance:
A. potassium C. amino acid
B. sodium D. magnesium
102. This ECG configuration represents ventricular depolarization:
A. P wave C. T wave
B. QRS complex D. u wave

Xavier University – Dr Jose P Rizal College of Medicine


Basic Biomedical Sciences

FINAL EXAM: Block 3 (Modules 21 – 29) March

INSTRUCTION: Write the letter of the BEST answer on the answer sheet. Use
CAPITAL letters in ink. NO ERASURES/SUPERIMPOSITIONS ALLOWED!
Write your name on this questionnaire and on the answer sheet.
NAME ____________________________

1. Metal ions like iron bind to this atom in the pyrrole ring:
A. carbon C. nitrogen
B. oxygen D. hydrogen
2. Which of the following is a naturally occurring porphyrin containing 4 acetate and 4
proprionate side chains?
A. protoporphyrins C. coproporphyrins
B. uroporphyrins D. none of the above
3. A complex enzyme system responsible for heme catabolism:
A. synthases C. oxygenases
B. dehydratases D. carboxylases
4. The following associations between conditions with jaundice and type of bilirubin
primarily increased are correct, EXCEPT:
A. hepatitis – unconjugated bilirubin
B. hemolytic anemia – conjugated bilirubin
C. biliary tree obstruction – conjugated bilirubin
D. neonatal physiologic jaundice – unconjugated bilirubin
5. The catabolism of heme at the reticuloendothelial system results in:
A. formation of biliverdin which is reduced to bilirubin
B. production of CO and the release of ferric iron
C. the consumption of 2 moles of NAPH
D. all of the above
6. The major site of heme synthesis occurs in the:
A. hepatocytes C. leucocytes
B. bone marrow D. mature erythrocytes
7. The rate controlling step in heme biosynthesis is catalyzed by:
A. ALA synthase C. heme ferrochelatase
B. ALA dehydratase D. protorphyrin oxidase
8. These components of bile are active in the digestive process, EXCEPT:
A. bile pigments C. lecithin
B. glycocholate D. deoxycholic acid
9. According  to  Bloor’s  classification  of  lipids,  esters  of fatty acid with glycerol as alcohol
are classified as:
A. simple lipids C. derived lipids
B. complex lipids D. miscellaneous group
10. Once inside the cell, triglycerides and cholesterol esters are reformed and packaged
as ________
A. free fatty acids C. monoglycerides
B. diglycerides D. chylomicrons
11. The following are products of the oxidation of fatty acyl CoA, EXCEPT:
A. AMP C. NADH
B. acetyl CoA D. FADH2
12. Esters of fatty acid with glycerol as the alcohol:
A. sphingolipids C. triglycerides
B. phospholipids D. waxes

13. Which of the following is NOT true of high density lipoprotein (HDL)?
A. contains phospholipids and cholesterol
B. its main apolipoprotein is apolipoprotein B
C. responsible for reverse cholesterol transport
D. synthesized and secreted from liver and intestine
14. The rate limiting step in fatty acid synthesis is catalyzed by :
A. fatty synthase C. acetyl CoA carboxylase
B. acyl carnitine transferase D. ketoacyl transferase
15. Hydrolysis of phospholipid by phospholipase A2 produces:
A. 3 moles of fatty acid + glycerol C. 2 moles fatty acid + phosphatidic acid
B. lysophospholipid + fatty acid D. plasmalogen + fatty acid
16. Which of the following statements describe correctly the cholesterol metabolism?
A. Bile salt synthesis takes place exclusively in the gallbladder.
B. The  rate  limiting  in  cholesterol  synthesis  is  the  activity  of  7  α  – hydroxylase
C. Reverse cholesterol transport involves HDL & L-CAT
D. all of the above
17. The committed step in cholesterol biosynthesis is :
A. the formation of mevalonate
B. catalyzed by HMGCoA synthase
C. the reaction between acetoacetyl-CoA & acetyl Co-A
D. requires thiamine diphosphate
18. Which of these is the correct sequence of lipoproteins from the most dense to the least
dense?
A. HDL>VLDL>chylomicrons>LDL C. HDL>LDL>VLDL>chylomicrons
B. LDL>chylomicrons>VLDL>HDL D. VLDL>chylomicrons>LDL.HDL
19. Esters of fatty acid with glycerol as the alcohol:
A. sphingolipids B. phospholipids C. triglycerides D. waxes
20. During ketogenesis, the increased fatty acid oxidation may produce all of the following
ketone bodies, EXCEPT:
A. acetone C. 3- B –OH butyrate
B. acetoacetate D. 3-OH CH3 glutarate
21. The following descriptions are true of glucose, EXCEPT:
A. an aldose monosaccharide
B. the most important monosaccharide biomedically
C. may be classified as hexose based on number of Carbon atoms it possesses
D. its straight –chain structural formula can account for all of its properties
22. The major products of starch hydrolysis by amylase include:
A. maltose, lactose, limit dextrans C. glucose, maltose, maltotriose
B. maltriose, lactose, glucose D. maltose, maltotriose,limit dextran
23. The transporter of glucose in facilitated diffusion across the basolateral membrane is:
A. SGLUT C. GLUT-2 transporter
B. GLUT-1 trnsporter D. GLUT-4 transporter
24. Conversion of pyruvate to oxaloacetate is catalyzed by pyruvate carboxylase in the
presence of the following, EXCEPT:
A. water C. biotin
B. ATP D. carbon dioxide
25. Gycogenolysis in muscle leads to formation of lactate instead of glucose due to the
absence of:
A. phosphorylase C. phosphoglucomutase
B. debranching ezyme D. glucose-6-phosphatase
26. In liver glycogenesis, glucose is phosphorylated to glucose-6-phosphate. This
reaction is catalyzed by:
A. hexokinase C. glycogen synthase
B. glucokinase D. glycogen phosphorylase

27. Which of the following integrates the regulation of glycogenolysis and glycogenesis by
simultaneous activation of phosphorylase and inhibition of glycogen synthase?
A. Ca++ C. cAMP
B. calmodulain D. glucokinase
28. The rate-limiting step in glycogenolysis is catalyzed by:
A. hexokinase C. glycogen synthase
B. glucokinase D. glycogen phosphorylase
29. These enzymes are involved in liver glycogenolysis, EXCEPT:
A. glycogen synthase C. glucan transferase
B. glycogen phosphorylase D. debranching enzyme
30. These substances are important substrates for gluconeogenesis during fasting,
EXCEPT:
A. glycerol C. biotin
B. acetyl CoA D. phosphoenolpyruvate
31. Which of the following is NOT true of the uronic acid pathway?
A. it is the source of active glucoronate
B. it is a source of ascorbic acid in man
C. it provides for interconversion of hexose to pentose
D. it is a source of active glucose for glycogen synthesis
32. The Hydrogen Breath Test is used to diagnose this condition:
A. galactase deficiency C. fructose deficiency
B. lactase deficiency D. biphosphatase deficiency
33. Which of these is true of protein digestion and absorption in the intestine?
A. digestion is initiated by the presence of bile in the duodenum
B. most of the aminopeptidases are secreted by the pancreatic cells
C. absorption requires amino acid transporter across mucosal cells
D. the terminal step in digestion occurs in the lumen of the duodenum
34. The ten nutritionally essential amino acids are necessary in the diet because:
A. humans cannot synthesize them in adequate amounts
B. they are necessary in blood coagulation
C. they are essential for digestion
D. all of the above
35. Amino acids are essentially composed of four basic components attached to the same
carbon atom.These are an amino group, a hydrogen atom, a side chain group & ___.
A. oxygen C. glycosidic group
B. pyrrole ring D. carboxylic acid group (COOH)
36. This refers to the multiple forms of an enzyme in a given specie:
A. isoenzymes C. proenzymes
B. zymogens D. coenzymes
37. The following are classification of proteins based on their biological function, EXCEPT:
A. enzymes C. VLDL
B. storage D. regulatory
38. The final digestion of proteins to amino acids occur in these locations, EXCEPT:
A. brush border C. cytoplasm of mucosa cells
B. intestinal lumen D. interstitial fluid bathing mucosal cells
39. These are ketogenic amino acids:
A. leucine, lysine, cystein, glycine
B. leucine, lysine, cystein
C. leucine, lysine
D. leucine
40. The first step in the catabolism of most amino acids is the removal of its amino group
by transamination, EXCEPT for this amino acid:
A. Gly C. Lys
B. Ala D. Val
41. Humans synthesize adequate amounts of all of the following amino acids, EXCEPT:
A. Met C. Arg
B. Gln D. Asp
42. Which of the following catalyzes the transfer of electrons?
A. transferases C. hydrolases
B. oxidoreductases D. ligases
43. Alpha ketoglutarate can be produced from all of the following amino acids, EXCEPT:
A. phenylalanine C. proline
B. histidine D. arginine
44. The following are end-products of glucogenic amino acid catabolism, EXCEPT:
A. aspartate C. arginine
B. fumarate D. proline
Module 25 – Genetics

45. The following usually make up nucleotides, EXCEPT:


A. carboxyl group C. purine or pyrimidine base
B. pentose sugar D. phosphate group
46. The primary regulatory enzyme of pyrimidine nucleotide synthesis in eukaryotic cells:
A. OPR transferase C. CTP synthase
B. CP synthetase-2 D. APR transferase
47. The following are removed from the ring during pyrimidine nucleotide catabolism,
EXCEPT:
A. ribose C. amino groups
B. phosphate D. carbon dioxide
48. The following describe both replication and transcription, EXCEPT:
A. phases/stages are similar in terms of initiation and elongation
B. direction of synthesis is the same
C. direction of elongation is the same
D. both make use of a primer
49. Which of these directs proper initiation of translation?
A. regulator C. operator
B. promoter D. repressor
50. The following factors are needed to initiate translation of mRNA in prokaryotes,
EXCEPT:
A. guanosine triphosphate C. 30 S ribosomal subunits
B. formylMet tRNA D. cap-binding protein
51. Which of the following is true of Okasaki fragments?
A. they are RNA-DNA hybrids C. they are removed by nuclease activity
B. they are double-stranded D. they contain covalent linked RNA & DNA
52. The hydrolytic step leading to release of a polypeptide from a ribosome is catalyzed
by:
A. stop codon C. hydrolases
B. release factors D. peptidases
53. which of these is the nicking-resealing enzyme in replication?
A. translocase C. DNA topoisomerase
B. frameshift mutase D. DNA-dependent RNA polymerase
54. Degeneracy of the genetic code means that:
A. there is no punctuation in the code sequence
B. the 3rd base in the codon is not important in coding
C. a given base triplet can code for more than one amino acid
D. a given amino acid can be coded by more than one base triplet
55. This structure in tRNA is responsible for recognition of the triplet nucleotide:
A. D arm C. extra arm
B. T U C arm D. anticodon arm
56. Which of the following enzymes in replication functions as proof-reading device?
A. 3’exonuclease C. polymerase
B. 5’exonuclease D. topoisomerase

Module 26 – Endocrine

57. The high hemoglobin levels of people who live in Baguio City is an example of:
A. adaptation C. biologic rhythm
B. acclimatization D. aging
58. Intercellular chemical messengers maybe any of the following, EXCEPT:
A. neurotransmitters C. holocrine hormones
B. endocrine hormones D. paracrine hormones
59. Which of the following second messengers activates calmodulin?
A. calcium C. diacyglycerol
B. cyclic AMP D. cyclic GMP
60. Which of these is common to all endocrine glands?
A. parynchema of epithelium
B. strom of loose connective tissue
C. fine network of ducts among capillaries
D. all of the above
61. The following are true of the pancreas, EXCEPT:
A. both an endocrine and exocrine gland
B. its venous supply drain to the portal system
C. lies in the right upper quadrant of the abdomen
D. its main duct opens into the 2nd part of the duodenum
62. The following describe the parathyroid gland, EXCEPT:
A. ovoid bodies
B. supplied by the inferior thyroid artery
C. supplied by the superior thyroid artery
D. intimately related to the inferior border of the thyroid cartilage
63. The following describe the pituitary gland, EXCEPT:
A. consists of two glands
B. weights about 0.5 gram
C. lies in the cavity of the sphenoid bone
D. blood supply is derived from the external carotid artery
64. Which of these associations between endocrine cell and secretory product is NOT
correct?
A. beta cell > insulin C. pituicytes > melatonin
B. fat cell > leptin D. follicular cells > thyroid hormone
65. Tumors in this gland can cause sustained or episodic hypertension as a result of high
levels of hormones released:
A. pancreas C. adrenal cortex
B. parathyroid D. adrenal medulla
66. Juan was walking down the street when his enemy suddenly accosted him. He
decided to turn and run for help. This corresponds to which of the following stages of
the general adaptation syndrome?
A. stage of shock C. stage of resistrance
B. stage of countershock D. stage of exhaustion
67. Low plasma level of cortisol stimulates the secretion of pituitary adrenocorticotropic
hormone. This is due to:
A. positive feedback C. short-loop negative feedback
B. simple feedback D. long-loop negative feedback

Module 27 – MRS
68. The following are true of the penis, EXCEPT:
A. made up of erectile tissue
B. body  is  enclosed  by  Buck’s  fascia
C. glans penis is formed by the corpora cavernosa
D. root is formed by the bulb and crura of the penis
69. The following are contents of the deep perineal pouch in males, EXCEPT:
A. prostate gland C. membranous part of the urethra
B. sphincter urethrae D. deep transverse perineal muscle
70. Which of the following is NOT correct regarding the male reproductive system?
A. glandular interstitial cells are stimulated by luteinizing hormone
B. sperm motility is essential for fertilization to occur under normal conditions
C. acrosome forms from golgi body during spermatid stage of spermatogenesis
D. rate of sperm production is influenced by frequency of ejaculation
71. Which of the following embryonic structures will differentiate into the penis?
A. urogenital sinus C. genital swelling
B. genital tubercle D. genital fold
72. Maturation of spermatids into spermatozoa takes place during this phase of
spermatogenesis:
A. golgi phase C. acrosome phase
B. cap phase D. maturation phase
73. Which of the following is NOT true of the seminal vesicles?
A. folded mucosa lined by cuboidal epithelium
B. secretes a viscid and yellowish secretion
C. has an elastic fiber-rich lamina propria
D. tubuloalveolar glands
74. Which of the following is NOT an action of testosterone?
A. exerts a protein-catabolic effect
B. exerts inhibitory feedback on pituitary LH secretion
C. develops and maintains male secondary sex characteristics
D. along with FSH, maintains spermatogenesis
75. Which of the following functions of Sertoli cells mediates negative feedback control of
FSH secretion?
A. excessive growth of semineferous tubules
B. aromatization of testosterone
C. synthesis of testosterone
D. synthesis of inhibin
76. The following are characteristics of human semen, EXCEPT:
A. pH is similar to plasma
B. average volume of ejaculate is 2.5 – 3.5 ml
C. average sperm count is 80 – 120 million per ml
D. most of its volume is contributed by the prostate
77. The first step in the biosynthesis of testosterone is the conversion of cholesterol to:
A. progesterone C. estradiol
B. pregnenolone D. dehydroepiandrosterone
78. Which of the following hormones is responsible for male-pattern balding?
A. estradiol C. dihydrotestosterone
B. testosterone D. androstenedione

Module 28 – FRS-1
79. The following describe the vagina, EXCEPT:
A. lined by keratinized stratified epithelium
B. absence of mucosal glands and sensory nerve endings
C. circular and longitudinal smooth muscles in the middle layer
D. two types of connective tissues are seen in the lamina propria
80. Which of these is NOT true of the uterus?
A. divided into body, fundus, and cervix
B. in most women the normal position is retroversion
C. body is related posteriorly to the pouch of Douglas
D. supplied by the uterine artery, a branch of the internal iliac artery
81. The following are components of the corpus luteum, EXCEPT:
A. granulosa cells C. theca cells
B. elastic fibers D. fibroblasts
82. Which of the following is correct regarding the endometrium at the proliferative phase
of the menstrual cycle?
A. stromal cells in active division C. swollen lamina propria
B. disappearance of spiral artery D. all of the above
83. Which of these is NOT correct regarding oogenesis?
A. the development of oogonia to mature egg cells
B. starting before birth, primordial follicles with primary oocyte grow
C. during ovulation, the first meiotic division is done forming secondary oocyte
D. secondary oocyte completes second meiotic division without fertilization
84. As regards fertilization and implantation, which of these statements is/are correct?
A. Fertilization normally occurs at the ampulla of the oviduct.
B. The fertilized embryo is a blastocyst when implanted
C. implantation occurs at the upper anterior or posterior wall of the uterus body
D. all of the above
85. Which of these cells is responsible for conversion of androgen to estradiol and
secretion of progesterone?
A. granulosa cells C. theca lutein cells
B. theca interna cells D. granulosa lutein cells
86. In a 28 day cycle, progressive increase in estrogen is seen during which of the
following phases?
A. menstrual phase C. ovulation phase
B. follicular phase D. secretory phase
87. A change in the color of the labia minora from bright red to burgundy occurs at which
of the following phases of the female sexual response cycle?
A. Excitement phase C. Orgasmic phase
B. Plateau phase D. Resolution phase
88. Estrogen dominance may be characterized by the following, EXCEPT:
A. Hyperglycemia C. Depression
B. Water retention D. Decreased sex drive
89. Maria has a regular 40 day menstrual cycle. Her last two menstrual periods started
on January 1 and February 9. Her ovulation most likely occurred on:
A. January 14 C. January 27
B. January 20 D. February 1
90. Basal cells of the glands of the stratum basalis begin to proliferate during which of
these phases of the endometrial cycle?
A. pre-menstrual phase C. proliferative phase
B. menstrual phase D. secretory phase

91. Which of the following is NOT true of ovulation?


A. characterized by an increase in basal body temperature
B. characterized by very thick cervical mucus
C. triggered by a surge in LH secretion by pituitary
D. occurs about 14 days prior to the next menstrual flow

Module 29 – Pregnancy
92. Which of the following sequential order of development of the zygote in the first week
of gestation is true?
A. zygote --> morula --> blastocyst --> yolk sac
B. zygote --> blastomere --> blastocyst --> morula
C. zygote --> morula --> blastomere --> blastocyst
D. zygote --> blastomere --> morula --> blastocyst
93. Completion of implantation of the blastocyst in the endometrium happens in which
week of gestation?
A. first C. third
B. second D. fourth
94. In response to the attaching blastocyst, the following events happen in the uterus,
EXCEPT:
A. edema formation
B. blood vessel constriction
C. increased capillary permeability
D. increased proliferation of glandular and epithelial cells
95. The following are produced by the corpus luteum of pregnancy:
A. progesterone, estrogen, relaxin, enkephalins
B. progesterone, estrogen, relaxin
C. progesterone, estrogen
D. progesterone
96. During pregnancy, estrogens cause the following, EXCEPT:
A. enlargement  of  mother’s  uterus
B. enlargement  of  mother’s  breasts
C. enlargement  of  mother’s  external  genitalia
D. contraction  of  mother’s  pelvic  ligaments
97. Relaxin helps maintain pregnancy by:
A. inhibiting myometrial contractions
B. causing the relaxation of the pelvic ligaments
C. preventing the opening of the cervix
D. relaxing the breasts
98. Which of these placental components would be gone by the late third trimester of
pregnancy?
A. Cytotrophoblasts C. Syncytiotrophoblasts
B. Capillaries D. Mesenchyme
99. In what  order  would  an  oxygen  molecule  from  Nanay’s  blood  encounter  the  following  
placental components as the oxygen makes its way to a first-trimester fetus:
1. Capillary bed 4. Mesenymal-like connective tissue
2. Chorionic plate 5. Syncytiotrophoblast
3. cytotrophoblast
A. 1,2,3,4,5 C. 3,1,2,5,4
B. 5,3,4,1,2 D. 2,4,3,1,5
100.Which change in the mammary gland normally associated with the first
trimester of pregnancy is most likely to also vary with the menstrual cycle?
C. ducts sprout and form new branches
D. interlobular connective tissue decreases
E. milk dilates alveoli
F. colostrums dilates alveoli
101.Which of the following is presumptive evidence of pregnancy?
A. Goodell’s  sign C. Mastodynia
B. Hegar’s  sign D. Ballottment
102. Which of the following methods of fertility control takes into consideration the
character of cervical mucus as indicative of the fertile period?
A. post-coital douche C.    Billing’s  method
B. calendar rhythm D. Temperature rhythym

XAVIER UNIVERSITY – DR JOSE P RIZAL COLLEGE OF MEDICINE


BASIC BIOMEDICAL SCIENCES

Final Exam: BLOCK 4 (Modules 30 – 36 & Special Topics) March 15, 2005

Write the letter of the BEST answer on the answer sheet. Use capital letters in ink.
NO ERASURES/SUPERIMPOSITIONS!
Write your name on this questionnaire and on the answer sheet.

NAME__________________________

19. The mesencephalon develops into which of the following


A. cerebral hemispheres C. midbrain
B. cerebellum D. pons
20. Which of the following is not true of the central nervous system?
A. its components are covered by meninges
B. of its cellular content, neurons number more than the neuroglia
C. made up of the brain and spinal cord
D. its parts are suspended in cerebrospinal fluid
21. This neuroglial cell is inactive in a normal central nervous system:
A. fibrous astrocyte C. oligodendroglia
B. microglia D. ependymocytes
22. A mechanoreceptor in hairless skin mediated by A fibers and with slow adaptability:
A. Meissner’s  corpuscle C.    Merkel’s  discs
B. Ruffini’s  corpuscle D. Free nerve endings
23. Higher centers in the brain influence spinal segmental reflexes via these tracts:
A. corticospinal tract, vestibulospinal tract, spinothalamic tract, fasciculus gracilis
B. corticospinal tract. vestibulospinal tract, spinothalamic tract
C. corticospinal tract, vestibulospinal tract
D. corticospinal tract
24. Aside from an effector organ, these are the components of a monosynaptic reflex arc:
A. receptor organ, afferent neuron, efferent neuron, interneuron/s
B. receptor organ, afferent neuron, efferent neuron
C. receptor organ, afferent neuron
D. receptor organ
25. Lesions of lower motor neurons occur in these parts:
A. abducens nerve, facial nerve, anterior horn, dorsal funiculus
B. abducens nerve, facial nerve, anterior horn
C. abducens nerve, facial nerve
D. abducens
26. Most of the fibers of the corticospinal tracts decussate at this level ;
A. junction of the medulla oblongata and the spinal cord
B. junction of the midbrain and the pons
C. at the cerebellum
D. none of the above
27. The following are indicative of lesions in the corticospinal (pyramidal) tracts EXCEPT:
A. Babinski sign is present C. clasp knife reaction
B. absent superficial abdominal reflex D. absent cremasteric reflex
10. With a lesion on the posterior funiculus, which function is likely impaired?
A. pain and temperature sensation C. tactile discrimination
B. movement of body parts D. unconscious propioception
11. Where do you find the cerebrospinal fluid in the spinal cord?
A. epidural space C. subarachnoid space
B. subdural space D.all of the above

12. Which of these structure to role association is NOT correct ?


A. ventral spinothalamic tract > light touch sensation
B. fasciculus gracilis > tactile discrimination of upper body parts
C. zone of Lissauer > pain and temperature sensation
D. reticulospinal tract > relay of autonomic information to preganglionic neurons
13. Trauma to the C6 spinal vertebra is likely to involve this spinal cord segment:
A. C4 B. C6 C. C8 D. C9
14.The following are true regarding the spinal cord, EXCEPT:
A. ends at lower border of L1 vertebral level in the adult
B. supported laterally by the denticulate ligaments
C. segments are in line with corresponding vertebral level
D. a bridge between the brain and peripheral nervous system
15. Contralateral hemiplegia is a likely presentation with lesions in these areas,
EXCEPT:
A. lateral funiculus C. pyramid
B. pars basalis D. precentral gyrus
16. The following are general functions of the brain stem, EXCEPT :
A. regulation of heart rate and respiration
B. control of sleep and wakefulness
C. channel for ascending and descending tracts
D. contains all the cranial nerve nuclei
17. What cavity is present at the level of the decussation of pyramids?
A. central canal C. third ventricle
B. fourth ventricle D. lateral ventricle
18. The spinal nucleus of cranial nerve V is present at which level/s of the medulla?
A. decussation of pyramids C. olives
B. decussation of medial lemnisci D. all of the above
19. These nuclei are found at the level of the facial colliculus of the pons but not at the
level of the trigeminal nuclei of the pons:
A. facial nucleus C. medial vestibular nucleus
B. abducent nucleus D. all of the above
20. The pons displays these elevations:
A. facial colliculus, locus ceruleus, eminentia pyramidalis, trigunum hypoglossi
B. facial colliculus, locus ceruleus, eminentia pyramidalis
C. facial colliculus, locus ceruleus
D. facial colliculus
21. These structures belong to the open medulla:
A. 4th ventricle, nucleus solitarius, inferior olivary nucleus, brachium pontis
B. 4th ventricle, nucleus solitarius, inferior olivary nucleus
C. 4th ventricle, nucleus solitarius
D. 4th ventricle
22. Which of the following is/are characteristics of the reticular formation?
A. contains monoaminergic cell groups that modulate mood and alertness
B. a collection of neurons and nerve fibers in the brain stem and upper spinal cord
C. project to the spinal cord, cerebellum, hypothalamus, cerebral cortex
D. all of the above
23. The cranial nerve nucleus involved with mastication is the:
A. motor nucleus of vagus C. motor facial nucleus
B. motor nucleus of trigeminal D. nucleus ambiguus
24. A lesion on the left pyramid and its emerging cranial nerve lateral to it can produce:
A. weakness, right tongue C. hyperactive tendon reflexes, right foot
B. paralysis, left extremities D. paralysis, left upper and lower face
25. Axons from the dorsal motor nucleus of the vagus are incorporated in these nerves:
A. VII, IX, X, XII C. VII, IX
B. VII, IX, X D. VII

26. Which of the following is/are characteristic/s of the extrapyramidal system?


A. controls crude movements
B. ends in motor cranial nerve nuclei
C. lesions in pathway can cause paralysis
D. all of the above
27. Which of the following structures is not found in the molecular layer of the cerebellum?
A. stellate cells C. Golgi type I cells
B. basket cells D. dendrites of Purkinje cells
28. Which of the following is not true of the granular layer?
A. contains small, multipolar cells
B. dendrites of granule cells synapse with climbing fibers
C. axons of granule cells proceed to the molecular layer
D. neuroglial cells are found throughout this layer
29. This fiber type forms the greater part of the cerebellar white matter:
A. Intrinsic C. Afferent
B. Efferent D. Extrinsic
30. The part of the cerebellum that receives projections from the cerebral hemisphere:
A. posterior lobe C. flocculi
B. anterior lobe D. nodulus
31. The outermost part of the cerebellar cortex contains the following:
A. basket cells, stellate cells, glial cells, axons of Purkinje cells
B. basket cells, stellate cells, glial cells
C. basket cells, stellate cells
D. basket cells
32. Alex, 12 years old, presented with unsteady stance and tendency to fall forward and
backward. The most likely structure to be involved is the:
A. right cerebellar peduncle C. cerebellar vermis
B. left brachium conjunctivum D. right red nucleus
33. Which of the following manifestations is characteristic of a unilateral lesion involving
the right cerebellar hemisphere?
A. dysarthria
B. atrophy of the right upper and lower limbs
C. paralysis of right upper and lower limbs
D. unsteady gait with tendency to reel to the right
34. The following signs are suggestive of cerebellar disease, EXCEPT:
A. intention tremor C. decreased muscle tone
B. wide base gait D. ipsilateral muscle fasciculations
35. Which of these deep cerebellar nuclei is involved in programming of repeated
movements?
A. fastigial C. globose
B. dentate D. emboliform
36. The cerebelum influences movement via these routes:
A. red nucleus, ventral lateral nucleus, vestibular nucleus, globus pallidus
B. red nucleus, ventral lateral nucleus, vestibular nucleus
C. red nucleus, ventral lateral nucleus
D. red nucleus
37. This tract is the only efferent pathway of the inferior cerebellar peduncle:
A. fastigiobulbar C. pontocerebellar
B. olivocerebellar D. trigeminocerebellar
38. These arteries supply the cerebellum:
A. posterior inferior cerebellar, anterior inferior cerebellar,superior cerebellar
B. posterior inferior cerebellar, anterior inferior cerebellar
C. posterior inferior cerebellar, superior cerebellar
D. posterior inferior cerebellar, anterior cerebral

39. The cavity between the thalami within the cerebrum:


A. 3rd ventricle C. lateral ventricle
th
B. 4 ventricle D. cerebral aqueduct
40. The origin of the projection fibers to the auditory area arise from which structure:
A. medial geniculate body C. mammillary bodies
B. lateral geniculate body D. superior colliculi
41. A cortical area which is important for planning or coordinating the articulatory
movements needed for speech:
A. Uncus C. Straight gyrus
B. Insula D. Postcentral gyrus
42. Which of the following manifestations is not referable to a basal ganglia dysfunction?
cortex?
A. Hemiballismus C. Chorea
B. Athetosis D. Agnosia
43. The structural pathology in a patient with conduction aphasia is:
A. disconnection  of  Wernicke’s  area  from  Broca’s  area
B. vascular lesions in the internal capsule
C. tumor in the postcentral gyrus
D. all of the above
44. Which of the following structures serves as a cell station to all the main sensory
systems except the olfactory pathway?
A. diencephalon C. hypothalamus
B. thalamus D. medulla
45. Which of these is true of the layers of the cerebral cortex?
A. conveniently divided into five layers
B. the most superficial layer is the molecular layer
C. Majority of the cells in the multiform layer are modified pyramidal cells
D. the internal granular layer is composed of small pyramidal cells
46. The thalamus serves as a relay station to these sensory systems:
A. sight, hearing, taste, smell
B. sight, hearing, taste
C. sight, hearing
D. sight
47. Which of the following is/are true of the choroid plexus?
A. produces cerebrospinal fluid
B. a vascular fringe composed of pia matter and dura matter
C. located in the lateral, third & fourth ventricles and in the cerebral aqueduct
D. all of the above
48. This is a characteristic of the basal ganglia:
A. a collection of nuclei in the gray matter of the cerebrum
B. globus pallidus is main site for receiving input to basal ganglia
C. caudate nucleus is main site from which output leaves basal ganglia
D. afferent connections come from cerebral cortex, thalamus, substantia
nigra
49. Which fibers in the centrum semiovale links the sensory language areas to the motor
speech areas?
A. projection C. association
B. commisural D. all of the above
50. In which layer of the cerebral cortex lie the cell bodies of the descending motor
pathways?
A. internal pyramidal C. external pyramidal
B. internal granular D. external granular
51. Which part of the basal ganglia contribute to the limbic system as part of the pleasure
center?
A. putamen C. globus pallidus
B. caudate nucleus D. nucleus accumbens

52. An 80 year old man who displays uncontrollable flinging movements of his right upper
limb is suspected to have a stroke; most likely the lesion is in the left:
A. motor area C. cerebellar hemisphere
B. subthalamic nucleus D. dorsolateral medulla
53. Which of these associations between anatomic structure and role is/are correct?
A. pars triangularis > production of words in language
B. anterior frontal gyrus > cognition, social and moral sense
C. inferior parietal lobule > comprehension of symbols/words in language
D. all of the above
54. A 54 year old stroke victim developed alexia, finger agnotia, agraphia without motor
weakness.    He  most  likely  sustained  a  lesion  in  Brodmann’s:
A. Area 44, right C. Area 4, right
B. Area 39, left D. Area 4, left
55. The mesencephalic nucleus of the V in the brain stem receives fibers mediating what
sensation?
A. touch & pressure C. proprioception
B. pain & temperature D. taste
56. First order neurons of sensory pathways from the body are in:
A. sensory ganglia of spinal nerves C. receptor
B. autonomic ganglia D. dorsal root ganglia
57.Sensory pathway that decussate at the level of the medulla:
A. pain & temperature from face C. proprioception from face
B. touch from body D. pain & temperature from body
58.The lateral cervical system forms part of the pathway for:
A. proprioception C. touch
B. pain D. audition
59. The second order neurons of the proprioceptive pathway from the body occurs at the
level of:
A. diencephalon C. medulla
B. pons D. spinal cord
60. Which sensory pathway/s from the body makes synapse at its point of entry in the
CNS?
A. light touch & pressure C. proprioception
B. pain & temperature D. all of these
61. The pathway for proprioception from the leg includes the following, EXCEPT:
A. medial lemniscus C. internal arcuate fibers
B. fasciculus cuneatus D. posterolateral ventral nucleus
62. After eating sour pickles, an individual noticed that the water he drank tastes sweet.
This is most probably due to:
A. simultaneous contrast C. positive after image
B. successive contrast D. negative after image
63. In an experiment, a student placed a red disc on a sheet of white paper. While staring
at the red disc, he noticed a ring of green surrounding the red disc. This is most
probably due to:
A. simultaneous contrast C. positive after image
B. successive contrast D. negative after image
64. The most common site of pain modulation is at the:
A. locus ceruleus
B. pontine raphe nucleus
C. rostral ventromedial medulla
D. ventrolateral periacqueductal gray
65. The following characterize the medial lemniscus:
A. arises from the posterior column nuclei
B. travels through the spinal cord and brain stem
C. ends in the posterolateral ventral nucleus of the thalamus
D. conveys sensations carried by the fasciculus gracilis only

66. In an experiment, a student placed a red disc on a sheet of white paper. While staring
at the red disc, he noticed a ring of green surrounding the red disc. This is most
probably due to:
A. simultaneous contrast C. positive after image
B. successive contrast D. negative after image
67.Which of the following structures is not common to the visual, pupillary and
accommodation pathways?
C. Retina C. Optic tract
D. Pretectal nucleus D. Optic nerve
68. Which of the following is not a test for vision?
A. Finger counting C. Flashlight test
B. Confrontation method D.    Snellen’s  chart
69. An injury to the right lower part of the geniculocalcarine tract will result to:
A. Left homonymous hemianopsia
B. Left homonymous superior quadrantanopsia
C. Left homonymous inferior quadrantanopsia
D. Left heteronymous hemianopsia
70. The efferent path of the pupillary light reflex begins with which of the following
structures
A. Edinger-Westphal nucleus C. Ciliary ganglion
B. Pretectal area D. Oculomotor nerve
71. An optical reflex test which signifies the sensitivity of the eye to touch
A. Ishihara C. Corneal reflex test
B. Reading cards D. Flashlight test
72. Right nasal hemianopsia can result from an injury to which of the following
A. optic chiasma C. right optic nerve
B. right optic tract D. right angle of the chiasma
73. Going down the ladder stimulates primarily the:
A. vertical semicircular canals C. saccule
B. horizontal semicircular canals D. utricle
74. The hair cells of the organ of Corti are stimulated when:
A. stereocilia are bent away from the limbus
B. stereocilia are bent away from the tallest cilium
C. organ of Corti is deflected downward
D. basilar membrane is pushed towards the scala tympani
75. The central artery of the retina is a direct branch of the:
A. ophthalmic artery C. supraorbital artery
B. internal carotid artery D. external carotid artery
76. Bilateral deafness may result in a lesion of:
A. spiral ganglion C. cochlear nerve
B. trapezoid body D. cochlear nuclei
77. A sensory pathway that is associated with the temporal lobe:
A. vision C. audition
B. taste D. touch
78. One of these structures is NOT associated with the vestibular pathway:
A. semicircular ducts C. utricle
B. spiral organ D. saccule
79. Second order neurons of the vestibular pathway are found in:
A. vestibular ganglion C. motor cranial nerve nuclei
B. vestibular nuclei D. flocculo-nodular lobe, cerebellum
80. The following tract/s carry vestibular fibers:
A. reticulospinal tract C. vestibulospinal tract
B. medial longitudinal fasciculus D. all of these
81.The medial olfactory stria ends at:
A. anterior perforated substance C. septal area
B. uncus D. hippocampus

82. Fibers from taste buds in the epiglottis is associated with which ganglion?
A. geniculate C. nodose
B. Gasserian D. petrosal
83.The nucleus in the brain stem that mediates taste sensation:
A. main sensory nucleus of V C. nucleus ambiguus
B. nucleus solitarius D. facial nucleus
84. When light passes through two apertures located close to each other, light waves that
emanate from the apertures tend to overlap at certain points producing:
A dispersion B. refraction C. reflection D. diffraction
85. Which of the following errors of refraction is commonly associated with the aging
process?
A. myopia C. presbyopia
B. hyperopia D. astigmatism
86.Aside from the eyeball the orbit contains the following:
A. connective tissues, nerves, blood vessels, extraocular muscles
B. connective tissues, nerves, blood vessels
C. connective tissues, nerves
D. connective tissues
87. Juan refuses to talk to his wife until she follows his wishes. This action constitute:
A. physical abuse C. social abuse
B. sexual abuse D. psychological abuse
88.Domestic violence is also known as:
A. wife battering C. woman abuse
B. marital violence D. all of the above
89. A married couple was having an argument. She felt his anger rising so she stopped arguing
because she felt that at any time now he would begin to hit her with his fist. He had done this
several times in the past. In the cycle of violence this is called the:
A. build-up phase C. violent phase
B. pursuit phase D. remorse phase
90. Juan often shouts at his elderly mother every time she refuses to take her medicines. He tells
to obey or else he would leave her. This behavior of Juan constitutes:
A. physical abuse C. psychological abuse
B. financial abuse D. sexual abuse
91.Which of the following is a physiological change associated with aging?
A. decrease in bronchial secretions C. decrease in acid secretion
B. decreased resistance to gas flow D. decrease in cholesterol secretion
92.Which of the following is an anatomical change associated with aging?
A. increase in liver mass and blood flow
B. increase in size of the thoracic cage
C. decrease in skin pigmentation
D. decrease in lean body mass
93.Which of the following taste sensations is the first to diminish with aging?
A. sweet C. salty
B. sour D. umami
94.The best way to synchronize the circadian rhythm to the exact 24 hours imposed on
man by the sun is to make use of:
A. entrainment C. drugs
B. zeitgebers D. muscle activity
95.Fluctuations in body temperature is an example of:
A. circadian rhythm C. ultradian rhythm
B. infradian rhythm D. circaseptan rhythm
96.Which of the following makes up the greater bulk of the sleep time of a young adult?
A. stage 1 C. delta sleep
B. stage 2 D. none of the above

97.Which of the following is most likely to take place upon ascent to an altitude of 10,000
feet?
A. mild tremors C. impaired judgment
B. reflex irritability D. personality changes
98.One of the most serious hazards of long term missions in aerospace is:
A. space adaptation syndrome C. muscle atrophy
B. decalcification of bone D. decrease in RBC by 5-15%
99he ability to resist fatigue is called:
A. fitness C.conditioning
B. endurance D. adaptation
100An oxygen consumption of 1-2 liters/min is typical of which of the following?
A. mild work C. hard work
B. moderate work D. maximal work
101.In order to be of value, carbohydrate-laden drinks should be taken:
A. 1 hour before exercise C. 1 hour after exercise
B. 30 minutes before exercise D. at intervals during exercise
102. Which of the following serves as part of warm-up exercise?
A. toe touch – 10 repetitions C. flutter kick – 30 repetitions
B. half knee bend – repetitions D. push-up – 4 repetitions

XAVIER UNIVERSITY – DR JOSE P RIZAL COLLEGE OF MEDICINE


BASIC BIOMEDICAL SCIENCES

Final Exam: BLOCK 4 (Modules 30 – 36 & Special Topics) March 15, 2007

Write the letter of the BEST answer on the answer sheet. Use capital letters in ink.
NO ERASURES/SUPERIMPOSITIONS!
Write your name on this questionnaire and on the answer sheet.
NAME__________________________
Module 30 (14) 1-14
28. The mesencephalon develops into which of the following
A. cerebral hemispheres C. midbrain
B. cerebellum D. pons
29. Which of the following is not true of the central nervous system?
A. its components are covered by meninges
B. of its cellular content, neurons number more than the neuroglia
C. made up of the brain and spinal cord
D. its parts are suspended in cerebrospinal fluid
30. This neuroglial cell is inactive in a normal central nervous system:
A. fibrous astrocyte C. oligodendroglia
B. microglia D. ependymocytes
31. Higher centers in the brain influence spinal segmental reflexes via these tracts:
A. corticospinal tract, vestibulospinal tract, spinothalamic tract, fasciculus gracilis
B. corticospinal tract. vestibulospinal tract, spinothalamic tract
C. corticospinal tract, vestibulospinal tract
D. corticospinal tract
32. Aside from an effector organ, these are the components of a monosynaptic reflex arc:
A. receptor organ, afferent neuron, efferent neuron, interneuron/s
B. receptor organ, afferent neuron, efferent neuron
C. receptor organ, afferent neuron
D. receptor organ
33. Lesions of lower motor neurons occur in these parts:
A. abducens nerve, facial nerve, anterior horn, dorsal funiculus
B. abducens nerve, facial nerve, anterior horn
C. abducens nerve, facial nerve
D. abducens
34. Most of the fibers of the corticospinal tracts decussate at this level ;
A. junction of the medulla oblongata and the spinal cord
B. junction of the midbrain and the pons
C. at the cerebellum
D. none of the above
35. The bundle of anterior and posterior roots of lumbar, sacral, and coccygeal
spinal nerves and filum terminale iscalled:
A. conus medullaris C. filum terminale
B. cauda equina D. ligamentum denticulatum
36. As regards the blood supply to the spinal cord, the following are true, EXCEPT:
A. there are two anterior spinal arteries
B. there are reinforcing arteries called radicular arteries
C. the posterior spinal arteries originate from the vertebral artery
D. its veins drain into the internal vertebral venous plesuxes
37. This nerve cell group in the posterior gray column of spinal cord receives
afferent fibers concerned with pain, temperature and touch from the posterior
root:
A. nucleus proprius C. nucleus dorsalis
B. substancia gelatinosa D. visceral afferent nucleus
11. With a lesion on the posterior funiculus, which function is likely impaired?
A. pain and temperature sensation C. tactile discrimination
B. movement of body parts D. unconscious propioception
12. Where do you find the cerebrospinal fluid in the spinal cord?
A. epidural space C. subarachnoid space
B. subdural space D.all of the above

13. Trauma to the C6 spinal vertebra is likely to involve this spinal cord segment:
A. C4 B. C6 C. C8 D. C9
14.The following are true regarding the spinal cord, EXCEPT:
A. ends at lower border of L1 vertebral level in the adult
B. supported laterally by the denticulate ligaments
C. segments are in line with corresponding vertebral level
D. a bridge between the brain and peripheral nervous system
Module 31 brain stem (11) 15-25

55. The following are general functions of the brain stem, EXCEPT :
A. regulation of heart rate and respiration
B. control of sleep and wakefulness
C. channel for ascending and descending tracts
D. contains all the cranial nerve nuclei
56. The elongated swelling found on each side of the posterior median sulcus ofthe
inferior half of the medulla is formed by the:
A. olive C. gracile tubercle
B. pyramid D. inferior cerebral peduncle
57. The centers of visual reflexes and the lower auditory center are located in this
part of the midbrain:
A. crus cerebri C. corpora quadrigemina
B. lateral geniculate body D. medial geniculate body
58. CN VIII takes its exit from the skull through what opening?
A. jugular foramen C. foramen magnum
B. internal auditory meatus D. hypoglossal
59. The spinal nucleus of cranial nerve V is present at which level/s of the medulla?
A. decussation of pyramids C. olives
B. decussation of medial lemnisci D. all of the above
60. The pons displays these elevations:
A. facial colliculus, locus ceruleus, eminentia pyramidalis, trigunum hypoglossi
B. facial colliculus, locus ceruleus, eminentia pyramidalis
C. facial colliculus, locus ceruleus
D. facial colliculus
61. These structures belong to the open medulla:
A. 4th ventricle, nucleus solitarius, inferior olivary nucleus, brachium pontis
B. 4th ventricle, nucleus solitarius, inferior olivary nucleus
C. 4th ventricle, nucleus solitarius
D. 4th ventricle
62. Which of the following is/are characteristics of the reticular formation?
A. contains monoaminergic cell groups that modulate mood and alertness
B. a collection of neurons and nerve fibers in the brain stem and upper spinal cord
C. project to the spinal cord, cerebellum, hypothalamus, cerebral cortex
D. all of the above
63. The cranial nerve nucleus involved with mastication is the:
A. motor nucleus of vagus C. motor facial nucleus
B. motor nucleus of trigeminal D. nucleus ambiguus
64. Axons from the dorsal motor nucleus of the vagus are incorporated in these nerves:
A. VII, IX, X, XII C. VII, IX
B. VII, IX, X D. VII
65. Which of the following is/are characteristic/s of the extrapyramidal system?
A. controls crude movements
B. ends in motor cranial nerve nuclei
C. lesions in pathway can cause paralysis
D. all of the above
Module 32 (cerebellum) (11) 26- 36
66. Which of the following structures is not found in the molecular layer of the cerebellum?
A. stellate cells C. Golgi type I cells
B. basket cells D. dendrites of Purkinje cells
67. Which of the following is not true of the granular layer?
A. contains small, multipolar cells
B. dendrites of granule cells synapse with climbing fibers
C. axons of granule cells proceed to the molecular layer
D. neuroglial cells are found throughout this layer
68. The part of the cerebellum that receives projections from the cerebral hemisphere:
A. posterior lobe C. flocculi
B. anterior lobe D. nodulus
69. The outermost part of the cerebellar cortex contains the following:
A. basket cells, stellate cells, glial cells, axons of Purkinje cells
B. basket cells, stellate cells, glial cells
C. basket cells, stellate cells
D. basket cells
70. Which of the following is/are descriptive of the cerebellar puduncles?
A. The superior peduncles connect the cerebellum to the midbrain
B. the middle peduncles consist almost entirely of afferent fibers
C. the inferior peduncles consist chiefly of afferent fibers
D. all of the above
71. Which of these is the largest of the cerebellar nuclei?
A. globose C. dentate
B. fastigial D. emboliform
72. Alex, 12 years old, presented with unsteady stance and tendency to fall forward and
backward. The most likely structure to be involved is the:
A. right cerebellar peduncle C. cerebellar vermis
B. left brachium conjunctivum D. right red nucleus
73. Which of the following manifestations is characteristic of a unilateral lesion involving
the right cerebellar hemisphere?
A. dysarthria
B. atrophy of the right upper and lower limbs
C. paralysis of right upper and lower limbs
D. unsteady gait with tendency to reel to the right
74. Which of these deep cerebellar nuclei is involved in programming of repeated
movements?
A. fastigial C. globose
B. dentate D. emboliform
75. This tract is the only efferent pathway of the inferior cerebellar peduncle:
A. fastigiobulbar C. pontocerebellar
B. olivocerebellar D. trigeminocerebellar
76. These arteries supply the cerebellum:
A. posterior inferior cerebellar, anterior inferior cerebellar,superior cerebellar
B. posterior inferior cerebellar, anterior inferior cerebellar
C. posterior inferior cerebellar, superior cerebellar
D. posterior inferior cerebellar, anterior cerebral
Module 33 –cerebrum (16) 37- 52
77. The cavity between the thalami within the cerebrum:
A. 3rd ventricle C. lateral ventricle
B. 4th ventricle D. cerebral aqueduct
78. The origin of the projection fibers to the auditory area arise from which structure:
A. medial geniculate body C. mammillary bodies
B. lateral geniculate body D. superior colliculi
79. A cortical area which is important for planning or coordinating the articulatory
movements needed for speech:
A. Uncus C. Straight gyrus
B. Insula D. Postcentral gyrus
80. Which of the following manifestations is referable to a cerebral cortex dysfunction?
A. Hemiballismus C. Chorea
B. Athetosis D. Agnosia
81. The structural pathology in a patient with conduction aphasia is:
A. disconnection  of  Wernicke’s  area  from  Broca’s  area
B. vascular lesions in the internal capsule
C. tumor in the postcentral gyrus
D. all of the above
82. Which of the following structures serves as a cell station to all the main sensory
systems except the olfactory pathway?
A. diencephalon C. hypothalamus
B. thalamus D. medulla
83. Which of these is true of the layers of the cerebral cortex?
A. conveniently divided into five layers
B. the most superficial layer is the molecular layer
C. Majority of the cells in the multiform layer are modified pyramidal cells
D. the internal granular layer is composed of small pyramidal cells
84. The thalamus serves as a relay station to these sensory systems:
A. sight, hearing, taste, smell
B. sight, hearing, taste
C. sight, hearing
D. sight
85. Which of the following is/are true of the choroid plexus?
A. produces cerebrospinal fluid
B. a vascular fringe composed of pia matter and dura matter
C. located in the lateral, third & fourth ventricles and in the cerebral aqueduct
D. all of the above
86. This is a characteristic of the basal ganglia:
A. a collection of nuclei in the gray matter of the cerebrum
B. globus pallidus is main site for receiving input to basal ganglia
C. caudate nucleus is main site from which output leaves basal ganglia
D. afferent connections come from cerebral cortex, thalamus, substantia
nigra
87. Which fibers in the centrum semiovale links the sensory language areas to the motor
speech areas?
A. projection C. association
B. commisural D. all of the above
88. In which layer of the cerebral cortex lie the cell bodies of the descending motor
pathways?
A. internal pyramidal C. external pyramidal
B. internal granular D. external granular
89. Which part of the basal ganglia contribute to the limbic system as part of the pleasure
center?
A. putamen C. globus pallidus
B. caudate nucleus D. nucleus accumbens
90. An 80 year old man who displays uncontrollable flinging movements of his right upper
limb is suspected to have a stroke; most likely the lesion is in the left:
A. motor area C. cerebellar hemisphere
B. subthalamic nucleus D. dorsolateral medulla
91. Which of these associations between anatomic structure and role is/are correct?
A. pars triangularis > production of words in language
B. anterior frontal gyrus > cognition, social and moral sense
C. inferior parietal lobule > comprehension of symbols/words in language
D. all of the above
92. The circle of Willis is formed by the following arteries:
A. basilar, anterior cerebral, posterior cerebral, middle cerebral
B. basilar, anterior cerebral, posterior cerebral
C. basilar, anterior cerebral, middle cerebral
D. anterior cerebral, middle cerebral, posterior cerebral
Module 34- sensory pathways (11) 53-63
53. The mesencephalic nucleus of the V in the brain stem receives fibers mediating what
sensation?
A. touch & pressure C. proprioception
B. pain & temperature D. taste
54. First order neurons of sensory pathways from the body are in:
A. sensory ganglia of spinal nerves C. receptor
B. autonomic ganglia D. dorsal root ganglia
55.Sensory pathway that decussate at the level of the medulla:
A. pain & temperature from face C. proprioception from face
B. touch from body D. pain & temperature from body
56.The lateral cervical system forms part of the pathway for:
A. proprioception C. touch
B. pain D. audition
57. The second order neurons of the proprioceptive pathway from the body occurs at the
level of:
A. diencephalon C. medulla
B. pons D. spinal cord
58. Which sensory pathway/s from the body makes synapse at its point of entry in the
CNS?
A. light touch & pressure C. proprioception
B. pain & temperature D. all of these
59. The pathway for proprioception from the leg includes the following, EXCEPT:
A. medial lemniscus C. internal arcuate fibers
B. fasciculus cuneatus D. posterolateral ventral nucleus
60. After eating sour pickles, an individual noticed that the water he drank tastes sweet.
This is most probably due to:
A. simultaneous contrast C. positive after image
B. successive contrast D. negative after image
61. In an experiment, a student placed a red disc on a sheet of white paper. While staring
at the red disc, he noticed a ring of green surrounding the red disc. This is most
probably due to:
A. simultaneous contrast C. positive after image
B. successive contrast D. negative after image
62. The most common site of pain modulation is at the:
A. locus ceruleus
B. pontine raphe nucleus
C. rostral ventromedial medulla
D. ventrolateral periacqueductal gray
63. The following are common causes of chronic pain in the elderly, EXCEPT:
A. appendicitis C. peripheral neuropathies
B. osteoporosis D. degenerative joint disease

Module 35-sense organs (24) 64-83


64. Which of the following structures is not common to the visual, pupillary and
accommodation pathways?
E. Retina C. Optic tract
F. Pretectal nucleus D. Optic nerve
65. Which of the following is not a test for vision?
C. Finger counting C. Flashlight test
D. Confrontation method D.    Snellen’s  chart
66. An injury to the right lower part of the geniculocalcarine tract will result to:
a. Left homonymous hemianopsia
b. Left homonymous superior quadrantanopsia
c. Left homonymous inferior quadrantanopsia
d. Left heteronymous hemianopsia
67. The efferent path of the pupillary light reflex begins with which of the following
structures
a. Edinger-Westphal nucleus C. Ciliary ganglion
b. Pretectal area D. Oculomotor nerve
68. An optical reflex test which signifies the sensitivity of the eye to touch
a. Ishihara C. Corneal reflex test
b. Reading cards D. Flashlight test
69. The possibility of pharyngitis (inflammation of the pharynx) to spread to the
middle ear can be explained anatomically due to the presence of the:
A. eustachian tube C. external auditory canal
B. internal auditory canal D. all of the above
70. Going down the ladder stimulates primarily the:
A. vertical semicircular canals C. saccule
B. horizontal semicircular canals D. utricle
71. The hair cells of the organ of Corti are stimulated when:
A. stereocilia are bent away from the limbus
B. stereocilia are bent away from the tallest cilium
C. organ of Corti is deflected downward
D. basilar membrane is pushed towards the scala tympani
72. The central artery of the retina is a direct branch of the:
A. ophthalmic artery C. supraorbital artery
B. internal carotid artery D. external carotid artery
73. Bilateral deafness may result in a lesion of:
A. spiral ganglion C. cochlear nerve
B. trapezoid body D. cochlear nuclei
74. A sensory pathway that is associated with the temporal lobe:
A. vision C. audition
B. taste D. touch
75. One of these structures is NOT associated with the vestibular pathway:
A. semicircular ducts C. utricle
B. spiral organ D. saccule
76. Second order neurons of the vestibular pathway are found in:
A. vestibular ganglion C. motor cranial nerve nuclei
B. vestibular nuclei D. flocculo-nodular lobe, cerebellum
77. The following tract/s carry vestibular fibers:
A. reticulospinal tract C. vestibulospinal tract
B. medial longitudinal fasciculus D. all of these
78.The medial olfactory stria ends at:
A. anterior perforated substance C. septal area
B. uncus D. hippocampus
79. Fibers from taste buds in the epiglottis is associated with which ganglion?
A. geniculate C. nodose
B. Gasserian D. petrosal
80.The nucleus in the brain stem that mediates taste sensation:
A. main sensory nucleus of V C. nucleus ambiguus
B. nucleus solitarius D. facial nucleus
81. When light passes through two apertures located close to each other, light waves that
emanate from the apertures tend to overlap at certain points producing:
A dispersion B. refraction C. reflection D. diffraction
82. Which of the following errors of refraction is commonly associated with the aging
process?
A. myopia C. presbyopia
B. hyperopia D. astigmatism
83. Aside from the eyeball the orbit contains the following:
A. connective tissues, nerves, blood vessels, extraocular muscles
B. connective tissues, nerves, blood vessels
C. connective tissues, nerves
D. connective tissues

Module 36 –domestic, etc (16) (84-100)


84. Juan refuses to talk to his wife until she follows his wishes. This action constitute:
A. physical abuse C. social abuse
B. sexual abuse D. psychological abuse
88.Domestic violence is also known as:
A. wife battering C. woman abuse
B. marital violence D. all of the above
89. A married couple was having an argument. She felt his anger rising so she stopped
arguing
because she felt that at any time now he would begin to hit her with his fist. He had
done this
several times in the past. In the cycle of violence this is called the:
A. build-up phase C. violent phase
B. pursuit phase D. remorse phase
90. Juan often shouts at his elderly mother every time she refuses to take her medicines.
He tells
to obey or else he would leave her. This behavior of Juan constitutes:
A. physical abuse C. psychological abuse
B. financial abuse D. sexual abuse
91.Which of the following is a physiological change associated with aging?
A. decrease in bronchial secretions C. decrease in acid secretion
B. decreased resistance to gas flow D. decrease in cholesterol secretion
92.Which of the following is an anatomical change associated with aging?
A. increase in liver mass and blood flow
B. increase in size of the thoracic cage
C. decrease in skin pigmentation
D. decrease in lean body mass
93.Which of the following taste sensations is the first to diminish with aging?
A. sweet C. salty
B. sour D. umami
94.The best way to synchronize the circadian rhythm to the exact 24 hours imposed on
man by the sun is to make use of:
A. entrainment C. drugs
B. zeitgebers D. muscle activity
95.Fluctuations in body temperature is an example of:
A. circadian rhythm C. ultradian rhythm
B. infradian rhythm D. circaseptan rhythm
96.Which of the following makes up the greater bulk of the sleep time of a young adult?
A. stage 1 C. delta sleep
B. stage 2 D. none of the above

97.Which of the following is most likely to take place upon ascent to an altitude of 10,000
feet?
A. mild tremors C. impaired judgment
B. reflex irritability D. personality changes
98.One of the most serious hazards of long term missions in aerospace is:
A. space adaptation syndrome C. muscle atrophy
B. decalcification of bone D. decrease in RBC by 5-15%
99he ability to resist fatigue is called:
A. fitness C.conditioning
B. endurance D. adaptation
100An oxygen consumption of 1-2 liters/min is typical of which of the following?
A. mild work C. hard work
B. moderate work D. maximal work
101.In order to be of value, carbohydrate-laden drinks should be taken:
A. 1 hour before exercise C. 1 hour after exercise
B. 30 minutes before exercise D. at intervals during exercise
102. Which of the following serves as part of warm-up exercise?
A. toe touch – 10 repetitions C. flutter kick – 30 repetitions
B. half knee bend – repetitions D. push-up – 4 repetitions

Xavier University – Dr Jose P Rizal College of Medicine


Basic Biomedical Sciences

FINAL EXAM: Block I (Modules 1 – 10) 13 March 2007


INSTRUCTIONS: Write the letter of the BEST answer on the answer sheet. Use
CAPITAL letters in ink. NO ERASURES/SUPERIMPOSITIONS ALLOWED!
Write your name on this questionnaire and on the answer sheet.

NAME ____________________________

1. In the absence of ATP this transport process cannot occur:


A. osmosis C. facilitated diffusion
B. bulk flow D. sodium-glucose cotransport
2. When a stimulus causes the membrane potential to increase, which of these would
most likely occur?
A. action potential C. resting membrane potential
B. graded potential D. diffusion potential
3. The rectifying property of the cell membrane is due to:
A. sodium influx C. sodium efflux
B. potassium efflux D. potassium influx
4. The cell is absolutely refractory to stimulation during these phases of the action
potential:
A. depolarization, overshoot, early repolarization, undershoot
B. depolarization, overshoot, early repolarization
C. depolarization, overshoot
D. depolarization
5. The plasma membranes of most cells in the body exhibit the following characteristics,
EXCEPT:
A. can store large amounts of ATP
B. can produce action potentials spontaneously
C. can go back to its original state after excitation
D. allows movement of molecules across the membrane
6. At its resting membrane potential (RMP), the nerve cell membrane:
A. is excitable C. allows net influx of Na+
B. allows net efflux of K+ D. is positive inside relative to outside
7. Which of the following is true of the Na+/K+ pump?
A. it is an extrinsic protein
B. it is an ATPase that catalyzes the hydrolysis of ATP
C. at RMP it moves 2 K+ along concentration gradient
D. at RMP it moves 3 Na+ along electrical gradient
8. Which of the following is true for both graded potential and action potential?
A. can be summated C. can occur spontaneously
B. can be propagated D. magnitude varies with stimulus size
9. When red cells are exposed to 10% Na CL solution, which of the following occurs?
A. hemolysis C. crenation
B. swelling D. cell volume remains the same
10. The reducing equivalents formed during the course of oxidation of acetyl CoA in the
TCA cycle:
A. enter the respiratory chain
B. are regenerated to form carbohydrate
C. are regenerated to form oxaloacetate
D. participate in the formation of fatty acid
11. The energy released by the oxidation of 1 mole of NADH will support the synthesis of
approximately:
A. 1 mole ATP C. 3 moles ATP
B. 2 moles ATP D. 12 moles ATP
12. Which of these is the terminal component of the respiratory chain responsible for the
reaction whereby electrons are transferred to their final acceptor?
A. cytochrome a3 oxidase C. CoQH2-cytochrome e-c reductase
B. succinate dehydrogenase D. NADH CoQ reductase
13. Which of the following is not true of the mitochondria?
A. accumulate in parts of the cell where energy use is higher
B. made up of an inner & outer membranes
C. cristae are folds in the outer mitochondrial membrane
D. low energy requiring cells have few mitochondria and short cristae
14. Which of the following is the major role of the cytoskeleton?
A. Stores secretory products
B. Maintains shape & for organelle movement
C. Excretes waste products
D. Supports metabolic processes
15. Transitory components of the cell which store colored substances
A. zymogen granules C. lipid droplets
B. pigments D. glycogen
16. The following are true of chromatin, EXCEPT
A. nuclear component
B. made up of coiled strands of DNA
C. in interphase, it maintains its coiled state
D. euchromatin is less coiled than heterochromatin
17. Which is not true of anaphase?
A. nucleolus disappears
B. migration of chromosomes to poles
C. beginning of cell division
D. appearance of cleavage furrow
18. Which phase in interphase is characterized by DNA synthesis and the start of
centrosome duplication?
A. G1 B. S C. G2 D. G0
19. As a result of meiosis 1 the amount of DNA is
A. Doubled C. Tripled
B. Remains the same D. Halved
20. Which of the following is not a basic structural component of a simple epithelial tissue?
A. lining epithelium C. muscularis mucosae
B. lamina propria D. basement membrane
21. The prostate gland is an example of which type of compound gland
A. Tubular C. Saccular
B. Acinar D. Coiled
22. An epithelial cell specialization found on the lateral surface whose role is for
communication
A. zonulae adherents C. gap junctions
B. hemidesmosomes D. tight junctions
23. The following connective tissue cells play a role in immunologic defense, EXCEPT
A. Plasma C. Mast cell
B. Osteoblast D. Basophil
24. A connective tissue type characterized by abundant ground substance, very few fibers,
fibroblasts predominate
A. Mucous C. Dense
B. Adipose D. Reticular
25. A cell member of the mononuclear phagocyte system found in the skin
A. Monocyte C. Langerhans
B. Melanocyte D. Dendritic
26. Which of the following is not true of the dermis?
A. Made up of loose and dense connective tissues
B. The reticular layer projects to the epidermis
C. Contains the arrector pili muscles
D. Where the sweat & sebaceous glands are found
27. The layer of the epidermis whose role is for protection and as a permeability barrier
A. stratum lucidum C. stratum corneum
B. stratum spinosum D. stratum basale
28. Which of the following is not true of the histologic structure of the neuron?
A. The perikaryon contains highly developed RER
B. Golgi complex is seen up to the axon
C. Dendrites are usually short and branched
D. May or may not have an axon
29. This neuroglial cell is found in the CNS and mainly functions for support, repair and
metabolism
A. Astrocyte C. Oligodendrocyte
B. Ependymal D. Microglia
30. Which of the following differentiates the spinal cord from the other CNS organs?
A. its white matter do not contain neuronal cell bodies
B. glial cells and dendrites are seen in the gray matter
C. arrangement of the gray and white matter
D. fiber tracts are found in the white matter
31. The smallest neurons in the body are found in this layer of the cerebrum
A. Molecular C. Granular
B. Purkinje D. White matter
32. Which of the following is true of the Schwann cell?
A. Serves several axons at one time
B. Also found in the cerebrum
C. Binds neurons to capillaries and the meninges
D. Non-branching & forms myelin around one axon
33. Which of the following is not true of the ganglia?
A. encapsulated by myelin sheath
B. contains nerve cell bodies and glial cells
C. has a connective tissue support
D. associated with either cranial or spinal nerves
34. As regards inhibitors at synapses, the following associations are true, EXCEPT:
A. hemicholinium --> inhibits acetylcholine synthesis
B. organophosphaste --> inhibits cholinesterase
C. prostigmine --> inhibits release of acetylcholine
D. succinylcholine --> competes with acetylcholine for receptor sites
35. Which of the following is true of the end plate potential?
A. occurs at the postsynaptic neuron
B. can trigger an action potential
C. due to quantal release of acetylcholine
D. involves a voltage change of 15 mV in the muscle membrane
36. These events lead to the production of an excitatory postsynaptic potential at the
postsynaptic neuron, EXCEPT:
A. diffusion of Ca++ into the terminal knob
B. release of neurotransmitter into the synaptic cleft
C. depolarization of the terminal knob of the presynaptic membrane
D. local increase in permeability to K+ at the postsynaptic neuron
37. Which of these events immediately precedes exocytosis and release of acetylcholine
at the neuromuscular junction?
A. calcium entry into the synaptic vesicles
B. depolarization of the terminal knob
C. opening of voltage-gated sodium channels
D. fusion of vesicles with membrane of terminal knob
38. Which of the following helps explain why stimulation of the sympathetic division gives
rise to a widespread response?
A. The sympathetic division innervates practically all organ-systems of the body.
B. The sympathetic chain consists of 22 interconnected ganglia.
C. The transmitter of the sympathetic division is inactivated slowly.
D. All preganglionic fibers of the sympathetics arise from the spinal cord.
39. Which of the following statements as regards the sympathetic division is most correct?
A. White rami communicantes are formed by preganglionic fibers and spinal
nerves.
B. All spinal nerves receive white rami communicantes.
C. Gray rami communicantes are formed by postganglionic fibers.
D. There are approximately 31 white rami communicantes.
40. Which of the following is most likely to occur upon stimulation of postganglionic fibers
that exit from the ciliary ganglion?
A. pupilloconstriction C. lacrimation
B. salivary secretion D. bradycardia
41. Which of the following is most likely to occur upon stimulation of the vagus?
A. secretion of scanty, mucoid saliva C. bronchodilation
B. increased gastric tone and motility D. penile erection
42. Which of the following cranial nerves contains autonomic efferent fibers only?
A. oculomotor B. facial C. glossopharyngeal D. vagus
43. Which of the following responses to sympathetic stimulation can be attributed to
stimulation of 1 receptors only?
A. liver glycogenolysis C. piloerection
B. decreased intestinal motility D. lipolysis
44. The microscopic appearance of osteoblasts during matrix deposition
A. reduced number of RER
B. spherical with acidophilc cytoplasm
C. columnar in shape with basophilic cytoplasm
D. large and becomes branched
45. A cross section of this bone type on gross observation shows dense areas with
absence of cavities
A. Developing C. Compact
B. Cancellous D. None of the above
46. Which of the following is true of cartilages?
A. all types are surrounded by perichondrium
B. fibrocartilage is the most common type
C. collagen type II is the principal fiber in all forms
D. variations in the composition of the matrix produce the 3 types
47. Which of the following is not true of endochondral ossification?
A. occurs in the presence of mesenchymal tissue
B. responsible for the formation of short and long bones
C. requires a intervening cartilage step
D. formation of primary and secondary ossification centers
48. A patient was brought to the emergency room because of a fracture of the
biggest short bone of the right foot. What bone is this?
A. First metatarsal C. first phalanx
B. Calcaneus D. talus
49. The metaphysis of a long bone is found between epiphyseal cartilage and the:
A. Epiphysi C. diaphysis
B. Periosteum D. endosteum
50. A vehicular accident victim had undergone surgical removal of one upper
extremity. Give the total number of bones left with the patient after the operation:
A. 174 B. 175 C. 176 D. 177
51. Fracture to this part of the humerus could injure the radial nerve:
A. Surgical neck C. medial epicondyle
B. Spiral groove D. bicipital groove
52. Which of the following joints permits supination and pronation movements?
A. Elbow C. metacarpophalangeal
B. Radiocarpal D. radioulnar
53. Which of the following muscles adducts the arm and rotates it medially?
A. Pectoralis minor C. teres major
B. Serratus anterior D. deltoid

54. Fracture of the olecranon process of the humerus could injure this muscle:
A. Coracobrachialis C. brachialis
B. pronator teres D. triceps brachii
55. Injury to the axillary nerve could paralyze this muscle:
A. Deltoid C. latissimus dorsi
B. Supraspinatus D. teres major
56. The following are muscles associated with respiration except:
A. Serratus posterior superior C. serratus posterior inferior
B. Serratus anterior D. levatores costarum
57. The following the constitute erector spinae muscles except:
A. Iliocostalis C. semispinalis
B. Longissimus D. spinalis
58. The following form the boundaries of the triangle of auscultation Except:
A. Latissimus dorsi C. trapezius
B. Rhomboideus major D. scapula
59. The number of processes arising from the vertebral arch of a vertebra:
A. Five C. seven
B. Six D. eight
60. Which of the following is characteristic of the thoracic vertebra?
A. The vertebral foramen is large and triangular
B. The spines are small and bifurcated
C. The body is large and kidney shaped
D. The transverse processes possesses a foramen
61. The following ligaments support atlanto-axial joints except:
A. Alar C. cruciate
B. Longitudinal D. Apical
62. Which of the following is not rue of skeletal muscle tissue?
A. made up of long, cylindrical multinucleated cells
B. each muscle fiber is ensheathed by connective tissue
C. muscle fibers show cross striations of alternating light and dark bands
D. the sarcoplasm is filled with intra and extrafusal fibers
63. Which of the following membrane junctions is not found in the intercalated discs?
A. fasciae adherents C. tight junctions
B. maculae adherents D. gap junctions
64. In skeletal muscle contraction, which of the following events immediately follows the
binding of Ca++ with troponin C?
A. release of more calcium from the sarcoplasmic reticulum (SR)
B. transport of calcium by the calcium pump back to the SR
C. movement of tropomyosin away from actin binding sites
D. binding of myosin heads to actin binding sites
65. Which of the following is immediately responsible for the opening of calcium channels
in the sarcoplasmic reticulum (SR)?
A. an action potential in the T tubule
B. an action potential in the SR membrane
C. hydrolysis of ATP in the SR membrane
D. calcium influx from the extracellular fluid
66. Calcium plays a significant role in the following events, EXCEPT:
A. muscle contraction C. bone formation
B. blood coagualtion D. nerve action potential propagation
67. Referred    to  as  the  “  dangerous  area”  of  the  scalp:
A. Aponeurosis C. dense connective tissue
B. Loose areolar tissue D. pericranium
68. Gives attachment to the ligamentum nuchae:
A. Superior nuchal line C. inferior nuchal line
B. Mastoid process D. external occipital protruberance
69. The nasal septum is formed largely by this bone:
A. Ethmoid C. vomer
B. Nasal D. maxilla
70. The following sinuses open into the middle meatus of the nasal cavity except:
A. Sphenoidal C. maxillary
B. Frontal D. ethmoidal
71. Which of the following skull openings transmits the ophthalmic artery?
A. Superior orbital fissure C. optic canal
B. Foramen spinosum D. foramen ovale
72. The following muscles elevate mandible except:
A. Lateral pterygoid C. medial pterygoid
B. Masseter D. temporalis
73. This muscle divides the neck into anterior and posterior triangles:
A. Omohyoid C. digastric
B. Sternohyoid D. sternocleidomastoid
74. Part of the pubis that joins ischial ramus below the obturator foramen:
A. Superior ramus C. inferior ramus
B. Body D. pubic tubercle
75. Tarsal bone that articulates with the tibia and fibula at the ankle joint:
A. Cuboid C. navicular
B. Calcaneus D. talus
76. Injury to the superior gluteal nerve causes paralysis of the following muscles
except:
A. Gluteus maximus C. gluteus minimus
B. Gluteus medius D. tensor fasciae latae
77. Which of the following muscles causes extension of leg at knee joint;
flexion of thigh at hip joint?
A. Rectus femoris C. semitendinosus
B. Semimembranosus D. sartorius
78. Injury to this nerve causes paralysis of the muscles in the anterior fascial
compartment of the leg:
A. Femoral C. obturator
B. Deep peroneal D. superficial peroneal
79. Which of the following muscles plantar flexes foot at ankle joint: everts foot
at subtalar and transverse tarsal joints?
A. Gastrocnemius C. soleus
B. Peroneus longus D. plantaris
80. The following are synovial, hinge joints except:
A. Elbow C. radioulnar
B. Knee D. ankle
81. The following describe the lumbar plexus except:
A. Formed in the psoas muscle
B. The sciatic is its largest branch
C. Formed by the anterior rami of the upper four lumbar nerves
D. The main nervous pathway supplying the lower limb
82. The male inguinal canal transmits the following structures except:
A. Pampiniform plexus C. vas deferens
B. Testicular artery D. epididymis
83. Which of the following is not a content of the popliteal fossa?
A. Short saphenous vein C. lymph nodes
B. Common peroneal nerve D. femoral vessels
84. The primary component of the buffy coat
C. Platelets C. Red blood cells
D. Leukocytes D. Clootign factors
85. Which of the following leucocytes is not a terminal cell?
A. Monocyte C. Basophil
B. Eosinophil D. Neutrophil
86. Biconcave, anucleated cells which never leave the circulatory system in normal
conditions
A. Platelets C. Erythrocytes
B. Reticulocytes D. Plasma cells
87. These agranular cells are spherical with similar morphologic characteristics, but
classified according to their distinct surface markers
A. Monocytes C. Lymphocytes
B. Macrophage D. Lymphoblast
88. Which of the following formed elements of blood possess phagocytic function, but die
soon after phagocytosis?
A. eosinophils B. basophils C. monocytes D. neutrophils
89. Which of the following conditions is most likely to give rise to neutropenia?
A. allergy B. viral infection C. stress D. chronic uremia
90. The principal hemopoietic organ at birth is the:
A. liver C. marrow of bones of the axial skeleton
B. spleen D. marrow of the proximal ends of long bones only
91. Which of the following multilineage growth factors enhances synthesis of other growth
factors?
A. IL 1 B. IL 3 C. IL 6 D. IL 11
92. Which of the following lineage-specific growth factors is chemotactic for neutrophils?
A. IL 2 B. IL 5 C. IL 7 D. IL 8
93 A lymphoepithelial organ found in the mediastinum and is of dual embryonic origin
E. Tonsils C. Bone marrow
F. Spleen D. Thymus
94. Which of the following is not divided into cortex and medulla?
G. Thymus C. Lymph nodes
H. Spleen D. None of the above
95. Which of the following is not true of MALT?
A. encapsulated aggregates of lymphoid tissues
B. found in the GIT, GUT and respiratory tracts
C. occur in the mucosa and submucosa
D. major cell component is the lymphocyte
96. Which of the following is basically capable of stimulating an immune response only
when bound to protein?
A. antigen B. antibody C. hapten D. cytokine
97. Which of the following cells are mainly responsible for a faster immune response upon
re-exposure to a foreign antigen?
A. effector T and B cells C. natural killer cells
B. memory T and B cells D. plasma cells
98. Which of the following serves as an indicator that the B cell is fully mature?
A. CD 25 B. CD 22 C. surface Ig M D. surface Ig D
99. Which of the following antibodies is most likely to be present in the serum of an
exclusively bottle-fed newborn?
A. Ig G B. Ig M C. Ig A D. Ig D
100.Which of the following is mainly responsible for electrical repulsion between red cells?
A. pH of blood C. ionic cloud
B. temperature of blood D. zeta potential
101. Which of the following is a primary organ of the immune system?
A. Spleen C. tonsils
B. Bone marrow D. appendix
102. The following are descriptive of the spleen except:
A. Situated at the right upper quadrant of the abdomen
B. Its arterial supply is derived from celiac artery
C. Largest lymphoid tissue in the body
D. Intraperitoneal organ

XAVIER UNIVERSITY – DR JOSE P RIZAL COLLEGE OF MEDICINE


BASIC BIOMEDICAL SCIENCES

Final Exam: BLOCK 3 (Modules 21 – 29) March 13, 2006

Write the letter of the BEST answer on the answer sheet. Use capital letters in ink.
NO ERASURES/SUPERIMPOSITIONS ALLOWED!

1. Which of the following statements describe correctly the cholesterol metabolism?


A. Bile salt synthesis takes place exclusively in the gallbladder.
B. The rate limiting in cholesterol synthesis  is  the  activity  of  7  α  – hydroxylase
C. Reverse cholesterol transport involves HDL & L-CAT
D. all of the above
2. The committed step in cholesterol biosynthesis is :
A. the formation of mevalonate
B. catalyzed by HMGCoA synthase
C. the reaction between acetoacetyl-CoA & acetyl Co-A
D. requires thiamine diphosphate
3. Alcoholism leads to fat accumulation in the liver resulting from oxidation of alcohol.
The oxidation of alcohol results in fatty liver because it:
A. increases adipose fatty acid synthesis & mobilization of fatty acid to the liver
B. forms the product acetyl-CoA the fatty acid precursor in the liver
C. produces NADH that ultimately inhibits fatty acid oxidation
D. it decreases the esterification of glycerol thus fatty acid is increase
4. Serves as the donor of methyl group in the synthesis of phosphatidylcholine from
cephalin:
A. H4folate B. glycine C. Serine D. S-adenosylmethionine
5. The activated precursor of phosphocholine, phosphoethanolamine and phosphoserine in
phospholipid synthesis contains the nucleotide:
A. CDP B. cAMP C. ATP D. cGMP
6. During ketogenesis, the increase fatty acid oxidation may produce all of the following
ketone bodies, EXCEPT:
A. acetone B. acetoacetate C. 3-  -OH butyrate D. 3-OH CH3 glutarate
7. Each round of the 4 core reactions in - oxidation of palmitate forms fatty acid with
1 Carbon less than the initial reactant plus:
A. 1 mole acetyl CoA, 1 mole FADH2 & 1 mole NADH
B. 4 moles “ ,  1      “ “ &4 “ “
C. 8 moles “ ,    2    “ “ &  8      “                      “
D.    1        “ “                      ,    1    “   “                  &  1  mole  ATP
8. During emergency situation TG hydrolysis by hormone sensitive lipase is stimulated
primarily by which of the following? A
A. glucagon, epinephrine & ACTH C. Thyroid hormone , insulin
B. growth hormone, ACTH & insulin D. glucagon and or insulin & ACTH
9. The rate limiting step in fatty acid synthesis is catalyzed by : C
A. fatty synthase C. acetyl CoA carboxylase
B. acyl carnitine transferase D. ketoacyl transferase
10. Hydrolysis of phospholipid by phospholipase A2 produces: B
A. 3 moles of fatty acid + glycerol C. 2 moles fatty acid + phosphatidic acid
B. lysophospholipid + fatty acid D. plasmalogen + fatty acid
11. Esters of fatty acid with glycerol as the alcohol:
A. sphingolipids B. phospholipids C. triglycerides D. waxes
12. The major product of starch hydrolysis by amylases include: D
A. maltose, lactose and limit dextrans
B. maltriose, lactose and glucose
C. glucose, maltose and maltotriose
D. maltose, maltotriose , limit dextrans

13. In anaerobic glycolysis, 2 moles of inorganic PO4 are used per mole of glucose
consumed. Which of the following enzymes catalyzes the uptake of Pi (inorganic
phosphate? B
A. hexokinase C. phosphofructokinase
B. glycerldehyde-3-PO4 dehydrogenase D. pyruvatae kinase
14. Each of the following substances are important substrates for gluconeogenesis during
fasting, EXCEPT:
A. glycerol B. acetyl CoA C. amino acids D. pyruvate
15.  A  patient  is  diagnosed  as  having  von  Gierke’s  disease.  This  patient  is  likely  to  exhibit  the
following symptoms, EXCEPT: B
A. an enlarged liver C. hypoglycemia
B. metabolic alkalosis D. hyperlipidemia
16. Glucose-6-PO4 is a substrate in the following pathways, EXCEPT: C
A. gluconeogenesis C. oxidative phosphorylation
B. glycogenolysis D. pentose phosphate pathway
17. Which of the following compounds is a positive allosteric regulator of the enzyme
Pyruvate carboxylase?
A. ATP B. acetyl CoA C. biotin D. phosphoenopyruvate
18. D-ribose-5-PO4 is formed in the pentose phosphate pathway from: A
A. ribulose-5-PO4 by an isomerase
B. xylulose-5-PO4 by an epimerase
C. interaction of sedoheptulose-7-PO4 and erythrose-4-PO4
D. interaction of xylulose -5-PO4 and erythrose -4-PO4
19. Which enzyme activity in PRPP requires thiamin pyrophosphate as cofactor? C
A. G-6-PO4 dehydrogenase C. transketolase
B. transaldolase D. phosphopentose isomerase
20. The following are functional significance of uronic acid pathway, EXCEPT: C
A. it is the source of active glucoronate
B. it is a source of active glucose for glycogen synthesis
C. it is a source of ascorbic acid in man
D. it provides for interconversion of hexose to pentose
21. Promotes liver phosphorylase activity:
A. insulin B. glucagon C. ADH D. all of the above
22. This compound transfers reducing equivalents from mitochondria to the cytosol during
gluconeogenesis: D
A. phosphoenolpyruvate C. oxaloacetate
B. glycerol-3-PO4 D. malate
23. All of the following carbon atoms of Glycine, Alanine, Cysteine, Serine & 2 carbon of
threonine form: C
A. directly acetyl CoA C. Pyruvate to acetyl CoA
B. acetoacetyl CoA D. fumarate to acetylCoA
24. This amino acid serves as a key protein-derived glucose precursor:
A. Ala B. Val C. Cys D. Ser
25. The first step in the catabolism of most amino acid is the removal of its amino group by
transamination EXCEPT for this amino acid:
A. Gly B. Ala C. Lys D. Val
26. A heriditary abnormality in tryptophan metabolism characterized by a pellagra-like skin
rash, intermittent cerebellar ataxia and mental retardation B
A. Classic phenylketonuria C. Primary hyperoxaluria
B. Hartnup disease D. Alkaptonuria
27. Human synthesizes adequate amount of all of the following amino acids, EXCEPT:
A. Met B. Gln C. Arg D. Asp
28. The N2 atom of urea are derived from:
A. NH3 & Glu B. Citroline & gln C. Ala & Gly D. NH3 & Glu
29. Beta-alanine is a constituent of:
A. Glutathione B. Carnosine C. FMN D. Biotin

30. Zymogen molecules are converted to the active enzyme as a result of: C
A. activation by ATP C. limited proteolysis
B. enzymatic dephosphorylation D. covalent modification
31. A double reciprocal plot use to determine the Km: C
A. Ping-pong phenomenon C. Lineweaver-burke equation
B. Michaelis –Menten equation D. Reversible inhibition
32. Refers to the multiple form of enzyme in a given specie:
A. Isoenzymes B. zymogen C. proenzyme D. coenzyme
33. In a zero order reaction:
A. half of the active sites of the enzyme are occupied
B. if the substrate is increase with all other factor constant Vmax is increase
C. the rate of the reaction is directly proportional to substrate concentration
D. the rate of enzyme catalyzed reaction is proportional to enzyme concentration
34. The rate controlling step in heme biosynthesis is catalyzed by: B
A. ALA dehydratase C. Heme ferrochelatase
B. ALA synthase D. Protoporphyrin oxidase
35. The most common type of porphyria, pophyria cutanea tarda is characterized by :
A. abdominal pain, absence of photosensitivity
B. presence of urinary porphobilinogen
C. deposition of coproporphyrin in the RBC and heme deficiency
D. absence of hepatic uroporphyrinogen decarboxylase
36. The catabolism of heme at the reticuloendotheial system results in:
A. production of CO and the release of ferric ion D
B. formation of biliverdin which is reduced to bilirubin
C. the consumption of 2 moles of NADPH
D. all of the above
37. A heterogenous group of disorder characterized by compensated hemolysis associated
with unconjugated bilirubin and defect in hepatic uptake of bilirubin:
A.  Gilbert’s  disease C.  Rotor’s  syndrome
B. Dubin-Johnson syndrome D.  Reye’s syndrome
38. Four days old John was suspected of having neonatal physiologic jaundice. Laboratory
test were done. Which of the following is the expected finding: B
A. urine bilirubin (-); fecal UBGN (+), indirect bilirubin trace
B. indirect bilirubin (); urine UBGN(); urine bilrubin (-); fecalUBGN()
C. () fecal UBGN; () urine UBGN ; indirect & direct bilirubin ()
D. () urine URBN; (+) urine bilirubin; normal indirect bilirubin
39. Crigler-Najjar syndrome is an autosomal disorder in humans that is primarily due to
Metabolic defect resulting from inherited absence of the enzyme: C
A. B-glucuronidase C. UDPGTransferase
B. Bilirubin Dismutase D. xanthine oxidase
40. A structural analogue of tRNA, this antibiotic is a potent inhibitor of protein synthesis:
A. streptomycin B. actinomycin C. puromycin D. Rifampicin
41. The following are end products of pyrimidine metabolism, EXCEPT:
A. glutamine B. B-alanine C. ammonia D. B-aminoisobutyric acid
42. The following may act as substrate for orotate phosphoribosyltransferase, EXCEPT: b
A. orotic acid B. OMP C. allopurinol D. PRPP
43. Catalyzes the committed step in purine nucleotide biosynthesis: C
A. glycinamide synthase C. PRPP glutamyl aminotransferase
B. PRPP synthase D. IMP dehydrogenase
44. Translation of mRNA into AA sequence of proteins is terminated by one of the
3 termination codons. The stop codon is recognized by a specific:
A. uncharged tRNA B. protein C. aminoacyl tRNA D. rRNA
45. Degeneracy of the genetic code means that:
A. a given base triplet can code for more than one amino acid
B. there is no punctuation in the code sequence
C. a given amino acid can be coded by more than one base triplet
D. the 3rd base in the codon is not important in coding.
46. The number of high energy phosphate bonds needed to be cleaved to form one peptide
bond during protein synthesis:
A. four B. two C. zero D. three
47. The following factors are needed to initiate translation of mRNA in prokaryotes, EXCEPT:
A. guanosine triphosphate C. 30S ribosomal subunits
B. formylMet tRNA D. cap-binding protein
48. The Okasaki fragments have which of the following properties? B
A. they are double-stranded
B. they contain covalent linked RNA and DNA
C. they are RNA-DNA hybrids
D. they ar eremoved by nuclease activity
49. If GCC is a codon in mRNA, which one of the following would be the anticodon in tRNA?
A. GCC B. CCG C. CGC D. GCG
50. The hydrolytic step leading to release of a polypeptide from a ribosome is catalyzed by:
A. stop codon B. hydrolase C. peptidase D. release factors
51. The main pancreatic duct opens into this part of the duodenum:
A. first part B. third C. second part D. fourth
52. The following are true of the thyroid gland Except:
A. consists of two lobes
B. a vascular organ
C. surrounded by the prevertebral layer of deep fascia (C)
D. its lymph drains mainly into the deep cervical lymph nodes
53. The following describe the parathyroid gland Except:
A. ovoid bodies
B. supplied by the superior thyroid artery
C. supplied by the inferior thyroid artery
D. intimately related to the inferior border of the thyroid cartilage
54. The following describe the pituitary gland Except:
A. lies in the cavity of the sphenoid bone
B. consists of two glands (D)
C. weighs about 0.5 grams
D. its blood supply is derived from the external carotid artery
55. Decreased secretion of hormones from this gland could lead to infertility in both
males and females:
A. anterior lobe, pituitary C. adrenal cortex
B. posterior lobe, pituitary D. adrenal medulla
56. Tumors in this gland can cause sustained or episodic hypertension as a result
of high levels of hormones released:
A. thyroid B. parathyroid C. adrenal cortex D. adrenal medulla
57. Which of these endocrine cell to secretion is NOT right?
A. beta cell > insulin C. fat cell > leptin
B. follicular cells > thyroid hormone D. pituicytes > melatonin
58. Which of these is common to all endocrine organs?
A. parenchyma of epithelium C. fine network of ducts among capillaries
B. stroma of loose connective tissue D. all of the above
59. An individual who has fever experiences chills. This is a form of:
A. negative feedback C. feedforward regulation
B. positive feedback D. acclimatization
60. The life span of a red cell is approximately 120 days. This is because of:
A. adaptation B. acclimatization C. apoptosis D. aging
61. Which of the following second messengers activates calmodulin?
A. diacylglycerol B. calcium C. cyclic AMP D. cyclic GMP
62.Which of the following is a steroid hormone?
A. prolactin C. epinephrine
B. thyroxine D. aldosterone

63.An individual was walking down the street when his enemy suddenly accosted him. He
decided to turn and run for help. This corresponds to which of the following stages of the
general adaptation syndrome?
A. stage of shock C. stage of resistance
B. stage of countershock D. stage of exhaustion
64.A 21 year old female has to walk down a dark alley on her way home. Since there have
been incidences of robbery in this area, she started experiencing tachycardia and
hyperventilation, the moment she started walking down the alley. Sweating was also
evident. These effects are mostly due to activity of the:
A. ACTH axis B. thyroid axis C. vasopressin axis D. sympathetic division
65. The following are true of the vas deferens Except:
A arises from the tail of the epididymis C. also called ejaculatory duct
B. passes through the inguinal canal D. joins the duct of the seminal vesicle
66. The following describe the penis Except:
A. the body consists of three cylinders of erectile tissue
B. the corpus cavernosum expands to form the glans penis
C. the root is formed by the bulb and crura of the penis
D. supplied by the pudendal nerve and the pelvic plexuses
67. The following statements are true Except:
A. the cremasteric fascia of the scrotum is derived from fascia transversalis
B. the left testis usually lies at a lower level than the right
C. the prostate gland lies between the neck of the bladder and the urogenital diaphragm
D. the epididymis is a much coiled tube that lies posterior to the testis
68. The following are contents of the deep perineal pouch in male Except:
A. membranous part of the urethra C. deep transverse perineal muscle
B. sphincter urethrae D. prostate gland
69. Which of the following is NOT correct regarding the male reproductive system?
A. glandular interstitial cells are stimulated by luteinizing hormone
B. sperm motility is essential for fertilization to occur under normal conditions
C. the acrosome forms from the golgi body during the spermatid stage of
spermatogegesis
D. the rate of sperm production by the seminiferous tubules is influenced by the
frequency of ejaculation
70. The following are true regarding the prostate gland, EXCEPT:
A. a compound tubuloalveolar gland C. is traversed by ejaculatory ducts
B. has stroma without smooth muscle cells D. corpora amylacea occur in alveoli
71. Which of the following regarding Sertoli cells is NOT correct?
A. rest on the basal lamina of seminiferous epithelium
B. secrete androgen binding protein
C. under the control of luteinizing hormone
D. contribute to the blood-testis barrier
72.A microscopic slide displays mucosal folds that branch and anastomose with thick
smooth muscles around. The organ is likely to be:
A. seminal vesicle B. prostate gland C. vas deferens D. epididymis
73. Which of the following embryonic structures will differentiate into the penis? B
A. urogenital sinus C. genital swelling
B. genital tubercle D. genital fold
74. Maturation of spermatids into spermatozoa takes place during which of the following
phases of spermatogenesis?
A. spermeation C. spermatocytogenesis
B. meiosis D. spermiogenesis
75. During the ___ phase of spermiogenesis, the acrosomal vesicle flattens and
spreads over the anterior half of the condensing nucleus.
A. Golgi phase C. acrosome phase
B. cap phase D. maturation phase

76.Semi-spastic contraction of facial muscles occurs during which of the following


phases of the male sexual response cycle?
A. excitement phase C. orgasmic phase
B. plateau phase D. resolution phase
77. Which of the following hormones is mainly responsible for male-pattern balding?
A. testosterone C. estradiol
B. dihydrotestosterone D. androstenedione
78. Hypogonadism associated with anosmia and unilateral renal agenesis is called:
A. mosaicism C.  Klinefelter’s  syndrome
B. XYY D. Kallmann syndrome
79.The following are true of the fallopian tube Except:
A. connects the peritoneal cavity with the cavity of the of the uterus
B. the infundibulum is the widest part of the tube
C. lies in the upper border of the broad ligament
D. measures about 10 cm long
80.The following describe the uterus Except:
A. divided into body, fundus, and cervix
B. the body is related posteriorly to the pouch of Douglas
C. in most women the normal position is retroversion (C)
D. supplied by the uterine artery, a branch of the internal iliac artery
81.The following ligaments play an important part in supporting the uterus and
keeping the cervix in position Except:
A. round ligament C. Cardinal ligament (A)
B. pubocervical ligament D. sacrocervical ligament
82. An organ presenting a mucosa having simple columnar epithelium with some cilia,
without glands in its lamina propria is likely to be:
A. vagina B. cervix C. uterus D. oviduct
83. Which of the following is correct regarding the endometrium at the proliferative phase of
the menstrual cycle?
A. stromal cells in active division C. disappearance of spiral artery
B. swollen lamina propria D. all of the above
84. Which of these is NOT correct regarding oogenesis?
A. the development of oogonia to mature egg cell
B. starting before birth, primordial follicles with primary oocyte grow
C. during ovulation, the first meiotic division is done forming secondary oocyte; polar body
D. secondary oocyte completes second meiotic division without fertilization
85. The characteristic cervical secretion during ovulation is:
A. clear and watery C. clear and mucoid
B. whitish and thick D. whitish and mucoid
86. Which of these statements is NOT correct?
A. fertilization normally occurs at the ampulla of the oviduct
B. the fertilized embryo is a blastocyst when implanted
C. implantation occurs at the upper anterior or posterior wall of the uterus body
D. the uterus is supportive of the growing embryo at proliferative phase of endometrium
87. Development of the breasts in females at age 8 in females is called: C
A. menarche B. pubarche C. thelarche D. puberty
88. Basal cells of the glands of the stratum basalis begin to proliferate during which of
the following phases of the endometrial cycle? B
A. pre-menstrual phase C. proliferative phase
B. menstrual phase D. secretory phase
89. Which of the following cells is responsible for conversion of androgen to estradiol
and secretion of progesterone? D
A. theca interna cells C. theca lutein cells
B. granulosa cells D. granulosa lutein cells

90. In a 28 day cycle, progressive increase in estrogen is seen during which of the
following phases? B
A. menstrual phase C. ovulation phase
B. follicular phase D. secretory phase
91. A change in color of the labia minora from bright red to burgundy occurs at which of
the following phases of the female sexual response cycle? B
A. excitement phase C. orgasmic phase
B. plateau phase D. resolution phase
92. Estrogen dominance may be characterized by the following, EXCEPT: A
A. hyperglycemia C. depression
B. water retention D. decreased sex drive
93. Which of these placental components would be gone by the late third trimester of
pregnancy?
A. cytotrophoblasts C. syncytiotrophoblasts
B. capillaries D. mesenchyme
94.  In  what  order  would  an  oxygen  molecule  from  Mom’s  blood  encounter  the  following
placental components as the oxygen makes its way to a first-trimester fetus: 1.) capillary
bed, 2.) chorionic plate, 3.) cytotrophoblast, 4.) mesencymal-like connective tissue,
5.) syncytiotrophoblast
A. 1,2,3,4,5 C. 3,1,2,5,4
B. 5,3,4,1,2 D. 2,4,3,1,5
95. Which change in the mammary gland normally associated with the first trimester of
pregnancy is most likely to also vary with the menstrual cycle?
A. ducts sprout and form new branches C. milk dilates alveoli
B. interlobular connective tissue decreases D. colostrums dilates alveoli
96. Which of the following hormones is secreted as early as 9 days after fertilization?
A. human chorionic gonadotropin C. estriol
B. human placental lactogen D. progesterone
97. Which of the following is a presumptive evidence of pregnancy?
A.  Goodell’s  sign C. mastodynia
B.  Hegar’s  sign D. ballottment
98. A 24 year old female consulted an obstetrician because of amenorrhea. Her last
normal menstrual period was Jan. 20, 2006. Assuming she has a normal pregnancy,
her expected date of confinement is:
A. Oct. 27, 2007 C. Sept. 27, 2007
B. Nov. 27, 2007 D. Dec. 27, 2007
99. In preparing mothers for breastfeeding, which of the following is recommended if the
nipples are flat or inverted?
A. expose the nipples to air C. use well-fitted support bras
B. perform Hoffman maneuver D. rub nipples briskly with a towel
100. Which of the following methods of fertility control takes into consideration the
character of cervical mucus as indicative of the fertile period?
A. post-coital douche C.  Billing’s  method
B. calendar rhythm D. temperature rhythm

XAVIER UNIVERSITY – DR JOSE P RIZAL COLLEGE OF MEDICINE


BASIC BIOMEDICAL SCIENCES

Final Exam: BLOCK 4 (Modules 30 – 36 & Special Topics) March 15, 2005
Write the letter of the BEST answer on the answer sheet. Use capital letters in ink.
NO ERASURES/SUPERIMPOSITIONS!
1. The following are true regarding the spinal cord, EXCEPT:
A. ends at lower border of L1 vertebral level in the adult
B. supported laterally by the denticulate ligaments
C. segments are in line with corresponding vertebral level
D. a bridge between the brain and peripheral nervous system
2. The neck is innervated by dorsal roots from which spinal cord segment?
A. C 2 C. C 4
B. C 3 D. all of the above
3. Control of muscle force requires participation of all of the following, EXCEPT:
A. recruitment of alpha motor neurons C. Renshaw cell
B. golgi tendon organ D. intermediolateral cell column
4. Which of these structure to role association is NOT correct ?
A. ventral spinothalamic tract > light touch sensation
B. fasciculus gracilis > tactile discrimination of upper body parts
C. zone of Lissauer > pain and temperature sensation
D. reticulospinal tract > relay of autonomic information to preganglionic neurons
5. Contralateral hemiplegia is a likely presentation with lesions in these areas, EXCEPT:
A. lateral funiculus C. pyramid
B. pars basalis D. precentral gyrus
6. With a lesion on the posterior funiculus, which function is likely impaired?
A. pain and temperature sensation C. tactile discrimination
B. movement of body parts D. unconscious propioception
7. The syndrome of flaccid paralysis, hyporeflexia, muscle atrophy is least likely due to a
lesion in the:
A. anterior horn C. facial colliculus
B. internal capsule D. midbrain tectum
8. A 50 year old hypertensive male developed sudden facial weakness. The right lower half of
the face sagged, but he could wrinkle his forehead symmetrically. Which of these is true?
A. facial weakness, peripheral type C. a lesion of facial motor nucleus
B. a lesion of the right facial nerve D. an upper motor neuron lesion
9. When an agonist muscle is stimulated, the action of the antagonist muscle is inhibited.
This is based on which of the following? C
A. Bell-Magendie law C. reciprocal innervation
B. autogenic inhibition D. central delay
10. The crossed extensor response is part of which of the following reflexes? B
A. stretch reflex C. inverse stretch reflex
B. withdrawal reflex D. monosynaptic reflex
11. Receptors found in the carotid sinus are examples of: D
A. nociceptors C. exteroceptors
B. proprioceptors D. interoceptors
12. Position sense is detected by:
A. mechanoreceptors C. interoceptors
B. proprioceptors D. nociceptors
13. The cranial nerve that emerges at the ventral midpons:c
A. III C. V
B. IV D. VI
14. The following elevations are found at the level of the medulla, EXCEPT:b
A. olivary body C. clava
B. colliculus facialis D. tuberculum cuneatum

15. The structure not seen in a cross-section of the lower midbrain: a


A. red nucleus C. substancia nigra
B. medial lemniscus D. cerebral aqueduct
16. Nuclei located in the central gray of the brain stem:c
A. nuclei of superior colliculi C. trochlear nuclei
B. hypoglossal nuclei D. all of the above
17. The following are classified as general somatic afferent nuclei, EXCEPT: d
A. spinal nucleus of V C. main sensory nucleus of V
B. mesencephalic nucleus of V D. vestibular nucleus
18. The following are observed in a lesion at the level of the mibrain , right side,
affecting the pyramidal tract and the emerging cranial nerve, EXCEPT: b
A. non-reactive right pupil C. ptosis of right eyelid
B. right hemiplegia D. no exception mentioned
19. A 60-year old hypertensive female developed suddenfacial weakness. The right lower
half of her face sagged but she could wrinkle her forehead symmetrically. Which is the
likely site of lesion? c
A. right facial nerve C. left corticobulbar tract
B. right corticospinal tract D. left facial motor nucleus
20. Muscle spindles serve as receptors of which of the following postural reflexes?
A. neck righting reflex C. tonic neck reflex
B. body on head righting reflex D. negative supporting reaction
21. A biologic rhythm that occurs every week is called: C
A. circadian C. circaseptan
B. circatrigintan D. ultradian
22. Which of the following components of the internal clock can bring about phase shifts? D
A. suprachiasmatic nucleus C. retina
B. intergeniculate leaflet D. pineal gland
23. Which of the following hormones will have peak levels at night?
A. cortisol C. prolactin
B. ACTH D. FSH
24. Which of the following waves in the ECG characterizes the awake state? B
A. alpha C. theta
B. beta D. delta
25. The awake state is made possible by activity of which of the following? D
A. serotoninergic nuclei of pontine raphe
B. gigantocellular tegmental field
C. nucleus ceruleus
D. ascending reticuloactivating system
26. Irresistible daytime sleepiness associated with increased appetite for food and sex is
known as: C
A. narcolepsy C. Kleine-Levin syndrome
B. periodic hypersomnia D. neutral state syndrome
27. Which of the following cerebellar lobes receive inputs from the spinocerebellar pathways?
A. Flocculonodular C. Anterior
B. Posterior D. Central
28. The following are functions of the vestibulocerebellum, EXCEPT:
A. fine tuning of postural reflexes C. inhibits the vestibule-ocular reflex
B. unconscious proprioception D. regulates reflex gain
29. Which of the following is NOT part of the granular layer?
A. Golgi type II cells C. granule cell axon terminals
B. Glomerulus D. mossy fibers
30. The following are true regarding Purkinje cells, EXCEPT:
A. dendrites lie in the molecular layer
B. axons send information to deep cerebellar nuclei
C. use GABA as neurotransmitter
D. has excitatory effect on its target

31. A 10 year old boy presented with unsteady stance and tendency to fall forward and
backward. The most likely are involved is the:
A. cerebellar vermis C. right cerebellar peduncle
B. right red nucleus D. left brachium conjunctivum
32. In the presence of dysmetria, executive function impairment, which of these cerebellar
structures is likely involved?
A. anterior lobe C. dentate nucleus
B. vermis D. fastigial nuclei
33. A cerebellar stroke could result to the following, EXCEPT:
A. asynergia C. dysarthria
B. emotionless face D. intention tremors
34.Which of the following regarding cerebellar connections is NOT correct?
A. cerebral cortex influences contralateral posterior cerebellar lobe
B. brachium conjunctivum projects to ipsilateral cerebral cortex
C. brachium pontis receives crossed afferents from contralateral pontine nuclei
D. restiform body contain afferent projections from body to cerebellum
35. The involvement of the cerebellum on cognitive processing is via:
A. motor association areas > lateral cerebellar hemisphere
B. lateral cerebellar hemisphere > Area 4
C. pyramidal tract collaterals > intermediate part, cerebellum
D. cerebellum > thalamocortical fibers > prefrontal cortex
36. The following tests use imaging dyes, EXCEPT:
A. CT scan C. Cerebral angiography
B. PET D. Doppler ultrasound
37. The middle cerebral artery supplies these cerebral parts, EXCEPT:
A. inferior part, temporal lobe C. lateral surface, parietal lobe
B. lateral surface, temporal lobe D. lateral surface, frontal lobe
38. Which of these anatomic structure to role association is NOT right?
A. anterior frontal gyrus > cognition, social & moral sense
B. pars triangularis > production of words in language
C. inferior parietal lobule > comprehension of symbols/words in language
D. posterior part, superior temporal gyrus > audition
39. The arcuate fasciculus links the sensory language cortical  areas  to  the  Broca’s  area. It is
a kind of:
A. projection fiber C. association fiber
B. commissural fiber D. both B and C
40. In which layer of the cerebral cortex lie the cell bodies of the descending motor
pathways?
A. internal pyramidal C. external pyramidal
B. internal granular D. external granular
41. An obstruction to the foramen of Monroe is likely to produce dilatation of:
A. lateral ventricle C. 4th ventricle
B. 3rd ventricle D. all of the above
42.The following are true regarding the basal ganglia, EXCEPT:
A. a collection of nuclei in the white matter of the cerebral hemisphere
B. its afferent connections come from cerebral cortex, thalamus, substantia nigra
C. its efferent connections mostly come from globus pallidus and substantia nigra
D. project only to the brain stem and spinal cord
43.Which part of the basal ganglia contribute to the limbic system as part of the pleasure
center
A. nucleus accumbens C. globus pallidus
B. caudate nucleus D. putamen
44.The following are suggestive of basal ganglia disease, EXCEPT:
A. bradykinesia C. rigidity of limbs
B. tremors D. emotionally expressive

45. Which of the following diencephalon structure to role association is NOT correct?
A. ventral lateral nucleus > cerebellar influence relay on cerebral cortex
B. supraoptic, paraventricular nucleus > vasopressin, oxytocin secretion
C. suprachiasmatic nucleus > sexual behavior regulation
D. anterior, posterior hypothalamus > body temperature regulation
46. A 70 year old man who displays uncontrollable flinging movements of his right upper limb
is suspected to have a stroke; most likely the lesion is in the left:
A. subthalamic nucleus C. cerebellar hemisphere
B. motor area D. dorsolateral medulla
47. The Papez circuit acting In harmonic fashion is responsible for the central functions of
emotion, as well as for its peripheral expressions. It is inclusive of these, EXCEPT:
A. cingulate gyrus > cingulum > hippocampus
B. hippocampus > fornix > hypothalamus, mammilary body
C. mammilary body > mammillothalamic tract > anterior thalamic nucleus
D. anterior thalamic nucleus > ventral striatum
48. A motor convulsive seizure that spreads from the face to arm to leg likely originates in:
A. alpha motor neurons in spinal cord C. supplementary motor cortex
B. brain stem D. primary motor cortex
49. A 50 year old stroke victim developed alexia, finger agnotia, agraphia without motor
weakness.  He  likely  sustained  a  lesion  in  Brodmann’s:        
A. Area 44, right C. Area 4, right
B. Area 39, left D. Area 4, left
50. Which of the following aspects of emotion involves mental processes? C
A. cognition C. conation
B. affect D. physical change
51. Emotional responses are most likely to be exaggerated in which of the following animals?
A. spinal animal C. decerebrate animal
B. decorticate animal D. decerebellate animal
52. Antigravity reflexes are most likely to be exaggerated in which of the following animals?
A. spinal animal C. decerebrate animal
B. decorticate animal D. decerebellate animal
53. Initiative and memory is retained in which of the following animals? D
A. spinal animal C. decerebrate animal
B. decorticate animal D. decerebellate animal
54. Processing of language which determines whether a sound is a word or not is believed to
take place at the: B
A. inferior gyrus of frontal lobe C. middle temporal gyrus
B. left temporal lobe D. superior temporal gyrus
55. When a person makes use of a descriptive phrase to substitute for a word which he
cannot recall, he might have: C
A. verbal aphasia C. nominal aphasia
B. syntactical aphasia D. semantic aphasia
56. The end product of motivated behavior, whether satisfactory or not is: D
A. learning B. memory C. speech D. emotion
57. The pathway for proprioception from the leg includes the following, EXCEPT:
A. fasciculus cuneatus C. medial lemniscus
B. internal arcuate fibers D. posterolateral ventral nucleus
58. These sensory functions end in the parietal lobe of the cerebrum, EXCEPT:
A. proprioception C. vision
B. vibratory sense D. pain & temperature sense
59. The following pathways decussate at the level of the medulla, EXCEPT:
A. proprioception from body C. pain & temperature from face
B. light touch & pressure from body D. no exception mentioned
60. The following basic principles of sensation are applicable to receptors, EXCEPT:
A. law of appropriate stimulus C. adaptation
B.  Mueller’s  doctrine   D. contrast

61. The most important factor that influences reaction to pain is the: C
A. personality of the sufferer C. significance of the pain
B. age of the sufferer D. nature of the pain
62. In examination of a peripheral neuropathy, which of the following involves
examination of
small unmyelinated fibers? D
A. light touch C. vibration sense
B. two point discrimination D. temperature perception
63. The sensory nerve to the eyeball passes through what opening of the skull?
A. optic canal C. superior orbital fissure
B. foramen ovale D. inferior orbital fissure
64. The middle ear communicates with the nasopharynx through which of the following?
A. mastoid antrum C. Eustachian tube
B. external auditory meatus D. internal auditory meatus
65. Which of the following extraocular muscles is supplied by CN VI (motor nerve supply)
A. medial rectus C. superior oblique
B. lateral rectus D. inferior oblique
66. Which of the following regarding the cochlea is NOT correct?
A. a bony labrinth C. contains endolymph
B. houses the cochlear duct D. spiral with central modiolus
67. Deafness in an airline mechanic who refused to wear earplug on the runway is
more likely to reflect:
A. perforation of the oval window C. damage of auditory nerve fibers
B. rupture of the eardrum D. loss of hair cell sterocilia
68. The following eye structure to role associations are correct, EXCEPT:
A. inner nonpigmented ciliary epithelium > aqueous humor secretion
B. choroid > vascular support for corneosclera
C. vitreous body > provides support, shape to eyeball
D. cornea > avascular refractive medium
69. Which of these retinal layer and contents is NOT correct?
A. photoreceptor layer > cell bodies of rods and cones
B.  inner  nuclear  layer  >  cell  bodies  of  bipolar,  horizontal,  amacrine,  Mueller’s  cells
C. ganglion cell layer > cell bodies of ganglion cells
D. nerve fiber layer > axons of ganglion cells

Questions 70-71
A patient came to the emergency room complaining of left eye blindness. Light
perception & finger counting tests were absent on the affected eye. Pupillary light reflex test
showed non-reactive pupils of both eyes when light was shone on the affected eye.

70. What is the possible visual field defect?


A. temporal hemianopsia, left C. Heteronymous hemianopsia
B. nasal hemianopsia, left D. Blindness, left eye
71. Which part of the visual pathway is affected?
A. optic nerve, right C. edge of optic chiasma. left
B. optic nerve, left D. optic tract, right
72. Which of the following layers of the retina is most important in preventing degradation of
the formed image by absorbing any stray light and internal reflections? C
A. internal limiting membrane C. pigment epithelium
B. external limiting membrane D. photoreceptor layer
73. The following are true as regards rods and cones, EXCEPT: D
A. the fovea centralis does not contain blue cones
B. the peripheral retina contains rods exclusively
C. both rods and cones contain light-sensitive pigments
D. rods resolve fine image detail

74. The basic of color television is:


A. flicker fusion frequency C. monocular parallax
B. color fusion D. binocular parallax
75. An individual confuses brown and orange, but distinguishes purple and yellow. He is most
probably a: B
A. protanope C. tritanope
B. deuteranope D. monochromat
76. Integration of disparate images of an object as seen by both eyes is called: D
A. flicker fusion frequency C. monocular parallax
B. color fusion D. binocular parallax
77. The rostral continuation of the fibers forming the trapezoid body:
A. auditory radiation C. lateral lemniscus
C. medial lemniscus D. lateral acoustic stria
78. The following cranial nerves mediate taste sensation, EXCEPT:
A. VII C. X
B. IX D, XI
79. These sensory pathways pass through the thalamus before reaching their primary cortical
centers, EXCEPT:
A. visual C. gustatory
B. olfactory D. auditory
80. The organ responsible for matching sound impedance in the middle and inner ears:
A. tympanic membrane C. ear ossicles
B. oval window D. inner ear fluid
81. Which of the following is NOT true of nystagmus?
A. initiated by visual impulses
B. slow component is opposite in direction to head rotation
C. quick component of same direction as head rotation
D. direction of nystagmus is determined by direction of quick component
82. The vestibulo-ocular reflex in response to head turning to the left in the upright posture
involves these structures:
A. horizontal semicircular ducts C. utricles
B. vertical semicircular ducts D. saccules
83. Juan refuses to talk to his wife until she follows his wishes. This action constitute:
A. physical abuse C. social abuse
B. sexual abuse D. psychological abuse
84. Domestic violence is also known as:
A. wife battering C. woman abuse
B. marital violence D. all of the above
85. A married couple was having an argument. She felt his anger rising so she stopped
arguing because she felt that at any time he would begin to hit her with his fist. He
had done this many times in the past. In the cycle of violence this is called the:
A. build-up phase C. violent phase
B. pursuit phase D. remorse phase
86. Juan often shouts at his elderly mother every time she refuses to take her medicines. He
tells her to obey or else he would leave her. This behavior of Juan constitutes:
A. physical abuse C. psychological abuse
B. financial abuse D. sexual abuse
87. According to this theory of aging, with advancing age, cells die and are not replaced: B
A. biologic clock theory C. wear and tear theory
B. apoptosis theory D. rate of living theory
88. The following are changes associated with aging, EXCEPT: C
A. increase in exposure to carcinogens
B. increase in bronchial secretions
C. increase in parietal cell mass
D. decrease in muscle strength

89. The most common manifestation of type I decompression sickness is: B


A. skin rash C. hearing loss
B. limb pain D. chest pain
90. At an alveolar P O2 of 30 mm Hg, one is likely to experience: D
A. impaired judgment C. personality changes
B. mild tremors D. unconsciousness
91. Which of the following is considered as a life-threatening emergency? B
A. high altitude pulmonary edema
B. high altitude cerebral edema
C. high altitude retinal hemorrhage
D. acute mountain sickness
92. The following are expected to occur in aerospace, EXCEPT:
A. decalcification of bone C. headward shift of fluid
B. muscle atrophy D. increase in red cell mass
93. An increase in the efficiency of the cardiovascular system and skeletal muscles as a
result of exercise is called: A
A. adaptation C. conditioning
B. endurance D. fitness
94. Which of the following sports is most likely to involve the anaerobic system as its principal
energy source? B
A. 100 meter dash C. 200 meter swim
B. 100 meter swim D. 1 mile run
95. The following are specific benefits of exercise, EXCEPT: D
A. heart beat becomes stronger C. reduction of chronic fatigue
B. increased endurance D. shallower breathing
96. One of the following exercises is designed to improve circulation:
A. knee lift C. skipping rope
B. sprinter D. push-up
97. Entrance of material into short term memory occurs in which of the following phases of
learning? B
A. apprehension phase C. recall phase
B. acquisition phase D. retention phase
98. The ability of an individual to discriminate facts is part of: A
A. apprehension phase C. recall phase
B. acquisition phase D. retention phase
99. Photographic memory is also known as:
A. short-term memory C. eidetic memory
B. long-term memory D. iconic memory
100. When an individual who recalls a past experience includes present data, the condition is
called: B
A. paramnesia C. anterograde amnesia
B. confabulation D. retrograde amnesia

XAVIER UNIVERSITY – JOSE P RIZAL COLLEGE OF MEDICINE


Basic Biomedical Sciences
Second Bimonthly Period
QUIZ # 4 (CVS 2)

NAME ____________________________ 22 September 2009 SCORE


____________
MULTIPLE CHOICE: write the letter of the BEST answer on the answer sheet. Use capital
letters in blue/black ink. NO EASURES/SUPERIMPOSITIONS ALLOWED!

1. This is true of central venous pressure (CVP)


A. average pressure is about 10 mmHg
B. refers to the pressure in the right and left atria
C. CVP is a little higher than peripheral venous pressure (PVP)
D. main determinant is the volume of blood in the right atrium
2. CVP tends to increase in which of these conditions?
A. Decreased PVP
B. Decreased heart rate
C. Increased venous return (VR)
D. increased right ventricular force of contraction
3. Which of the following occurs when a person rises from a lying to an upright posture?
A. CVP remains the same C. veins in skull collapse
B. Neck veins distend D. PVP in arms is higher than in legs
4. An increase in which of the following tends to increase venous return?
A. CVP C. mean systemic filling pressure
B. venous dilatation D. arteriolar constriction
5. Peripheral veins can accommodate large amounts of blood with relatively small
increases in PVP because:
A. Venomotor tone is high
B. Walls of veins are highly distensible
C. veins contain 65% of the total blood volume
D. the pressure of an empty vein is practically zero
6. Capillary filtration is enhanced in which of these conditions?
A. hypertension
B. increased plasma proteins
C. increased interstitial hydrostatic pressure
D. constriction of precapillary sphincter
7. The main form of transport across the capillary wall is by:
A. diffusion C. filtration
B. facilitated transport D. transcytosis
8. Lymph flow tends to increase in which of these conditions?
A. decreased capillary permeability
B. increased capillary hydrostatic pressure
C. increased plasma colloid osmotic pressure
D. decreased activity of the lymphatic pump
9. This is true of the lymphatic vascular system.
A. begins in the heart and ends in the heart
B. lymph passes through at least one or more lymph nodes
C. lymph is finally drained into the right and left axillary veins
D. lymph from right & left chest drains into the thoracic duct
10. When the force of contraction of the right ventricle is abnormally diminished (right
ventricular failure), which of the following occurs?
A.

19. During a physiology exercise, Rocky did the Valsalva maneuver. His group mates
noticed that the changes in his BP and heart rate were as expected. Which of the
following are the expected results during the maneuver?
A. increased BP, increased heart rate
B. increased BP, decreased heart rate
C. decreased BP, increased heart rate
D. decreased BP, decreased heart rate

Finals, Block 4 - 2006

Mod 30 – spinal cord (8Qs)

1. The following are true regarding the spinal cord, EXCEPT:


A. ends at lower border of L1 vertebral level in the adult
B. supported laterally by the denticulate ligaments
C. segments are in line with corresponding vertebral level
D. a bridge between the brain and peripheral nervous system
2. The neck is innervated by dorsal roots from which spinal cord segment?
A. C 2 C. C 4
B. C 3 D. all of the above
3. Control of muscle force requires participation of all of the following, EXCEPT:
A. recruitment of alpha motor neurons C. Renshaw cell
B. golgi tendon organ D. intermediolateral cell column
4. Which of these structure to role association is NOT correct ?
A. ventral spinothalamic tract > light touch sensation
B. fasciculus gracilis > tactile discrimination of upper body parts
C. zone of Lissauer > pain and temperature sensation
D. reticulospinal tract > relay of autonomic information to preganglionic neurons
5. Contralateral hemiplegia is a likely presentation with lesions in these areas, EXCEPT:
A. lateral funiculus C. pyramid
B. pars basalis D. precentral gyrus
6. With a lesion on the posterior funiculus, which function is likely impaired?
A. pain and temperature sensation C. tactile discrimination
B. movement of body parts D. unconscious propioception
7. The syndrome of flaccid paralysis, hyporeflexia, muscle atrophy is least likely due to a
lesion in the:
A. anterior horn C. facial colliculus
B. internal capsule D. midbrain tectum
8. A 50 year old hypertensive male developed sudden facial weakness. The right lower half of
the face sagged, but he could wrinkle his forehead symmetrically. Which of these is true?
A. facial weakness, peripheral type C. a lesion of facial motor nucleus
B. a lesion of the right facial nerve D. an upper motor neuron lesion

Mod 33 – cerebrum (13 Qs)

1. The middle cerebral artery supplies these cerebral parts, EXCEPT:


A. inferior part, temporal lobe C. lateral surface, parietal lobe
B. lateral surface, temporal lobe D. lateral surface, frontal lobe
2. Which of these anatomic structure to role association is NOT right?
A. anterior frontal gyrus > cognition, social & moral sense
B. pars triangularis > production of words in language
C. inferior parietal lobule > comprehension of symbols/words in language
D. posterior part, superior temporal gyrus > audition
3.  The  arcuate  fasciculus  links  the  sensory  language  cortical  areas  to  the  Broca’s  area.  It  is  a
kind of:
A. projection fiber C. association fiber
B. commissural fiber D. both B and C
4. In which layer of the cerebral cortex lie the cell bodies of the descending motor pathways?
A. internal pyramidal C. external pyramidal
B. internal granular D. external granular
5. An obstruction to the foramen of Monroe is likely to produce dilatation of:
A. lateral ventricle C. 4th ventricle
rd
B. 3 ventricle D. all of the above
6. The following are true regarding the basal ganglia, EXCEPT:
A. a collection of nuclei in the white matter of the cerebral hemisphere
B. its afferent connections come from cerebral cortex, thalamus, substantia nigra
C. its efferent connections mostly come from globus pallidus and substantia nigra
D. project only to the brain stem and spinal cord
7. Which part of the basal ganglia contribute to the limbic system as part of the pleasure
center
A. nucleus accumbens C. globus pallidus
B. caudate nucleus D. putamen
8. The following are suggestive of basal ganglia disease, EXCEPT:
A. bradykinesia C. rigidity of limbs
B. tremors D. emotionally expressive
9. Which of the following diencephalon structure to role association is NOT correct?
A. ventral lateral nucleus > cerebellar influence relay on cerebral cortex
B. supraoptic, paraventricular nucleus > vasopressin, oxytocin secretion
C. suprachiasmatic nucleus > sexual behavior regulation
D. anterior, posterior hypothalamus > body temperature regulation
10. A 70 year old man who displays uncontrollable flinging movements of his right upper limb
is suspected to have a stroke; most likely the lesion is in the left:
A. subthalamic nucleus C. cerebellar hemisphere
B. motor area D. dorsolateral medulla
11. The Papez circuit acting In harmonic fashion is responsible for the central functions of
emotion, as well as for its peripheral expressions. It is inclusive of these, EXCEPT:
A. cingulate gyrus > cingulum > hippocampus
B. hippocampus > fornix > hypothalamus, mammilary body
C. mammilary body > mammillothalamic tract > anterior thalamic nucleus
D. anterior thalamic nucleus > ventral striatum
12. A motor convulsive seizure that spreads from the face to arm to leg likely originates in:
A. alpha motor neurons in spinal cord C. supplementary motor cortex
B. brain stem D. primary motor cortex
13. A 50 year old stroke victim developed alexia, finger agnotia, agraphia without motor
weakness.  He  likely  sustained  a  lesion  in  Brodmann’s:        
A. Area 44, right C. Area 4, right
B. Area 39, left D. Area 4, left

Xavier University – Dr Jose P Rizal College of Medicine


Basic Biomedical Sciences – First Bimonthly Exam
Set A: Modules 1 – 6

NAME ________________________ August 7, 2006 SY 2006 – 07

MULTIPLE CHOICE: Write the letter of the BEST answer in CAPITAL letter in ink on
the answer sheet. NO ERASURES/SUPERIMPOSITIONS ALLOWED!

1. Which of the following is a passive carrier-mediated transport process?


A. osmosis C. fluid-phase pinocytosis
B. facilitated transport D. receptor-mediated pinocytosis
2. The following are true of aqueous channels, EXCEPT:
A. Open all the time
B. made up of transmembrane proteins
C. open wide when electrically stimulated
D. allow passage of molecules less than 7 Angstroms
3. The rate of diffusion of small solutes across the plasma membrane is directly
proportional to the following, EXCEPT:
A. thickness of the membrane C. lipid solubility of the solute
B. surface area of the membrane D. concentration gradient
4. This moves large substances out of the cell without having to pass through the
membrane molecules:
A. exocytosis C. pinocytosis
B. phagocytosis D. endocytosis
5. Which of the following is true only to secondary active transport but not to primary
active transport:
A. requires ATP C. requires an ion concentration gradient
B. requires carrier D. can be saturated and inhibited
6. Which of the following would bring back the membrane to its resting potential from the
undershoot phase of the action potential (AP)?
A. closure of the n gates C. closure of the m gates
B. closure of the h gates D. closure of the h and n gates
7. Which of these is true of the upstroke phase of the AP?
A. membrane polarity usually remains negative
B. begins at the start of stimulation
C. membrane cannot be stimulated
D. due to rapid and large K+ influx
8. An AP at one area in the cell membrane is conducted to all the T-tubules because of
this property:
A. Condenser C. rectifier
B. core conductor D. resistor
9. The following are true of the relative refractory period of a nerve cell, EXCEPT:
A. K+ conductance is low
B. usually, cell cannot fire an AP
C. an AP can occur if applied stimulus is larger than normal
D. occurs during the latter part of the depolarization phase of the AP
10. The oxidation of one mole of glucose by anaerobic glycolysis yields a net of:
A. 2 moles of lactate and 2 moles of ATP
B. 2 moles of lactate, 2moles of FAD, and 2 moles of ATP
C. 2 moles of lactate, 2 moles of NAD+, and 6 moles of ATP
D. 2 moles of pyruvate, 2 moles of NADH, and 4 moles of ATP
11. The terminal electron acceptor in the respiratory chain is:
A. cytochrome a C. coenzyme Q
B. oxygen D. iron sulfur protein
12. The reaction catalyzed by this enzyme in the TCA results in the formation of high
energy compound:
A. citrate synthase C. isocitrate dehydrogenase
B. succinyl CoA synthetase D. succinate dehydrogenase
13. The only oxidation reaction in glycolysis is in the stage of ATP synthesis. this reaction
is catalyzed by:
A. hexokinase
B. phosphohexose isomerase
C. 3-phosphoglycerate kinase
D. glyceraldehyde-3-PO4dehydrogenase
14. As regards inhibitors of mitochondrial ATP synthesis, which of the following
associations in NOT correct?
A. uncoupler - aspirin
B. proton pump inhibitor – cyanide
C. binds to ATP synthase – oligomycin
D. ADP/ATP translocase inhibitor – carbon monoxide
15. A motor unit is composed of:
A. a motor neuron and all the muscle fibers it innervates
B. a motor neuron and one muscle fiber it innervates
C. a motor axon collateral and the muscle fiber it innervates
D. a motor axon collateral and all the muscle fibers it innervates
16. An action potential arrives at the terminal knob of a motor neuron causing
depolarization of the knob. Which of the following events immediately follows?
A. calcium influx into the knob
B. formation of an end plate potential
C. exocytosis and release of acetylcholine
D. fusion of synaptic vesicles with knob membrane
17. The following are true of chemical synapses, EXCEPT:
A. direct propagation of signals from pre- to postsynaptic neuron
B. signal propagation is slower than in electrical synapses
C. signal propagation is usually unidirectional
D. involves activity of neurotransmitters
18. Release of neurotransmitters from an axon terminal of presynaptic neuron is
immediately preceded by:
A. depolarization of the axon terminal
B. calcium influx into the axon terminal
C. opening of voltage-gated calcium channels
D. fusion of synaptic vesicles with terminal knob membrane
19. Which of the following will most likely initiate an AP in the adjacent membrane?
A. excitatory postsynaptic potential (EPSP)
B. inhibitory postsynaptic potential (IPSP)
C. miniature end plate potential (MEPP)
D. end plate potential (EPP)
20. Two excitatory stimuli arrived simultaneously at two different points on the same axon,
and this resulted in a depolarization that is bigger than if there was only one stimulus.
This phenomenon is called:
A. spatial summation C. convergence
B. temporal summation D. divergence
21. Which of the following prevents synaptic transmission by preventing the synthesis of
the transmitter?
A. Hemicholinium C. botulinum toxin
B. Organophosphate D. succinylcholine
22. Which of the following are transmitters at postganglionic sympathetic neurons?
A. epinephrine, norepinephrine, acetylcholine, and dopamine
B. epinephrine, norepinephrine, and acetylcholine
C. epinephrine and norepinephrine
D. epinephrine
23. Which of the following is the electrochemical basis of the development of EPSP?
A. local chloride efflux C. local sodium influx
B. local calcium influx D. local potassium efflux
24. Which of the following is NOT a classical neurotransmitter?
A. dopamine C. epinephrine
B. acetylcholine D. enkephalin
25. The following hormones favor bone formation, EXCEPT:
A. growth hormone C. estrogen
B. parathyroid hormone D. calcitonin
26. The final step in the synthesis of 1,25-dihydroxyvitamin D (calcitriol) from vitamin D2 or
Vitamin D3 occurs in the:
A. liver C. kidneys
B. spleen D. bone
27. When an AP reaches the T-tubule, which of the following immediately follows?
A. stimulation of the calcium pump in the sarcoplamic reticulum (SR)
B. opening of calcium channels in the SR
C. binding of calcium to troponin-C
D. Calcium entry into the cell
28. In the relaxed state of a skeletal muscle fiber, which of the following is true?
A. binding sites in actin are not covered by tropomyosin
B. myosin heads are attached to actin
C. cytosolic calcium level is low
D. calcium is bound to troponin-C
29. During muscular contraction, which of the following would shorten?
A. actin filaments C. sarcomere and myosin filaments
B. sarcomere D. sarcomere and actin filaments

XAVIER UNIVERSITY – DR JOSE P RIZAL COLLEGE OF MEDICINE


BASIC BIOMEDICAL SCIENCES
Final Exam: BLOCK 1 (Modules 1 – 10) March 7, 2006
Write the letter of the BEST answer on the answer sheet. Use capital letters in ink.
NO ERASURES/SUPERIMPOSITIONS ALLOWED!

1. The organelle that is characteristic of steroid secreting cells:


A. smooth endoplasmic reticulum C. lysosomes
B. rough endoplasmic reticulum D. golgi complex
2. In which phase of the cell cycle does DNA synthesis occur?
A. G1 C. G2
B. S D. Go
3. Which of these associations is NOT right?
A. intermediate filaments > cell support C. microbodies > inclusions
B. microfilaments > cell motility D. none
4. This property of the cell membrane allows the spread of excitation from one part to
the rest of the membrane.
A. Semipermeability C. condenser
B. core conductor D. rectifier
5. The Na+ channels involved in the upstroke of the action potential are:
A. aqueous channels C. voltage-gated channels
B. ligand-gated channels D. G protein-gated channels
6. Which of the following processes cannot occur in the absence of ATP?
A. osmosis C. facilitated transport
B. diffusion D. sodium-gludose cotransport
7. When red cells are exposed to a hypertonic solution, which of the following occurs?
A. crenation C. hemolysis
B. swelling D. cell volume remains the same
8. Which of these does NOT describe the membrane at its resting potential?
A. it cannot be stimulated
B. it is negative inside relative to the outside
C. it is more permeable to K+ than to Na+
D. there is no net movement of Na+ and K+ across the membrane
9. The reducing equivalents formed during the course of oxidation of acetyl CoA in the TCA
cycle: D
A. are regenerated to form carbohydrate C. participate in the formation of fatty acid
B. are regenerated to form oxaloacetate D. enter the respiratory chain
10. The oxidation of one mole of glucose by anaerobic glycolysis yields a net of:
A. 2 moles of lactate and 2 moles of ATP A
B. 2 moles of lactate, 2 moles of NADH & 2 moles of ATP
C. 2 moles of lactate, 2 moles of NAD, & 6 moles of ATP
D. 2 moles of pyruvate, 2 moles of lactate, & 2 moles of NADH
11. Which of the following enzymes catalyzes the reaction in the TCA cycle that results in the
formation of high energy intermediate? B
A. isocitrate dehydrogenase C. succinate dehydrogenase
B. succinyl CoA synthetase D. citrate synthase
12. The following are characteristics of transitional epithelium, EXCEPT:
A. multi layered tissue C. capillaries at basal layers
B. large dome shaped superficial cells D. no exception mentioned
13. The following epithelium to role associations are correct, EXCEPT:
A. stratified squamous > protection C. simple squamous>transport of substances
B. simple columnar epithelium > absorption D. no exception mentioned
14. Which epithelial cell specialization is well developed in absorptive cells?
A. microvilli C. demosomes
B. stereocilia D. nexus

15.Which of these is a serous membrane?


A. serous glandular epithelium C. mesothelium with areolar tissue
B. tissue with serous glands D. all of the above
16.This gland pours out its secretion to a body surface; it is:
A. exocrine C. endocrine
B. merocrine D. any of the above
17. Which of the following cell to role associations is NOT right?
A. macrophage > secretion of cytokines, phagocytosis of foreign agents
B. plasma cell > direct cytotoxicity of foreign agents
C. mast cell > liberation of pharmacologically active substances
D. fibroblast > secretion of fibers and ground substance
18. In the adult, the most common connective tissue supporting the organs is:
A. adipose C. dense irregular
B. areolar D. reticular
19. Which glycoprotein is involved in interactions of cells, collagen, and glycosaminoglycans,
mediating normal cell adhesion and migration?
A. laminin C. fibronectin
B. chondronectin D. all of the above
20. The layer of the epidermis that provides for newer cells is stratum:
A. germinativum C. corneum
B. granulosum D. spinosum
21. Which of these epidermal cells is involved in immune responses?
A.  keratinocytes                                                C.  Merkel’s
B.  Langerhans’                                                    D.  melanocytes
22. The reticular layer of the dermis may demonstrate the following, EXCEPT:
A. sebaceous, sweat glands C. hair follicles
B. smooth muscle D. Meissner’s  corpuscle
23. Which of these is NOT related to the hypodermis?
A. areolar and adipose tissue C. small blood vessels and nerves
B. Pacinian corpuscles D. muscle spindle
24. This part of the epidermis is characterized by abundant desmosomes:
A. stratum basale C. stratum granulosum
B. stratum spinosum D. all of the above
25. The following relates to the astrocytes, EXCEPT:
A. blocks the brain against poisons and foreign invaders
B. provides structural support to the brain
C.  secrete  growth  factors  for  neuron’s  maintenance
D. actively phagocytic and produce scar tissue after damage
26. A microscopic section of the gray matter would show the following, EXCEPT:
A. nuclei C. glial cells
B. nerve D. no exception mentioned
27. Most neurons in the body are:
A. pseudounipolar C. multipolar
B. bipolar D. any of the above
28. Which of these is NOT correct as to the neuron?
A. large spherical to angular cell bodies C. Nissl bodies in perikarya
B. rough endoplasmic reticulum in axonal cytoplasm D. euchromatic nucleus
29. In ganglia, the supporting cells of their component nerve fibers is:
A.  Schwann’s C. oligoglia
B. capsule cell D. all of the above
30. These events lead to the production of an EPSP in the postsynaptic neuron, EXCEPT:
A. depolarization of the terminal knob of the presynaptic neuron
B. diffusion of Ca++ into the terminal knob
C. release of neurotransmitter into the synaptic cleft
D. local increase in permeability to K+ at the postsynaptic neuron

31. Synaptic transmission may be blocked by this mechanism:


A. hemicholinium prevents release of acetylcholine
B. botulinum competes with acetylcholine for the receptor
C. organophosphate inhibits degradation of acetylcholine
D. succinylcholine prevents synthesis of acetylcholine
32. Which of the following events immediately precedes exocytosis and release of
acetylcholine at the neuromuscular junction?
A. depolarization of the terminal knob membrane
B. calcium entry into the synaptic vesicles
C. fusion of vesicles with terminal knob membrane
D. opening of voltage-gated sodium channels
33. Which of the following differentiates the somatic nervous system (SNS) from the
autonomic nervous system (ANS)? The SNS: C
A. innervates involuntary structures
B. consists of preganglionic and postganglionic neurons
C. when stimulated, produces excitatory effects
D. when sectioned, leads to paralysis of structure innervated
34.The following are characteristics of the autonomic nervous system in general, EXCEPT:
A. innervates smooth muscles, cardiac muscles and gland cells
B. possesses homeostatic function
C.  involved  in  “fight  or  flight”  responses
D. its transmitter is exclusively acetylcholine
35.The portion of the autonomics concerned with emergency function is the: A
A. sympathetic division C. enteric division
B. parasympathetic division D. cranial outflow
36.Stimulation of postganglionic fibers that exit from the inferior cervical ganglion will
produce: D
A. lacrimation B. mydriasis C. salivation D. tachycardia
37.Stimulation of postganglionic fibers that exit from the hypogastric ganglion will produce:
A. decreased intestinal motility C. ejaculation
B. acetylcholine release D. defecation
38.Stimulation of postganglionic fibers that exit from the ciliary ganglion will produce: B
A. lacrimation B. miosis C. salivation D. bradycardia
39.The following are characteristics of the sympathetic nervous system, EXCEPT: D
A. preganglionic fibers are generally short
B. postganglionic fibers are non-myelinated
C. effect of stimulation is generalized and long-lasting
D. removal is incompatible with life
40.Which of the following is most likely to take place upon stimulation of the vagus?
A. pupilloconstriction C. bronchoconstriction
B. abundant saliva D. erection
41.  What  is  the  “dangerous  area  “  of  the  scalp?
A. aponeurotic layer C.dense connective tissue layer
B.skin D. loose connective tissue layer
42. Which of the following is referred to as  the  “  blowing  “  muscle?    
A. buccinator C. corrugator supercilli
B. zygomaticus D. risorius
43. The following are paired facial bones EXCEPT:
A. maxilla C. inferior concha
B. vomer D. lacrimal
44. One of the following sinuses does not open into the middle nasal meatus:
A.frontal C.maxillary
B.sphenoid D. ethmoid, anterior and middle

45. The following are contents of the carotid sheath EXCEPT:


A.internal jugular vein C.common carotid artery
B. vagus nerve D.phrenic nerve
46. The thyroid gland, larynx. trachea and esophagus are contents of :
A.digastric triangle C.carotid triangle
B.muscular triangle D.submental triangle
47. The following muscle tissue associations are correct, EXCEPT:
A. smooth muscle > fusiform densities
B. skeletal muscle > multinucleated cells
C. cardiac muscle > branching fibers
D. no exception mentioned
48. Which of these is common to all muscle tissues?
A. the type of junctions between muscle cells C. the microfilaments in the sarcoplasm
B. the location of the nucleus in cells D. the amount of sarcoplasm in cells
49. In skeletal muscle contraction, the event which immediately precedes the binding of
myosin heads to actin binding sites is the: D
A. arrival of an action potential in the muscle fiber
B. release of calcium from the sarcoplasmic reticulum
C. binding of calcium with troponin C
D. movement of tropomyosin away from the actin binding sites
50. These structures shorten during muscular contraction, EXCEPT: C
A. H band C. A band
B. I band D. sarcomere
51. Which is a common site of intramuscular injection?
A. biceps brachii C.deltoid
B. latissimus dorsi D.triceps brachii
52. These are lateral rotators of the arm and stabilize shoulder joint EXCEPT:
A. supraspinatus C.infraspinatus
B. teres minor D.teres major
53. Which does not belong to the erector spinae group of muscles?
A spinalis C.iliocostalis
B. semispinalis D. longissimus
54. The following are characteristics of the thoracic vertebra EXCEPT:
A. transverse foramen C. costal facets
B. long pointed spine D. heart-shaped, meduim sized body
55. Which is not a boundary of the triangle of auscultation?
A. scapula , inferior angle C. trapezius
B.latissimus dorsi D. external oblique
56. The following are parts of the scapula EXCEPT:
A.coronoid process C.spine
B.glenoid cavity D.acromion
57. What nerve passes the back of medial epicondyle of the humerus?
A.radial C.ulnar
B. musculocutaneous D.median
58. Median nerve supplies the following anterior compartment muscles of the
forearm EXCEPT:
A. pronator teres C. flexor carpi radialis
B. palmaris longus D.flexor carpi ulnaris
59. What is the common nerve supply to the posterior muscles of forearm?
A. radial C.ulnar
B.median D.musculocutaneous
60. One of the following muscles of the thumb is not supplied by the median nerve:
A. opponens pollicis C. adductor pollicis
B flexor pollicis brevis D. abductor pollicis brevis

61. What is the artery of taking the blood pressure?


A.radial C.brachial
B.ulnar D.axillary
62. Which of the following superficial veins is most stable thereby site of blood extraction?
A.median cubital C.cephalic
B.basilic D. dorsal veno
63. Which of these associations is correct?
A. elastic cartilage > homogenous extracellular matrix
B. fibrocartilage > matrix secreted by fibroblasts
C. cancellous bone > osteoblasts, osteocytes, osteoclasts
D. developing bone > bone trabeculae, hyaline cartilage
64. Which of these forms the structural and functional unit of compact bone?
A. the concentric bone lamellae with central vascular channel
B. the  angular  bone  matrix  with  Volkmann’s  canal
C. the circumferential lamellae beneath the periosteum and endosteum
D. all of the above
65. Which of these is NOT correct regarding bone cells?
A. osteoblasts are basophilic cuboidal cells along bone surfaces
B. osteocytes lie in lacunae and maintain mature bones
C. osteoclasts secrete collagenase for matrix breakdown
D. all the bone cells are present in young and mature bone
66. Calcitriol can influence calcium level by the following mechanisms, EXCEPT: C
A. enhances osteoclastic activity of PTH
B. activates calcium-binding proteins in bone
C. inhibits distal tubular reabsorption of calcium
D. enhances calcium absorption in intestinal tissue
67. Parathyroid hormone causes an increase in the following, EXCEPT: B
A. synthesis of 1,25-dihydroxycholecalciferol
B. phosphate reabsorption in the proximal tubule
C. absorption of calcium from GIT
D. bone resorption
68. Which of these is true of calcium in bone? d
A. only 1% of bone calcium is freely exchangeable with ECF calcium
B. bone calcium constitutes 99% of the total body calcium
C. it forms hydroxyapatite crystals in bone
D. all of the above
69. The following are branches of the posterior cord of brachial plexus EXCEPT:
A. radial C.axillary
B. thoracodorsal D. ulnar
70. The following are inserted into the greater trochanter of femur EXCEPT:
A.quadratus femoris C.piriformis
B. obturator internus D. gemellus superior
71. What is the nerve supply to the gluteus maximus?
A.inferior gluteal C.obturator
B.superior gluteal D.femoral
72. What is the common action of the quadriceps femoris group?
A. flexion of the leg C. extends the thigh
B. extension of the leg D. all of the above
73. One serves as the common origin of the biceps femoris group:
A. ischial tuberosity C. sciatic notch
B. ischial spine D. iliac crest
74. Which of the following can plantar flex and evert the foot?
A.tibialis anterior C.peroneus longus
B. peroneus tertius D. extensor digitorum longus

75. The following form the tendo calcaneus EXCEPT:


A.popliteus C. gastrocnemius
B.plantaris D. soleus
76. The following bones form the knee joint EXCEPT:
A.femur C.patella
B. tibia D.fibula
77. These are classified as hinge type of joints EXCEPT:
A.knee C. wrist
B.ankle D. elbow
78. Which is not a branch of the sacral plexus?
A.sciatic C.pudendal
B. superior gluteal D. obturator
79. First layer muscles of the sole of the foot are the following except:
A. quadratus plantae C. abductor hallucis
B. flexor digitorum brevis D. abductor digiti minimi
80. Contents of the Hunters canal include the following EXCEPT:
A.femoral nerve C.saphenous nerve
B.femoral artery & vein D.nerve to vastus medialis
81. The following are characteristics of neutrophils, EXCEPT:
A. multiple nuclear lobes C. abundant lysosomes
B. coarse lilac specific staining granules D. 70% of circulating leukocytes
82. The following are true of monocytes, EXCEPT:
A. large cell with abundant cytoplasm
B. round or indented nucleus with less condensed chromatin
C. can differentiate into macrophages
D. normally makes up about 30% of total leukocytes
83. Which leukocyte will be elevated in acute bacterial infections?
A. monocyte C. neutrophil
B. lymphocyte D. eosinophil
84. Which of these is NOT right regarding blood cells?
A. few nucleated erythrocytes can sometimes normally occur in peripheral blood
B. the blood platelets are derived from the cytoplasm of megakaryocytes
C. all leukocytes possess cytoplasmic azurophilic granules
D. the granulocytes and nongranulocytes develop from the same precursor cell
85. The earliest cell stage of a basophil is the:
A. myeloblast C. megakaryoblast
B. proerythroblast D. promyelocyte
86. The following are true regarding the bone marrow, EXCEPT:
A. developing blood cells lie in the parenchyma
B. reticular cells secrete its matrix
C. developing bone cells form part of its functional part
D. principal blood forming organ in the adult
87. This interleukin is believed to be responsible for chemotaxis of neutrophils: D
A. IL 5 C. IL 7
B. IL 6 D. IL 8
88. Which of these tests is most useful in determination of the morphologic type of anemia?A
A. red cell indices C. peripheral blood smear
B. hemoglobin and hematocrit D. red cell and white cell counts
89. The rate limiting enzyme, ALAS in heme synthesis requires the active form of this
vitamin:C
A. Niacin C. Pyridoxine
B. Riboflavin D. Biotin
90. Globin synthesis of hemoglobin takes place in the ribosomes of: B
A. eryhtrocytes C. rubriblast
B. rubricytes D. lymphocytes

91. What is normal location of the spleen?


A.right hypochondriac C. right lumbar
B.left hypochondriac D. left lumbar
92. The thymus gland is located in what mediastinum?
A.superior C.middle
B. anterior D.both A & B
93. What is considered as the longest vein of the body?
A.small saphenous C.great saphenous
B.femoral D. popliteal
94. This lymphoid organ is structurally designed for filtration of lymph:
A. spleen C. tonsil
B. lymph node D. thymus
95. Which of these organs demonstrate a lymph nodule with an associated arteriole?
A. thymus C. spleen
B. tonsil D. lymph node
96. Which of these functions is common to all lymphoid organs?
A. blood filtration C. lymphopoiesis
B. lymph filtration D. all of the above
97.Which of the following immediately precedes secretion of interleukin 1 by the antigen-
presenting cell (APC)? C
A. phagocytosis of foreign antigen
B. release of inflammatory mediators
C. epitope is presented to T helper cell
D. processing of epitope by APC
98.Cell-mediated immune response includes the following, EXCEPT: A
A. synthesis of immunoglobulins M and G
B. proliferation of effector T cells
C. determination of response specificity
D. release of cytokines by effector cells
99.When the amount of antigen is greater than the amount of antibody, an antigen-antibody
reaction may not take place because of: D
A. high zone tolerance C. prozone phenomenon
B. low zone tolerance D. postzone phenomenon
100. Down-regulation of immunologic response to forbidden clones is called: C
A. clonal deletion C. clonal anergy
B. clonal abortion D. clona

XAVIER UNIVERSITY – JOSE P RIZAL COLLEGE OF MEDICINE


Basic Biomedical Sciences
Third Bimonthly Period
QUIZ # 4 (Genetics)

NAME ____________________________ 3 Dec 2007 SCORE ____________


MULTIPLE CHOICE: write the letter of the BEST answer on the answer sheet. Use capital
letters in blue/black ink. NO EASURES/SUPERIMPOSITIONS ALLOWED!

1. Many codons may decode for the same amino acid. This is because the genetic code
is:
A. Unambiguous C. degenerate
B. Universal D. nonoverlapping
2. Which of the following occurs during protein synthesis?
A. message  in  mRNA  is  read  from  3’  to  5’  direction
B. translation  begins  near  the  5’  terminal  of  the  mRNA
C. free amino acids attach to corresponding codons in mRNA
D. tRNA dictates the amino acid sequence of the protein to be synthesized
3. Which of these is the first step in the initiation phase of translation?
A. formation of the 43S preinitiation complex
B. formation of the 43S initiation complex
C. formation of the 80S initiation complex
D. dissociation of the ribosomes into its two subunits
4. These are the three termination codons:
A. UAA, UGA, AAU C. UAA, AUA, AAU
B. UAA, UAG, UGA D. UAA, AGU, GAU
5. Which of these events is associated with the termination of translation?
A. tRNA anticodon recognizes and binds with the stop codon
B. stop codon of mRNA appears in the A site of the ribosome
C. releasing factor RF1 releases mRNA from the ribosome
D. an amino acid is added to release tRNA from the P site
6. Which of these phases in translation immediately preceeds the release of the
synthesized protein from the ribosome?
A. activation of the amino acid
B. folding of the polypeptide chain
C. elongation of the polypeptide chain
D. initiation of the polypeptide chain
7. Which of these post-translational modifications help protect the native conformation of
proteins form denaturation?
A. formation of S-S cross links C. addition of prosthetic groups
B. metylation of R groups D. addition of CHO side chains
8. Which of the following occurs during the initiation phase of translation?
A. mRNA binds to 40S preinitiation complex
B. mRNA is read

Module 36
5,1 – 5.2

1. The non-pharmacologic forms of therapy for lipid disorders include the following: C
A. weight loss and exercise
B. weight loss, exercise, and meditation
C. weight loss, exercise, meditation and diet
D. weight loss, exercise, meditation, diet and Vit. B 3
2. Drug treatment to lower LDL-C levels is indicated in these patients: D
A. patients with established CHD
B. patients with a 10-year absolute CHD risk of > 20%
C. patients with low absolute CHD risk but with markedly high LDL – C level
D. all of the above
3. STATINS are HMG-CoA reductase inhibitors and thus, they tend to lower the plasma levels of
the following: C
A. cholesterol
B. cholesterol and LDL -C
C. cholesterol, LDL -C and triglyceride
D. cholesterol, LDL -C, triglyceride and HDL -C
4. The cholesterol-lowering drug of choice in children, pregnant women and lactating mothers is: B
A. omega-3 fatty acids C. statins
B. bile acid sequestrants D. fibrates
5. The drug of choice in patients with severe hypertriglyceridemia is: D
A. omega-3 fatty acids C. statins
B. bile acid sequestrants D. fibrates

Module 37
I.O. # 2.11 -2.14

1. Prophylactic long-term administration of low-dose antibiotics may be indicated in the following


patients, EXCEPT: D
A. women with symptomatic UTIs three or more episodes per year
B. pregnant women with asymptomatic bacteriuria during the first trimester
C. men undergoing prostatectomy and during the postoperative period
D. men with acute bacterial prostatitis
2. 54 year old Ms Rena, a diabetic, was admitted to the hospital because of fever, chills, and pain
in  the  right  flank.    Urinalysis  showed  50  rbc/hpf.    A  “ring  shadow”  was  seen  on  pyelography.    
Based on these limited data, your most likely diagnosis is: C
A. chronic pyelonephritis C. papillary necrosis
B. acute bacterial prostatitis D. emphysematous pyelonephritis
3. Emphysematous pyelonephritis is best confirmed and localized by: C
A. urinalysis C. computed tomography
B. pyelography D. ultrasound

4. Mr Pelvy, 35 years old, came for consultation because of intermittent perineal and low-back pain
and difficulty of starting urine flow. He has no previous history of bacterial prostatitis. Physical
examination was practically negative, CBC was normal and urine culture was negative. Based
on these limited data, your most likely diagnosis is: D
A. Acute bacterial prostatitis C. inflammatory CPPS
B. chronic pelvic pain syndrome (CPPS) D. noninflammatory CPPS
5. Which of these conditions is most associated with an indwelling catheter? A
A. Acute bacterial prostatitis C. papillary necrosis
B. emphysematous cystitis D. chronic pelvic pain syndrome

XAVIER UNIVERSITY – JOSE P RIZAL COLLEGE OF MEDICINE


Basic Biomedical Sciences
Third Bimonthly Period
QUIZ # 2 (Lipids)

NAME ____________________________ 20 Nov 2007 SCORE ____________


MULTIPLE CHOICE: write the letter of the BEST answer on the answer sheet. Use capital
letters in blue/black ink. NO EASURES/SUPERIMPOSITIONS ALLOWED!

47. Regular exercise helps prevent the development of hypercholesterolemia by this


mechanism.
A. formation of smaller VLDL particles
B. Increasing catabolic rate of LDL
C. up-regulation of LDL receptors
D. raising HDL level
48. Which  of  the  following  factors  exert  the  greatest  influence  on  an  individual’s  serum        
cholesterol concentration?
A. Diet C. exercise
B. Environment D. heredity
49. Which of the following have cholesterol-lowering effect?
C. Sucrose C. corn oil
D. palm oil D. beef fat
50. Type I primary hyperlipidemia is characterized by high levels of:
A. Triglycerides C. cholesterol and triglycerides
B. Cholesterol D. either triglycerides or cholesterol
XAVIER UNIVERSITY – JOSE P RIZAL COLLEGE OF MEDICINE
Basic Biomedical Sciences
Second Bimonthly Period
QUIZ # 1 (Module 8)

NAME ____________________ 19 August 2008 SCORE ____________


MULTIPLE CHOICE: write the letter of the BEST answer on the answer sheet. Use capital
letters in blue/black ink. NO EASURES/SUPERIMPOSITIONS ALLOWED!

1. Which of these associations between blood components and functions is correct?


A. erythrocytes – phagocytosis C. neutrophils – antibody
B. platelets – blood coagulation D. lymphocytes – transport oxygen
2. A few hours after Jessa ate rice and shrimps for lunch, she complained of itchiness
throughout her body and her eyelids were swollen. She attributed this to the shrimps she
ate. She went to the school clinic for treatment. If a CBC is done at this time, which blood
cell type is expected to be increased?
A. Neutrophils C. eosinophils
B. Lymphocytes D. monocytes
3. Polycythemia vera is characterized by an absolute increase in:
A. erythrocytes C. plasma proteins
B. leukocytes D. platelets
4. Neutrophils are attracted to go to the area of infection. This is due to the activity of which
of the following interleukins?
A. interleukin 2 C. interleukin 6
B. interleukin 4 D. interleukin 8

XAVIER UNIVERSITY – JOSE P RIZAL COLLEGE OF MEDICINE


Basic Biomedical Sciences
Second Bimonthly Period
QUIZ # 36 (Respiratory 2 & Acid-base)

NAME ____________________________ 6 October 2009 SCORE ____________


MULTIPLE CHOICE: write the letter of the BEST answer on the answer sheet. Use capital
letters in blue/black ink. NO EASURES/SUPERIMPOSITIONS ALLOWED!

1. Which of the following will most likely result from exercise?


A. dyspnea B. hyperpnea C. hyperventilation D. polypnea
2. Which of the following is characterized by irregular periods of apnea, and may be seen
in patients with meningitis?
A. hypoventilation C.  Biot’s  respiration
B. Kussmaul respiration D. Cheyne-Stokes respiration
3. Which of the following may occur normally during sleep?
A. hypoventilation C.  Biot’s  respiration
B. Kussmaul respiration D. Cheyne-Stokes respiration
4. Which of the following gas laws states the inverse relationship between gas volume
and pressure?
A. Boyle’s  law C.  Dalton’s  law
B.  Charles’  law D.  Graham’s  law    
5. Which of the following gas laws considers the direct relationship between gas volume
and absolute temperature?
A.  Avogadro’s  law C. Charles’  law
B.  Boyle’s  law D.  Dalton’s  law
6. Laminar flow normally occurs at the:
A. alveoli B. bronchi C. trachea D. branch points of tracheobronchial
tree
7. Diffusion of gas through the alveolo-capillary membrane occurs in accordance with:
A.  Charles’  law C.  Graham’s  law
B.  Dalton’s  law D.  Henry’s  law
8. Which of the following determinants  of  gas  diffusion  involves  application  of  Graham’s  
law?
A. gas temperature C. solubility coefficient
B. molecular weight of the gas D. pulmonary capillary blood flow
9. Which of the following determinants of gas diffusion, when increased, will increase the
rate of gas diffusion?
A. gas temperature C. diffusion distance
B. molecular weight of the gas D. liquid viscosity
10. Which of the following best explains the normal shape of the oxyhemoglobin
dissociation curve?
A. it involves the relationship between P O2 and hemoglobin saturation
B. the chemical reaction between hemoglobin and oxygen is reversible
C. the reaction of the subunits of hemoglobin with oxygen occur one after the other
D. the reaction of the subunits of hemoglobin with oxygen occur simultaneously and
fuse
11. Which of the following, when decreased, will shift the oxyhemoglobin dissociation
curve to the right?
A. blood pH B. carbon dioxide C. blood temperature D. 2,3 DPG
12. The relationship between carbon dioxide and oxyhemoglobin saturation is called:
A. Haldane effect C. shift to the left
B. Bohr effect D. shift to the right
13. Which of the following is the main transport form of carbon dioxide?
A. bicarbonate C. carboxyhemoglobin
B. carbonic acid D. carbamino compound
14. Which of the following is partly responsible for the sigh response?
A. Hering-Breuer inflation reflex C. Paradoxic reflex of Head
B. Hering-Breuer deflation reflex D. Arterial chemoreceptor reflex
15. Which of the following is most likely to happen in the pulmonary capillary when
bicarbonate enters the red cell?
A. bicarbonate binds with hemoglobin C. chloride moves out of the red cell
B. water moves into the red cell D. bicarbonate will form carbamino
compounds
16. Which of the following is most likely to exist during normal ventilation?
A. lower P O2 at the alveolus than at the pulmonary capillary
B. higher P O2 at the alveolus than at the pulmonary capillary
C. high P CO2 at the alveolus and pulmonary capillary
D. low P CO2 at the alveolus and pulmonary capillary
17. Which of the following will most likely happen when the ventilation-perfusion ratio of a
lung unit decreases? The alveoli in that unit will have:
A. higher P O2 and P CO2 C. higher P O2 and lower P CO2
B. lower P O2 and P CO2 D. higher P CO2 and lower P O2
18. The condition in # 16 will best be compensated by:
A. bronchoconstriction C. regional vasoconstriction
B. bronchodilation D. regional vasodilation
19. Which of the following cells serve as the major efferent pathway of the cerebral
cortex?
A. stellate cells B. fusiform cells C. pyramidal cells D. cuboidal cells
20. Which of the following centers involved in respiration will primarily be affected in an
individual delivering a speech?
A. area 4 B.  Broca’s  area C. limbic system D. pneumotaxic center
21. Which of the following centers involved in respiration functions only during forced
expiration?
A. nucleus tractus solitarius C. nucleus ambiguous
B. nucleus parabrachialis medialis D. nucleus retroambigualis
22. Which of the following centers involved in respiration will most likely discharge as a
consequence of spontaneous depolarization of its neurons?
A. medullary dorsal respiratory group C. pre-Botzinger complex
B. medullary ventral respiratory group D. Botzinger complex
23. Which of the following centers involved in respiration serves as the principal initiators
of activity of the diaphragm?
A. medullary dorsal respiratory group C. pre-Botzinger comlex
B. medullary ventral respiratory group D. Botzinger complex
24. Which of the following will be the effect of stimulation of the caudal neurons of the
nucleus retroambigualis?
A. neurons project to the spinal cord to drive the external intercostals
B. neurons project to the dorsal respiratory group to inhibit inspiratory neurons
C. neurons project to the spinal cord to drive the internal intercostals
D. neurons are vagal motorneurons that bring about patency of the airways
25. Which of the following is primarily responsible for processing of sensory inputs from
peripheral chemoreceptors?
A. medullary dorsal respiratory group C. pontine respiratory group
B. medullary ventral respiratory group D. apneustic center
26. Which of the following neurons/nerves will directly influence activity of the extrafusal
fibers of the diaphragm?
A. intercostal nerve B. alpha motor neuron C. vagus nerve D. gamma
efferents
27. Which of the following will be an effect of lung inflation?
A. hyperpnea B. bronchoconstriction C. cough D. increased heart
rate
28. Which of the following will be an effect of stimulation of J receptors in pulmonary
vessels?
A. tachypnea B. bronchodilation C. vasoconstriction D. vasodilation
29. Which of the following is bound to happen when an individual washes his face?
A. bronchoconstriction B. apnea C. increased heart rate D. vasodilation
30. Which of the following, when stimulated can give rise to inspiration?
A. stretch receptors in the trachea and bronchi
B. stretch receptors in the lungs
C. juxtapulmonary capillary receptors
D. receptors in the nasal mucosa and face
31. The single most important buffer system in body fluids is the;
A. HbO2 buffer system C. plasma HCO3 – buffer system
B. Plasma proteins D. organic phosphate buffer system
32. This is true of the Hb and HbO2 buffer systems.
A. present in erythrocytes and plasma
B. involved in buffering increased or decreased plasma [HCO3-]
C. HHb is the predominant form of hemoglobin in arterial blood
D. HbO2- is the predominant form of hemoglobin in venous blood
33. Pure buffer response to a gain in strong base in body fluids results in the following
directional changes at the new equilibrium.
A. [HCO3-] = increased C. [Buf -] = decreased
B. [H2CO3] = increased D. pH = decreased
34. Pure buffer response to a primary gain in CO 2 in body fluids results in an increase in
which of these parameters at the new equilibrium?
A. [HCO3-] and [Buf -] C. [HBuf] and [HCO3-]
-
B. [HBuf] and [Buf ] D. pH and [HCO3-]
35. Pure buffer response to loss of [HCO3-] in body fluids results in an increase in which of
these parameters at the new equilibrium?
A. pH = C. [Buf-]
B. PCO2 D. [HBuf]
36. Body responses to metabolic alkalosis secondary to ingestion of large amounts of
NaHCO3 tablets include:
A. renal addition of new HCO3-] C. decreased rate of ventilation
B. renal excretion of acid D. buffering by the HCO3- buffer system
37. Body responses to respiratory alkalosis include:
A. renal addition of new HCO3-]
B. maximal renal reabsorption of HCO3-]
C. buffering by the bicarbonate buffer system
D. buffering by the non-bicarbonate buffer system
38. Which of these conditions can give rise to respiratory acidosis?
A. Hysteria C. hypoxemia
B. Salicylate poisoning D. pulmonary edema
39. Compensation of respiratory acidosis involves which of the following?
A. Alkalinization of the urine
B. decreased rate of ventilation
C. renal addition of new bicarbonate
D. Buffering by the non-bicarbonate buffer system
40. Plasma PCO2 is increased in which of these conditions?
A. completely compensated respiratory acidosis
B. partially compensated respiratory alkalosis
C. partially compensated metabolic acidosis
D. uncompensated metabolic alkalosis
41. Jose, a five year old boy, was hospitalized and treated for dehydration and acid-base
disturbance secondary to diarrhea of three days duration. Dr Neil ordered some
laboratory tests to determine his acid-base status. Which of the following lab results
will tell you that the disturbance is already compensated?
A. pH = 7.321 C. [HCO3-] = 18 mM/L
B. PCO2 = 30 mmHg D. [BE] = (-) 4 mM/L
42. Renal addition of new bicarbonate to plasma is associated with:
A. alkalinization of urine
B. reabsorption of bicarbonate
C. buffering of secreted H+ with HPO4=
D. buffering of secreted H+ with HCO3-
43. Renal addition of new HCO3- into the plasma is a compensatory response to:
A. respiratory acidosis C. metabolic acidosis
B. respiratory alkalosis D. metabolic alkalosis

44. What will be the compensatory response to the acid-base disturbance secondary to
vomiting?
A. increased respiratory rate C. renal reabsorption of HCO3-
B. decreased respiratory rate D. renal addition of new HCO3-
45. Dr Caryl showed her students the following laboratory results from her patient with an
acid-base disorder: pH = 7.549, [HCO3-] = 25.2 mM/L, PCO2 = 28.8 mmHg, [BE] =
(+) 1.99 mM/L. Dr Caryl then asked one student to give his diagnosis based on the
laboratory results. If you were the student, which of the following would be your
answer?
A. partially compensated metabolic alkalosis
B. partially compensated respiratory alkalosis
C. uncompensated respiratory alkalosis
D. uncompensated metabolic alkalosis
46. Dr Jhoanna looked at the laboratory results of her patient: pH = 7.715, [HCO3-] = 34.5
mM/L, PCO2 = 26.9 mmHg, and [BE] = 13.2 mM/L. Based on these results Dr
Jhoanna concluded that her patient has a mixed acid-base disturbance which involves:
A. respiratory alkalosis and metabolic acidosis
B. respiratory alkalosis and metabolic alkalosis
C. respiratory acidosis and metabolic alkalosis
D. respiratory acidosis and metabolic acidosis
47. A medical student showed the following laboratory results of his mother to Dr Jude: pH
= 7.271, [HCO3-] = 34 mM/L, PCO2 = 78.1 mmHg, and [BE] = (+) 7.2 mM/L. Based
on the laboratory results, Dr Jude said that his mother has:
A. partially compensated respiratory acidosis
B. partially compensated metabolic acidosis
C. partially compensated respiratory alkalosis
D. partially compensated metabolic acidosis
48. Dr  Amer  asked  one  of  his  students,  “Which  of  these  laboratory  results  will  differentiate  
partially compensated metabolic alkalosis from completely compensated metabolic
alkalosis?”  If  you  were  the  student,  your  answer  would  be:
A. pH = 7.335 C. [HCO3-] = 16.2 mM/L
B. PCO2 = 31.4 mmHg D. [BE] = (-) 8.3 mM/L
Problem: (Please write your answers at the back of your answer sheet.)

1. Mr Camaro is suffering from renal failure and consequently, he is unable to excrete the
strong acids resulting from his tissue metabolism. Dr Anel suspected that Mr Camaro
is also suffering from an acid-base disorder. Based on this information, answer the
following questions: (0.5 each)

1.1 What acid-base disturbance is most likely present in Mr Camaro?

1.2 What buffer system/s will respond to this disturbance?

1.3 What will be the compensatory mechanism/s?

1.4 What will be the corrective mechanism/s?

XAVIER UNIVERSITY – JOSE P RIZAL COLLEGE OF MEDICINE


Basic Biomedical Sciences
Second Bimonthly Period
QUIZ # 3 (CVS 1)

NAME ____________________________ 1 September 2008 SCORE ____________


MULTIPLE CHOICE: write the letter of the BEST answer on the answer sheet. Use capital
letters in blue/black ink. NO EASURES/SUPERIMPOSITIONS ALLOWED!

1. The SA node is the normal pacemaker of the heart because;


A. Its pacemaker cells are located centrally
B. It generates action potentials spontaneously
C. it is located at the highest portion of the heart
D. it has the fastest rate of spontaneous impulse generation
2. Jasmine was diagnosed to have a sick sinus node. Which of these structures will
most likely take over as the pacemaker of her heart?
A. AV node C. left bundle branch
B. Bundle of His D. terminal Purkinje network
3. Which of the following associations between structure and their main cell components
is correct?
A. SA node – purkinje cells C. internodal path – P cells
B. AV node – transitional cells D. bundle of His – working cells
4. Impulse conduction is fastest in the:
A. AV node C. AV bundle and branches
B. Internodal tracts D. ventricular muscles
5. Which of the following associations between the phases of the fast response action
potential (FRAP) and their primary causes is correct?
A. phase 0 – rapid and large Ca++ inward current
B. phase 1 – closure of Ca++ gates and hence decreased Ca++ entry
C. phase 2 – slow and sustained Ca++ current via Ca++ L channels
D. phase 3 – rapid outward Ca++ current via fast Ca++ channels
6. Slow response action potentials (SRAP) differs from FRAP in that SRAP has a:
A. slower velocity of the upstroke phase
B. more negative resting membrane potential
C. more negative threshold potential
D. longer plateau phase
7. This is true of SRAP.
A. depolarization is primarily due to inward Ca++ current via L channels
B. repolarization is primarily due to Ca++ efflux
C. phases 1, 2 and 3 are well demarcated
D. exhibited by atrial and ventricular cells
8. According to Guyton, self-excitation or automaticity of nodal cells in the SA node is
primarily due to:
A. opening of the calcium T channels
B. inherent leakiness of P cells to Na+ ions
C. opening of the calcium-sodium channels
D. time-dependent decline in K+ efflux
9. According to Ganong, the second part of the pacemaker potential is mainly due to:
A. a decrease in membrane permeability to K+
B. opening of sodium-calcium slow channels
C. opening of calcium T channels
D. opening of calcium L channels
10. These cells exhibit pacemaker potentials.
A. P cells, Purkinje cells, atrial muscles, and ventricular muscles
B. P cells, Purkinje cells, and atrial muscles
C. P cells and Purkinje cells
D. P cells only
11. The discharge rate of pacemaker cells may be decreased by increasing the:
A. slope of diastolic depolarization
B. threshold potential
C. initial resting potential
D. sympathelic stimulation
12. The absolute refractory period of fast response cells includes this/these phase/s of the
FRAP.
A. phase 0, 1, 2, and part of 3
B. phases 0, 1, and part of 2
C. phases 0 and 1
D. phase 0 only
13. Maria is diagnosed to have right atrial enlargement. If an ECG is done on her, which of
these ECG intervals will be prolonged?
A. PR interval C. QT interval
B. QRS interval D. RR interval
14. As regards the ECG, which of the following is correct?
A. a recording of electrical activity as recorded from the surface of the heart
B. Each lead is a different view of the electrical activity of the heart
C. The wire attached to the left leg is used as ground
D. Routine ECG consists of 14 leads
15. End systolic volume is equal to:
A. end diastolic volume (EDV) minus reserve volume RV) plus stroke volume (SV)
B. EDV minus systolic reserve volume (SRV)
C. EDV minus SRV plus SV plus RV
D. EDV minus SV
16. End diastolic volume refers to the volume of blood present in the ventricle during this
phase of the cardiac cycle.
A. isovolumetric contraction phase C. protodiastole
B. isovolumetric relaxation phase D. diastasis
17. Ventricular pressure is greater than aortic pressure during this phase of the cardiac
cycle.
A. isovolumetric contraction C. rapid ejection
B. isovolumetric relaxation D. atrial systole
18. All valves are closed during these phases of the cardiac cycle.
A. phases I and V C. phases III and VII
B. phases II and VI D. phases IV and VIII
19. The atria act as reservoirs during these phases of the cardiac cycle.
A. phases I, II, III, IV, and V C. phases II and III only
B. phases VI, VII, VIII D. phases V and VI only
20. Which of these events occur earlier in the right than in the left side of the heart?
A. onset of atrial contraction
B. onset of ventricular contraction
C. closure of semilunar valve
D. closure of A-V valve
21. This is true of the first heart sound (S1).
A. Sharp in quality
B. due to closure of the semilunar valves
C. clinically marks the beginning of systole
D. best heard over the 2nd left intercostal space along sternal border
22. During a physiology laboratory exercise, Rochel listened for the heart sounds of his
obese classmate Ike. Assuming Ike is normal, which heart sound/s did Rochel hear
when he placed the stethoscope over the 5th intercostal space along the left
midclavicular line?
A. S1, S2, S3, and S4 C. S1 and S2
B. S1, S2, and S3 D. S1 only

23. During inspiration the venous return to the right side of the heart is increased. This can
result to splitting of the second heart sound. The splitting of S2 is due to:
A. early A2 C. delayed A2
B. early P2 D. delayed P2
24. Which of these conditions can give rise to a diastolic murmur?
A. mitral valve prolapse C. aortic stenosis
B. tricuspid insufficiency D. papillary muscle weakness
25. Applying  Starling’s  law  of  the  heart,  increasing  the  _______  increases  the  force  of  
myocardial contraction, and thus the stroke volume.
A. End diastolic volume C. afterload stress
B. aortic blood pressure D. heart rate
26. Which of these associations between cardiac reflexes and their effect on heart rate is
correct?
A. Bainbridge reflex – bradycardia
B. McDowall reflex – tachycardia
C. Goltz reflex – tachycardia
D. Ashner-Dagnini reflex – tachycardia
27. Which of these hormones exert a positive inotropic effect on the heart?
A.

XAVIER UNIVERSITY – JOSE P RIZAL COLLEGE OF MEDICINE


Basic Biomedical Sciences
Second Bimonthly Period
QUIZ # 4 (CVS 2)

NAME ____________________________ 8 September 2008 SCORE ____________


MULTIPLE CHOICE: write the letter of the BEST answer on the answer sheet. Use capital
letters in blue/black ink. NO EASURES/SUPERIMPOSITIONS ALLOWED!

1. Which of the following blood vessels functions as a windkessel vessel?


A. large artery C. vein
B. small artery D. venule
2. Blood behaves as a Newtonian fluid in which of the following blood vessels?
A. capillaries C. large arteries
B. venules D. large veins
3. The smaller the radius of a blood vessel, the lesser the viscosity. This is explained on
the basis of:
A. Fahreus-Lindqvist phenomenon C. shear thinning
B. Poiseuille-Hagen formula D. plasma skimming
4. Which of the following best applies to the pulmonary circulation only?
A.  Poiseuille’s  law C. continuity principle
B.  Laplace’s  law D. Bernouilli principle
5. Which of the following is the most critical factor influencing turbulent flow?
A. blood vessel diameter C. blood viscosity
B. Reynolds number D. blood temperature
6. Which of the following will most likely happen if the intraluminal pressure in a blood
vessel is much lower than the tissue pressure?
A. tension on the wall will increase causing the blood vessel to balloon
B. blood flow within the blood vessel will increase
C. the blood vessel will collapse and the flow stops
D. the relationship between pressure and flow will remain linear
7. Which of the following, when diminished, will increase blood viscosity?
A. hematocrit C. vessel radius
B. temperature D. fibrinogen
8. Which of the following will most likely happen if the lumen of a blood vessel is lesser
than the diameter of red cells?
A. Bernouilli principle C. shear thinning
B. tank treading D. plasma skimming
9. The major determinant of resistance in a blood vessel is the:
A. driving pressure C. hydrostatic pressure
B. transmural pressure D. axial pressure difference
10. The blood pressure (BP) of Franz is 138/90 mmHg. What is his mean arterial
pressure (MAP)?
A. 106 mmHg C. 120 mmHg
B. 116 mmHg D. 136 mmHg
11. Which of these is true of arterial blood pressure determination?
A. Is a measure of the average pressure of all arteries of the body
B. The direct and indirect measurements will give the same values/results
C. the Flush method measures BP of young patients in coma
D. the Riva-Rocci method estimates systolic but not the diastolic pressure
12. In  which  method  of  arterial  BP  determination  is  the  “auscultatory  gap”  a  problem?
A. Riva-Rocci method C. oscillatory method
B. Auscultatory method D. direct method
13. When this is increased, arterial BP tends to decrease.
A. vessel radius C. blood volume
B. hematocrit D. cardiac output

14. In which of these conditions is the pulse pressure decreased?


A. Patent ductus arteriosus C. aortic stenosis
B. aortic insufficiency D. arteriosclerosis
15. Central venous pressure (CVP) refers to the pressure in the:
A. right atrium C. right and left atria
B. left atrium D. right atrium and right ventricle
16. An increase of which of the following will most likely cause a decrease in CVP?
A. venous return
B. blood volume
C. peripheral venous pressure
D. force of right ventricular contraction
17.When a person is in the upright posture:
A. his CVP is higher than when he is in supine posture
B. the venous pressure in his legs are lower than in his arms
C. the veins inside his skull are collapsed
D. his neck veins are collapsed
20. When this is increased, the venous return is decreased.
A. blood volume C. arteriolar dilatation
B. venous dilatation D. mean systemic filling pressure
21. Which of these conditions tends to decrease net filtration pressure?
A. venous obstruction C. decreased plasma protein
B. hypertension D. precapillary sphincter constriction
22. The main form of transport across the capillary wall is by:
A. diffusion C. filtration
B. facilitated transport D. transcytosis
23. A decrease in which of the following tends to increase lymph flow?
A. capillary hydrostatic pressure C. activity of the lymphatic pump
B. plasma colloid osmotic pressure D. capillary permeability
24. This is true of the lymphatic system.
A. a one-way route for movement of interstitial fluid to the blood
B. Lymph from the right and left chest drains into the thoracic duct
C. Lymph is finally drained into the right and left brachial veins
D. Total lymph flow is 100 ml per hour
25. During a physiology exercise, Rocky did the Valsalva maneuver. His group mates
noticed that the changes in his BP and heart rate were as expected. Which of the
following are the expected results during the maneuver?
A. increased BP, increased heart rate
B. increased BP, decreased heart rate
C. decreased BP, increased heart rate
D. decreased BP, decreased heart rate
26. Jerry runs around the XU oval every Sunday morning. He knows that every time he
runs, his blood flow to his lower extremities is markedly increased to meet the needs of
his extremities. And he correctly calls this response:
A. reactive hyperemia C. Oxygen lack theory
B. active hyperemia D. increased tissue vascularity
27. Lina loves eating salty foods and yet her blood volume and blood pressure stay within
normal limits because of the activity of the:
A. carotico-aortic sinus reflex C. vasopressin system
B. carotico-aortic body reflex D. rennin-angiotensin system
28. Increased concentration of this ion causes vasoconstriction.
A. Hydrogen C. calcium
B. Magnesium D. potassium
29. Ms Cruz was so emotionally disturbed that her arterial BP shoot up to 180/100 mmHg.
Which of the following reflexes was immediately stimulated to bring down her BP
towards normal?
A. carotico-aortic body reflex C. vasopressin system
B. carotico-aortic sinus reflex D. rennin-angiotensin system
30. The greatest resistance to flow resides in the:
A. Veins C. arterioles
B. Arteries D. capillaries
31. The hypertension of Mr Cruz is due to an increase in blood volume. Which of these
substances will reflexly increase to bring down his BP towards normal?
A. antidiuretic hormone C. atrial natriuretic factor
B. aldosterone D. angiotensin II
32. The organ/s primarily involved in long-term regulation of arterial BP is/are the:
A. Liver C. kidneys and liver
B. Kidneys D. kidneys and gastrointestinal tract
31. Mr Rey complained of headache unrelieved by analgesics. He consulted a physician,
and was told that his BP was 164/110 mmHg. Repeat BP determination showed the
same value. Mr Rey claimed that this was the first time his BP was high. Two days
later  he  went  back  to  the  doctor’s  office.  On  this  second  visit,  his  BP  was  162/108  
mmHg. If you were the physician you would diagnose him as:
A. prehypertensive C. hypertensive, stage 2
B. hypertensive, stage 1 D. hypertensive, stage 3
32. The main component of the tunica adventitia
A. Collagen and elastic fibers
B. Amorphous ground substance
C. Smooth muscle tissue
D. Endothelial cells
33. The vasa vasorum can be found in which of the following layers?
A. Intima & media C. Adventitia and intima
B. Media and adventitia D. Intima, media and adventiita
34. Which of the following best describes large arteries?
A. Intimal thickness is the same as in medium arteries
B. Smooth muscle cells & reticular fibers in media
C. absent internal elastic lamina
D. very thick tunica adventitia
35. Which of the following is characteristic of an arteriole?
A. thick adventitia, large lumen
B. large amounts of elastic fibers in intima
C. muscular media, absent external lamina
D. thick subendothelial layer
36. Which of the following is true of veins?
A. prominent subendothelial layer
B. smooth muscle cells, elastic & reticular fibers are seen in the media
C. large veins have the thickest media
D. intimal folds are seen in all sizes of veins
37. Which capillary type has a continuous basal lamina and a diaphragm covering large
fenestrae?
A. Somatic C. Sinusoidal
B. Visceral D. Lymphatic
38. Which of the following is true of the lymphatic vessels?
A. elastic fibers bind lymphatic capillaries to the tissues
B. layers of the vessel are well-defines
C. presence of valves is less compared to the veins
D. vasa vasorum is found in the media
39. Part of the aorta that gives origin to the coronary arteries:
A. Ascending C. Descending
B. Aortic arch D. Thoracic
40. Which of the following is a branch of the brachiocephalic artery?
A. Right subclavian C. Left common carotid
B. Vertebral D. Axillary

41. Which of the following branches of the thoracic aorta supplies the lower nine
intercostals spaces?
A. Anterior intercostal C. Superior intercostal
B. Inferior intercostal D. Posterior intercostal
42. A branch of the aorta that supplies the liver, stomach, and spleen:
A. Common iliac C. Celiac trunk
B. Superior mesenteric D. Inferior mesenteric
43. Arterial supply to the second half of the large intestine:
A. Celiac trunk C. Common iliac
B. Superior mesenteric D. Inferior mesenteric
44. This vein receives blood from the arm through the axillary vein:
A. Brachial C. Subclavian
B. Basilic D. Cephalic
45. Vein that drains the dural sinuses of the brain:
A. Internal jugular C. External jugular
B. Vertebral D. Brachiocephalic
46. Drains venous blood from the liver to the systemic circulation:
A. Hepatic C. Hepatic portal
B. Superior mesenteric D. Inferior mesenteric
47. The great saphenous vein is a tributary of this vein:
A. Internal iliac C. External iliac
B. Femoral D.Popliteal
48. Forms the hepatic portal vein:
A. Splenic and hepatic veins
B. Superior mesenteric and hepatic veins
C. Splenic and superior mesenteric veins
D Superior and inferior mesenteric veins

ESSAY

1. What is the effect of right ventricular failure on rate of lymph flow?


ENCIRCLE your answer. 0.5 pt

INCREASED / DECREASED

2. In outline form give the sequence of events that lead to such an effect. Correction
stops at the point where you commit a mistake. 1.5 pts

XAVIER UNIVERSITY – JOSE P RIZAL COLLEGE OF MEDICINE


Basic Biomedical Sciences
Second Bimonthly Period
QUIZ # 4 (CVS 2)

NAME ____________________________ 8 September 2008 SCORE ____________


MULTIPLE CHOICE: write the letter of the BEST answer on the answer sheet. Use capital
letters in blue/black ink. NO EASURES/SUPERIMPOSITIONS ALLOWED!

17. Which of the following blood vessels functions as a windkessel vessel?


A. large artery C. vein
B. small artery D. venule
18. Blood behaves as a Newtonian fluid in which of the following blood vessels?
A. capillaries C. large arteries
B. venules D. large veins
19. The smaller the radius of a blood vessel, the lesser the viscosity. This is explained on
the basis of:
A. Fahreus-Lindqvist phenomenon C. shear thinning
B. Poiseuille-Hagen formula D. plasma skimming
20. Which of the following best applies to the pulmonary circulation only?
A.  Poiseuille’s  law C. continuity principle
B.  Laplace’s  law D. Bernouilli principle
21. Which of the following is the most critical factor influencing turbulent flow?
A. blood vessel diameter C. blood viscosity
B. Reynolds number D. blood temperature
22. Which of the following will most likely happen if the intraluminal pressure in a blood
vessel is much lower than the tissue pressure?
A. tension on the wall will increase causing the blood vessel to balloon
B. blood flow within the blood vessel will increase
C. the blood vessel will collapse and the flow stops
D. the relationship between pressure and flow will remain linear
23. Which of the following, when diminished, will increase blood viscosity?
A. hematocrit C. vessel radius
B. temperature D. fibrinogen
24. Which of the following will most likely happen if the lumen of a blood vessel is lesser
than the diameter of red cells?
A. Bernouilli principle C. shear thinning
B. tank treading D. plasma skimming
25. The major determinant of resistance in a blood vessel is the:
A. driving pressure C. hydrostatic pressure
B. transmural pressure D. axial pressure difference
26. The blood pressure (BP) of Franz is 138/90 mmHg. What is his mean arterial
pressure (MAP)?
A. 106 mmHg C. 120 mmHg
B. 116 mmHg D. 136 mmHg
27. Which of these is true of arterial blood pressure determination?
A. Is a measure of the average pressure of all arteries of the body
B. The direct and indirect measurements will give the same values/results
C. the Flush method measures BP of young patients in coma
D. the Riva-Rocci method estimates systolic but not the diastolic pressure
28. In which method of arterial BP determination is  the  “auscultatory  gap”  a  problem?
A. Riva-Rocci method C. oscillatory method
B. Auscultatory method D. direct method
29. When this is increased, arterial BP tends to decrease.
A. vessel radius C. blood volume
B. hematocrit D. cardiac output

30. In which of these conditions is the pulse pressure decreased?


A. Patent ductus arteriosus C. aortic stenosis
B. aortic insufficiency D. arteriosclerosis
31. Central venous pressure (CVP) refers to the pressure in the:
A. right atrium C. right and left atria
B. left atrium D. right atrium and right ventricle
32. An increase of which of the following will most likely cause a decrease in CVP?
A. venous return
B. blood volume
C. peripheral venous pressure
D. force of right ventricular contraction
17.When a person is in the upright posture:
A. his CVP is higher than when he is in supine posture
E. the venous pressure in his legs are lower than in his arms
F. the veins inside his skull are collapsed
G. his neck veins are collapsed
33. When this is increased, the venous return is decreased.
A. blood volume C. arteriolar dilatation
B. venous dilatation D. mean systemic filling pressure
34. Which of these conditions tends to decrease net filtration pressure?
A. venous obstruction C. decreased plasma protein
B. hypertension D. precapillary sphincter constriction
35. The main form of transport across the capillary wall is by:
A. diffusion C. filtration
B. facilitated transport D. transcytosis
36. A decrease in which of the following tends to increase lymph flow?
A. capillary hydrostatic pressure C. activity of the lymphatic pump
B. plasma colloid osmotic pressure D. capillary permeability
37. This is true of the lymphatic system.
A. a one-way route for movement of interstitial fluid to the blood
B. Lymph from the right and left chest drains into the thoracic duct
C. Lymph is finally drained into the right and left brachial veins
D. Total lymph flow is 100 ml per hour
38. During a physiology exercise, Rocky did the Valsalva maneuver. His group mates
noticed that the changes in his BP and heart rate were as expected. Which of the
following are the expected results during the maneuver?
A. increased BP, increased heart rate
B. increased BP, decreased heart rate
C. decreased BP, increased heart rate
D. decreased BP, decreased heart rate
39. Jerry runs around the XU oval every Sunday morning. He knows that every time he
runs, his blood flow to his lower extremities is markedly increased to meet the needs of
his extremities. And he correctly calls this response:
A. reactive hyperemia C. Oxygen lack theory
B. active hyperemia D. increased tissue vascularity
40. Lina loves eating salty foods and yet her blood volume and blood pressure stay within
normal limits because of the activity of the:
A. carotico-aortic sinus reflex C. vasopressin system
B. carotico-aortic body reflex D. rennin-angiotensin system
41. Increased concentration of this ion causes vasoconstriction.
A. Hydrogen C. calcium
B. Magnesium D. potassium
42. Ms Cruz was so emotionally disturbed that her arterial BP shoot up to 180/100 mmHg.
Which of the following reflexes was immediately stimulated to bring down her BP
towards normal?
A. carotico-aortic body reflex C. vasopressin system
B. carotico-aortic sinus reflex D. rennin-angiotensin system
43. The greatest resistance to flow resides in the:
A. Veins C. arterioles
B. Arteries D. capillaries
44. The hypertension of Mr Cruz is due to an increase in blood volume. Which of these
substances will reflexly increase to bring down his BP towards normal?
A. antidiuretic hormone C. atrial natriuretic factor
B. aldosterone D. angiotensin II
45. The organ/s primarily involved in long-term regulation of arterial BP is/are the:
A. Liver C. kidneys and liver
B. Kidneys D. kidneys and gastrointestinal tract
31. Mr Rey complained of headache unrelieved by analgesics. He consulted a physician,
and was told that his BP was 164/110 mmHg. Repeat BP determination showed the
same value. Mr Rey claimed that this was the first time his BP was high. Two days
later  he  went  back  to  the  doctor’s  office.  On  this  second  visit,  his  BP  was  162/108  
mmHg. If you were the physician you would diagnose him as:
A. prehypertensive C. hypertensive, stage 2
B. hypertensive, stage 1 D. hypertensive, stage 3
32. The main component of the tunica adventitia
E. Collagen and elastic fibers
F. Amorphous ground substance
G. Smooth muscle tissue
H. Endothelial cells
39. The vasa vasorum can be found in which of the following layers?
A. Intima & media C. Adventitia and intima
B. Media and adventitia D. Intima, media and adventiita
40. Which of the following best describes large arteries?
A. Intimal thickness is the same as in medium arteries
B. Smooth muscle cells & reticular fibers in media
C. absent internal elastic lamina
D. very thick tunica adventitia
41. Which of the following is characteristic of an arteriole?
A. thick adventitia, large lumen
B. large amounts of elastic fibers in intima
C. muscular media, absent external lamina
D. thick subendothelial layer
42. Which of the following is true of veins?
A. prominent subendothelial layer
B. smooth muscle cells, elastic & reticular fibers are seen in the media
C. large veins have the thickest media
D. intimal folds are seen in all sizes of veins
43. Which capillary type has a continuous basal lamina and a diaphragm covering large
fenestrae?
A. Somatic C. Sinusoidal
B. Visceral D. Lymphatic
44. Which of the following is true of the lymphatic vessels?
A. elastic fibers bind lymphatic capillaries to the tissues
B. layers of the vessel are well-defines
C. presence of valves is less compared to the veins
D. vasa vasorum is found in the media
39. Part of the aorta that gives origin to the coronary arteries:
A. Ascending C. Descending
B. Aortic arch D. Thoracic
40. Which of the following is a branch of the brachiocephalic artery?
A. Right subclavian C. Left common carotid
B. Vertebral D. Axillary

41. Which of the following branches of the thoracic aorta supplies the lower nine
intercostals spaces?
A. Anterior intercostal C. Superior intercostal
B. Inferior intercostal D. Posterior intercostal
42. A branch of the aorta that supplies the liver, stomach, and spleen:
A. Common iliac C. Celiac trunk
B. Superior mesenteric D. Inferior mesenteric
43. Arterial supply to the second half of the large intestine:
A. Celiac trunk C. Common iliac
B. Superior mesenteric D. Inferior mesenteric
44. This vein receives blood from the arm through the axillary vein:
A. Brachial C. Subclavian
B. Basilic D. Cephalic
45. Vein that drains the dural sinuses of the brain:
A. Internal jugular C. External jugular
B. Vertebral D. Brachiocephalic
46. Drains venous blood from the liver to the systemic circulation:
A. Hepatic C. Hepatic portal
B. Superior mesenteric D. Inferior mesenteric
47. The great saphenous vein is a tributary of this vein:
A. Internal iliac C. External iliac
B. Femoral D.Popliteal
48. Forms the hepatic portal vein:
C. Splenic and hepatic veins
D. Superior mesenteric and hepatic veins
C. Splenic and superior mesenteric veins
D Superior and inferior mesenteric veins

ESSAY

3. What is the effect of right ventricular failure on rate of lymph flow?


ENCIRCLE your answer. 0.5 pt
INCREASED / DECREASED

4. In outline form give the sequence of events that lead to such an effect. Correction
stops at the point where you commit a mistake. 1.5 pts

XAVIER UNIVERSTIY – JOSE P RIZAL COLLEGE MEDICINE


BASIC BIOMEDICAL SCIENCES
SECOND BIMONTHLY PERIOD
QUIZ # 2: MODULE 13

NAME ____________________________ Sept. 4, 2006

Write the letter of the BEST answer on the answer sheet in CAPITAL letters in INK.
NO ERASURES/SUPERIMPOSITIONS ALLOWED!

1. The ability to generate an action potential spontaneously is a characteristic of these


cells: C
A. P cells C. P cells and Purkinje cells
B. Purkinje cells D. P cells and ventricular muscles
2. The SA node is the normal pacemaker of the heart because it: D
A. is composed of P cells C. exhibits automaticity
B. is located at right atrium D. has highest rate of impulse generation
3. From the atrial muscle cells, the impulse is conducted to the: B
A. internodal tracts C. ventricular muscles
B. AV node D. Bundle of His
4. Impulse conduction is fastest in:
A. AV node C. ventricular muscles
B. SA node D. terminal Purkinje network
5. The following are true of the plateau phase of the fast response action potential
(FRAP), EXCEPT: C
A. there is very little change in the membrane potential
B. it prolongs the duration of the action potential (AP)
C. primarily due to slow inward Ca++ current through transient (T) channels
D. slow outward K+ current contributes to its development
6. Repolarization of the FRAP includes the following phases: B
A. phases 1, 2, 3, 4 C. phases 1, 2
B. phases 1, 2, 3 D. phase 1 only
7. The following are true of the slow response action potential (SRAP), EXCEPT: A
A. phase 0 is primarily due to slow inward Na+ current
B. repolarization phase is a single slow curve, not well demarcated
C. peak depolarization is about 0 mV
D. exhibited by nodal cells
8. Compared to FRAP, the SRAP has: B
A. more negative RMP C. longer duration of AP
B. slower velocity of upstroke D. longer plateau phase
9. The following are true of the pacemaker potential (PP), EXCEPT: D
A. also termed the prepotential
B. characterized by unstable RMP
C. does not require an outside stimulus to occur
D. normally present in atrial and ventricular muscle fibers
10. Ventricular muscle starts to be relatively refractory to stimulation during the latter part
of the FRAP: D
A. phase 0 C. phase 2
B. phase 1 D. phase 3
11. The rate of discharge of pacemaker cells can be increased by increasing the following,
EXCEPT: C
A. rate of spontaneous depolarization of the resting potential
B. threshold potential
C. initial resting potential
D. sympathetic stimulation

12. The following are true of the P wave, EXCEPT: A


A. represents atrial repolarization
B. should not be more than 0.10 sec.
C. becomes prolonged when atria are enlarged
D. measured from the beginning to the end of P wave
13. The following association between ECG lead and location of positive electrode are
true, EXCEPT: C
A. lead I – left arm C. lead III – right leg
B. lead II – left leg D. lead aVR – right arm
14. If there are 2.5 small squares from the onset of the Q wave to the termination of the S
wave, then the QRS interval is _____ second. C
A. 0.05 C. 0.1
B. 0.5 D. 1.0
15. The volume of blood present in the ventricle during isovolumetric relaxation phase of
the cardiac cycle is the: B
A. end diastolic volume C. residual volume
B. end systolic volume D. systolic reserve volume
16. Aortic pressure is higher than ventricular pressure during these phases of the cardiac
cycle, EXCEPT: C
A. isovolumetric contraction C. rapid ejection
B. isovolumetric relaxation D. reduced ejection
17. The heart is at its most complete rest during this phase of the cardiac cycle: C
A. isovolumetric relaxation C. reduced filling
B. rapid filling D. atrial contraction
18. The following events occur earlier in the left than in the right side of the heart,
EXCEPT: D
A. onset of ventricular contraction C. closure of the AV valve
B. closure of the semilunar valve D. opening of the semilunar valve
19. These are true of the first heart sound (S 1), EXCEPT: C
A. louder and longer in duration than S 2
B. clinically marks the onset of systole
C. heard only at the mitral and tricuspid valve areas
D. occurs a few msecs after onset of isovolumetric contraction phase
20. Which heart sound may be normally split during inspiration? B
A. S 1 B. S 2 C. S 3 D. S 4
21. Which of the following may cause systolic murmur? A
A. aortic stenosis C. mitral stenosis
B. pulmonary insufficiency D. tricuspid stenosis
22. Applying  Starling’s  law  of  the  heart,  which  of  the  following  when  increased  will  result  in  
greater force of ventricular contraction? A
A. end diastolic volume C. heart rate
B. residual volume D. cardiac reserve
23. Stimulation of the following reflexes results in bradycardia, EXCEPT: B
A. Goltz reflex C. carotico-aortic sinus reflex
B. McDowall reflex D. Ashner-Dagnini reflex
24. In which of these conditions may the heart stop in diastole? C
A. hyperkalemia, hypokalemia, hypercalcemia, hypocalcemia
B. hyperkalemia, hypokalemia, hypercalcemia
C. hyperkalemia, hypokalemia
D. hyperkalemia
25. Which of the following substances exert a positive inotropic effect? A
A. catecholamine, thyroid hormone, glucagon, angiotensin II
B. catecholamine, thyroid hormone, glucagon
C. catecholamine, thyroid hormone
D. catecholamine

XAVIER UNIVERSITY – JOSE P RIZAL COLLEGE OF MEDICINE


Basic Biomedical Sciences
Second Bimonthly Period
QUIZ # 2 (Blood Groups & Hemostasis)

NAME ____________________________ 1 September 2009 SCORE ____________


MULTIPLE CHOICE: write the letter of the BEST answer on the answer sheet. Use capital
letters in blue/black ink. NO EASURES/SUPERIMPOSITIONS ALLOWED!

1. These are the major factors involved to effect hemostasis:


A. coagulation factors, blood vessels, platelets, & white blood cells
B. coagulation factors, blood vessels, & platelets
C. coagulation factors & blood vessels
D. coagulation factors & platelets
2. To effect primary hemostasis, which of these should occur?
A. vasodilatation
B. platelets become flattened discs
C. platelet adhere to collagen and aggregate
D. formation of fibrin network surrounding platelets
3. Collagen in injured blood vessels directly bind to platelets via these platelet receptors;
A. GP Ia/IIa C. GP Ib/IX
B. GP IIb/IIIa D. GP IV
4. Fibrinogen forms a bridge between two opposing platelets by binding to these platelet
receptors;
A. GP Ia/IIa C. GP Ib/IX
B. GP IIb/IIIa D. GP IV
5. This substance reinforces the adhesion of platelets to collagen by binding to collagen
and to a receptor on platelets.
A. thromboxane A2 C. fibronectin
B. thrombospondin D. von Willebrand factor
6. These coagulation factors are present both in plasma and in serum.
A. Factors XII, XI, X, & IX C. Factors II, IV, VII, & VIII
B. Factors V, VII, VIII & IX D. Factors VII, VIII, IX, & X
7. A patient who develops vitamin K deficiency may also develop a bleeding disorder
secondary to a deficiency in any one or all of these coagulation factors.
A. factors I, V, VIII, & XIII C. I, II, VII, & X
B. factors II, V, VII, & X D. II, VII, IX, & X
8. Mr Bladdy was suspected of having a bleeding disorder secondary to factor IX
deficiency. Dr Kaye, the resident physician, ordered the following laboratory tests to be
done on him. If indeed Mr Bladdy has factor IX deficiency, which of these tests is/are
expected to be abnormal?
A. Bleeding time (BT) and tourniquet test (Tor T)
B. Thrombin time (TT)
C. Prothrombin time (PT)
D. activated partial thromboplastin time (APTT)
9. Don, a seven year old boy, developed rashes and petechiae over his extremities after
three days of fever. He also complained of nose bleeding just before coming to the
hospital. After taking the history and doing the physical exam, Dr Beltran ordered
laboratory tests to be done on him and these are the results: BT = prolonged, Tor T =
positive; APTT = normal; and PT = normal. Based on these data, which of the
following  is  most  likely  to  be  deficient  to  explain  Don’s  bleeding  disorder?
A. Platelets C. any one of the factors of the intrinsic pathway
B. red blood cells D. any one of the factors of the extrinsic pathway
10. After taking the history and doing the physical exam on Mr Ree, Dr Cyril suspected
that Mr Ree was suffering from a hemorrhagic disorder. The following laboratory tests
were done and these are the results: BT = normal; APTT = normal; PT = prolonged;
and TT = normal. Based on these data, which of the following is most likely to be
deficient?
A. Factors VII, V, X, & Platelets C. factors VII & V
B. factors VII, V, & platelets D. factor VII
11. Five  ml  of  Carlo’s    blood was placed in a test tube and was observed for clot
retraction. The clot was reduced to one-half its original mass after one hour; the
exuded serum was clear. The clot began to dissolve after two days. The correct
interpretation of this test result is:
A.

ESSAY

1. During a physiology laboratory exercise, John volunteered to be the subject for the
bleeding time test (Ivy method). Unfortunately, he did not tell his group mates that he
is suffering from hemophilia A (factor VIII deficiency). Bleeding from his punctured
wound lasted for five minutes. However, the classmates noticed that the wound bled
again a few minutes after it had stopped bleeding. Based on these data, answer the
following questions:

Is his bleeding time normal? Encircle your answer. 0.5 pt

YES / NO

In ONE sentence explain why there was recurrence of bleeding from his punctured
wound. (1.5 pts)

XAVIER UNIVERSITY – JOSE P RIZAL COLLEGE OF MEDICINE


Basic Biomedical Sciences
Second Bimonthly Period
QUIZ # 2 (Blood Groups & Hemostasis)

NAME ____________________________ 1 September 2009 SCORE ____________


MULTIPLE CHOICE: write the letter of the BEST answer on the answer sheet. Use capital
letters in blue/black ink. NO EASURES/SUPERIMPOSITIONS ALLOWED!

12. These are the major factors involved to effect hemostasis:


A. coagulation factors, blood vessels, platelets, & white blood cells
B. coagulation factors, blood vessels, & platelets
C. coagulation factors & blood vessels
D. coagulation factors & platelets
13. To effect primary hemostasis, which of these should occur?
A. vasodilatation
B. platelets become flattened discs
C. platelet adhere to collagen and aggregate
D. formation of fibrin network surrounding platelets
14. Collagen in injured blood vessels directly bind to platelets via these platelet receptors;
A. GP Ia/IIa C. GP Ib/IX
B. GP IIb/IIIa D. GP IV
15. Fibrinogen forms a bridge between two opposing platelets by binding to these platelet
receptors;
A. GP Ia/IIa C. GP Ib/IX
B. GP IIb/IIIa D. GP IV
16. This substance reinforces the adhesion of platelets to collagen by binding to collagen
and to a receptor on platelets.
A. thromboxane A2 C. fibronectin
B. thrombospondin D. von Willebrand factor
17. These coagulation factors are present both in plasma and in serum.
A. Factors XII, XI, X, & IX C. Factors II, IV, VII, & VIII
B. Factors V, VII, VIII & IX D. Factors VII, VIII, IX, & X
18. A patient who develops vitamin K deficiency may also develop a bleeding disorder
secondary to a deficiency in any one or all of these coagulation factors.
A. factors I, V, VIII, & XIII C. I, II, VII, & X
B. factors II, V, VII, & X D. II, VII, IX, & X
19. Mr Bladdy was suspected of having a bleeding disorder secondary to factor IX
deficiency. Dr Kaye, the resident physician, ordered the following laboratory tests to be
done on him. If indeed Mr Bladdy has factor IX deficiency, which of these tests is/are
expected to be abnormal?
A. Bleeding time (BT) and tourniquet test (Tor T)
B. Thrombin time (TT)
C. Prothrombin time (PT)
D. activated partial thromboplastin time (APTT)
20. Don, a seven year old boy, developed rashes and petechiae over his extremities after
three days of fever. He also complained of nose bleeding just before coming to the
hospital. After taking the history and doing the physical exam, Dr Beltran ordered
laboratory tests to be done on him and these are the results: BT = prolonged, Tor T =
positive; APTT = normal; and PT = normal. Based on these data, which of the
following  is  most  likely  to  be  deficient  to  explain  Don’s  bleeding  disorder?
A. Platelets C. any one of the factors of the intrinsic pathway
B. red blood cells D. any one of the factors of the extrinsic pathway
21. After taking the history and doing the physical exam on Mr Ree, Dr Cyril suspected
that Mr Ree was suffering from a hemorrhagic disorder. The following laboratory tests
were done and these are the results: BT = normal; APTT = normal; PT = prolonged;
and TT = normal. Based on these data, which of the following is most likely to be
deficient?
A. Factors VII, V, X, & Platelets C. factors VII & V
B. factors VII, V, & platelets D. factor VII
22. Five  ml  of  Carlo’s    blood  was  placed  in  a  test  tube  and  was  observed  for  clot  
retraction. The clot was reduced to one-half its original mass after one hour; the
exuded serum was clear. The clot began to dissolve after two days. The correct
interpretation of this test result is:
A.

ESSAY

2. During a physiology laboratory exercise, John volunteered to be the subject for the
bleeding time test (Ivy method). Unfortunately, he did not tell his group mates that he
is suffering from hemophilia A (factor VIII deficiency). Bleeding from his punctured
wound lasted for five minutes. However, the classmates noticed that the wound bled
again a few minutes after it had stopped bleeding. Based on these data, answer the
following questions:

Is his bleeding time normal? Encircle your answer. 0.5 pt

YES / NO
In ONE sentence explain why there was recurrence of bleeding from his punctured
wound. (1.5 pts)

XAVIER UNIVERSITY – JOSE P RIZAL COLLEGE OF MEDICINE


Basic Biomedical Sciences
Second Bimonthly Period
QUIZ # 2 (Module 9)

NAME ____________________ 26 August 2008 SCORE ____________


MULTIPLE CHOICE: write the letter of the BEST answer on the answer sheet. Use capital
letters in blue/black ink. NO EASURES/SUPERIMPOSITIONS ALLOWED!

1. The platelet membrane component involved in many steps in clot formation is the:
A. Glycoproteins C. thrombosthenin
B. Phospholipids D. actin and myosin
2. The rate-limiting step in clot formation is the:
A. formation of the prothrombin converting complex (PCC)
B. conversion of prothrombin to thrombin by the PCC
C. conversion of fibrinogen to fibrin
D. stabilization of fibrin
3. A blood clot is composed of a meshwork of fibrin fibers together with the entrapped:
A. platelets, red blood cells, white blood cells and plasma
B. platelets, red blood cells, and white blood cells
C. platelets and red blood cells
D. platelets only
4. The ability of platelets to contract the clot is due to the presence of these components:
A. actin, myosin, thrombosthenin and prostaglandin
B. actin, myosin and thrombosthenin
C. actin and myosin
D. actin
5. Tissue thromboplastin complexes with this factor to activate factor X.
A. factor V C. factor VIII
B. factor VII D. factor IX
6. This coagulation factor causes cross linkages between adjacent fibrin fibers to
strengthen the clot.
A. Stuart Prower factor C. Christmas factor
B. Antihemophilic factor D. Fibrin-stabilizing factor
7. Ca++ is needed in the activation of these factors?
A. IX, X, XI and XII C. IX and X
B. IX, X and XI D. IX only
8. Plasma protein C which acts as an anticoagulant is activated by:
A. Thrombin C. prothrombin-thrombomodulin cmplex
B. Thrombomodulin D. thrombin-thrombomodulin complex
9. The effectiveness of antithrombin III as an anticoagulant is greatly enhanced by:
A. Plasmin C. heparin
B. protein C D. calcium
10. Which of the following associations between factors/substances and their binding
sites on platelet membrane is correct?
A. fibrinogen – GP IV C. fibrinogen – IIb-IIIa
B. collagen – GP Ib-IX D. von .Willebrand factor - Ia-IIa
11. As regards formation of the temporary hemostatic plug, which of these statements is
true?
A. Vasodilation immediately follows blood vessel injury.
B. von Willebrand factor links platelets to collagen.
C. Fibrinogen is activated as a result of exposure to collagen.
D. Platelets become flattened discs and are spiculated
12. Which of these is primarily responsible for secondary platelet aggregation?
A. thromboxane A2 C. fibrinogen
B. von Willebrand factor D. thrombospondin

13. These factors act as cofactors in the intrinsic pathway of coagulation.


A. factors V, VIII, and Williams factor
B.    factors  V,  VII,  VIII,  and  William’s  factor
C.    factors  XI,  IX,  VIII,  and  William’s  factor
D.    factors  V,  IX,  XIII,  and  William’s  factor
14. Thrombin activates these factors/substances.
A. factors XIII, X, V, II, and plasminogen
B. factors I, V, VIII, IX and plasminogen
C. factors I, II, V, VIII and plasminogen
D. factors
15. The complex in the intrinsic pathway that activates factor X is composed of:
A. factors XIa, VIIIa, platelet phospholipid (PL), and Ca++
B. factors IXa, XIa, PL and Ca++
C. factors IXa, VIII, PL and CA++
D. factors XIIa, XIa, PL and Ca++
16. This is a characteristic of platelets.
A. are also involved in secondary hemostasis
B. when activated, they become flattened discs
C. normal concentration in the blood is 100,000 – 250,000 per cu mm
D. make direct membrane to membrane contact during platelet aggregation
17. Rose was suspected of having a hemorrhagic disorder. Laboratory tests were done
and these were the results: Bleeding time (BT) = normal; activated thromboplastin
time (APTT) = normal; prothrombin time (PT) = prolonged; thrombin time (TT) =
normal. Based on these results, we may that Rose is most likely deficient in
this/these factor/s.
A. factors VII, V, X and II C. factors VII and V
B. factors VII, V and X D. factor VII only
18. Aton is known to be suffering from Christmas factor deficiency. If these laboratory
tests are done, which will be prolonged?
A. BT C. PT
B. APTT D. TT
19. Which of the following tests would be prolonged in the presence of von Willebrand
factor deficiency?
A. BT C. PT
B. APTT D. TT
20. Which of these substances is/are inhibitory to both coagulation and fibrinolysis
A. antithrombin III, alpha 2 macroglobulin, alpha 1 antitrypsin, protein C and S
B. antithrombin III, alpha 2 macroglobulin, alpha 1 antitrypsin
C. antithrombin III, alpha 2 macroglobulin
D. antithrombin III
21. In the lab exercise to you performed, you placed one drop each of anti-A and anti-B
typing serum. How much blood should you add to the typing sera to get optimum
results?
A. one-half drop C. one drop
B. one-forth drop D. two drops
22. The blood pressure (BP) of Nikki is 120/80 mmHg. To do a tourniquet test, you must
inflate the BP cuff to _______ mmHg and maintain this pressure for five minutes.
A. 120 C. 100
B. 110 D. 90
23. Which of the following genes of the ABO system determines the existence of the
basic precursor on red cells?
A. A gene C. H gene
B. B gene D. Z gene
24. Which of the following ABO genes is involved in activating galactosyl transferase?
A. A gene C. H gene
B. B gene D. Z gene

25. Which of the following genes of the Rh system is of doubtful existence?


A. D gene C. d gene
B. c gene D. C gene
26. Which of the following genes of the ABO system are egalitarian?
A. A and B genes C. B and O genes
B. A and O genes D. H and Z genes
27. Which of the following is most likely to take place if a type A newborn is exposed to
agglutinins of a type B individual?
A. an antigen-antibody reaction resulting in agglutination
B. the newborn will develop anti-B
C. the newborn will develop anti-A
D. nothing, since agglutinins are antibodies and not antigens
28. Which of the following is most likely to take place if a D positive individual is
transfused with D negative blood?
A. an antigen-antibody reaction between D antigen and anti-D agglutinin
B. nothing if it is the first exposure to D negative blood
C. nothing because D agglutinins are not naturally occurring
D. hemolytic anemia because D antigen is very antigenic
29. A medical student wants to identify the ABO agglutinins present in the serum of his
classmate. Which of the following would be most useful?
A. mixing of known cells with unknown sera at 20 oC
B. mixing of known sera with unknown cells at 20 oC
C. mixing of known cells with unknown sera at 37 oC
D. mixing of known sera with unknown cells at 37oC
30. In the Philippines, which of the following procedures in Rh typing is most practical?
A. mixing of anti-D with unknown cells
B. mixing of anti-e with unknown cells
C. mixing of D cells with unknown sera
D. mixing of e cells with unknown sera
31. Which of the following anti-sera is most useful in detection of sub-types of A?
A. anti-A C. lectin from Ulex europaeus
B. anti-A1 D. group O cells
32. To make sure that the results of Rh typing are reliable, the best time to perform Rh
typing is:
A. immediately after birth C. 3-6 months after birth
B. anytime after birth D. upon adolescence
33. Which of the following antibodies is detected only during the anti-globulin phase of
the cross-match?
A. anti-A C. anti-K
B. anti-M D. anti-E
34. Cryptagglutinoids are best detected in the:
A. minor cross-match C. thermo phase
B. anti-globulin phase D. protein phase
35. Which of the following is a statement of the third law of Mendelian inheritance?
A. A parent of group O cannot have a child of group AB
B. A parent of group AB cannot have a child of group O
C. factor A cannot be present in a child unless present in one parent
D. factor B cannot be present in a child unless present in one parent
36. In order to prevent problems in blood grouping, which of the following should be
done when performing blood typing of a patient with multiple myeloma?
A. perform the test at 37oC C. gentle agitation of the mixture
B. perform the test at 4oC D. perform anti-globulin test
37. Which of the following is most likely to occur in an individual with bacterial infection?
A. rouleaux formation C. polyagglutination
B. non-agglutination D. panagglutination

38. The results of blood typing of an individual showed agglutination with A cells and
anti-B only. The individual is:
A. type A C. type AB
B. type B D. type O
39. The results of blood typing of an individual showed absence of agglutination with B
cells, O cells and anti-A only. The individual is:
A. type A C. type AB
B. type B D. type O
40. The results of blood typing of a Filipino male showed absence of agglutination with A
cells, B cells, O cells, anti-c and anti-E. The phenotype is most probably:
A. type O, cde C. type O, CDe
B. type AB, CDE D. type AB, Cde
41. If the parents are genotypes rr and Ror’  respectively,   the   most   possible   genotype   of  
50% of the offsprings would be:
A. cde/cde C. Cde/cDe
B. cde/Cde D. cDe/cDe
42. If the genotypes of the parents are rr and RoRz respectively, the phenotype of 50%
y

of the offsprings would be:


A. ccdee C. CcDEE
B. ccDee D. CcDEe
43. The parents are homozygous for A and B respectively. This means that all of their
children would be:
A. type A C. type AB
B. type B D. type O
44. Which of the following results of Rh typing of pregnant women will most likely result
in isoimmunization to the D factor?
A. anti-Rho (+), anti-rh’  (+),  anti-rh”  (+),  anti-hr’  (+),  anti-hr”  (+)    
B. anti-Rho (+), anti-rh’  (+),  anti-rh”  (+),  anti-hr’  (-), anti- hr”  (+)
C. anti-Rho (+), anti-rh’  (-), anti-rh”  (-), anti-hr’  (+),  anti-hr”  (+)
D. anti-Rho (-), anti-rh’  (+),  anti-rh”  (+),  anti-hr’  (-), anti-hr”  (-)
45. Which of the following results of Rh typing of pregnant women will most likely result
in isoimmunization to the c factor?
A. anti-Rho (-), anti-rh’  (+),  anti-rh”  (+),  anti-hr’  (-), anti-hr”  (+)
B. anti-Rho (+), anti-rh’  (-), anti-rh”  (+),  anti-hr’  (+),  anti-hr”  (+)
C. anti-Rho (-), anti-rh’  (+),  anti-rh”  (-), anti-hr’  (+),  anti  hr”  (+)
D. anti-Rho (+), anti-rh’  (+),  anti-rh”  (-), anti-hr’  (+),  anti-hr”  (+)
46. A   mother   is   type   AB,   cDe   and   her   baby   is   type   A,   Cde.   The   father’s   blood   type   is  
most probably:
A. type O, CcDEe C. type O, Ccdee
B. type A, CCDEE D. type A, ccDee
47. Which of the following potential blood donors can donate blood to a type A, cde
patient?
A. anti-A (+), anti-B (-), anti-D (-), anti-c (+), anti-e (-)
B. anti-A (-), anti-B (+), anti-D (-), anti-c (+), anti-e (+)
C. anti-A (+), anti-B (-), anti-D (-), anti-c (-), anti-e (+)
D. anti-A (+), anti-B (-), anti-D (-), anti-c (-), anti-e (-)
48. Which of the following potential blood donors should not donate but may donate
blood to a type A patient if there is no other blood available?
A. anti-A (+), anti-B (-), A cells (-), B cells (+), O cells (-)
B. anti-A (-), anti-B (+), A cells (+), B cells (-), O cells (-)
C. anti-A (-), anti-B (-), A cells (+), B cells (+), O cells (-)
D. anti-A (+), anti-B (+), A cells (-), B cells (-), O cells (-)
TEST II

1. Diagram the second stage of the coagulation pathway. 2pts


14. Which of the following is most likely to take place in a person with extensive burns?
A. decrease in ECF volume, normal ECF osmolarity
B. decrease in ECF volume, decrease in ECF osmolarity
C. decrease in ECF volume, increase in ECF osmolarity
D. increase in ECF volume, decrease in ECF
14. Pure buffer response to a primary gain in HCO3- results in the following directional
changes when the new equilibrium is reached, EXCEPT: A
C. PCO2 = increased C. [BB] = increased
D. [HBuf] = decreased D. pH = increased
15.Renal formation of new HCO3- is a compensatory response in: C
A. metabolic acidosis C. respiratory acidosis
B. metabolic alkalosis D. respiratory alkalosis
16.Ten year old Tom was admitted to the hospital because of severe diarrhea. Aside
from dehydration, he also had an acid-base disturbance. Body responses to such a
disturbance would include following, EXCEPT: D
E. hyperventilation
F. renal addition of new HCO3-
G. renal excretion of acid
H. buffering by the bicarbonate and non-bicarbonate systems
17. Marie was admitted to the hospital during an asthmatic attack. Initial laboratory
tests showed mild acid-base disturbance. Three days later repeat lab tests were
done and the results were the following: pH = 7.39; [HCO3-] = 25.89 mmls/L;
PCO2 = 44.97 mmHg; [BE] = (+) 1.9 mmols/L. Based on these second lab results,
you can say that Marie has a: D
E. completely compensated respiratory acidosis
F. partially compensated respiratory acidosis
G. completely compensated metabolic alkalosis
H. normal acid-base status
18. PCO2 is increased in the following conditions, EXCEPT: A
A. uncompensated respiratory acidosis
B. partially compensated respiratory alkalosis
C. partially compensated metabolic alkalosis
D. completely compensated metabolic alkalosis
19. Which of the following is true of the buffers systems in the body fluids?
A. these are evenly distributed between the red cells and plasma
B. the single most important buffer is the plasma HCO3 buffer system
C. the greatest contributor to the NBBS is the proteins
D. both the BBS and NBBS buffer losses of HCO- and CO2
Module 31 Brain Stem

1. Which of these cranial nerves is both sensory and motor?


A. Trochlear C. facial
B. Hypoglossal D. vestibulocochlear
2. Which of the following associations between nuclei and cranial nerves is correct?
A. dorsal motor nucleus – glossopharyngeal nerve
B. inferior salivatory nucleus – facial nerve
C. superior dalivatory nucleus – vagus
D. Edinger-Westphal nucleus – oculomotor nerve
3. This is true of the trigeminal nerve.
A. has three nuclei
B. the smallest cranial nerve
C. sensory component carries sensations of touch and pressure
D. motor component supplies the muscles of the lower part of the face
4. This cranial nerve supplies the GIT from pharynx to splenic flexure of colon.
A. Hypoglossal C. glossopharyngeal
B. Vagus D. accessory cranial root
5. Which of these cranial nerve nuclei receive afferent fibers from the semicircular
canals of the ear?
A. Facial C. cochlear
B. Trigeminal D. vestibular
6. One of the manifestations of lateral medullary syndrome of Wallenberg is dysphagia.
This is due to the involvement of this structure in the lateral part of the medulla
oblongata :
A. nucleus ambiguus
B. vestibular nuclei
C. nucleus of the trigeminal nerve
D. spinal lemniscus-spinothalamic tract
7. Weber’s  syndrome  is  characterized  by  ipsilateral  ophthalmoplegia  and  contralateral  
paralysis of the lower part of the face, the tongue and the arm and leg. These
manifestations are due to involvement of the:
A. medial lemniscus C. nucleus and spinal tract of trigeminal nerve
B. red nucleus D. oculomotor nerve and the crus cerebri
8. Contralateral hemiparesis which is a manifestation of medial medullary syndrome is
due to involvement of the:
A. medial lemniscus C. spinothalamic tract
B. pyramidal tract D. medial longitudinal fasciculus
9. A lesion of the right corticonuclear fibers innervating the facial nerve will result in
paralysis of the:
A. entire right side of the face C. lower part of the right side of the face
B. entire left side of the face D. lower part of the left side of the face
10. Which of the following manifestations of pontine hemorrhage will make you highly
suspect that the pons is severely damaged?
A. facial paralysis on the side of the lesion
B. paralysis of the limbs on the opposite side
C. conjugate ocular deviation
D. poikilothermia
11. This condition is due to necrosis of the medial lemniscus and red nucleus and is
characterized by contralateral hemianesthesis and involuntary movements of the limbs
of the opposite side.
A. infarction of the pons C. Arnold-Chiari phenomenon
B. Benedikt’s  syndrome D. blockage of cerebral aqueduct

Module 34 Quiz

1. As regards Herpes zoster (HZ), which of the following is NOT true?


A. condition is not fatal
B. caused by human herpes virus-3
C. usually occurs in older patients or immunocompromised patients
D. occurs in patients with or without previous varicella infection
2. The following are the usual manifestations of herpes zoster, EXCEPT:
A. high grade fever with general body malaise
B. painful vesicular eruptions in a dermatomal distribution
C. light touch of affected area can lead to severe pain
D. pain is minimal or absent in patients less than 50 years of age
3. The persistence of pain even after the eruptions are healed is known as postherpetic
neuralgia. This could be due to:
A. injury to peripheral nerves
B. hyperexcitability of peripheral nociceptors
C. functional changes in central neurons that receive input from primary afferents
D. all of the above
4. The herpes zoster virus are usually harbored in these locations where they remain
dormant for several years:
A. geniculate, trigeminal, dorsal root ganglion, anterior root ganglion
B. geniculate, trigeminal, dorsal root ganglion
C. geniculate, trigeminal
D. geniculate
5. If you have a patient with the typical manifestations of herpes zoster, which laboratory
studies should you request/perform to help you in the diagnosis and management of
the case?
A. X-ray of affected part C. MRI of the brain and spinal cord
B. CSF evaluation D. none of the above
6. Which of the following drugs may be given to patients with herpes zoster to help
shorten the clinical course, prevent complications and prevent recurrences?
A. Amitryptyline C. lidocaine
B. famciclovir D. dexamethasone
Module 35

1. Which of these is true of sound waves? A


A. it is produced by a vibrating source
B. it travels in waves and straight lines
C. it can be transmitted through air and through a vacuum
D. its frequency determines the loudness of sound
2. Which of the following events occur in air conduction but not in bone conduction of
sound? A
A. vibration of the tympanic membrane
B. vibration of the perilymph
C. vibration of the endolymph
D. vibration of the organ of Corti
3. These events lead to stimulation of the hair cells in the organ of Corti, EXCEPT: C
A. fluid in inner ear vibrates C. stereocilia bend towards the limbus
B. organ of Corti moves up D. K+ channels open
4. Auditory masking is greatest when the masking tone is: C
A. at higher level than the masked tone
B. identical in frequency to that of the masked tone
C. close but not identical in frequency to that of the masked tone
D. presented to one ear and the masked tone is presented to the other ear
5. The following events could lead to stimulation of the right horizontal semicircular
duct, EXCEPT: D
A. the head is turned to the right
B. the stereocilia are bent towards the utricle
C. the head has just started to turn
D. the head moves at constant speed
6. Pedro was sitting on a swivel chair when his friend suddenly rotated it to the right.
Which of the following would Pedro experience as a result? A
A. nystagmus to the right at the start of rotation
B. nystagmus continues until the rotation stops
C. nystagmus to the right at the end of rotation
D. all of the above
7. JR was staring at the family picture on the wall in front of him. He turned his head
about 40 degrees to the right, but kept his gaze on the picture. This action results in
stimulation of the: B
A. left horizontal semicircular duct C. left oculomotor nerve
B. left abducens nerve D. none of the above
Module 35

1. Which of the following is characteristic of sound?


A. sound energy travels in waves
B. one wavelength is measured from the crest to next trough
C. the higher the amplitude of the sound waves, the louder the sound
D. humans hear most keenly sounds with frequency between 100 to 4000 Hz
2. Which of these events would cause the depolarization of the cochlear hair cell?
A. relaxing the tip links between hair cells
B. bending of the stereocilia away from the limbus
C. bending of the stereocilia towards the kinocilium
D. pushing the basilar membrane towards the scala vestibuli
3. In which of the following situations is masking most likely to occur?
A. frequency of masked tone is identical to that of the masking tone
B. frequency of masked tone is close to that of the masking tone
C. amplitude of masked tone is close to that of the masking tone
D. masked and masking tones are presented to different ears
4. Localization to the left ear occurs when:
A. sound wave frequency reaching left ear is greater than that in right ear
B. sound waves travel faster at the left side than at the right side
C. sound waves reach the left ear earlier than the right ear
D. all of the above
5. A high school coed attempted to copy a ballerina, rotated in a counterclockwise
direction and collapsed immediately after. If the eyes were examined immediately
after the rotation, which of the following is most likely to be exhibited by the coed?
A. nystagmus to the left C. nystagmus to the left then to the right
B. nystagmus to the right D. nystagmus to the right then to the left
6. A 21 year old student was traveling by bus from Bukidnon to Cagayan de Oro. As she
was about to get off from the bus, she noticed that as her right foot was about to touch
the ground, she felt a stiffening of her right thigh and leg. This is most probably due to
stimulation of the:
A. horizontal semicircular duct C. utricle
B. vertical semicircular duct D. saccule
7. Which of the following is most likely to give rise to putrid odor?
A. rotten eggs C. urine
B. feces D. vinegar
8. Which of the following is/are characteristic/s of olfactory receptor cells?
A. these are bipolar neurons
B. receptors for odorant molecules are located in their cilia
C. may generate an action potential when depolarized to threshold
D. all of the above
9. Which of the following will give rise to decreased sensitivity of olfactory receptors?
FINALS
A. Attentiveness C. sex
B. Hunger D. smoking
10. Which of the following is a characteristic of taste receptor cells:
A. these are located in taste buds which are mostly found in gums
B. increased intracellular calcium occurs only when these are depolarized
C. increased intracellular calcium triggers release of neurotransmitters
D. these generate action potentials when stimulated to threshold
11. Which of the following taste sensations is most likely to be activated by a second
messenger system?
A. Sour C. bitter via bitter 1 receptor
B. Sweet D. umami
12. Lola Maria noticed that her taste sensitivity has been diminished for the last few
months. Her condition is known as: FINALS
A. anosmia C. ageusia
B. hyposmia D. hypogeusia

BB MUSCLE QUIZ

1. When the myofibril contracts, the following structures shorten.


A. sarcomere, H band, I band, and A band
B. sarcomere, H band, and I band only
C. sarcomere and H band only
D. sarcomere only
2. Actin filaments but not myosin filaments are found in this/these structure/s.
A. A band C. I band
B. H band D. I band and H band
3. As regards myosin filaments, which of the following is true?
A. filaments are arranged in a head-to-tail configuration
B. filaments are attached to the Z line by the protein Titin
C. heads are found at the two ends of the filaments
D. tropomyosin covers the heads of the myosin
4. Which of the following is true regarding the structure of the triad in skeletal muscles?
A. Formed by one cisterna of sarcoplasmic reticulum (SR) & 2 adjacent T-tubules
B. The space between cisterna and T-tubule is filled with extracellular fluid
C. One dihydropyridine receptor in T-tubule faces one ryanodine receptor in SR
D. Dihydropyridine receptors are voltage-gated L-type calcium channel
5. To effect release of Ca++ from the SR, the depolarization of the T-tubule is
immediately followed by:
A. Opening of calcium-release channels
B. conformational change in the L-type calcium channels
C. conformational change in the calcium release channels
D. coupling between the L-type calcium channels and Ca++ release channels
6. Which of the following immediately precedes binding of myosin heads to binding sites
in actin?
A. binding of calcium to Troponin-C
B. binding of ATP to myosin heads
C. hydrolysis of ATP on myosin heads
D. exposure of myosin binding sites in actin
7. Which of these causes detachment of myosin heads from actin?
A. Binding of new ATP to myosin head
B. Hydrolysis of ATP to ADP and Pi
C. tilting of the myosin head away from actin
D. removal of ADP and inorganic phosphate from myosin head
8. Muscle relaxation begins when:
A. new ATP binds to myosin head
B. Ca++ is pumped out of the cell
C. Ca++ is pumped back into the SR
D. Ca++ enters cell through L-type calcium channels

THIRD BIMONTHLY EXAMINATION

PORPHYRINS
1. Metal ions like iron bind to this atom in the pyrrole ring
A. Carbon C. Nitrogen
B. Oxygen D. Hydrogen
2. Which of the following is a naturally occurring porphyrin containing 4 acetate and 4
propionate side chains?
A. Protoporphyrins C. Coproporphyrins
B. Uroporphyrins D. None of the above
3. The following are true of heme biosynthesis, EXCEPT
A. begins with succinyl CoA & glycine
B. glycine is activated by pyridoxal phosphate
C. the rate-limiting step is catalyzed by a dehydratase
D. does not occur in red blood cells
4. Which is not true of erythropoietic protoporhyria?
A. enzyme involved is a uroporphyrinogen III cosynthase
B. photosensitivity is a major complaint
C. presence of fecal and red cell protoporphyrin
D. happens when conversion of protoporphyrin to heme is blocked
5. A complex enzyme system responsible for heme catabolism
A. Synthases C. Oxygenases
B. Dehydratases D. Carboxylases

LIPIDS
1. Which of the following is true of fatty acid synthesis?
A. major carbon source is dietary galactose
B. happens in the mitochondria and cytosol
C. involves carboxylation, condensation and reduction reactions
D. NADPH is sourced from the citrate shuttle and PP pathway
2. The following are descriptive of Acetyl-CoA carboxylase, EXCEPT
A. rate limiting enzyme in fatty acid synthesis
B. it is by inhibited by citrate
C. palmitoyl CoA prevents its polymerization
D. regulated by phosphorylation
3. The following are products of the oxidation of fatty acyl CoA, EXCEPT
A. acetyl CoA C. NADH
B. AMP D. FADH2
4. Which of the following paired enzymes catalyze the first two reactions in ketogenesis?
A. carnitine acyltransferase and acetyl CoA carboxylase
B. cholesterol acyltransferase and HMG CoA reductase
C. thiolase and HMG CoA synthase
D. ferrochelatase and UDPG transferase
5. Which of the following enzymes is not involved in ketone oxidation?
A. β-OHbutyrate dehydrogenase
B. succinyl CoA:acetoacetate CoA transferase
C. thiolase
D. HMG CoA synthase

CARBOHYDRATES
1. Skeletal muscles cannot release glucose into the blood after gluconeogenesis
because of the absence of which of the following enzymes
A. pyruvate carboxylase C. glucokinase
B. glucose 6 phosphatase D. fructokinase

2. Conversion of pyruvate to oxaloacetate is catalyzed by pyruvate carboxylase in the


presence of the following, EXCEPT
A. Water C. Biotin
B. ATP D. Carbon dioxide
3. The following are descriptive of mechanisms of homeostatic glucose regulation,
EXCEPT
A. Almost all of the extrahepatic tissue cells are relatively impermeable to glucose
B. Hexokinase phosphorylates glucose that may enter extrahepatic tissues
C. (-) feedback is exerted by inhibition of hexokinase by glucose 6-PO4
D. Insulin causes glycogenolysis by activating phosphorylase
4. The following enzymes catalyze irreversible reactions which prevent reversal of
glycolysis, EXCEPT
A. pyruvate carboxylase C. fructose 1,6-biphosphatase
B. PEP carboxylase D. glucose 6-phosphatase
5. If a patient is glycosuric his blood glucose is
A. >9.5-10 mmol/L C. <10 mmolL
B. 8-10 mmol/L D. 6-10 mmol/L

PROTEINS
1. The following are descriptive of protein degradation, EXCEPT
A. proteins with short half lives follow the PEST sequence
B. all of nitrogen form urea
C. excess proteins are degraded not stored
D. ubiquitin is needed for degradation of short-lived proteins
2. Which of the following happens first in amino acid catabolism?
A. Transamination C. Translation
B. Deamination D. Hydration
3. The following are end-products of glucogenic amino acid catabolism, EXCEPT
A. α-ketoglutarate C. acetyl CoA
B. fumarate D. succinyl CoA
4. Which of the following amino acids involves a non-enzymatic reaction in its
catabolism?
A. Aspartate C. Arginine
B. Glutamine D. Proline

GENETICS
1. Which of the following enzymes catalyze the formation  of  5’phosphoribosylamine  in  
purine synthesis?
A. glutamine PRPP amidotransferase
B. ribonucleotide reductase
C. thioredoxin reductase
D. carbamy phosphate synthetase
2. Which is of the following is not true of the salvage pathway of purines?
A. its role is to avoid high energy demand on the cell during synthesis
B. requires only 4 high energy PO4 bonds
C. catalyzed by the enzymes HGPR Trasnferase & APR Transferase
D. APRT has the highest activity compared to HGPRT
3. The primary regulatory enzyme of pyrimidine nucleotide synthesis in eukaryotic cells
A. OPR Transferase C. CTP synthase
B. CP synthetase-2 D. APR Transferase

4. Which of the following enzyme catalyze the reaction of adding a methyl group to
dUMP to form dTMP?
A. Adenylate cyclase C. Thymidylate cyclase
B. Nucelotidase D. Kinase
5. The following are removed from the ring during pyrimidine nucleotide catabolism,
EXCEPT
A. amino groups C. phosphate
B. ribose D. carbon dioxide
6. The following are descriptive of gout, EXCEPT
A. Symptoms appear as blood uric acid levels increase
B. Elevated blood levels of uric acid
C. Monosodium urate crystals deposit in tissues
D. Increased incidence in females after menopause

ENDOCRINE
1. Which of the following is not true of the function of insulin?
A. increases glucose entry to adipose tissue
B. increases glycogen synthesis in muscle tissue
C. decreases hepatic ketogenesis
D. increase release of glucose by liver cells
2. The high hemoglobin levels of people who live in Baguio is an example of
A. Adaptation C. Biologic rhythm
B. Acclimatization D. Aging

MRS
1. The following are the effects of dihydrotestosterone, EXCEPT
A. stimulates the Sertoli cells to secrete MIF
B. catalyzed  from  testosterone  via  the  action  of  5  α  reductase
C. influences the differentiation of the genital folds to corpus spongiosum
D. controls the changing of the genital tubercle into the penis in males
2. The Wolffian duct differentiates into the following, EXCEPT
A. Epididymis C. Seminal vesicles
B. Ductus deferens D. Prostate
3. The following are microscopic features of the testes, EXCEPT
A. fibrous septa from the tunica albuginea divides the testes into lobules
B. seminiferous tubules are enmeshed in loose connective tissue
C. a complex stratified epithelium lines the tubules
D. spermatogenic cells are arranged in layers
4. Which of the following is not true of the seminal vesicles?
A. folded mucosa lined by cuboidal epithelium
B. has an elastic fiber-rich lamina propria
C. secretes a viscid and yellowish secretion
D. tubuloalveolar glands
5. The following are descriptive of the prostate, EXCEPT
A. branched tubuloalveolar glands
B. its central zone occupies 70% of the gland
C. has a rich fibromuscular stroma
D. covered by a fibroelastic capsule
6. All of the following describe the penis, EXCEPT
A. contains three cylindrical masses
B. ventrally located corpus spongiosum
C. mostly pseudostratified columnar epithelium lines the penile urethra
D. contains serous-secreting Littre glands

7. The following are true of the functions of Sertoli cells, EXCEPT


A. secretes inhibin
B. supports mechanically developing sperm
C. secretes mitosis inhibiting factor in fetal life
D. secretes androgen-binding protein

FRS-1
1. The following are descriptive of the ovary, EXCEPT
A. divided into cortex and medulla
B. its cortex contain follicles of varying shapes
C. loose connective & blood vessels are found in the medulla
D. lined by a continuous sheet of squamous epithelium
2. Which of the following is not true of the oviduct?
A. has a mesothelium-lined outermost layer
B. elaborately branched mucosal folds
C. its middle layer contains skeletal muscles
D. its mucosa is line d by simple columnar ciliated epithelium
3. All of the following is true of the uterus, EXCEPT
A. the myometrium is divided into 4 sublayers
B. mucosal epithelium is made up of secretory and ciliated cells
C. its outermost layer is mostly covered by peritoneum
D. the endometrial sublayers are sloughed off during menses
4. The following describe the vagina, EXCEPT
A. lined by keratinized stratified epithelium
B. two types of connective tissues are seen in the lamina propria
C. absence of mucosal glands and sensory nerve endings
D. circular and longitudinal smooth muscles in the middle layer
5. The following are true of the second stage of oogenesis, EXCEPT
A. starts after menarche
B. formation of the antrum
C. the oocyte is covered by theca cells
D. formation of the antral follicle
6. The following are the components of a corpus luteum, EXCEPT
A. granulose cells C. theca cells
B. elastic fibers D. fibroblasts

FRS-2
1. Which of the following sequential order of development of the zygote in the first week
of gestation is true?
A. zygote→morula→blastocyst→yolk  sac
B. zygote→blastomere→blastocyst→morula
C. blastomere→morula→blastocyst→hypoblast
D. zygote→blastomere→morula→blastocyst
2. The following major events happen during the 4 th to the 8th week of gestation,
EXCEPT
A. all major organ systems form from three germ layers
B. formation of head and caudal eminence
C. the yolk sac dislodges from its stalk
D. expansion of the amnion envelops the yolk stalk and allantois
3. Completion of implantation of the blastocyst in the endometrium, happens in which
week of gestation?
A. 1st B. 2nd C. 3rd D. 4th
4. The following are descriptive of placental development, EXCEPT
A. the placenta and the fetal membranes originate from the zygote
B. the deciduas basalis forms the fetal part of the placenta
C. the villous chorion is the larger fetal placental part
D. presence of a smooth chorion, the avascular bare area
5. Appearance of rapid eye movements and the blink-startle reflex characterize which
time of fetal development?
A. 17-20 weeks C. 21-23 weeks
B. 24-25 weeks D. 13-16 weeks

XAVIER UNIVERSITY – JOSE P RIZAL COLLEGE OF MEDICINE


Basic Biomedical Sciences
Third Bimonthly Period
QUIZ # 1

NAME ____________________________ 12 Nov 2007 SCORE ____________


MULTIPLE CHOICE: write the letter of the BEST answer on the answer sheet. Use capital
letters in blue/black ink. NO EASURES/SUPERIMPOSITIONS ALLOWED!

1. Which of the following is true of porphyrins?


A. cyclic compounds of four pyrrole rings linked by methyne bridges
B. form complexes with metal ions bound to one of the side chains
C. have six hydrogen atoms which may be substituted
D. only one type of side chain may be present in one porphyrin
2. The fluorescence emitted by porphyrins is due to the:
A. nitrogen atoms of the pyrrole rings
B. cyclic arrangement of the pyrrole rings
C. asymmetric arrangement of the side chains
D. double bonds linking the pyrrole rings
3. The substituents of coproporphyrin are:
A. methyl and acetate C. methyl and propionate
B. acetate and propionate D. all acetates
4. Uroporphyrin I and uroporphyrin III differ in that the substituents of:
A. uroporphyrin I are acetate and propionate
B. uroporphyrin III are asymmetrically arranged in ring IV
C. uroporphyrin I are asymmetrically arranged in ring IV
D. uroporphyrin III are asymmetrically arranged in ring I
5. Which of the following is the most abundant type of porphyrin found in nature?
A. type I C. type III
B. type II D. type IV
6. What type of porphyrin is the heme of hemoglobin?
A. type I C. type III
B. type II D. type IV
7. This type of porphyria is inherited in an autosomal recessive manner.
A. ALA dehydratase deficiency C. acute intermittent porphyria
B. porphyria cutanea tarda D. congenital erythropoietic porphyria
8. Which of these conditions is characterized by photosensitivity, abdominal pain and
neuropsychiatric symptoms?
A. Protoporphyria C. porphyria cutanea tarda
B. variegate porphyria D. hereditary coproporphyria
9. Which of these is an erythropoietic type of porphyria?
A. Protoporphyria C. hereditary coproporphyria
B. variegate porphyria D. acute intermittent porphyria
10. Which of the following associations between type of porphyria and enzyme involved is
correct?
A. porphyria cutanea tarda – uroporphyrinogen decarboxylase
B. acute intermittent porphyria – protoporphyrinogen oxidase
C. hereditary coproporphyria – ferrochelatase
D. protoporphyria – ALA dehydratse

XAVIER UNIVERSITY – JOSE P RIZAL COLLEGE OF MEDICINE


Basic Biomedical Sciences
Third Bimonthly Period
QUIZ # 3 (Proteins)

NAME ____________________________ 26 Nov 2007 SCORE ____________


MULTIPLE CHOICE: write the letter of the BEST answer on the answer sheet. Use capital
letters in blue/black ink. NO EASURES/SUPERIMPOSITIONS ALLOWED!

1. This  amino  acid  has  to  be  included  in  a  person’s  diet  because  it  cannot  be  synthesized  
by the human body?
A. Alanine C. leucine
B. Cysteine D. glutamine
2. Which of the following is true as regards protein digestion?
A. Enzymatic digestion of proteins starts in the stomach
B. Most digestion of proteins is completed in the large intestine
C. Pepsin splits denatured protein to its component amino acids
D. Dipeptidases complete digestion of dipeptides to free amino acids
3. Alanine, glycine, glycine and many other amino acids are transported across the
intestinal mucosal cells via this transport system.
A. neutral amino acid C. dibasic aminoacid
B. acidic amino acid D. imino acids and glycine
4. This is true of the digestion and absorption of proteins. - D
A. Amino acids are passively transported across luminal border of cells
B. Pepsin, a pancreatic enzyme, starts digestion of protein to large peptides
C. digestion and absorption is mainly completed in the large intestine
D. short chains of two or three amino acids may also be absorbed

XAVIER UNIVERSITY – JOSE P RIZAL COLLEGE OF MEDICINE


Basic Biomedical Sciences
Fourth Bimonthly Period
QUIZ # 1 (Spinal Cord)

NAME ____________________ 28 January 2008 SCORE ____________


MULTIPLE CHOICE: write the letter of the BEST answer on the answer sheet. Use capital
letters in blue/black ink. NO EASURES/SUPERIMPOSITIONS ALLOWED!

11. Which of the following may not be present in a reflex arc?


A. afferent neuron C. effector organ
B. interneuron D. receptor organ
12. Spinal shock syndrome is followed by decerebrate rigidity which in turn is followed by
paraplegia in extension. This last condition is due to severance of all descending
tracts to the spinal cord but with persistence of which of these tracts?
A. vestibulospinal tract C. reticulospinal tract
B. corticospinal tract D. tectospinal tract
13. Which of the following associations between reflex and center is correct?
A. biceps reflex – C5, C6 C. cilio-spinal reflex – C7, C8
B. wrist reflex – C4, C5 D. plantar reflex – S3, S4
14. Which of the following associations between reflex and normal response is correct?
A. triceps reflex – flexion of forearm
B. corneal reflex – opening of eyes
C. patellar reflex – flexion of knee
D. ankle reflex – plantar flexion of foot
15. The pyramidal tract refers to the:
A. vestibulospinal tract C. reticulospinal tract
B. corticospinal tract D. tectospinal tract
16. Fibers of the corticospinal tract arise as axons of pyramidal cells in the cerebral cortex.
These fibers then descend and converge in the:
A. Pyramids C. internal capsule
B. corona radiate D. superior cerebellar peduncle
17. Fibers of the lateral corticospinal tract terminate in the anterior gray column of:
A. the spinal cord segments in the cervical and upper thoracic regions
B. the spinal cord segments in the upper thoracic region only
C. the spinal cord segments in the cervical region only
D. all the spinal cord segments
18. Fibers of the anterior corticospinal tract terminate in the anterior gray column of:
A. the spinal cord segments in the cervical and upper thoracic regions
B. the spinal cord segments in the upper thoracic region only
C. the spinal cord segments in the cervical region only
D. all the spinal cord segments
19. The afferent fast acting pain fibers stimulate the second-order neurons of the:
A. lateral spinothalamic tract C. posterior spinocerebellar tract
B. anterior spinothalaamic tract D. internal arcuate fibers
20. Axons of the second-order neurons of the light touch and pressure pathway form the:
A. lateral spinothalamic tract C. posterior spinocerebellar tract
B. anterior spinothalamic tract D. internal arcuate fibers
11. Which of the following is not true of the dermatome?
A. Area of skin supplied by a single spinal nerve
B. On the trunk, it extends round the body
C. Area of skin supplied by a single segment of spinal cord
D. Does not overlap with adjacent dermatomes
12. To produce complete anesthesia in the region of the trunk, at least____ contiguous
spinal nerves have to be sectioned:
A. One C. three
B. Two D. four
13. Wallerian degeneration refers to changes following injury:
A. That occur in the cell body
B. In the middle segment of the axon
C. Distally from the site of the lesion
D. In the proximal segment of the axon
14. Referred to as enlargements of the spinal cord:
A. Cervical and thoracic C. cervical and lumbar
B. Thoracic and lumbar D. thoracic and sacral
15. Which of the following is not true of the spinal cord?
A. Inferiorly tapers off into filum terminale
B. 31 pairs of spinal nerves are attached to it
C. Superiorly continuos with medulla oblongata
D. Surrounded by three meninges
16. In adult, the spinal cord terminates at the level of the lower border of the:
A. First lumbar vertebra C. third lumbar vertebra
B. Second lumbar vertebra D. fourth lumbar vertebra
17. The space occupied by the cerebrospinal fluid:
A. Subdural C. subarachnoid
B. Epidural D. arachnoid
18. The arterial blood supply of the spinal cord is derived from:
A. Internal carotid C. external carotid
B. Vertebral D. basilar
19. An ultrastructure seen in a X-section of a myelinated peripheral nerve fiber consisting
of 2 inner protein layers of the Schwann cell membrane fused together:
A. mesaxon C. major dense line
B. node of Ranvier D. incisure of Schmidt-Lanterman
20. The fiber type, classified according to size, of a nonmyelinated peripheral nerve
fiber, 0.4-1.2 um diameter, mediating deep pain & temperature:
A. A beta C. B
B. A delta D. C
21. The corticobulbar tract accompanies the corticospinal tract and terminates in:
A. spinal cord C. parasympathetic nuclei of brain stem
B. motor cranial nerve nuclei D. motor nuclei of diencephalon
22. A lesion of the right corticobulbar tract, at the level of the medulla, will most likely
cause affection of the:
A. face C. eye
B. tongue D. none of the above
23. The corticobulbar tract terminates only contralaterally on which cranial nerve
nucleus?
A. IV C. VI
B. V D. VII, supplying lower face
24. Lower motor neuron paralysis can result in a lesion of which structure?
A. motor area of cerebrum C. corticobulbar tract
B. motor cranial nerve nucleus D. corticospinal tract
25. One of the following signs characterize a lower motor neuron lesion:
A. muscle atrophy C. spasticity of affected muscles
B. (+) Babinski sign D. absent cremasteric reflex
26. The following clinical sign is suggestive of an upper motor neuron lesion:
A. muscular fasciculation
B. flaccid paralysis
C. muscular contracture
D. loss of performance of fine skilled voluntary movements
27. One of the following tracts is part of the extrapyramidal motor system:
A. reticulospinal tract C. corticobulbar tract
B. corticospinal tract D. corticopontine tract

28. Joe, a 47-year old security guard, was admitted due to a bullet wound in his left
lower back, level of L1. A flaccid paralysis of the muscles in the segment of
the lesion was noted. What is the likely structure affected?
A. ventral horn cells C. ventral corticospinal tract
B. lateral corticospinal tract D. corticobulbar tract
29. Sally came for consultation with the following clinical sign: spastic paralysis of
the left half of her body. The most likely structure involved is:
A. left ventral horn cells, SC C. right corticospinal tract, medulla
B. left corticopontine tract, pons D. right corticobulbar tract, midbrain
30. Following a motorcycle accident, Ben was admitted to the nearest hospital.
Physical examination revealed the following findings: spastic paralysis
of right lower extremity as well as flaccid paralysis of muscles, level of T12, right;
One-sided loss of sensations were also noted. What is the most likely
affection?
A. Central cord syndrome
B. Brown-Sequard syndrome
C. anterior cord syndrome
D. complete cord transection syndrome
31. Which of these structures is derived from neural crest cells?
A. gray matter of the spinal cord C. lateral gray column
B. posterior root ganglia D. pineal body
32. This structure develops from the midbrain vesicle.
A. hippocampus C. tegmentum
B. cerebellum D. basal ganglia
33. The third ventricle of the brain is located in the:
A. telencephalon C. mesencephalon
B. diencephalons D. metencephalon
34. The part of the brain which represents the main connection between the cerebral
hemispheres:
A. corpus callosum C. fornix
B. pons D. corona radiata
35. In the adult, the spinal cord ends at the level of
A. 12th thoracic vertebra C. 3rd lumbar vertebra
B. 5th lumbar vertebra D. 1st lumbar vertebra
36. Purkinje cells are found in the
A. cerebral cortex C. posterior root ganglia
B. cerebellar cortex D. gray matter of the spinal cord
37. Responsible for the formation of myelin in the central nervous system
A. astrocytes C. oligodendrocytes
B. Schwann cells D. tanycytes
XAVIER UNIVERSITY – JOSE P RIZAL COLLEGE OF MEDICINE
Basic Biomedical Sciences
Second Bimonthly Examination
PHYSIOLOGY

NAME ____________________________ 6 CTOBER 2008 SCORE ____________


MULTIPLE CHOICE: write the letter of the BEST answer on the answer sheet. Use capital
letters in blue/black ink. NO EASURES/SUPERIMPOSITIONS ALLOWED!

R S was suspected of having right bundle branch block (RBBB), a condition wherein the
impulse cannot pass through the right bundle of the bundle of His. Assuming that RS has
RBBB, answer the next 2 questions:
1. What heart sound/s may be split?
A. S 1 C. S 1 and S 2
B. S 2 D. no split sound is expected
2. Which of these ECG configuration/s is/are likely to be prolonged?
A. P wave C. P-R interval
B. T wave D. QRS interval
1. Blood flow to a particular organ increases when this is increased.
A. diameter of arterioles C. blood viscosity
B. number of open capillaries D. mean arterial pressure
2. Mean arterial pressure (MAP) tend to increase when this/these is/are increased.
A. heart rate, blood volume, hematocrit, and vasoconstriction
B. heart rate, blood volume and hematocrit
C. heart rate and blood volume
D. heart rate only
3. In the presence of right ventricular failure, which of the following will be decreased?
A. peripheral venous pressure C. venous return
B. central venous pressure D. lymph flow
4. The principal cause of the edema in varicose veins is the increase in:
A. mean arterial pressure C. central venous pressure
B. peripheral venous pressure D. capillary permeability
5. Increasing the following tends to increase venous return.
A. hfv
B. kjgfnd,

ACID-BASE
1. Which of the following is true of body water?
A. makes up about 80% of the body weight
B. most of the body water is found in the plasma
C. intracellular fluid includes fluid in all cells except that in blood cells
D. the percentage of body water in females is greater than in males
2. Most of the ECF is found in this compartment.
A. intravascular fluid compartment
B. interstitial fluid compartment
C. transcellular fluid compartment
D. lymph
3. Compared to interstitial fluid, the plasma (intravascular) has:
A. a higher pH C. more protein
B. more HCO3- D. more Na+
4. The volume of the extracellular fluid compartment is primarily dependent on its content
of:
A. Water C. chloride
B. Sodium D. protein
5. It was Rammadan and Jani being a Muslim, refused to drink water even though she
was already very thirsty. Which of these substances are reflexly increased at this
time to maintain her fluid and electrolyte balance?
A. Angiotensisn II and ADH
B. Angiotensin II and serotonin
C. atrial natriuretic factor and ADH
D. atrial natriuretic factor and angiotensin II
6. In preparation for an ultrasound examination of her abdomen, Vanni drank 1.5 liters of
distilled water. This is likely to result in:
A. An increase in ICF volume and osmolarity
B. An increase in ECF volume and osmolarity
C. A decrease in ECF osmolarity and a decrease in ICF osmolarity
D. A decrease in ECF osmolarity and an increase in ICF osmolarity
7. Infusion of one liter of 0.9% NaCl solution to Lina would most likely result in:
A. increased ECF volume without change in ECF osmolarity
B. increased ECF volume and decreased ECF osmolarity
C. increased ICF volume and decreased ICF osmolarity
D. increased ICF volume without change in ICF osmolarity
8. As regards buffer system, which of the following is true?
A. plasma bicarbonate buffer system (BBS) is the most important
B. inorganic phosphate is found only in the plasma
C. organic phosphates is found both in red cells and plasma
D. the nonbicarbonate buffer system (NBBS) is primarily due to plasma proteins
9. In which of the following conditions is the plasma [HCO3-] increased?
A. compensated respiratory alkalosis
B. compensated respiratory acidosis
C. uncompensated respiratory acidosis
D. uncompensated respiratory alkalosis
10. The following are the laboratory results of Ms Ace Demi who was suspected of having
diabetic acidosis. Which of these results will show that her acidosis is not
compensated?
A. pH = 7.32 C. PCO2 = 42 mmHg
B. [HCO3-] = 16.1 D. [BE] = (-) 8.1
11. As a result of pure buffer response to a primary gain in HCO3-, which of the following
parameters will be decreased at the new equilibrium?
A. pH C. [HBuf]
B. S.PCO2 D. [BB]
12. Body responses to the acid-base disturbance secondary to renal failure will result in:
A. decreased respiratory rate
B. acidification of urine
C. renal formation of new bicarbonate
D. buffering by the BBS and NBBS
13. Aside from dehydration, M S was suspected of suffering from an acid-base
disturbance secondary to severe diarrhea. Laboratory tests were ordered and these
are the results: pH = 7.301; [HCO3] = 10.1 mmol/L; [BE] = (-)16.5 mmol/L and
PCO2 = 22.52 mmHg. Based on her history and lab exams, she is likely to have:
A. partially compensated respiratory acidosis
B. uncompensated respiratory acidosis
C. partially compensated metabolic acidosis
D. uncompensated metabolic acidosis
14. R N, a first year medical student was shown this laboratory result of a friend who was
suspected of having an acid-base disturbance: pH = 7.474; [HCO3-] = 23.4 mmol; [BE]
= (-) 1.5 mmol/L; PCO2 = 28.67 mmHg. Based on these lab results, R N can say that
his friend is likely to have:
A. uncompensated respiratory alkalosis
B. partially compensated respiratory alkalosis
C. uncompensated metabolic alkalosis
D. partially compensated metabolic alkalosis
15. Which of the following can give rise to respiratory alkalosis?
A. acute pulmonary edema
B. salicylate poisoning
C. acute pulmonary edema
D. paralysis of the diaphragm

GIT

1. Which of these muscles are responsible for opening the jaw during mastication/
A. masseter and medial pterygoid C. digastric and medial pterygoid
B. masseter and lateral pterygoid D. digastric and lateral pterygoid
2. Which of these statements regarding the slow waves in the GIT is true?
A. They are actually action potentials.
B. They cause the entry of Ca++ into the muscle fiber.
C. Each slow wave is usually followed by a muscle contraction.
D. Slow wave frequency determines rhythm of the GI contractions
3. Which of the following is true of intestinal peristalsis?
A. Occurs only in the small intestine
B. distention of the gut initiates peristalsis
C. requires the presence of an intact submucosal plexus
D. direction of peristalsis is primarily toward the stomach
4. Which of these movements is the most essential in preventing food from entering the
trachea during swallowing?
A. palatopharyngeal folds are pulled medially
B. upward movement of the larynx
C. vocal cords are tightly approximated
D. epiglottis swings backward and downward
5. These enzymes are present in the brush border of enterocytes
A. Trypsin, chymotrypsin, carboxypolypeptidase, dipeptidase
B. Trypsin, chymotrypsin, and carboxypolypeptidase
C. trypsin and chymotrypsin
D. trypsin only
6. The final digestion of peptides into amino acids occur in the _____ of enterocytes
lining the small intestine.
A. brush border C. endoplasmic reticulum
B. cytosol D. golgi apparatus
7. Which of the following is true regarding carbohydrate digestion and absorption?
A. digestion starts in the stomach or in duodenum
B. galactose is the final digestion product of most carbohydrate food
C. Glucose enters and leaves epithelial cells via facilitated diffusion
D. Enzymes for disaccharide digestion are located in microvilli of
enterocytes
8. As regards fat digestion and absorption, which of these is true?
A. Almost all fat digestion occurs in the stomach
B. Bile salts and lecithin digest the fat globules into fatty acids
C. Bile salt micelles transport fat digestion products to brush border of
enterocytes
D. Digested fat products are formed into Chylomicrons that enter into portal blood
9. Which of the following can stimulate the pancreas to release large amounts of
bicarbonate-rich fluid?
A. histamine C. gastrin
B. cholecystokinin (CCK) D. secretin
10. Which of these substances is the most potent stimulus for gallbladder contraction?
A. Histamine C. gastrin
B. CCK D. secretin
11. Gastrin enhances gastric HCl secretion by:
A. increasing gastric motility
B. directly stimulating the parietal cells
C. stimulating chief cells to secrete pepsinogen
D. stimulating enterochromaffin-like cells to secrete histamine
12. Entrance of food into the stomach reflexly increases activity of the terminal ileum
resulting in ejection of chyme into the cecum. This is known as the:
A. ileogastric reflex C. gastrocolic reflex
B. gastroileal reflex D. enterogastric reflex
13. This is true of gastric secretion.
A. it is inhibited by secretin
B. cephalic phase accounts for about 70% of the total gastric secretion
C. gastric phase results from signals coming from the cerebral cortex
D. intestinal phase is due to vasovagal reflex from stomach to brain & back to
stomach
14. Ms Jaja came to your clinic to consult you about her baby. She said she is worried
because her baby defecates some minutes after each meal. You get a history and do
a physical examination and these were all normal. You then assure Ms Jaja that her
baby’s  frequent  defecation  is  just  due  to  a  strong: P/U
A. ileogastric reflex C. gastrocolic reflex
B. gastroileal reflex D. enterogastric reflex
15. Respiration is inhibited during this/these phase/s of swallowing. R
A. oral phase C. esophageal phase
B. pharyngeal phase D. pharyngeal and esophageal phases

16. As regards salivary secretion, which of the following is true? R


A. Average daily secretion of saliva is about 2,000 liters per day
B. Salivation is primarily regulated by the sympathetic nervous system
C. of the salivary glands, the parotids secrete the largest amount of mucus
D. at rest, K+ concentration in saliva is greater than that in plasma
17. Which of the following conditions enhance rate of gastric emptying? R
A. Increased food volume in the stomach
B. Highly acidic duodenal chyme
C. Hypertonic fluids in duodenum
D. Protein breakdown products in duodenum
18. Being a Muslim, Janni observed Ramadan. She noticed that she would experience
hunger contractions any time during the day. She learned later that these contractions
are most associated with: U
A. Peristalsis C. retropulsion
B. Segmentation D. migrating motor complex
19. Which of these motor activities is present in the esophagus down to the colon?
A. Peristalsis C. haustral shuttling
B. Segmentation D. mass movement
20. Which of the following will most likely lead to the defecation reflex?
A. rhythmic peristalsis C. haustral shuttling
B. mass movement D. migrating motor comples
21. Mucus is secreted in this/these segment/s of the gastrointestinal tract.
A. esophagus, stomach, small intestine, large intestine
B. esophagus, stomach and small intestine
C. esophagus and stomach
D. esophagus only
22. Which of these salivary glands secrete ptyalin?
A. Parotid C. sublingual
B. Submandibular D. parotid and submandibular
23. This is true of the gallbladder.
A.
XAVIER UNIVERSTIY – JOSE P RIZAL COLLEGE MEDICINE
BASIC BIOMEDICAL SCIENCES
SECOND BIMONTHLY PERIOD
QUIZ # 2: MODULE 13

NAME ____________________________ Sept. 4, 2006

Write the letter of the BEST answer on the answer sheet in CAPITAL letters in INK.
NO ERASURES/SUPERIMPOSITIONS ALLOWED!

1. The ability to generate an action potential spontaneously is a characteristic of these


cells: C
A. P cells C. P cells and Purkinje cells
B. Purkinje cells D. P cells and ventricular muscles
2. The SA node is the normal pacemaker of the heart because it: D
A. is composed of P cells C. exhibits automaticity
B. is located at right atrium D. has highest rate of impulse generation
3. From the atrial muscle cells, the impulse is conducted to the: B
A. internodal tracts C. ventricular muscles
B. AV node D. Bundle of His
4. Impulse conduction is fastest in:
A. AV node C. ventricular muscles
B. SA node D. terminal Purkinje network
5. The following are true of the plateau phase of the fast response action potential
(FRAP), EXCEPT: C
A. there is very little change in the membrane potential
B. it prolongs the duration of the action potential (AP)
C. primarily due to slow inward Ca++ current through transient (T) channels
D. slow outward K+ current contributes to its development
6. Repolarization of the FRAP includes the following phases: B
A. phases 1, 2, 3, 4 C. phases 1, 2
B. phases 1, 2, 3 D. phase 1 only
7. The following are true of the slow response action potential (SRAP), EXCEPT: A
A. phase 0 is primarily due to slow inward Na+ current
B. repolarization phase is a single slow curve, not well demarcated
C. peak depolarization is about 0 mV
D. exhibited by nodal cells
8. Compared to FRAP, the SRAP has: B
A. more negative RMP C. longer duration of AP
B. slower velocity of upstroke D. longer plateau phase
9. The following are true of the pacemaker potential (PP), EXCEPT: D
A. also termed the prepotential
B. characterized by unstable RMP
C. does not require an outside stimulus to occur
D. normally present in atrial and ventricular muscle fibers
10. Ventricular muscle starts to be relatively refractory to stimulation during the latter part
of the FRAP: D
A. phase 0 C. phase 2
B. phase 1 D. phase 3
11. The rate of discharge of pacemaker cells can be increased by increasing the following,
EXCEPT: C
A. rate of spontaneous depolarization of the resting potential
B. threshold potential
C. initial resting potential
D. sympathetic stimulation
12. The following are true of the P wave, EXCEPT: A
A. represents atrial repolarization
B. should not be more than 0.10 sec.
C. becomes prolonged when atria are enlarged
D. measured from the beginning to the end of P wave
13. The following association between ECG lead and location of positive electrode are
true, EXCEPT: C
A. lead I – left arm C. lead III – right leg
B. lead II – left leg D. lead aVR – right arm
14. If there are 2.5 small squares from the onset of the Q wave to the termination of the S
wave, then the QRS interval is _____ second. C
A. 0.05 C. 0.1
B. 0.5 D. 1.0
15. The volume of blood present in the ventricle during isovolumetric relaxation phase of
the cardiac cycle is the: B
A. end diastolic volume C. residual volume
B. end systolic volume D. systolic reserve volume
16. Aortic pressure is higher than ventricular pressure during these phases of the cardiac
cycle, EXCEPT: C
A. isovolumetric contraction C. rapid ejection
B. isovolumetric relaxation D. reduced ejection
17. The heart is at its most complete rest during this phase of the cardiac cycle: C
A. isovolumetric relaxation C. reduced filling
B. rapid filling D. atrial contraction
18. The following events occur earlier in the left than in the right side of the heart,
EXCEPT: D
A. onset of ventricular contraction C. closure of the AV valve
B. closure of the semilunar valve D. opening of the semilunar valve
19. These are true of the first heart sound (S 1), EXCEPT: C
A. louder and longer in duration than S 2
B. clinically marks the onset of systole
C. heard only at the mitral and tricuspid valve areas
D. occurs a few msecs after onset of isovolumetric contraction phase
20. Which heart sound may be normally split during inspiration? B
A. S 1 B. S 2 C. S 3 D. S 4
21. Which of the following may cause systolic murmur? A
A. aortic stenosis C. mitral stenosis
B. pulmonary insufficiency D. tricuspid stenosis
22. Applying  Starling’s  law  of  the  heart,  which  of  the  following  when  increased  will  result  in  
greater force of ventricular contraction? A
A. end diastolic volume C. heart rate
B. residual volume D. cardiac reserve
23. Stimulation of the following reflexes results in bradycardia, EXCEPT: B
A. Goltz reflex C. carotico-aortic sinus reflex
B. McDowall reflex D. Ashner-Dagnini reflex
24. In which of these conditions may the heart stop in diastole? C
A. hyperkalemia, hypokalemia, hypercalcemia, hypocalcemia
B. hyperkalemia, hypokalemia, hypercalcemia
C. hyperkalemia, hypokalemia
D. hyperkalemia
25. Which of the following substances exert a positive inotropic effect? A
A. catecholamine, thyroid hormone, glucagon, angiotensin II
B. catecholamine, thyroid hormone, glucagon
C. catecholamine, thyroid hormone
D. catecholamine
NAME______________________________

1. Give the 3 main criteria for a good research problem.

1.

2.

3.

2. Given the following hypotheses, identify the type. Encircle the letter of the best
answer.

2.1 There is a high incidence of goiter among the residents, aged 5 years and above,
in Jasaan, Misamis Oriental.

A. one-tailed hypothesis
B. two-tailed hypothesis
C. null hypothesis

2.2 Clotrimazole and ketoconazole have equal efficacy in the treatment of fungal
infections.

A. one-tailed hypothesis
B. two-tailed hypothesis
C. null hypothesis

Xavier University – Dr Jose P Rizal College of Medicine

Physiology

INSTRUCTIONS: Write the letter of the BEST answer on the answer sheet. Use
CAPITAL letters in ink. NO ERASURES/SUPERIMPOSITIONS ALLOWED!
Write your name on this questionnaire and on the answer sheet.

84. In the absence of ATP this transport process cannot occur:


A. osmosis C. facilitated diffusion
B. bulk flow D. sodium-glucose cotransport
85. When red cells are exposed to 10% Na CL solution, which of the following occurs?
A. hemolysis C. crenation
B. swelling D. cell volume remains the same
86. Which of these events immediately precedes exocytosis and release of acetylcholine
at the neuromuscular junction?
A. calcium entry into the synaptic vesicles
B. depolarization of the terminal knob
C. opening of voltage-gated sodium channels
D. fusion of vesicles with membrane of terminal knob
87. Which of the following helps explain why stimulation of the sympathetic division gives
rise to a widespread response?
A. The sympathetic division innervates practically all organ-systems of the body.
B. The sympathetic chain consists of 22 interconnected ganglia.
C. The transmitter of the sympathetic division is inactivated slowly.
D. All preganglionic fibers of the sympathetics arise from the spinal cord.
88. Which of the following is immediately responsible for the opening of calcium channels
in the sarcoplasmic reticulum (SR)?
A. an action potential in the T tubule
B. an action potential in the SR membrane
C. hydrolysis of ATP in the SR membrane
D. calcium influx from the extracellular fluid
89. Which of the following formed elements of blood possess phagocytic function, but die
soon after phagocytosis?
A. eosinophils B. basophils C. monocytes D. neutrophils
7. Which of the following cells are mainly responsible for a faster immune response upon
re-exposure to a foreign antigen?
A. effector T and B cells C. natural killer cells
B. memory T and B cells D. plasma cells
8. Which of the following antibodies is most likely to be present in the serum of an
exclusively bottle-fed newborn?
A. Ig G B. Ig M C. Ig A D. Ig D
9. Which of the following donor blood types will most likely give rise to a compatible major
cross-match but an incompatible minor cross-match if the recipient is type A?
A. type A B. type B C. type O D. type AB
14. Mr Ree accidentally cut his jaw while shaving. The bleeding continued for several
minutes. He became alarmed because he has a family history of hemophilia A (factor VIII
deficiency). He went to the hospital and these lab tests were done. Which of these are
expected to be prolonged?
c. APTT, PT, TT, BT C. APTT, PT
d. APTT, PT, TT D. APTT
15. When the SA node is diseased, which of the following is likely to take its place as the
pacemaker of the heart?
e. bundle of His C. ventricular muscles
f. terminal Purkinje network D. AV node
16. The plateau phase of the fast response action potential (FRAP) is primarily due to slow
and sustained:
a. Na+ influx via slow Na+ channels C. Ca++ influx via L channels
b. K+ efflux via slow K+ channels D. Ca influx via T channels
17. Clinically, cardiac systole is from:
a. S1 to S2 C. S1 to the next S1
b. S2 to the next S1 D. S2 to the next S2
18. Which of the following conditions may cause diastolic murmur?
a. aortic insufficiency and mitral stenosis
b. pulmonary stenosis and mitral stenosis
c. tricuspid stenosis and pulmonary stenosis
d. pulmonary insufficiency and tricuspid insufficiency
19. Mean arterial pressure (MAP) tends to increase when the following are increased:
a. stroke volume, blood volume, blood viscosity, arteriolar radius
b. stroke volume, blood volume, blood viscosity
c. stroke volume, blood volume
d. stroke volume
20. Capillary filtration tends to increase in these conditions:
a. increased PVP, venous obstruction, hypoproteinemia, arteriolar constriction
b. increased PVP, venous obstruction, hypoproteinemia
c. increased PVP, venous obstruction
d. increased PVP
21. Which of the following would most likely lead to a decrease in venous return?
a. right ventricular failure
b. decreased CVP
c. increased blood volume
d. increased mean systemic filling pressure
22. Which of the following pressures, when increased, will enhance air flow into the
alveoli?
A. intrathoracic pressure C. transthoracic pressure
B. intrapulmonic pressure D. transpulmonary pressure
63. A subject exhales quietly. He then exhales maximally. The air that is expelled is called:
A. tidal volume C. functional residual capacity
B. residual volume D. expiratory reserve volume
64. Which of the following pressures, when increased, will enhance air flow into the
alveoli?
A. intrathoracic pressure C. transthoracic pressure
B. intrapulmonic pressure D. transpulmonary pressure
79. Majority of the absorbable substances filtered across the glomeruli are reabsorbed at
the:
A. proximal tubule C. loop of Henle
B. distal tubule D. collecting duct
80. Which of the following best explains why fluid that enters the distal tubule is generally
hypo-osmolar? B
A. at the loop of Henle, about 25% of water is reabsorbed, while 15% of salt is
reabsorbed
B. at the loop of Henle, about 15% of water is reabsorbed, while 25% of salt is
reabsorbed
C. at the distal tubule water reabsorption is greater than salt reabsorption
D. at the distal tubule salt reabsorption is greater than water reabsorption
81. The bladder volume of an individual is 200 ml. At this volume, which of the following is
most likely to take place? C
A. voluntary control of micturition is lost
B. sensation of bladder filling is experienced
C. first desire to void is experienced
D. rise in intravesical pressure
82. Excessive ingestion of pure water could result in an increase in the following:
A. ECF volume, ICF volume, ECF osmolarity, ICF osmolarity
B. ECF volume, ICF volume, ECF osmolarity
C. ECF volume, ICF volume
D. ECF volume
83. A patient was admitted to the hospital due to abdominal pain and diarrhea. Aside from
dehydration he also suffered from an acid-base disturbance. After three days of
treatment, acid-base parameters were again determined and the results were: pH =
7.358; [HCO3-] = 24 mM/L; pCO2 = 37 mmHg; [BE] = (-) 1.1 mM/L. At this time the
patient has a:
A. partially compensated metabolic acidosis
B. fully compensated metabolic acidosis
C. fully compensated metabolic alkalosis
D. normal acid-base status
84. A patient was admitted to the hospital due to abdominal pain and diarrhea. Aside from
dehydration he also suffered from an acid-base disturbance. After three days of
treatment, acid-base parameters were again determined and the results were: pH =
7.358; [HCO3-] = 24 mM/L; pCO2 = 37 mmHg; [BE] = (-) 1.1 mM/L. At this time the
patient has a:
A. partially compensated metabolic acidosis
B. fully compensated metabolic acidosis
C. fully compensated metabolic alkalosis
D. normal acid-base status
85. Which of the following hormones is responsible for male-pattern balding?
A. estradiol C. dihydrotestosterone
B. testosterone D. androstenedione
86. Maria has a regular 40 day menstrual cycle. Her last two menstrual periods started on
January 1 and February 9. Her ovulation most likely occurred on:
A. January 14 C. January 27
B. January 20 D. February 1
75.Which of the following methods of fertility control takes into consideration the
character of cervical mucus as indicative of the fertile period?
A. post-coital douche C.    Billing’s  method
C. calendar rhythm D. Temperature rhythym
The hair cells of the organ of Corti are stimulated when:
A. stereocilia are bent away from the limbus
B. stereocilia are bent away from the tallest cilium
C. organ of Corti is deflected downward
D. basilar membrane is pushed towards the scala tympani
Going down the ladder stimulates primarily the:
A. vertical semicircular canals C. saccule
B. horizontal semicircular canals D. utricle

The best way to synchronize the circadian rhythm to the exact 24 hours imposed on
man by the sun is to make use of:
A. entrainment C. drugs
B. zeitgebers D. muscle activity
The best way to synchronize the circadian rhythm to the exact 24 hours imposed on
man by the sun is to make use of:
A. entrainment C. drugs
B. zeitgebers D. muscle activity
In order to be of value, carbohydrate-laden drinks should be taken:
A. 1 hour before exercise C. 1 hour after exercise
B. 30 minutes before exercise D. at intervals during exercise

XAVIER UNIVERSITY – JOSE P RIZAL COLLEGE OF MEDICINE


Basic Biomedical Sciences
Third Bimonthly Period
QUIZ # 4 (Genetics)

NAME ____________________________ 4 Dec 2007 SCORE ____________


MULTIPLE CHOICE: write the letter of the BEST answer on the answer sheet. Use capital
letters in blue/black ink. NO EASURES/SUPERIMPOSITIONS ALLOWED!

1. Which of the following associations between each layer of the adrenal gland and
cellular content is least likely true?
A. zona glomerulosa – closely packed columnar cells
B. zona fasciculata – lipofuscin laden polyhedral cells
C. zona reticularis – anastomosing irregularly shaped-cells
D. medulla – polyhedral cells supported by reticular tissue
2. Which of the following hormones secreted by the adrenal cortex play a role in
glycogenesis?
A. Mineralocorticoids C. Glucocorticoids
B. Androgens D. Catecholamines
3. Which of the following is true of the islets of Langerhans?
A. encapsulated by reticular fibers
B. staining divides the islet cells into three types
C. equally distributed throughout the entire pancreas
D. tight junctions allows communication between cells
4. This islet of Langerhans cell type secretes hormones which inhibit secretion of other
islet cell hormones:
A. A B. B C. C D. D
5. Which of the following best describes the parafollicular cells?
A. arrange in anastomosing cords
B. abundant rough endoplasmic reticulum
C. has cytoplasmic granules containing hormones
D. stain more intensely than thyroid follicular cells
6. Which of the following is true of the parathyroid gland?
A. Septated C. reticular tissue capsule
B. Oxyphil cells secrete PTH D. irregular cell arrangement
7. Which of the following is true of the adenohyphosis?
A. funnel-shaped pars intermedia
B. pars distalis contains 3 cell types
C. pars tuberalis cells secrete FSH & LH
D. cord-like arrangement of the pars distalis
8. An endocrine organ covered by pia mater:
A. pineal gland C. thyroid gland
B. pituitary gland D. adrenal medulla
9. One of the following statements is NOT true regarding homeostasis:
A. It refers to the dynamic steady state of the extracellular fluid.
B. It maintains physical and/or chemical parameters of the body relatively
constant.
C. Feedback is the only type of regulation which exists in the human
homeostatic system.
D. Homeostatic control minimizes changes in the internal environment when there
are changes in the external environment.
10. This type of regulation in homeostatic systems anticipates changes in the external
environment causing a faster response to the change:
A. negative feedback C. positive feedback
B. feedforward regulation D. anticipated feedback

11. An example of inputs and outputs maintaining the stability of an internal environment
is:
A. normal heart rate C. normal blood pressure
B. normal body temperature D. normal sleep patterns
12. This term refers to the rhythmic changes which occur in body functions.
A. biologic rhythm C. acclimatization
B. apoptosis D. none of the above
13. Acclimatization is most associated with one of the following:
A. adaptation C. biologic rhythm
B. apoptosis D. aging
14. This programmed cell response plays a major role in growth and
development of organisms by removing undesirable cells:
A. aging C. acclimatization
B. apoptosis D. biological rhythm
15.The intercellular messengers secreted into the extracellular fluid and act on
neighboring cells of a different type are called:
A. endocrine agents C. paracrine agents
B. autocrine agents D. neuroendocrine agents
16. The first step in the action of any intercellular chemical messenger is:
A. alter the membrane permeability of the target cell
B. bind to its specific receptor
C. signal the activation of cAMP
D. trigger the activation of protein kinase
17. As regards upregulation of messenger receptors, which of these is true?
A. takes place in the presence of a high concentration of messenger substance
B. makes  the  target  cell  more  sensitive  to  the  messenger’s  effect
C. involves the sequestration of the receptor molecules to the inside of the cell
D. is brought about by inactivation of receptor molecules.
18. G-proteins are named for their ability to bind with:
A. guanosine nucleotides C. STAT proteins
B. gamma amino butyrate D. glucagon receptor proteins
19. Steroid hormones bind with receptor proteins located:
A. on the cell membrane surface C. in the cell nucleus
B. in the cytosol D. on the nuclear envelope
20. Norepinephrine and acetylcholine bind with:
A. G-protein linked hormone receptors
B. Enzyme linked hormone receptors
C. Ion-channel linked hormone receptors
D. Nucleotide linked hormone receptors
21. Example of tyrosine-derived hormones
A. growth hormone C. adrenocorticotrophic hormone
B. luteinizing hormone D. thyroid hormone
22. When G-protein is activated, one of its trimeric components dissociates to bind with
other intracellular signaling proteins. This component is the
A. alpha component C. gamma component
B. beta component D. GTP component
23. Which of these is true of all G-protein linked receptors?
A. are proteins that penetrate through the cell membrane only once
B. have segments that penetrate the cell membrane seven times
C. alter the ion permeability of the cell membrane
D. function directly as enzymes
24. There are no known hormones that exist as:
A. Polysaccharides C. steroids
B. Polypeptides D. amino acids
25. This is one of the general characteristic of a receptor.
A. acts as a signal C. non-specifically binds to a ligand
B. may be a protein or a lipoprotein D. is almost always an ion channel
26. In the adenyl cyclase-cAMP second messenger system, which of the following
immediately follows the binding of the hormone to its receptor?
A. Phosphorylation of specific proteins
B. activation of protein kinases
C. G protein stimulates adenyl cyclase
D. adenyl cyclase catalyzes conversion of ATP to cAMP
27. “Down  regulation”  refers  to  the  decrease in the:
A. synthesis of hormones by the endocrine cells
B. effects of the hormones on the target tissues
C. sensitivity of the endocrine organ to stimulation
D. number of receptors associated with increased hormone level
28. This is one of the general characteristics of hormones.
A. act as first messengers
B. Circulate in the lymphatic system
C. Circulate at very high concentrations
D. binds directly to DNA to stimulate transcription
29. Given: Endocrine organ (EO) #1 releases hormone (H) #1 which stimulates EO #2 to
release H #2. H #2 in turn stimulates EO #3 to release H #3. A decrease in plasma
level of H #3 stimulates EO #1 to release more H #1, and this eventually leads to the
release of more H #3 to bring its plasma level towards normal. This type of hormonal
control is known as:
A. simple feedback C. long-loop negative feedback
B. positive feedback D. short-loop negative feedback
30. Which of the following can act as second messenger?
A. cAMP C. diacylglycerol
B. inositol triphosphate D. all of the above
31. Which of these hormones are generally not stored but released as they are
synthesized?
A. Steroids C. thyroid hormones
B. proteins D. catecholamines
32. Which of the following groups of hormones is classified as steroids?
A. cortisol, testosterone, aldosterone
B. growth hormone, gastrin, insulin
C. vasopressin, rennin, cholecystokinin
D. epinephrine, dopamine, thyroid hormone
33. Which of the following is the first to be activated when a stressor confronts an
individual?
A. sympathetic nervous system C. pituitary gland
B. hypothalamus D. adrenal gland
34. Which of the following, when activated, will tend to modulate the responses to stress?
A. sympathetic nervous system C. hypothalamus
B. immune system D. endocrine system
35. Which of the following can function as a psychological stressor?
A. problem with faculty C. peer pressure
B. threat of terrorism D. traffic
36. Which if the following is most probably taking place if, as a result of a stressor, a
normal  female  suddenly  behaves  like  a  “superwoman”?
A. shock phase C. stage of resistance
B. countershock phase D. stage of exhaustion
37. Which of the following is an immediate effect of stress?
A. increased mental activity C. increased blood glucose
B. increased heart rate D. decreased level of immunity
38. Which of the following is attributable to the effects of cortisol only?
A. bronchodilation C. hyperglycemia
B. vasoconstriction D. increased muscle strength

39. Which of the following is considered as a negative effect of stress?


A. decrease in coagulation time C. decrease in lymphocytes
B. increase in red cells D. increase in blood glucose
40. Which of the following, when activated, will indicate that the effects of stress are still
positive?
A. ACTH axis C. pituitary axix
B. thyroid axis D. discrete autacoids
41. Which of the following can be considered as a personal coping skill?
A.  learn  how  to  say  “no” C. develop relations with co-workers
B. develop hobbies D. respond rather than react to stressor
42. The most rostral region of the hypothalamus:
A. Anterior hypothalamus C. tuberal
B. Preoptic D. mamillary
43. Which of the following is not true of the Pituitary gland?
A. Its blood supply is derived from the external carotid artery
B. Consists of two glands
C. Lies in the sella turcica of the sphenoid bone
D. Weighs about 0.5 g.
44. A muscular band that connects the pyramidal lobe of the thyroid gland
to the hyoid bone:
A. Prevertebral layer of deep fascia
B. Isthmus
C. Pretracheal layer of deep fascia
D. Levator glandulae thyroidea
45. Which of the following thyroid veins is a tributary of the brachiocephalic vein?
A. Highest C. Middle
B. Superior D. inferior
46. Which of the following describe the right adrenal gland?
A. Crescentic in shape
B. Lies behind the pancreas
C. Supplied by a single artery
D. A retroperitoneal organ
47. Which of the following is true of the parathyroid glands?
A. Are intimately related to the posterior border of the thyroid glands
B. Usually two in number
C. Measure about 6 cm long
D. The superior gland lie close to the superior poles of the thyroid gland
48. Which of the following structures is medially related to the thyroid gland?
A. Recurrent laryngeal nerve
B. Common carotid artery
C. Vagus nerve
D. Internal jugular vein
49. This is an endocrine gland that secretes growth hormone:
A. Adenohypophysis C. posterior pituitary
B. Hypothalamus D. thyroid
50. This organ secretes erythropoieten:
A. Liver C. kidney
B. Heart D. spleen

XAVIER UNIVERSITY – DR JOSE P RIZAL COLLEGE OF MEDICINE


BASIC BIOMEDICAL SCIENCES

Final Exam: BLOCK 2 (Modules 11 – 20) March 9, 2006


Write the letter of the BEST answer on the answer sheet. Use capital letters in ink.
NO ERASURES/SUPERIMPOSITIONS ALLOWED!

1. The temporary hemostatic plug is primarily due to:


A. red cells B. white cells C. platelets D. fibrin
2. Bladdy was suspected of having a hemorrhagic disorder. Laboratory tests were performed
and these are the results: APTT = abnormal; Prothrombin time = normal; thrombin time =
normal. Based on these limited data, Bladdy is most probably deficient in this factor:
A. factor XII B. factor VII C. factor VIII D. factor I
3. Which of the following genes of the ABO system is responsible for activity of the H
gene on the basic precursor on red cells?
A. A gene B. B gene C. Z gene D. Se gene
4. A subject has negative results for Anti-A and Anti-B, as well as with A, B and O cells.
He is most probably:
A. a type AB newborn C. a type AB elderly with leukemia
B. a type O newborn D. a type O elderly with multiple myeloma
5. The following are contents of the middle mediastinum Except:
A. arch of azygos vein C. ascending aorta
B. heart D. esophagus
6. The following are true of the superior mediastinum Except:
A. bounded posteriorly by the first four thoracic vertebrae
B. opens above to the thoracic outlet
C. further subdivided into anterior, middle, and posterior parts
D contains the trachea
7. Which of the following surfaces/parts of the heart is formed mainly by the right
and left ventricles?
A. sternocostal surface C. posterior surface
B. diaphragmatic surface D. apex
8. Which of the following types of trabeculae carnae conveys the right branch of the
atrioventricular bundle?
A. type I B. type II C. type III D. type IV
9. The following are descriptive of the pericardium Except:
A. a fibroserous sac
B. encloses the heart and the roots of the great vessels
C. a pericardial cavity is formed between its fibrous and serous parts
D. its visceral layer is often called the epicardium
10.The following are structural characteristics of the cardiac muscle, EXCEPT:
A .made up of branching muscle cells
B. muscle fibers are arranged in laminae
C. cells have one or two oval central nuclei
D. banding of fibers differs from that of skeletal muscle
11.Which of the following is the primary cell type in the AV node?
A. P B. transitional C. Purkinje D. ordinary
12.Which of the following membrane junction types allow nutrients or ions to pass
directly from one cardiac cell to another?
A. nexus C. zonula occludens
B. macula adherens D. fascia adherens
13.The following are true of the epicardium, EXCEPT:
A. continuous with the intima of the large arteries
B. lined by simple squamous epithelium
C. has loose connective tissue with adipose cells
D. glands are absent

14. When the SA node is diseased, which of the following is most likely to take its place
as the cardiac pacemaker?
C. atrial muscles C. bundle of His
D. AV node D. terminal Purkinje network
15. When the right atrium is enlarged, which of these ECG configurations will be prolonged?
A. PR interval and QRS interval C. P wave duration and PR interval
B. P wave duration and ST segment D. QRS interval and QT interval
16. If the P vector is located at (-) 10 degrees, the P wave is tallest in this ECG lead:
A. lead I B. lead II C. lead aVL D. lead aVR
17. Peak aortic (systolic) pressure occurs at the end of this phase of the cardiac cycle:
A. isovolumetric contraction C. protodiastole
B. isovolumetric relaxation D. rapid ejection
18. The following events occur during cardiac diastole, EXCEPT:
A. ventricular filling C. atrial contraction
B. ventricular relaxation D. opening of semilunar valves
19. If the chordae tendinae in the left ventricle are cut, the following would occur,
EXCEPT:
A. blood regurgitates into the left atrium during systole
B. mitral valve would be everted during systole
C. systolic murmur would be heard
D. stroke volume would be increased
20. The heart may stop in systole in which of these conditions?
A. hypercalcemia C. hyperkalemia
B. hypocalcemia D. hypokalemia
21. Which of the following gives origin to the right coronary artery?
A. anterior aortic sinus C. right posterior aortic sinus
B. left posterior aortic sinus D. posterior aortic sinus
22. The following are direct arterial branches of the arch of the aorta Except:
A. right common carotid artery C. left common carotid artery
B. right subclavian artery D. brachiocephalic artery
23. The following veins drain directly to the superior vena cava Except:
A. right brachiocephalic vein C. left brachiocephalic vein
B. internal jugular vein D. azygos vein
24. The following are direct tributaries of the inferior vena cava Except:
A. left testicular vein C. hepatic veins
B. renal veins D. common iliac veins
25. The hepatic portal vein is formed by the union of the:
A. inferior mesenteric and superior mesenteric veins
B. superior mesenteric and hepatic veins
C. inferior mesenteric and splenic veins
D. superior mesenteric and splenic veins
26. The following are contents of the tunica adventitia of blood vessels, EXCEPT:
A. collagen & elastic fibers C. vas vasorum & lymphatic capillaries
B. adipose cells D. layer of endothelial cells
27. The following describe medium arteries, EXCEPT:
A. areolar tissue in adventitia
B. prominent internal elastic lamina
C. abundant elastic fibers in media
D. similar thickness of media & adventitia
28. Which of the following is true as regards histologic features of capillaries?
A. made up only of tunica intima C. gap junction between endothelial cells
B. collagen fibers in tunica media D. endothelial cells rest on a basal lamina
29. The following characterize large veins, EXCEPT:
A. well-developed tunica media C. abundant loose connective tissue
B. indistinct wall boundaries D. has the thickest tunica adventitia

30.The following describe lymphatic vessels, EXCEPT:


A. endothelium-lined C. fenestrated capillaries
B. underdeveloped adventitia D. absence of basal lamina in capillaries
31. Stimulation of the vasopressin system reflexly results in the following, EXCEPT:
A. stimulation of the thirst center C. increased renal reabsorption of water
B. increased release of ADH D. dilute urine
32. When right ventricular contractions are weak, which of the following would increase?
A. central venous pressure (CVP) and peripheral venous pressure (PVP)
B. right ventricular stroke volume (SV) and venous return (VR)
C. CVP, PVP, VR and SV
D. CVP, PVP, and VR
33. Capillary filtration tends to increase in these conditions, EXCEPT:
A. hypoproteinemia C. venous obstruction
B. arteriolar constriction D. increased PVP
34. Mean arterial pressure tends to increase when the following are increased, EXCEPT:
A. blood viscosity C. stroke volume
B. arteriolar radius D. blood volume
35. Which of the following explains the relationship between flow velocity and lateral
pressure in blood vessels?
A.    Critical  Reynold’s  number                                      C.  Law  of  Laplace
B. Poiseulle-Hagen  formula                                            D.  Bernoulli’s  principle
36. The following form the bony framework of the external nose Except:
A. frontal processes of maxilla C. nasal bones
B. body of the sphenoid D. nasal part of the frontal bone
37. The following are true of the respiratory region of the nasal cavity Except:
A. constitutes the majority of the cavity
B. its lateral wall forms the main wall of the nasal cavity
C. receives the openings of all the paranasal sinuses
D. its medial wall is marked by three elevations called conchae
38. The following are true of the cartilages of the larynx as described Except:
A. epiglottis-a thin leaf like plate of elastic cartilage
B. thyroid cartilage- largest cartilage
C. cuneiform cartilage- has two processes, vocal and muscular processes
D. cricoid cartilage-forms a complete ring around the airway
39. The following muscles controls the movements of the vocal cords Except:
A. thyroepiglottic C. thyroarytenoid
B. lateral cricoarytenoid D. posterior cricoarytenoid
40. The right principal bronchus differ from the left principal bronchus by the
following Except:
A. wider C. more vertical
B. longer D. gives off two lobar bronchi
41. The following relates to the histologic features of the lung, EXCEPT:
A. contains pulmonary and bronchial vessels
B. invested with mesothelium
C. outer covering contain serous glands
D. show intrapulmonary bronchus and bronchioles
42. These are histologic features of the trachea, EXCEPT:
A. lined by pseudostratified columnar ciliated epithelium
B. has thickest basement membrane in the body
C. mixed glands in submucosa
D. skeletal muscles in tunica adventitia
43. The last segment of the respiratory tract to display smooth muscle cells:
A. alveolar duct C. alveolar wall
B. respiratory bronchiole D. terminal bronchiole

44. The following are true of the bronchiole, EXCEPT:


A. simple columnar to cuboidal epithelium
B. loose connective tissue in lamina propria
C. smooth muscle in mucosa
D. mixed glands in ordinary bronchioles
45. The following cells to role association are correct, EXCEPT:
A. type I pneumocyte > diffusion of gases
B. type II pneumocyte > lung surfactant
C. septal cell > maintenance of lung interstitium
D. no exception mentioned
46. Mechanical or chemical irritation of the nasal mucosa will produce the following
effects, EXCEPT:
A. sneeze C. bronchoconstriction
B. rise in BP D. tachycardia
47.Expansion of the alveoli is directly attributed to a change in:
A. intrathoracic pressure C. transpulmonary pressure
B. intrapulmonary pressure D. transthoracic pressure
48. Which of the following muscles plays a part in forced expiration?
A. serratus posterior superior C. levatores costarum
B. serratus posterior inferior D. sternocleidomastoid
49. The amount of air that can be forcibly inhaled from the end expiratory level is called:
A. inspiratory reserve volume C. inspiratory capacity
B. total lung capacity D. vital capacity
50. Which of the following is the only method capable of measuring trapped gas?
A. wet spirometry C. closed circuit method
B. body plethysmography D. open circuit method
51. Which of the following gas laws describes the inverse relationship between the size of the
gas molecules and the rate of gas diffusion?
A.  Avogadro’s  law C.  Charles’  law
B. Boyle’s  law D.  Dalton’s  law
52. One of the following, when decreased, will increase the rate of gas diffusion:
A. gas temperature C. solubility coefficient
B. membrane surface area D. diffusion distance
53. The following are true as regards ventilation-perfusion ratio, EXCEPT:
A. transpulmonary pressure is greater at the upper lung
B. greater expansion of the lung at the hilus than at the periphery
C. greater perfusion of the left lung than of the right
D. lung apices have more carbon dioxide than lung apices
54. Which of the following shifts the oxyhemoglobin curve to the right by causing stabilization
of the deoxy configuration of hemoglobin?
A. hydrogen ions C. increased 2,3 diphosphoglycerate
B. high temperature D. increased carbon dioxide
55. Carbon dioxide unloading at the tissues is called:
A. Bohr effect C. positive cooperativity
B. Haldane effect D. blood buffer curve
56. The most ventral structure at the hilum of the kidney:
A. renal artery C. renal vein
B. ureter D. renal pelvis
57. Forms the outermost covering of the kidney:
A. renal fascia C. perirenal fat
B. pararenal fat D. fibrous
57. The ureters joined the urinary bladder at its:
A. apex B. neck C. base D. superior surface
58. The following are true of the membranous urethra in male Except:
A. least dilatable portion of the urethra C. lies within the urogenital diaphragm
B. widest portion of the urethra D. shortest portion of the urethra

59. The left kidney is related anteriorly to the following structures Except:
A. suprarenal gland C. liver
B. pancreas D. Spleen
60. The following structures are seen in the renal pyramid, EXCEPT:
A. convoluted tubules C. vasa recta
B. loop of Henle D. collecting duct
61. The phagocytic cells that clean up the basal lamina of the glomerulus:
A. podocytes C. macula densa
B. mesangial D. lacis cells
62. The following make up the juxtaglomerular apparatus, EXCEPT:
A. macula densa C. mesangial cells
B. juxtaglomerular cell D. lacis cells
63. The filtration barrier is formed by the following, EXCEPT:
A. all visceral epithelial cells C. basal lamina of collagen, glycoproteins
B. filtration slits between pedicels D. fenestrated endothelial cells
64. The following are general histologic pattern of the urinary passages, EXCEPT:
A. transitional lining epithelium
B. mucous membrane without glands
C. inner longitudinal, outer circular smooth muscle arrangement
D. no exception mentioned
65. The most important mechanism regulating glomerular filtration is:
A. myogenic mechanism C. metabolic mechanism
B. tissue pressure mechanism D. tubulo-glomerular feedback mechanism
66. Substance X was used to measure glomerular filtration rate of a normal male subject. The
result showed that the glomerular filtration rate was 110 ml/min. This means that:
A. substance X is freely filtered
B. substance X is not degraded by the tubules
C. substance X is not synthesized by the tubules
D. substance X is reabsorbed by the tubules
67. Substance Y is reabsorbed via a Tm-limited mechanism. If substance Y appears in the
urine of a normal subject, the following are logical conclusions, EXCEPT:
A. there might be another substance that competes with Y for carriers
B. substance Y may have been converted to anionic or cationic form
C. the tubular load of substance Y may have exceeded its Tm
D. different nephrons may have different Tm values for substance Y
68. Majority of reabsorbable substances are reabsorbed at the:
A. proximal tubule C. loop of Henle
B. distal tubule D. collecting duct
69. Countercurrent multiplication of osmolar concentration takes place at the:
A. proximal tubule C. loop of Henle
B. distal tubule D. vasa recta
70. Which of the following is most likely to take place if bladder urine volume is 350 ml in a
normal subject?
A. sensation of bladder filling is experienced
B. the first desire to void is experienced
C. voluntary control of micturition is lost
D. mucosal folds of the bladder unfold
71. Jane has passed 8 watery stools for the past 6 hours. She feels thirsty and tired. Based
on this short history we can say that the following are the likely changes occurring in her
body fluids:
E. decreased ECF volume and ICF volume,
F. decreased ECF volume and ECF osmolarity
G. decreased ECF volume, no change in ECF osmolarity, no change in ICF volume
H. decreased ECF volume and osmolarity and no change in ICF volume and osmolarity

72. Mia was admitted to the hospital because of severe asthmatic attack. On admission, the
results of laboratory exams also showed an acid base disturbance. After three days,
laboratory exams were again done and these were the results: pH = 7.352; [HCO3-] =
25.9 mmols/liter; PCO2 = 49.98 mmHg; [BE] = 1.89 mmols/liter. Based on these results,
we can say that Mia has a:
C. normal acid-base status C. fully compensated metabolic alkalosis
D. fully compensated respiratory acidosis D. fully compensated respiratory alkalosis
73. Body responses to metabolic alkalosis secondary to ingestion of a bottle of bicarbonate
tablets include the following, EXCEPT:
C. buffering by the bicarbonate buffer system (BBS)
D. buffering by the non-BBS
E. decreased alveolar ventilation
F. alkalinization of theurine
74. Given the following laboratory results of Gino: pH = 7.438; [HCO3-] = 20.2 mmols/liter;
PCO2 =30.8 mmHG; [BE] = (-) 2 mmols/liter. Based on these results,he is likely to have:
C. fully compensated metabolic alkalosis C. uncompensated metabolic alkalosis
D. fully compensated respiratory alkalosis D. normal acid-base status
75. The deep membranous layer of superficial fascia of the abdomen is called:
A.  Colles’  fascia                 C.  Camper’s    fascia
B.  Scarpa’s  fascia D. transversalis fascia
76. The following aponeuroses of muscles form the rectus sheath Except:
A. rectus abdominis C. transversus abdominis
B. external oblique D. internal oblique
77. The following are sites of constrictions of the esophagus Except:
A. at the level of the 6th cervical vertebra
B. at the level of the arch of the aorta
C. at the level of the right primary bronchus
D. at the level of the 10th thoracic vertebra
78. The following are contents of the of the lesser omentum Except:
A. hepatic artery C. portal vein
B. hepatic vein D. common bile duct
79. The following are intraperitoneal organs Except:
A. pancreas C. Spleen
B. stomach D. vermiform appendix
80. The following veins belong to the portal venous system Except:
A. superior mesenteric vein C. Splenic vein
B. inferior mesenteric vein D. hepatic vein
81. The following organs are supplied by the superior mesenteric artery Except:
A. spleen C. jejunum
B. vermiform appendix D. ascending colon
82. The following are correct matches of digestive organs to their distinct features,
EXCEPT:
A. large intestine > gland of Lieberkuhn
B.  duodenum  >  Brunner’s  glands                                  
C. stomach, body > fundic gland
D. esophagus > stratified squamous epithelium, mucous glands
83. A microscopic slide displays a mucosa with simple columnar epithelial cells with
thin striate border, without goblet cells; loose connective tissue without glands.
The organ is likely to be:
A. stomach, cardia B. jejunum C. appendix D. gallbladder
84. The small intestine may be distinguished from the large intestine by the presence
of the following, EXCEPT:
A. intestinal villi C. intestinal glands
B.  Paneth’  cells                                                             D. distribution of goblet cells
85. The  following  ‘cells  to  secretions’  associations are correct, EXCEPT:
A. parietal > hydrochloric acid C. enteroendocrine > gastrin
B. chief > gastric intrinsic factor D.  Paneth’s  >  lysozyme

86. Which of these is NOT right regarding the liver?


A. liver cells are arranged in cords between sinusoids
B. hepatic sinusoids are lined with endothelial and von Kuppfer cells
C. hepatocytes around central vein have best access to nutrients
D. branches of bile duct, portal vein, hepatic artery form the portal triad
87. Which of these pancreatic cells secrete enzymes?
A. acinar C. beta
B. alpha D. delta
88. Which of these phases of deglutition is under voluntary control?
A. oral phase C. esophageal phase
B. pharyngeal phase D. all of the above
89. Glucose absorption from the intestinal lumen is enhanced by this substance:
A. amino acid C. magnesium
B. potassium D. sodium
90. Which of the following events is unique in the digestion of fats?
A. CCK secretion C. micelle formation
B. emulsification D. secretion of pancreatic juice
91. Bile is reabsorbed in this portion of the GIT:
A. duodenum C. ileum
B. jejunum D. ascending colon
92. Which of the following is the most frequent type of movement in the small intestine?
A. peristalsis C. migrating motor complex
B. segmentation D. haustral shuttling
93. The following are true of gastric acid secretion, EXCEPT:
A. Chief cells secrete HCl
B. Gastrin stimulates gastric acid secretion
C. acid secretion between meals is low
D. entry of food into the stomach initiates the gastric phase of acid secretion
94. Meniong, a 29 year old, 5 feet tall, Filipino athlete keeps record of his average body
weight every year. The following is his average weight for the past four years. At what
year is Meniong considered overweight?
A. 2000 – 40 kg C. 2003 – 65 kg
B. 2001 – 54 kg D. 2002 – 80 kg
95. Juana, 20 year old cashier asked her brother, a medical student, how much iron does
she need everyday. The best answer is:
A. 5 mg /day B. 10 mg/day C. 30 mg/day D. 50 mg/day
96. Cherry, who has just delivered her baby, questions the pediatrician why she was told to
breastfeed her baby soon after delivery. The best answer would be:
A.  The  colustrum  has  higher  immune  globulin  than  cow’s  milk
B. Breastmilk has a laxative effect because of the protein content
C.  Breastmilk  has  more  vitamin  content  than  cow’s  milk
D. all of the above
97. Which of the following vitamins is needed as cofactor in a large number of oxidation-
reduction reactions?
A. Thiamin B. Niacin C. Pyridoxine D. Cobalamin
98. Supplemental amount of this vitamin is recommended immediately after birth to prevent
bleeding:
A. vit. A B. Vit. E C. vit. K D. vit. D
99. The characteristic flaky paint dermatosis is seen in:
A. Marasmus C. Beriberi
B. Kwashiorkor D. Pellagra
100. Unless carbohydrate is restricted fruits may be given at least 3 exchanges per day. The
calorie derived from fruit is:
A. 12 calories C. 30 calories
B. 120 calories D. 10 calories

Module 13 CVS 1

_____The following are contents of the middle mediastinum Except:


A. arch of azygos vein C. ascending aorta
B. heart D. esophagus (D)
_____The following are true of the superior mediastinum Except:
A. bounded posteriorly by the first four thoracic vertebrae
B. opens above to the thoracic outlet
C. further subdivided into anterior, middle, and posterior parts
D. contains the trachea (C)
_____Which of the following surfaces/parts of the heart is formed mainly by the right
and left ventricles?
A. sternocostal surface C. posterior surface ( B )
B. diaphragmatic surface D. apex
_____Which of the following types of trabeculae carnae conveys the right branch of the
atrioventricular bundle?
A. type I C. type III (B)
B. type II D. type IV
_____The following are descriptive of the pericardium Except:
A. a fibroserous sac
B. encloses the heart and the roots of the great vessels ( C )
C. a pericardial cavity is formed between its fibrous and serous parts
D. its visceral layer is often called the epicardium

Module 14 CVS 2

_____Which of the following gives origin to the right coronary artery?


A. anterior aortic sinus C. right posterior aortic sinus (A)
B. left posterior aortic sinus D. posterior aortic sinus
_____The following are direct arterial branches of the arch of the aorta Except:
A. right common carotid artery C. left common carotid artery
B. right subclavian artery D. brachiocephalic artery (A)
_____The following veins drain directly to the superior vena cava Except:
A. right brachiocephalic vein C. left brachiocephalic vein
B. internal jugular vein D. azygos vein ( B)
_____The following are direct tributaries of the inferior vena cava Except:
A. left testicular vein C. hepatic veins
B. renal veins D. common iliac veins ( A)
_____The hepatic portal vein is formed by the union of the:
A. inferior mesenteric and superior mesenteric veins
B. superior mesenteric and hepatic veins
C. inferior mesenteric and splenic veins (D)
D. superior mesenteric and splenic veins
_____The following are branches of the celiac trunk Except:
A. splenic artery C. pancreatic artery
B. common hepatic artery D. left gastric artery (C)

Module 15 Resp. 1

_____The following form the bony framework of the external nose Except:
A. frontal processes of maxilla C. nasal bones
B. body of the sphenoid D. nasal part of the frontal bone (B)
_____ The following are true of the respiratory region of the nasal cavity Except:
A. constitutes the majority of the cavity
B. its lateral wall forms the main wall of the nasal cavity
C. receives the openings of all the paranasal sinuses (D)
D. its medial wall is marked by three elevations called conchae
_____The following are true of the cartilages of the larynx as describe Except:
A. epiglottis-a thin leaf like plate of elastic cartilage
B. thyroid cartilage- largest cartilage (C)
C. cuneiform cartilage- has two processes, vocal and muscular processes
D. cricoid cartilage-forms a complete ring around the airway
_____The following muscles controls the movements of the vocal cords Except:
A. thyroepiglottic C. thyroarytenoid
B. lateral cricoarytenoid D. posterior cricoarytenoid ( A )
_____The following muscles are supplied by the recurrent laryngeal nerve Except:
A. transverse arytenoid C. lateral cricoarytenoid
B. cricoarytenoid D. oblique arytenoids (B)
_____ The right principal bronchus differ from the left principal bronchus by the
following Except:
A. wider C. more vertical (D)
B. longer D. gives off two lobar bronchi

Module 17 Urinary

_____The most ventral structure at the hilum of the kidney:


A. renal artery C. renal vein
B. ureter D. renal pelvis (C)
_____Forms the outermost covering of the kidney:
A. renal fascia C. perirenal fat
B. pararenal fat D. fibrous capsule (B)
_____The ureters joined the urinary bladder at its:
A. apex C. base (C)
B. neck D. superior surface
_____The following are true of the membranous urethra in male Except:
A. least dilatable portion of the urethra
B. widest portion of the urethra (B)
C. lies within the urogenital diaphragm
D. shortest portion of the urethra
_____The left kidney is related anteriorly to the following structures Except:
A. suprarenal gland C. liver
B. pancreas D. Spleen (C)
Module 19 GIS
_____ The deep membranous layer of superficial fascia of the abdomen is called:
A.  Colles’  fascia                C.  Camper’s    fascia
B.  Scarpa’s  fascia            D. transversalis fascia (B)
_____The following aponeuroses of muscles form the rectus sheath Except:
A. rectus abdominis C. transversus abdominis ( A )
B. external oblique D. internal oblique
_____The following are sites of constrictions of the esophagus Except:
E. at the level of the 6th cervical vertebra
F. at the level of the arch of the aorta
G. at the level of the right primary bronchus (C)
th
H. at the level of the 10 thoracic vertebra
_____The following are contents of the of the lesser omentum Except:
A. hepatic artery C. portal vein
B. hepatic vein D. common bile duct (B)
_____The following are intraperitoneal organs Except:
A. pancreas C. Spleen
B. stomach D. vermiform appendix (A)
_____The following veins belong to the portal venous system Except:
A. superior mesenteric vein C. Splenic vein
B. inferior mesenteric vein D. hepatic vein ( D )
_____The following organs are supplied by the superior mesenteric artery Except:
A. spleen C. jejunum
B. vermiform appendix D. ascending colon (A)

Module 26 Endocrine

_____The main pancreatic duct opens into this part of the duodenum:
A. first part C. second part
B. third D. fourth (C)
_____The following are true of the thyroid gland Except:
A. consists of two lobes
B. a vascular organ
C. surrounded by the prevertebral layer of deep fascia (C)
D. its lymph drains mainly into the deep cervical lymph nodes
_____The following describe the parathyroid gland Except:
A. ovoid bodies
B. supplied by the superior thyroid artery ( D )
C. supplied by the inferior thyroid artery
D. intimately related to the inferior border of the thyroid cartilage
_____The following describe the pituitary gland Except:
A. lies in the cavity of the sphenoid bone
B. consists of two glands (D)
C. weighs about 0.5 grams
D. its blood supply is derived from the external carotid artery

Module 27 Male RS
_____The following are true of the vas deferens Except:
A. arises from the tail of the epididymis
B. passes through the inguinal canal
C. also called ejaculatory duct (C)
D. joins the duct of the seminal vesicle
_____The following describe the penis Except:
A. the body consists of three cylinders of erectile tissue
B. the corpus cavernosum expands to form the glans penis ( B )
C. the root is formed by the bulb and crura of the penis
D. supplied by the pudendal nerve and the pelvic plexuses
_____The following statements are true Except:
A. the cremasteric fascia of the scrotum is derived from fascia transversalis
B. the left testis usually lies at a lower level than the right ( A )
C. the prostate gland lies between the neck of the bladder and the urogenital
diaphragm
D. the epididymis is a much coiled tube that lies posterior to the testis
_____The following are contents of the deep perineal pouch in male Except:
A. membranous part of the urethra
B. sphincter urethrae
C. deep transverse perineal muscle (D)
D. prostate gland

Module 28 FRS

_____The following are true of the fallopian tube Except:


A. connects the peritoneal cavity with the cavity of the of the uterus
B. the infundibulum is the widest part of the tube
C. lies in the upper border of the broad ligament (B)
D. measures about 10 cm long
_____The following describe the uterus Except:
A. divided into body, fundus, and cervix
B. the body is related posteriorly to the pouch of Douglas
C. in most women the normal position is retroversion (C)
D. supplied by the uterine artery, a branch of the internal iliac artery
_____The following ligaments play an important part in supporting the uterus and
keeping the cervix in position Except:
A. round ligament C. Cardinal ligament (A)
B. pubocervical ligament D. sacrocervical ligament

BBS Finals – Blocks 1 & 2

Module 4

1. Which of the following differentiates the somatic nervous system (SNS) from the
autonomic nervous system (ANS)? The SNS: C
A. innervates involuntary structures
B. consists of preganglionic and postganglionic neurons
C. when stimulated, produces excitatory effects
D. when sectioned, leads to paralysis of structure innervated
2. The following are characteristics of the autonomic nervous system in general,
EXCEPT: D
A. innervates smooth muscles, cardiac muscles and gland cells
B. possesses homeostatic function
C.  involved  in  “fight  or  flight”  responses
D. its transmitter is exclusively acetylcholine
3. The portion of the autonomics concerned with emergency function is the: A
A. sympathetic division C. enteric division
B. parasympathetic division D. cranial outflow
4. Stimulation of postganglionic fibers that exit from the inferior cervical ganglion will
produce: D
A. lacrimation B. mydriasis C. salivation D. tachycardia
5. Stimulation of postganglionic fibers that exit from the hypogastric ganglion will
produce: C
A. decreased intestinal motility C. ejaculation
B. acetylcholine release D. defecation
6. Stimulation of postganglionic fibers that exit from the ciliary ganglion will produce: B
A. lacrimation B. miosis C. salivation D. bradycardia
7. The following are characteristics of the sympathetic nervous system, EXCEPT: D
A. preganglionic fibers are generally short
B. postganglionic fibers are non-myelinated
C. effect of stimulation is generalized and long-lasting
D. removal is incompatible with life
8. Which of the following is most likely to take place upon stimulation of the vagus?
A. pupilloconstriction C. bronchoconstriction
B. abundant saliva D. erection

Module 10

1. Which of the following immediately precedes secretion of interleukin 1 by the antigen-


presenting cell (APC)? C
A. phagocytosis of foreign antigen
B. release of inflammatory mediators
C. epitope is presented to T helper cell
D. processing of epitope by APC
2. Cell-mediated immune response includes the following, EXCEPT: A
A. synthesis of immunoglobulins M and G
B. proliferation of effector T cells
C. determination of response specificity
D. release of cytokines by effector cells
3. When the amount of antigen is greater than the amount of antibody, an antigen-
antibody reaction may not take place because of: D
A. high zone tolerance C. prozone phenomenon
B. low zone tolerance D. postzone phenomenon
4. Down-regulation of immunologic response to forbidden clones is called: C
A. clonal deletion C. clonal anergy
B. clonal abortion D. clonal suppression
Module 11

1. Which of the following genes of the ABO system is responsible for activity of the H
gene on the basic precursor on red cells? C
A. A gene B. B gene C. Z gene D. Se gene
2. A subject has negative results for Anti-A and Anti-B, as well as with A, B and O cells.
He is most probably: B
A. a type AB newborn C. a type AB elderly with leukemia
B. a type O newborn D. a type O elderly with multiple myeloma

Module 15

1. Mechanical or chemical irritation of the nasal mucosa will produce the following
effects, EXCEPT: D
A. sneeze C. bronchoconstriction
B. rise in BP D. tachycardia
2. Expansion of the alveoli is directly attributed to a change in: C
A. intrathoracic pressure C. transpulmonary pressure
B. intrapulmonary pressure D. transthoracic pressure
3. Which of the following muscles plays a part in forced expiration? B
A. serratus posterior superior C. levatores costarum
B. serratus posterior inferior D. sternocleidomastoid
4. The amount of air that can be forcibly inhaled from the end expiratory level is called:
A. inspiratory reserve volume C. inspiratory capacity *
B. total lung capacity D. vital capacity
5. Which of the following is the only method capable of measuring trapped gas? B
A. wet spirometry C. closed circuit method
B. body plethysmography D. open circuit method

Module 16

1. Which of the following gas laws describes the inverse relationship between the size of
the gas molecules and the rate of gas diffusion? A
A.  Avogadro’s  law C. Charles’  law
B.  Boyle’s  law D.  Dalton’s  law
2. One of the following, when decreased, will increase the rate of gas diffusion: D
A. gas temperature C. solubility coefficient
B. membrane surface area D. diffusion distance
3. The following are true as regards ventilation-perfusion ratio, EXCEPT: B
A. transpulmonary pressure is greater at the upper lung
B. greater expansion of the lung at the hilus than at the periphery
C. greater perfusion of the left lung than of the right
D. lung apices have more carbon dioxide than lung apices
4. Which of the following shifts the oxyhemoglobin curve to the right by causing
stabilization of the deoxy configuration of hemoglobin? C
A. hydrogen ions C. increased 2,3 diphosphoglycerate
B. high temperature D. increased carbon dioxide
5. Carbon dioxide unloading at the tissues is called: B
A. Bohr effect C. positive cooperativity
B. Haldane effect D. blood buffer curve

Module 17

1. The most important mechanism regulating glomerular filtration is: D


A. myogenic mechanism C. metabolic mechanism
B. tissue pressure mechanism D. tubulo-glomerular feedback mechanism
2. Substance X was used to measure glomerular filtration rate of a normal male subject.
The result showed that the glomerular filtration rate was 110 ml/min. This means that:
D
A. substance X is freely filtered
B. substance X is not degraded by the tubules
C. substance X is not synthesized by the tubules
D. substance X is reabsorbed by the tubules
3. Substance Y is reabsorbed via a Tm-limited mechanism. If substance Y appears in the
urine of a normal subject, the following are logical conclusions, EXCEPT: B
A. there might be another substance that competes with Y for carriers
B. substance Y may have been converted to anionic or cationic form
C. the tubular load of substance Y may have exceeded its Tm
D. different nephrons may have different Tm values for substance Y
4. Majority of reabsorbable substances are reabsorbed at the: A
A. proximal tubule C. loop of Henle
B. distal tubule D. collecting duct
5. Countercurrent multiplication of osmolar concentration takes place at the: C
A. proximal tubule C. loop of Henle
B. distal tubule D. vasa recta
6. Which of the following is most likely to take place if bladder urine volume is 350 ml in a
normal subject? D
A. sensation of bladder filling is experienced
B. the first desire to void is experienced
C. voluntary control of micturition is lost
D. mucosal folds of the bladder unfold

Block 3 – Module 26

1. An individual who has fever experiences chills. This is a form of: A


A. negative feedback C. feedforward regulation
B. positive feedback D. acclimatization
2. The life span of a red cell is approximately 120 days. This is because of: C
A. adaptation C. apoptosis
B. acclimatization D. aging
3. Which of the following second messengers activates calmodulin? B
A. diacylglycerol C. cyclic AMP
B. calcium D. cyclic GMP
4. Which of the following is a steroid hormone? D
A. prolactin C. epinephrine
B. thyroxine D. aldosterone
5. An individual was walking down the street when his enemy suddenly accosted him. He
decided to turn and run for help. This corresponds to which of the following stages of
the general adaptation syndrome? A
A. stage of shock C. stage of resistance
B. stage of countershock D. stage of exhaustion
6. A 21 year old female has to walk down a dark alley on her way home. Since there
have been incidences of robbery in this area, she started experiencing tachycardia
and hyperventilation, the moment she started walking down the alley. Sweating was
also evident. These effects are mostly due to activity of the: D
A. ACTH axis C. vasopressin axis
B. thyroid axis D. sympathetic division
XAVIER UNIVERSITY – JOSE P RIZAL COLLEGE OF MEDICINE
Basic Biomedical Sciences
Third Bimonthly Period
QUIZ # 4 (Genetics)

NAME ____________________________ 4 Dec 2007 SCORE ____________


MULTIPLE CHOICE: write the letter of the BEST answer on the answer sheet. Use capital
letters in blue/black ink. NO EASURES/SUPERIMPOSITIONS ALLOWED!

1. If the ribose in a nucleotide  has  a  hydrogen  atom  attached  to  its  C2’  instead  of    OH  the  
nucleotide is part of a strand of:
A. DNA C. tRNA
B. mRNA D. rRNA
2. The naturally occurring nucleotide that is a component of the precursor of glycogen is
derived from:
A. Adenosine C. uridine
B. Guanosine D. inosine
3. The backbone of a DNA strand is composed of:
A. bases and phosphates
B. sugars and bases
C. phosphates and nucleotides
D. phosphates and sugars
4. A synthetic nucleotide derivative is able to prevent the formation of uric acid from
purine due to its resemblance to the base of:
A. Inosine C. cytidine
B. Adenosine D. thymidine
5. Base pairs of DNA are stabilized by:
A. Van  der  Waal’s  forces C. ester linkages
B. Polar bonds D. hydrogen bonds
6. Which of the following precursors of purine and pyrimidine nucleotide de novo
synthesis is derived from pentose phosphate pathway?
A. Glycine C. aspartate
B. PRPP D. H4Folate
7. The enzyme that catalyzes the committed step in de novo synthesis of purine
nucleotide is:
A. PRPP synthase C. amidophosphoribosyl transferase
B. CAP synthase D. aspartate transcarbomylase
8. The formation of the first true pyrimidine nucleotide via de novo pathway is catalyzed
by:
A. orotydylic acid decarboxylase C. dihydroorotase
B. dihydroorotic acid dehydrogenase D. CAP synthase
9. The synthesis of purine deoxyribonucleotide occurs by:
A. decarboxylation of the corresponding ribonucleotide
B. addition of ribose-PO4 after the ring is formed
C. direct  reduction  of  the  2’C  for  the  sugar  moiety  of  corresponding  nucleotide
D. direct transfer of deoxyribose sugar for ribose moiety of the nucleotide
10. Purine nucleotide is synthesized via salvage pathway of the base or nucleotide in the:
A. Hepatocytes C. red blood cells
B. cardiac muscles D. endocrine cells
11. This nucleotide serves as intermediate in the synthesis of the major purine nucleotide.
A. AMP C. IMP
B. GMP D. UMP
12. This enzyme catalyzes the conversion of purine nucleotide intermediate to form uric
acid.
A. Uricase C. adenosine kinase
B. xanthine oxidase D. guanase
13. Deficiency of this enzyme causes the hyperuricemia in gout.
A. adenosine kinase C. HGPR transferase
B. guanosine kinase D. adenosine transferase
14. The fundamental unit of genetic information is:
A. DNA C. nucleotides
B. genes D. nucleosides
15. The genetic code contained in DNA is basically determined by the
____________ of nucleotides.
A. shape C. sequence
B. number D. all of the above
16. One of the following statements is NOT TRUE regarding RNA:
A. The sugar moiety in RNA is ribose.
B. Instead of thymine, RNA contains uracil.
C. Purine nucleotides does not necessarily equal the number of pyrimidine
nucleotides.
D. RNA exists as a double helix like DNA.
17. The messenger RNA serves as a messenger between these two
organelles of the cell:
A. cell membrane and nucleus C. nucleus and ribosomes
B. nucleus and cytosol D. cell membrane and mitochondria
18. The type of bonds which hold A to T and C to G in DNA is:
A. sulfide C. hydroxyl
B. hydrogen D. sulfhydryl
19.  This  substance  can  hydrolyze  RNA  to  2’,  3’  cyclic  diesters  of  its
mononucleotides:
A. alkali C. water
B. acid D. hydrogen
20. The length of DNA unwinding per polymerase molecule in the elongation process in
terms of base pairs:
A. 20 B. 25 C. 30 D. 35
21. The initial step involved in DNA replication:
A. formation of the replication fork
B. reconstitution of chromatin structure
C. identification of the origins of replication
D. initiation of DNA synthesis
22. The enzyme responsible for unwinding of duplex DNA segments:
A. DNA topoisomerase C. ligase
B. helicase D. polymerase gamma
23. Term which refers to a single-base mutation characterized by changes from purine to
either of the two pyrimidines:
A. transversion C. missense mutation
B. transition D. nonsense codon
24. Replication of the DNA genome occurs at what specified time during the
life span of the cell:
A. S phase C. M phase
B. G1 phase D. G2 phase
25. The enzyme which initiates the synthesis of RNA primers:
A. DNA polymerase C. DNA primase
B. helicase D. topoisomerase
26. Which of the following events least likely happens in RNA synthesis?
B. utilization of ribonucleotides
C. involvement of a primer
D. absence of proofreading
E. copying of only a portion of the genome
27. The activity of this enzyme prevents the formation of superhelical complexes:
A. Hydroxylase C. Epimerase
B. Transferase D. Topoisomerase

28. Which of the following is least likely true of the TATA box?
A. about 30 base pairs upstream from the transcription start site
B. its consensus sequence is generally TATAAA
C. attached by binding proteins
D. needs additional cis elements at all times
29. Which of the following best describes the CAAT box?
A. involves DNA sequences
B. binds Sp1 protein via DNA binding domains
C. requires strict spacing of trans elements
D. decreases the rate of transcription
30. The main function poly(A) tail is to:
A. efficiently start translation
B. shield  the  3’  end  of  mRNA  from  an  exonuclease
C. protect  the  5’  end  from  an  exonuclease
D. initiation site for splicing
31. Which of the following is true of the termination process of transcription?
A. dependent on DNA cis elements
B. modulated by cis-acting protein factors
C. signaled by RNA splicing of exons
D. formation  of  5’  to  3’  tail
32. Which of the following occurs during protein synthesis?
A. message  in  mRNA  is  read  from  3’  to  5’  direction
B. translation  begins  near  the  5’  terminal  of  the  mRNA
C. free amino acids attach to corresponding codons in mRNA
D. tRNA dictates the amino acid sequence of the protein to be synthesized
33. Which of these is the first step in the initiation phase of translation?
A. formation of the 43S preinitiation complex
B. formation of the 43S initiation complex
C. formation of the 80S initiation complex
D. dissociation of the ribosomes into its two subunits
34. These are the three termination codons:
A. UAA, UGA, AAU C. UAA, AUA, AAU
B. UAA, UAG, UGA D. UAA, AGU, GAU
35. Which of these events is associated with the termination of translation?
A. tRNA anticodon recognizes and binds with the stop codon
B. stop codon of mRNA appears in the A site of the ribosome
C. releasing factor RF1 releases mRNA from the ribosome
D. an amino acid is added to release tRNA from the P site
36. Which of these phases in translation immediately precedes the release of the
synthesized protein from the ribosome?
A. activation of the amino acid
B. folding of the polypeptide chain
C. elongation of the polypeptide chain
D. initiation of the polypeptide chain
37. Which of the following is the biological importance of replication?
A. viability of the species C. prevents cancer
B. production of normal proteins D. prevents mutation
38. Which of the following enzyme participates in replication in a specialized manner like
repair?
A.  3’  exonuclease C. polymerase enzyme
B.  5’  exonuclease D. all of these
39. A group of coordinately regulated genes that function in gene expression refers to ?
A. exon C. cistron
B. operon D. intron
40. Which of the following is NOT true of the operator region of the lac operon?
A. lies adjacent to initiator C. can block transcription
B. spans transcriptional site D. repressor proteins bind to this site
41. Which of the following is responsible for the synthesis of the lac repressor molecules?
A. Z gene C. A gene
B. Y gene D. I gene
42. Which of the following can bind to repressor molecules ?
A. CAP C. lactose
B. cAMP D. promoter
43. Which portion of the DNA directs proper initiation of transcription ?
A. operator C. initiator
B. regulator D. promoter
44. Which inhibitor is a structural analog of aminoacyl tRNA that prematurely terminates
elongation ?
A. puromycin C. ricin
B. streptomycin D. chloramphenicol
45. Which of the following is required for activation of amino acids to their respective
tRNA’s  ?
A. ATP C. CTP
B. GTP D. cAMP
46. Which enzyme is capable of proofreading and correcting errors during translation ?
A.  3’exonuclease C. polymerase
B.  5’  exonuclease D. aminoacyl tRNA synthetase
47. DNA replicates in a semi-conservative manner. If a completely radioactive dsDNA
undergoes 2 rounds of replication in a solution free of radioactive label, what is the
radioactivity status of the resulting 4 DNA molecules ?
A. All should contain radioactivity
B. half should contain NO radioactivity
C. half should contain radioactivity in both strands
D. one should contain radioactivity in both strands
48. Which of the following is true of DNA replication ?
A. it is initiated on RNA primer
B. it is unidirectional in prokaryotes
C. it involves only the formation of phosphodiester bonds
D. it proceeds birectionally from a single initiation point on each chromosome in
eukaryotic cell
49.    DNA  polymerase  synthesizes  DNA  only  in  the  5’  to  3’  direction.  Yet,  at  the  replication  
fork, both strands of parental DNA are being replicated with the synthesis of new
DNA.
While one strand is synthesized  in  the  5’  to  3’  direction,  the  other  strand  strand  
appears
to  be  synthesized  in  the  3’  to  5’  direction.  Which  of  the  following  explains  this  paradox  
?
A.  3’  to  5’  replicase
B. presence of Okazaki fragment
C. lack of RNA primer on one of the strands
D. immediate replication and transfer of the leading strand
50. Which of the following is the function of rho factor ?
A. increase the rate of RNA synthesis
B. bind catabolite repressor to the promoter region
C. participate in the proper termination of transcription
D. eliminate the binding of RNA polymerase to the promoter

XAVIER UNIVERSITY – DR JOSE P RIZAL COLLEGE OF MEDICINE


BASIC BIOMEDICAL SCIENCES

Final Exam: BLOCK 2 (Modules 11 – 20) March 9, 2006


Write the letter of the BEST answer on the answer sheet. Use capital letters in ink.
NO ERASURES/SUPERIMPOSITIONS ALLOWED!

1. The temporary hemostatic plug is primarily due to:


A. red cells B. white cells C. platelets D. fibrin
2. Bladdy was suspected of having a hemorrhagic disorder. Laboratory tests were performed
and these are the results: APTT = abnormal; Prothrombin time = normal; thrombin time =
normal. Based on these limited data, Bladdy is most probably deficient in this factor:
A. factor XII B. factor VII C. factor VIII D. factor I
3. Which of the following genes of the ABO system is responsible for activity of the H
gene on the basic precursor on red cells?
A. A gene B. B gene C. Z gene D. Se gene
4. A subject has negative results for Anti-A and Anti-B, as well as with A, B and O cells.
He is most probably:
A. a type AB newborn C. a type AB elderly with leukemia
B. a type O newborn D. a type O elderly with multiple myeloma
5. The following are contents of the middle mediastinum Except:
A. arch of azygos vein C. ascending aorta
B. heart D. esophagus
6. The following are true of the superior mediastinum Except:
A. bounded posteriorly by the first four thoracic vertebrae
B. opens above to the thoracic outlet
C. further subdivided into anterior, middle, and posterior parts
D contains the trachea
7. Which of the following surfaces/parts of the heart is formed mainly by the right
and left ventricles?
A. sternocostal surface C. posterior surface
C. diaphragmatic surface D. apex
8. Which of the following types of trabeculae carnae conveys the right branch of the
atrioventricular bundle?
A. type I B. type II C. type III D. type IV
9. The following are descriptive of the pericardium Except:
A. a fibroserous sac
E. encloses the heart and the roots of the great vessels
F. a pericardial cavity is formed between its fibrous and serous parts
G. its visceral layer is often called the epicardium
10.The following are structural characteristics of the cardiac muscle, EXCEPT:
A .made up of branching muscle cells
B. muscle fibers are arranged in laminae
C. cells have one or two oval central nuclei
D. banding of fibers differs from that of skeletal muscle
11.Which of the following is the primary cell type in the AV node?
A. P B. transitional C. Purkinje D. ordinary
12.Which of the following membrane junction types allow nutrients or ions to pass
directly from one cardiac cell to another?
A. nexus C. zonula occludens
B. macula adherens D. fascia adherens
13.The following are true of the epicardium, EXCEPT:
A. continuous with the intima of the large arteries
B. lined by simple squamous epithelium
C. has loose connective tissue with adipose cells
D. glands are absent

14. When the SA node is diseased, which of the following is most likely to take its place
as the cardiac pacemaker?
E. atrial muscles C. bundle of His
F. AV node D. terminal Purkinje network
15. When the right atrium is enlarged, which of these ECG configurations will be prolonged?
A. PR interval and QRS interval C. P wave duration and PR interval
B. P wave duration and ST segment D. QRS interval and QT interval
16. If the P vector is located at (-) 10 degrees, the P wave is tallest in this ECG lead:
A. lead I B. lead II C. lead aVL D. lead aVR
17. Peak aortic (systolic) pressure occurs at the end of this phase of the cardiac cycle:
A. isovolumetric contraction C. protodiastole
B. isovolumetric relaxation D. rapid ejection
18. The following events occur during cardiac diastole, EXCEPT:
A. ventricular filling C. atrial contraction
B. ventricular relaxation D. opening of semilunar valves
19. If the chordae tendinae in the left ventricle are cut, the following would occur,
EXCEPT:
A. blood regurgitates into the left atrium during systole
B. mitral valve would be everted during systole
C. systolic murmur would be heard
D. stroke volume would be increased
20. The heart may stop in systole in which of these conditions?
A. hypercalcemia C. hyperkalemia
B. hypocalcemia D. hypokalemia
21. Which of the following gives origin to the right coronary artery?
A. anterior aortic sinus C. right posterior aortic sinus
B. left posterior aortic sinus D. posterior aortic sinus
22. The following are direct arterial branches of the arch of the aorta Except:
A. right common carotid artery C. left common carotid artery
B. right subclavian artery D. brachiocephalic artery
23. The following veins drain directly to the superior vena cava Except:
A. right brachiocephalic vein C. left brachiocephalic vein
B. internal jugular vein D. azygos vein
24. The following are direct tributaries of the inferior vena cava Except:
A. left testicular vein C. hepatic veins
B. renal veins D. common iliac veins
25. The hepatic portal vein is formed by the union of the:
A. inferior mesenteric and superior mesenteric veins
B. superior mesenteric and hepatic veins
C. inferior mesenteric and splenic veins
D. superior mesenteric and splenic veins
26. The following are contents of the tunica adventitia of blood vessels, EXCEPT:
A. collagen & elastic fibers C. vas vasorum & lymphatic capillaries
B. adipose cells D. layer of endothelial cells
27. The following describe medium arteries, EXCEPT:
A. areolar tissue in adventitia
B. prominent internal elastic lamina
C. abundant elastic fibers in media
D. similar thickness of media & adventitia
28. Which of the following is true as regards histologic features of capillaries?
A. made up only of tunica intima C. gap junction between endothelial cells
B. collagen fibers in tunica media D. endothelial cells rest on a basal lamina
29. The following characterize large veins, EXCEPT:
A. well-developed tunica media C. abundant loose connective tissue
B. indistinct wall boundaries D. has the thickest tunica adventitia

30.The following describe lymphatic vessels, EXCEPT:


A. endothelium-lined C. fenestrated capillaries
B. underdeveloped adventitia D. absence of basal lamina in capillaries
31. Stimulation of the vasopressin system reflexly results in the following, EXCEPT:
A. stimulation of the thirst center C. increased renal reabsorption of water
B. increased release of ADH D. dilute urine
32. When right ventricular contractions are weak, which of the following would increase?
A. central venous pressure (CVP) and peripheral venous pressure (PVP)
B. right ventricular stroke volume (SV) and venous return (VR)
C. CVP, PVP, VR and SV
D. CVP, PVP, and VR
33. Capillary filtration tends to increase in these conditions, EXCEPT:
A. hypoproteinemia C. venous obstruction
B. arteriolar constriction D. increased PVP
34. Mean arterial pressure tends to increase when the following are increased, EXCEPT:
A. blood viscosity C. stroke volume
B. arteriolar radius D. blood volume
35. Which of the following explains the relationship between flow velocity and lateral
pressure in blood vessels?
A.    Critical  Reynold’s  number                                      C.  Law  of  Laplace
B. Poiseulle-Hagen  formula                                            D.  Bernoulli’s  principle
36. The following form the bony framework of the external nose Except:
A. frontal processes of maxilla C. nasal bones
B. body of the sphenoid D. nasal part of the frontal bone
37. The following are true of the respiratory region of the nasal cavity Except:
A. constitutes the majority of the cavity
B. its lateral wall forms the main wall of the nasal cavity
C. receives the openings of all the paranasal sinuses
D. its medial wall is marked by three elevations called conchae
38. The following are true of the cartilages of the larynx as described Except:
A. epiglottis-a thin leaf like plate of elastic cartilage
B. thyroid cartilage- largest cartilage
C. cuneiform cartilage- has two processes, vocal and muscular processes
D. cricoid cartilage-forms a complete ring around the airway
39. The following muscles controls the movements of the vocal cords Except:
A. thyroepiglottic C. thyroarytenoid
B. lateral cricoarytenoid D. posterior cricoarytenoid
40. The right principal bronchus differ from the left principal bronchus by the
following Except:
A. wider C. more vertical
B. longer D. gives off two lobar bronchi
41. The following relates to the histologic features of the lung, EXCEPT:
A. contains pulmonary and bronchial vessels
B. invested with mesothelium
C. outer covering contain serous glands
D. show intrapulmonary bronchus and bronchioles
42. These are histologic features of the trachea, EXCEPT:
A. lined by pseudostratified columnar ciliated epithelium
B. has thickest basement membrane in the body
C. mixed glands in submucosa
D. skeletal muscles in tunica adventitia
43. The last segment of the respiratory tract to display smooth muscle cells:
A. alveolar duct C. alveolar wall
B. respiratory bronchiole D. terminal bronchiole

44. The following are true of the bronchiole, EXCEPT:


A. simple columnar to cuboidal epithelium
B. loose connective tissue in lamina propria
C. smooth muscle in mucosa
D. mixed glands in ordinary bronchioles
45. The following cells to role association are correct, EXCEPT:
A. type I pneumocyte > diffusion of gases
B. type II pneumocyte > lung surfactant
C. septal cell > maintenance of lung interstitium
D. no exception mentioned
46. Mechanical or chemical irritation of the nasal mucosa will produce the following
effects, EXCEPT:
A. sneeze C. bronchoconstriction
B. rise in BP D. tachycardia
47.Expansion of the alveoli is directly attributed to a change in:
A. intrathoracic pressure C. transpulmonary pressure
B. intrapulmonary pressure D. transthoracic pressure
48. Which of the following muscles plays a part in forced expiration?
A. serratus posterior superior C. levatores costarum
B. serratus posterior inferior D. sternocleidomastoid
49. The amount of air that can be forcibly inhaled from the end expiratory level is called:
A. inspiratory reserve volume C. inspiratory capacity
B. total lung capacity D. vital capacity
50. Which of the following is the only method capable of measuring trapped gas?
A. wet spirometry C. closed circuit method
B. body plethysmography D. open circuit method
51. Which of the following gas laws describes the inverse relationship between the size of the
gas molecules and the rate of gas diffusion?
A.  Avogadro’s  law C.  Charles’  law
B.  Boyle’s  law D.  Dalton’s  law
52. One of the following, when decreased, will increase the rate of gas diffusion:
A. gas temperature C. solubility coefficient
B. membrane surface area D. diffusion distance
53. The following are true as regards ventilation-perfusion ratio, EXCEPT:
A. transpulmonary pressure is greater at the upper lung
B. greater expansion of the lung at the hilus than at the periphery
C. greater perfusion of the left lung than of the right
D. lung apices have more carbon dioxide than lung apices
54. Which of the following shifts the oxyhemoglobin curve to the right by causing stabilization
of the deoxy configuration of hemoglobin?
A. hydrogen ions C. increased 2,3 diphosphoglycerate
B. high temperature D. increased carbon dioxide
55. Carbon dioxide unloading at the tissues is called:
A. Bohr effect C. positive cooperativity
B. Haldane effect D. blood buffer curve
56. The most ventral structure at the hilum of the kidney:
A. renal artery C. renal vein
B. ureter D. renal pelvis
57. Forms the outermost covering of the kidney:
A. renal fascia C. perirenal fat
B. pararenal fat D. fibrous
57. The ureters joined the urinary bladder at its:
A. apex B. neck C. base D. superior surface
58. The following are true of the membranous urethra in male Except:
A. least dilatable portion of the urethra C. lies within the urogenital diaphragm
B. widest portion of the urethra D. shortest portion of the urethra

59. The left kidney is related anteriorly to the following structures Except:
A. suprarenal gland C. liver
B. pancreas D. Spleen
60. The following structures are seen in the renal pyramid, EXCEPT:
A. convoluted tubules C. vasa recta
B. loop of Henle D. collecting duct
61. The phagocytic cells that clean up the basal lamina of the glomerulus:
A. podocytes C. macula densa
B. mesangial D. lacis cells
62. The following make up the juxtaglomerular apparatus, EXCEPT:
A. macula densa C. mesangial cells
B. juxtaglomerular cell D. lacis cells
63. The filtration barrier is formed by the following, EXCEPT:
A. all visceral epithelial cells C. basal lamina of collagen, glycoproteins
B. filtration slits between pedicels D. fenestrated endothelial cells
64. The following are general histologic pattern of the urinary passages, EXCEPT:
A. transitional lining epithelium
B. mucous membrane without glands
C. inner longitudinal, outer circular smooth muscle arrangement
D. no exception mentioned
65. The most important mechanism regulating glomerular filtration is:
A. myogenic mechanism C. metabolic mechanism
B. tissue pressure mechanism D. tubulo-glomerular feedback mechanism
66. Substance X was used to measure glomerular filtration rate of a normal male subject. The
result showed that the glomerular filtration rate was 110 ml/min. This means that:
A. substance X is freely filtered
B. substance X is not degraded by the tubules
C. substance X is not synthesized by the tubules
D. substance X is reabsorbed by the tubules
67. Substance Y is reabsorbed via a Tm-limited mechanism. If substance Y appears in the
urine of a normal subject, the following are logical conclusions, EXCEPT:
A. there might be another substance that competes with Y for carriers
B. substance Y may have been converted to anionic or cationic form
C. the tubular load of substance Y may have exceeded its Tm
D. different nephrons may have different Tm values for substance Y
68. Majority of reabsorbable substances are reabsorbed at the:
A. proximal tubule C. loop of Henle
B. distal tubule D. collecting duct
69. Countercurrent multiplication of osmolar concentration takes place at the:
A. proximal tubule C. loop of Henle
B. distal tubule D. vasa recta
70. Which of the following is most likely to take place if bladder urine volume is 350 ml in a
normal subject?
A. sensation of bladder filling is experienced
B. the first desire to void is experienced
C. voluntary control of micturition is lost
D. mucosal folds of the bladder unfold
71. Jane has passed 8 watery stools for the past 6 hours. She feels thirsty and tired. Based
on this short history we can say that the following are the likely changes occurring in her
body fluids:
I. decreased ECF volume and ICF volume,
J. decreased ECF volume and ECF osmolarity
K. decreased ECF volume, no change in ECF osmolarity, no change in ICF volume
L. decreased ECF volume and osmolarity and no change in ICF volume and osmolarity

72. Mia was admitted to the hospital because of severe asthmatic attack. On admission, the
results of laboratory exams also showed an acid base disturbance. After three days,
laboratory exams were again done and these were the results: pH = 7.352; [HCO3-] =
25.9 mmols/liter; PCO2 = 49.98 mmHg; [BE] = 1.89 mmols/liter. Based on these results,
we can say that Mia has a:
E. normal acid-base status C. fully compensated metabolic alkalosis
F. fully compensated respiratory acidosis D. fully compensated respiratory alkalosis
73. Body responses to metabolic alkalosis secondary to ingestion of a bottle of bicarbonate
tablets include the following, EXCEPT:
G. buffering by the bicarbonate buffer system (BBS)
H. buffering by the non-BBS
I. decreased alveolar ventilation
J. alkalinization of theurine
74. Given the following laboratory results of Gino: pH = 7.438; [HCO3-] = 20.2 mmols/liter;
PCO2 =30.8 mmHG; [BE] = (-) 2 mmols/liter. Based on these results,he is likely to have:
E. fully compensated metabolic alkalosis C. uncompensated metabolic alkalosis
F. fully compensated respiratory alkalosis D. normal acid-base status
75. The deep membranous layer of superficial fascia of the abdomen is called:
A.  Colles’  fascia                 C.  Camper’s    fascia
B. Scarpa’s  fascia D. transversalis fascia
76. The following aponeuroses of muscles form the rectus sheath Except:
A. rectus abdominis C. transversus abdominis
B. external oblique D. internal oblique
77. The following are sites of constrictions of the esophagus Except:
I. at the level of the 6th cervical vertebra
J. at the level of the arch of the aorta
K. at the level of the right primary bronchus
L. at the level of the 10th thoracic vertebra
78. The following are contents of the of the lesser omentum Except:
A. hepatic artery C. portal vein
B. hepatic vein D. common bile duct
79. The following are intraperitoneal organs Except:
A. pancreas C. Spleen
B. stomach D. vermiform appendix
80. The following veins belong to the portal venous system Except:
A. superior mesenteric vein C. Splenic vein
B. inferior mesenteric vein D. hepatic vein
81. The following organs are supplied by the superior mesenteric artery Except:
A. spleen C. jejunum
B. vermiform appendix D. ascending colon
82. The following are correct matches of digestive organs to their distinct features,
EXCEPT:
A. large intestine > gland of Lieberkuhn
B.  duodenum  >  Brunner’s  glands                                  
C. stomach, body > fundic gland
D. esophagus > stratified squamous epithelium, mucous glands
83. A microscopic slide displays a mucosa with simple columnar epithelial cells with
thin striate border, without goblet cells; loose connective tissue without glands.
The organ is likely to be:
A. stomach, cardia B. jejunum C. appendix D. gallbladder
84. The small intestine may be distinguished from the large intestine by the presence
of the following, EXCEPT:
A. intestinal villi C. intestinal glands
B.  Paneth’  cells                                                                                        D.  distribution  of  goblet  cells
85. The  following  ‘cells  to  secretions’  associations are correct, EXCEPT:
A. parietal > hydrochloric acid C. enteroendocrine > gastrin
B. chief > gastric intrinsic factor D.  Paneth’s  >  lysozyme

86. Which of these is NOT right regarding the liver?


A. liver cells are arranged in cords between sinusoids
B. hepatic sinusoids are lined with endothelial and von Kuppfer cells
C. hepatocytes around central vein have best access to nutrients
D. branches of bile duct, portal vein, hepatic artery form the portal triad
87. Which of these pancreatic cells secrete enzymes?
A. acinar C. beta
B. alpha D. delta
88. Which of these phases of deglutition is under voluntary control?
A. oral phase C. esophageal phase
B. pharyngeal phase D. all of the above
89. Glucose absorption from the intestinal lumen is enhanced by this substance:
A. amino acid C. magnesium
B. potassium D. sodium
90. Which of the following events is unique in the digestion of fats?
A. CCK secretion C. micelle formation
B. emulsification D. secretion of pancreatic juice
91. Bile is reabsorbed in this portion of the GIT:
A. duodenum C. ileum
B. jejunum D. ascending colon
92. Which of the following is the most frequent type of movement in the small intestine?
A. peristalsis C. migrating motor complex
B. segmentation D. haustral shuttling
93. The following are true of gastric acid secretion, EXCEPT:
A. Chief cells secrete HCl
B. Gastrin stimulates gastric acid secretion
C. acid secretion between meals is low
D. entry of food into the stomach initiates the gastric phase of acid secretion
94. Meniong, a 29 year old, 5 feet tall, Filipino athlete keeps record of his average body
weight every year. The following is his average weight for the past four years. At what
year is Meniong considered overweight?
A. 2000 – 40 kg C. 2003 – 65 kg
B. 2001 – 54 kg D. 2002 – 80 kg
95. Juana, 20 year old cashier asked her brother, a medical student, how much iron does
she need everyday. The best answer is:
A. 5 mg /day B. 10 mg/day C. 30 mg/day D. 50 mg/day
96. Cherry, who has just delivered her baby, questions the pediatrician why she was told to
breastfeed her baby soon after delivery. The best answer would be:
A.  The  colustrum  has  higher  immune  globulin  than  cow’s  milk
B. Breastmilk has a laxative effect because of the protein content
C.  Breastmilk  has  more  vitamin  content  than  cow’s  milk
D. all of the above
97. Which of the following vitamins is needed as cofactor in a large number of oxidation-
reduction reactions?
A. Thiamin B. Niacin C. Pyridoxine D. Cobalamin
98. Supplemental amount of this vitamin is recommended immediately after birth to prevent
bleeding:
A. vit. A B. Vit. E C. vit. K D. vit. D
99. The characteristic flaky paint dermatosis is seen in:
A. Marasmus C. Beriberi
B. Kwashiorkor D. Pellagra
100. Unless carbohydrate is restricted fruits may be given at least 3 exchanges per day. The
calorie derived from fruit is:
A. 12 calories C. 30 calories
B. 120 calories D. 10 calories

BM 4

Audition/vestib

1. Hair cells of the organ of Corti are stimulated when;


A. Organ of Corti is deflected downward
B. stereocilia are bent toward the spiral limbus
C. stereocilia are bent away from the spiral ligament
D. basilar membrane is deflected toward the scala vestibule
2. Which of these structures must vibrate to effect ossicular route of sound conduction
but not the osseous route of sound conduction?
A. round window C. basilar membrane
B. tympanic membrane D. organ of Corti
3. When a sound with a frequency of 18,000 Hz is presented to the ears of _____, which
of the following is expected to occur?
A. She is not able to hear the sound
B. She hears a low-pitched sound
C. tympanic membrane vibrates at a lower frequency
D. basilar membrane vibration peaks near the base
4. Dr _____ tried to fix his gaze on a picture frame on the wall as he moved his head to
his left. Which of these nerves were stimulated as he started to do this?
A. left vestibular nerve and right abducent nerve
B. left vestibular nerve and right oculomotor nerve
C. right vestibular nerve and left oculomotor nerve
D. right vestibular nerve and right abducent nerve
5. _____ was traveling by bus from Marawi to Cagayan de Oro. As she was about to get
off from the bus, she noticed that as her right foot was about to touch the ground, she
felt a stiffening of her right thigh and leg. This is most probably due to stimulation of
the:
A. horizontal semicircular duct C. utricle
B. vertical semicircular duct D. saccule

Spinal cord

21. Higher centers in the brain influence spinal segmental reflexes via these tracts:
A. corticospinal tract, vestibulospinal tract, spinothalamic tract, fasciculus gracilis
B. corticospinal tract. vestibulospinal tract, spinothalamic tract
C. corticospinal tract, vestibulospinal tract
D. corticospinal tract
22. Aside from the effector organ, these are the components of a monosynaptic reflex arc:
A. receptor organ, afferent neuron, efferent neuron, interneuron/s
B. receptor organ, afferent neuron, efferent neuron
C. receptor organ, afferent neuron
D. receptor organ
23. Which of the following is/are manifestations of lower motor neuron lesions?
A. flaccid paralysis and atrophy of the muscles supplied
B. Loss of reflexes including the cremasteric reflex
C. spastic paralysis and dystonia
D. Spasms and Babinski sign

Brain stem

1. A 28-year old man went to the office of Dr Irene for consultation. He complained of
paralysis of the lower half of his left face. His speech was slurred. There was no
other pertinent finding. Dr Irene correctly diagnosed that the patient had a lesion in
the:
A. right facial nerve C. right corticonuclear fibers
B. left facial nerve D. left corticonuclear fibers

Cerebellum

1. Which of these is true of the cerebellum?


A. divided into four main lobes
B. located behind the spinal cord
C. covered superiorly by the tentorium cerebelli
D. cerebellar hemispheres are joined by the corpus callosum
2. The bodies of these cells are located in the granular layer of the cerebellar cortex.
A. granular cells, golgi cells, basket cells, and stellate cells
B. granular cells, golgi cells and basket cells only
C. granular cells and golgi cells only
D. granular cells only
3. The vermal cortex of the cerebellum influences movements of the following:
A. neck, thorax, shoulders, and arms
B. neck, thorax, and shoulders only
C. neck and thorax only
D. neck only
4. Which of these afferent cerebellar pathways eventually rich the cerebellar cortex via
climbing fibers?
A. Corticopontocerebellar C. anterior spinocerebellar
B. cerebro-olivocerebellar D. posterior spinocerebellar
5. The efferent fibers of this cerebellar nucleus form a large part of the superior
cerebellar peduncle
A. Emboliform C. Fastigial
B. Globose D. Dentate
6. The axons of which of these cells carry the entire output of the cerebellar cortex?
A. basket cells C. Purkinje cells
B. stellate cells D. granule cells
7. The cerebellum receives input regarding voluntary movement from which of these
structures?
A. muscles, tendons, joints, and cerebral cortex
B. muscles, tendons, and joints only
C. muscles and tendons only
D. muscles only
8. Fibers from which of these intracerebellar nuclei leave the cerebellum via the inferior
cerebellar peduncle?
A. Dentate C. globose
B. Emboliform D. fastigial
9. Which of the intracerebellar nuclei project on the neurons of the lateral vestibular
nucleus?
A. Dentate C. globose
B. Emboliform D. fastigial
10. Which of these statements apply to all the intracerebellar nuclei?
A. They are located in the cerebellar cortex
B. They are composed of large bipolar neurons
C. Their axons pass through the three cerebellar peduncles
D. They indirectly influence ipsilateral motor neurons in spinal cord
11. Dr Potri was seated in her office when a 12-year old boy entered. She immediately
noticed that he walked with a wide base and tended to fall forward. When he was
seated, he had difficulty holding his head in an upright position. Based on these
manifestations, Dr Potri would most likely think that the boy has a lesion in the:
A. cerebellar vermis C. right cerebellar peduncle
B. right red nucleus D. right and left cerebellar hemispheres

Cerebrum
1. thalamus serves as a relay station to these sensory systems:
A. ht, hearing, taste, and smell
B. sight, hearing, and taste only
C. sight and hearing only
D. sight only
2. Which of these is true of the diencephalon?
A. Lateral surface is formed by the fornix
B. Medial surface is formed by the choroids plexus
C. inferior surface is formed by the thalamus
D. superior surface is formed by the roof of the third ventricle
3. A 34 year old patient complains that he is extremely thirsty most of the time, drinks
large amounts of water, and passes out excessively large amounts of urine. His blood
sugar level is normal. His physician, Dr Danica, suspects that he is suffering from
diabetes insipidus associated with absent or very low plasma levels of vasopressin,
which in turn may be due to a lesion in this hypothalamic nucleus.
A. Supraoptic C. anterior hypothalamic
B. paraventricular D. lateral hypothalamic
4. Which of these hypothalamic nuclei control the parasympathertic system?
A. Posterior and lateral nuclei C. paraventricular nucleus
B. preoptic and anterior nuclei D. supraoptic nucleus
5. This condition is characterized by opisthotonus, extended arms and legs with pointed
feet and curled toes.
A. decerebrate rigidity C. decerebellate rigidity
B. decorticate rigidity D. spinal transection
6. The limbic system is concerned with:
A. visual experience C. pain perception
B. olfactory function D. fear and anger reactions
7. The reticular formation extends from the:
A. medulla to the cerebrum C. spinal cord to cerebrum
B. spinal cord to midbrain D. medulla to midbrain

BM2 HEMOSTASIS – BM 2
18. Vitamin K is essential for liver synthesis of all factors in which of these groups?
a. II, VII, IX and X C. II, V, VIII and XI
b. I, II, VIII and X D. IX, XI, XII and XIII

SPECIAL EXAM
SECOND BIMONTHLY SY 2006-2007

NAME ________________________

1. Diagram the steps in secondary hemostasis. 5 pts


2. Compare the pressures in the left ventricle (LV), left atrium (LA), and aorta (Ao) during
the 8 phases of the cardiac cycle. Write the name of each phase and under it, arrange
the three vessels (LV, LA, Ao) from highest pressure to lowest pressure. 5 pts
3. Given a case of metabolic acidosis due to renal failure, discuss the body responses to
bring the acid-base status towards normal. Indicate the buffer system/s involved, the
compensatory mechanism, and the corrective mechanism. 5pts
4. In outline form write the steps involved in the digestion and absorption of triglycerides
from the time it is ingested until it enters the lacteals. 5pts

XAVIER UNIVERSITY – JOSE P RIZAL COLLEGE OF MEDICINE


Basic Biomedical Sciences
Second Bimonthly Examination
PHYSIOLOGY
NAME ____________________________ 8 CTOBER 2008 SCORE ____________
MULTIPLE CHOICE: write the letter of the BEST answer on the answer sheet. Use capital
letters in blue/black ink. NO EASURES/SUPERIMPOSITIONS ALLOWED!

51. The first step in heme biosynthesis is the condensation of succinyl-CoA and;
A. Serine C. proline
B. Glycine D. cysteine
52. Which of these interleukins influences activities of cells from the pluripotential stem cell
to the mature progeny of the myeloid cell line?
A. interleukin 1 C. interleukin 3
B. interleukin 2 D. interleukin 4
53. Which of the following events occur to effect primary hemostasis?
A. injured blood vessel dilate
B. platelets become flattened discs
C. von Willebrand factor (VWF) binds platelets to fibrinogen
D. fibrinogen forms a bridge between two opposing platelets
54. This substance is synthesized by endothelial cells and prevents activated platelets
from adhering to uninjured endothelium.
A. prostacyclin C. adenosine diphosphatase
B. antithrombin III D. thromboxane A2
55. Gay’s  laboratory  test  results  are  as  follows:  BT  =  normal;;  APTT  =  prolonged;;
PT = normal; TT = normal. Based on these results, she could be deficient in any one
or a combination of these factors:
A. V, VII, X C. VIII, IX, XI
B. II, X, V D. V, VIII, IX
56. Thrombin acts as an anticoagulant when it combines with;
A. Heparin C. thrombospondin
B. Thrombomodulin D. thrombosthenin
57. Impulse conduction is slowest in the:
A. atrial muscles C. Bundle of His
B. AV node D. right bundle branch
58. Both hyper- and hypokalemia may show which of the following effects?
A. more negative resting membrane potential
B. fast conduction of action potential
C. heart may stop in diastole
D. shortened repolarization
59. Which of these can give rise to a systolic murmur/
A. mitral stenosis C. tricuspid insufficiency
B. aortic insufficiency D. pulmonary insufficiency
R S was suspected of having right bundle branch block (RBBB), a condition wherein the
impulse cannot pass through the right bundle of the bundle of His. Assuming that RS has
RBBB, answer the next 2 questions (#10 and #11):

60. What heart sound/s may be split?


A. S 1 C. S 1 and S 2
B. S 2 D. no split sound is expected
61. Which of these ECG configurations is/are likely to be prolonged?
A. P wave C. PR interval
B. T wave D. QRS interval
62. The following events occur during the isovolumetric contraction phase of the cardiac
cycle.
A. opening of semilunar valves C. decrease in ventricular volume
B. rapid rise in ventricular pressure D. second heart sound is heard
63. The BP of Mr Cruz was taken from the brachial and popliteal arteries while he was
lying down. Assuming that Mr Cruz has a normal BP, it is expected that the BP taken
from the brachial artery will be _______ that taken from the popliteal artery.
A. higher than C. more or less the same as
B. lower than D. incomparable with
64. Which of the following is true of body water?
A. Total body water makes up about 80% of the body weight
B. most of the body water is found in the plasma
C. intracellular fluid includes fluid in all cells except that in blood cells
D. the percentage of body water in males is greater than in females
65. Most of the ECF is found in this compartment.
A. intravascular fluid compartment
B. interstitial fluid compartment
C. transcellular fluid compartment
D. lymph
66. Compared to interstitial fluid, plasma (intravascular fluid) has:
A. a higher pH C. more protein
B. more HCO3- D. more Na+
67. The volume of the extracellular fluid compartment is primarily dependent on its content
of:
A. Water C. chloride
B. Sodium D. protein
68. It was Rammadan and Jani being a Muslim, refused to drink water even though she
was already very thirsty. Which of these substances are reflexly increased at this
time to maintain her fluid and electrolyte balance?
A. Angiotensisn II and ADH
B. Angiotensin II and serotonin
C. atrial natriuretic factor and ADH
D. atrial natriuretic factor and angiotensin II
69. In preparation for an ultrasound examination of her abdomen, Vanni drank 1.5 liters of
distilled water. This is likely to result in:
A. An increase in ICF volume and osmolarity
B. An increase in ECF volume and osmolarity
C. A decrease in ECF osmolarity and a decrease in ICF osmolarity
D. A decrease in ECF osmolarity and an increase in ICF osmolarity
70. Infusion of one liter of 0.9% NaCl solution to Lina would most likely result in:
A. increased ECF volume without change in ECF osmolarity
B. increased ECF volume and decreased ECF osmolarity
C. increased ICF volume and decreased ICF osmolarity
D. increased ICF volume without change in ICF osmolarity
71. As regards buffer system, which of the following is true?
A. plasma bicarbonate buffer system (BBS) is the most important
B. inorganic phosphate is found only in the plasma
C. organic phosphates is found both in red cells and plasma
D. the nonbicarbonate buffer system (NBBS) is primarily due to plasma proteins
72. In which of the following conditions is the plasma [HCO3-] increased?
A. compensated respiratory alkalosis
B. compensated respiratory acidosis
C. uncompensated respiratory acidosis
D. uncompensated respiratory alkalosis
73. The following are the laboratory results of Ms Ace Demi who was suspected of having
diabetic acidosis. Which of these results will show that her acidosis is not
compensated?
A. pH = 7.32 C. PCO2 = 42 mmHg
B. [HCO3-] = 16.1 D. [BE] = (-) 8.1

74. As a result of pure buffer response to a primary gain in HCO3-, which of the following
parameters will be decreased at the new equilibrium?
A. pH C. [HBuf]
B. S.PCO2 D. [BB]
75. Body responses to the acid-base disturbance secondary to renal failure will result in:
A. decreased respiratory rate
B. acidification of urine
C. renal formation of new bicarbonate
D. buffering by the BBS and NBBS
76. Aside from dehydration, M S was suspected of suffering from an acid-base
disturbance secondary to severe diarrhea. Laboratory tests were ordered and these
are the results: pH = 7.301; [HCO3] = 10.1 mmol/L; [BE] = (-)16.5 mmol/L and
PCO2 = 22.52 mmHg. Based on her history and lab exams, she is likely to have:
A. partially compensated respiratory acidosis
B. uncompensated respiratory acidosis
C. partially compensated metabolic acidosis
D. uncompensated metabolic acidosis
77. Which of the following can give rise to respiratory alkalosis?
A. acute pulmonary edema
B. salicylate poisoning
C. acute pulmonary edema
D. paralysis of the diaphragm
78. R N, a first year medical student was shown this laboratory result of a friend who was
suspected of having an acid-base disturbance: pH = 7.474; [HCO3-] = 23.4 mmol; [BE]
= (-) 1.5 mmol/L; PCO2 = 28.67 mmHg. Based on these lab results, R N can say that
his friend is likely to have:
A. uncompensated respiratory alkalosis
B. partially compensated respiratory alkalosis
C. uncompensated metabolic alkalosis
D. partially compensated metabolic alkalosis
79. Which of these muscles are responsible for opening the jaw during mastication?
A. masseter and medial pterygoid C. digastric and medial pterygoid
B. masseter and lateral pterygoid D. digastric and lateral pterygoid
80. Which of these statements regarding the slow waves in the GIT is true?
A. They are actually action potentials.
B. They cause the entry of Ca++ into the muscle fiber.
C. Each slow wave is usually followed by a muscle contraction.
D. Slow wave frequency determines rhythm of the GI contractions
81. Which of the following is true of intestinal peristalsis?
A. Initiated by distention of the gut
B. can be enhanced by sympathetic stimulation
C. requires the presence of an intact submucosal plexus
D. direction of peristalsis is primarily toward the stomach
82. Which of these movements is the most essential in preventing food from entering the
trachea during swallowing?
A. palatopharyngeal folds are pulled medially
B. upward movement of the larynx
C. vocal cords are tightly approximated
D. epiglottis swings backward and downward
83. These enzymes are present in the brush border of enterocytes
A. Trypsin, chymotrypsin, carboxypolypeptidase, dipeptidase
B. Trypsin, chymotrypsin, and carboxypolypeptidase
C. trypsin and chymotrypsin
D. trypsin only

84. The final digestion of peptides into amino acids occur in the _____ of enterocytes
lining the small intestine.
A. brush border C. endoplasmic reticulum
B. cytosol D. golgi apparatus
85. Which of the following is true regarding carbohydrate digestion and absorption?
A. digestion starts in the stomach or in duodenum
B. galactose is the final digestion product of most carbohydrate food
C. Glucose enters and leaves epithelial cells via facilitated diffusion
D. Enzymes for disaccharide digestion are located in microvilli of
enterocytes
86. As regards fat digestion and absorption, which of these is true?
A. Almost all fat digestion occurs in the stomach
B. Bile salts and lecithin digest the fat globules into fatty acids
C. Bile salt micelles transport fat digestion products to brush border of
enterocytes
D. Digested fat products are formed into Chylomicrons that enter into portal blood
87. Which of the following can stimulate the pancreas to release large amounts of
bicarbonate-rich fluid?
A. histamine C. gastrin
B. cholecystokinin (CCK) D. secretin
88. Which of these substances is the most potent stimulus for gallbladder contraction?
A. Histamine C. gastrin
B. CCK D. secretin
89. Gastrin enhances gastric HCl secretion by: U
A. increasing gastric motility
B. directly stimulating the parietal cells
C. stimulating chief cells to secrete pepsinogen
D. stimulating enterochromaffin-like cells to secrete histamine
90. This is true of gastric secretion.
A. it is inhibited by secretin
B. cephalic phase accounts for about 70% of the total gastric secretion
C. gastric phase results from signals coming from the cerebral cortex
D. intestinal phase is due to vasovagal reflex from stomach to brain & back to
stomach
91. Ms Jaja came to your clinic to consult you about her baby. She said she is worried
because her baby defecates some minutes after each meal. You get a history and do
a physical examination and these were all normal. You then assure Ms Jaja that her
baby’s  frequent  defecation  is  just  due  to  a  strong:    P/U
A. ileogastric reflex C. gastrocolic reflex
B. gastroileal reflex D. enterogastric reflex
92. As regards salivary secretion, which of the following is true? R
A. Average daily secretion of saliva is about 2,000 liters per day
B. Salivation is primarily regulated by the sympathetic nervous system
C. of the salivary glands, the parotids secrete the largest amount of mucus
D. sodium concentration in saliva is lesser than that in plasma
93. Which of the following conditions enhance rate of gastric emptying? R
A. Increased food volume in the stomach
B. Highly acidic duodenal chyme
C. Hypertonic fluids in duodenum
D. Protein breakdown products in duodenum
94. Which of these motor activities is present in the esophagus down to the colon?
A. Peristalsis C. haustral shuttling
B. Segmentation D. mass movement
95. Which of the following will most likely lead to the defecation reflex?
A. rhythmic peristalsis C. haustral shuttling
B. mass movement D. migrating motor complex
96. Being a Muslim, Hannan observed Ramadan. She noticed that she would experience
hunger contractions any time during the day. She learned later that these contractions
are most associated with: U
A. Peristalsis C. retropulsion
B. Segmentation D. migrating motor complex
97. Mucus is secreted in this/these segment/s of the gastrointestinal tract.
A. esophagus, stomach, small intestine, large intestine
B. esophagus, stomach and small intestine
C. esophagus and stomach
D. esophagus only
98. This is true of pancreatic juice. R
A. acinar cells secrete bicarbonate solution
B. ducts secrete the digestive enzymes
C. vagal stimulation increases enzyme production
D. it is secreted into the jejunum together with bile from the liver
99. Which of the following contract/s during vomiting? R
A. Diaphragm C. pylorus and fundus
B. Pylorus and diaphragm D. cardia and fundus
100. Entry of this/these substance/s across the luminal surface of enterocytes is by
sodium cotransport mechanism.
A. glucose, galactose, peptides and amino acids
B. glucose, galactose and peptides
C. glucose and galactose
D. glucose only
101. Which of the following CD markers differentiates a mature thymocyte from an
intermediate thymocyte?
A. CD 1 C. CD4
B. CD 3 D. CD 8
102. Which of the following cytokines are involved in differentiation of leukocyte
precursors?
A. interferons C. colony stimulating factors
B. interleukins D. transforming growth factors
103. Which of the following is directly responsible for dephosphorylation of
phosphate from NF-AT during T cell activation?
A. tyrosine kinase Fyn C. calcineurin
B. tyrosine kinase Lck D. PI-3 kinase
104. Chemically, red cell agglutinogens of the ABO system are:
A. lipoproteins C. glycoproteins
B. lipopolysaccharides D. glycolipids
105. Which of the following is most likely to lead to a fatal hemolytic disease of the
newborn?
A. a type O mother bearing her first type A baby
B. a type O mother bearing her second type A baby
C. an Rh negative mother bearing her first Rh positive baby
D. an Rh negative mother bearing her second Rh negative baby
106. Which of the following techniques is most useful in identification of rouleaux
formation during the slide method of ABO grouping?
A. gentle agitation C. perform the procedure at 37o C
B. add bovine albumin D. perform the procedure at 4oC
107. The smaller the radius of a blood vessel, the lesser the viscosity. This is:
A. plasma skimming C. Fahreus-Lindqvist phenomenon
B. shear thinning D.  Poiseuille’s  law
108. Blood behaves as a thixotropic fluid in the:
A. aorta C. capillary
B. small artery D. vein
109. Which of the following is most likely to influence diastolic pressure?
A. cardiac output C. amount of blood in the arteries
B. viscosity of blood D. peripheral resistance
110. Which of the following relationships is most correct?
A. a capillary has a small radius, low pressure but high tension
B. a capillary has a small radius, high pressure but low tension
C. a capillary has a small radius, low pressure and low tension
D. a capillary has a large radius, low pressure and high tension
111. Which of the following is the most important capillary exchange mechanism?

A. filtration C. active transport


B. diffusion D. transcytosis
112. Increased flow to an area following removal of an occluding force is called:

A.  Cushing’s  reaction C. active hyperemia


B. vasodilator theory D. reactive hyperemia
113. Which of the following is an afferent fiber of the sneeze reflex?
A. trigeminal nerve C. glossopharyngeal nerve
B. abducens nerve D. vagus nerve
114. Which of the following is directly involved in airflow to the lungs?
A. an increase in intrathoracic negative pressure
B. a subatmospheric intrapulmonic pressure
C. an increase in transpulmonary pressure
D. an increase in transthoracic pressure
115. Which of the following muscles of respiration is capable of increasing both the
superoinferior and anteropostarior diameters of the thorax?
A. external intercostals C. diaphragm
B. internal intercostals D. serratus posterior superior
116. A medical student inhaled normally, then exhaled normally into the mouthpiece
of a spirometer. The air that would remain in his lungs would be the:
A. vital capacity C. expiratory reserve volume
B. residual volume D. functional residual capacity
117. Which of the following methods of determination of lung volumes can measure
trapped gas?
A. Benedict-Roth spirometry C. nitrogen washout technique
B. helium dilution method D. body plethysmography
118. Which of the following is mainly responsible for small alveoli to have lower
pressure and low surface tension?
A. law of Laplace C. compliance
B. surfactant D. elastance
119. Which of the following is closely associated with diabetic ketoacidosis?
A. polypnea C. Cheyne-Stokes respiration
B.  Biot’s  respiration D. Kussmaul respiration
120. Which of the following defines the relationship between gas volume and
absolute temperature?
A.  Avogadro’s  law C.  Charles’  law
B.  Boyle’s  law D.  Dalton’s  law
121. Which of the following gas laws allows continuous diffusion of oxygen from the
alveoli to the pulmonary capillaries by maintaining low oxygen pressure in the
capillaries?
A.  Dalton’s  law C. General gas law
B.  Graham’s  law D.  Henry’s  law
122. Movement of gases through the conducting airways is made possible via:
A. simple diffusion C. diffusion-limited movement
B. bulk flow D. perfusion-limited movement
123. Which of the following best explains why carbon dioxide diffuses at a much
faster rate than oxygen?
A.  Avogadro’s  law C.  Graham’s  law
B.  Dalton’s  law D.  Henry’s  law
124. Which of the following, when increased, will increase the rate of gas diffusion?

A. molecular weight C. diffusion distance


B. gas temperature D. liquid viscosity

125. Which of the following will most likely have lower carbon dioxide?
A. lung bases C. right lung
B. lung apices D. left lung
126. Which of the following is most likely to happen when an individual
hypoventilates?
A. less oxygen diffuses into the pulmonary capillary
B. less carbon dioxide diffuses into the pulmonary alveolus
C. more carbon dioxide diffuses into the pulmonary alveolus
D. respiratory alkalosis may occur
127. Which of the following relationships is most correct?
A. the upper lung has lesser ventilation and lesser blood flow
B. the upper lung has greater ventilation but lesser blood flow
C. the upper lung has greater ventilation and greater blood flow
D. the upper lung has lesser ventilation but greater blood flow
128. Which of the following best explains why the oxyhemoglobin dissociation curve
is S-shaped?
A. it is a plot of how oxygen affects interaction with hemoglobin
B. it is a plot of the relationship between oxygen and hemoglobin
C. it is a plot of four reactions of hemoglobin with oxygen
D. it is a plot of reactions of 1 subunit of hemoglobin and 4 units of oxygen
129. Which of the following, when increased, will shift the oxyhemoglobin
dissociation curve to the left?
A. hydrogen ion concentration C. carbon dioxide
B. 2,3 diphosphoglycerate D. carbon monoxide
130. Which of the following is most descriptive of the carbon dioxide dissociation
curve?
A. a high partial pressure of oxygen shifts the curve to the left
B. a high partial pressure of oxygen shifts the curve to the right
C. it allows more carbon dioxide loading at the tissues
D. it allows more carbon dioxide unloading in the tissues
131. Which of the following is the end result of the Haldane effect?
A. more oxygen is delivered to the tissues by blood
B. less oxygen is delivered to the tissues by blood
C. less carbon dioxide is unloaded in the tissues by blood
D. more carbon dioxide is loaded at the tissues by blood
132. Which of the following values is considered as normal?
A. P CO2 = 35 mm Hg; P O2 = 90 mm Hg; HCO3 = 25 mmol/liter
B. P CO2 = 30 mm Hg; P O2 = 83 mm Hg; HCO3 = 20 mmol/liter
C. P CO2 = 40 mm Hg; P O 2 = 100 mm Hg; HCO3 = 30 mmol/liter
D. P CO2 = 46 mm Hg; P O2 = 80 mm Hg; HCO3 = 20 mmol/liter
133. Which of the following is presently believed to be the origin of respiratory
rhythmicity?
A. Botzinger complex C. dorsal respiratory group
B. preBotzinger complex D. ventral respiratory group
134. Which of the following serves to drive the diaphragm upon stimulation?
A. pontine respiratory group C. dorsal respiratory group
B. apneustic center D. ventral respiratory group
135. Which of the following activates the expiratory muscles during forced
respiration?
A. pontine respiratory group C. dorsal respiratory group
B. apneustic center D. ventral respiratory group
136. Which of the following will result from stimulation of receptors in the upper
airways and tracheobronchial tree?
A. decreased heart rate C. increased blood pressure
B. bronchoconstriction D. tachypnea
137. Which of the following will result from pulmonary vascular congestion?

A. apnea C.tachypnea
B. hyperpnea D. hyperventilation
138. Which of the following is a natural consequence of lung inflation? A
A. bronchodilation C. hyperpnea
B. tachypnea D. slight vasodilation
139. Ammonia is secreted in the tubular lumen via:
A. simple diffusion C. active Tm-limited secretion
B. diffusion trapping D. active gradient-time limited secretion
140. In the majority of instances, active tubular reabsorption takes place at the:
A. luminal surface of the tubular cell
B. basolateral membrane of the tubular cell
C. interface of the interstitium and endothelial cell
D. luminal surface of the endothelial cell
141. Complete inhibition of active sodium reabsorption would cause an increase in
the excretion of:
A. water, urea and chloride C. water, chloride and glucose
B. urea, glucose and creatinine D. chloride, uric acid and water
142. Hydrogen is secreted in the tubular lumen via:
A. simple diffusion C. active Tm-limited secretion
B. diffusion trapping ` D. active gradient-time limited secretion
143. Hippurate and oxalate are both secreted via an active Tm-limited mechanism. If
both are present in blood and secretion of one is diminished by the presence of the
other, which of the following would be the best explanation?
A. Tm is an average value C. enzyme carriers are non-specific
B. enzymes are substrate dependent D. Tm of one is lower than the other
144. Which of the following would most likely be the result of complete inhibition of
chloride transport at the ascending loop of Henle?
A. chloride would diminish in the urine
B. inability to excrete a concentrated urine
C. net reabsorption of sodium at the ascending loop
D. net reabsorption of water at the descending loop
145. At what point along the nephron does the tubular fluid reach the lowest
osmolarity in anti-diuretic states?
A. end of proximal tubule C. tip of the loop of Henle
B. end of cortical collecting duct D. distal end of thick ascending limb
146. Which of the following is true as regards countercurrent multiplication of
osmolar concentration?
A. intratubular fluid becomes progressively dilute as it flows down the descending limb
B. there is a greater osmotic gradient from the top of the medulla to the bottom
C. the countercurrent multiplier system dilutes the interstitial fluid of the medulla
D. intratubular fluid becomes progressively concentrated at the ascending limb
147. Which of the following statements is true as regards countercurrent exchange?

A. blood entering the arteriolar end of the vasa recta has the greatest osmolarity
B. blood at the venular end of the vasa recta has the least osmolarity
C. the capillary loop does not create a medullary gradient but only protects it
D. movement into the loop is passive while movement out of the loop is active
148. Which of the following is a result of the action of ADH? `
A. decrease in intracellular formation of cAMP
B. insertion of preformed water channels into collecting duct luminal membrane
C. increase in water permeability of the thick ascending loop of Henle
D. decrease in the transport of sodium chloride along the collecting duct
149. A normal subject drinks 1 liter of pure water. Which of the following will happen
to ADH secretion?
A. stimulation of ADH secretion due to decreased ECF volume
B. stimulation of ADH secretion due to reflexes induced by decreased ECF volume
C. inhibition of ADH secretion due to reduced osmolarity of ECF
D. unpredictable because of decreased ECF volume and ECF osmolarity
100. Which of the following, when done in experimental animals, will result in micturition
only at high urine volume?
A. spinal cord transection C. supracollicular transection
B. intercllicular transection D. transhypothalamic transaction

101.The most common microorganism that can infect the urinary tract is:
A. Staphylococcus aureus C. Streptococcus
B. Klebsiella D. Escherichia coli
102 Males with urinary tract infection associated with prostatic hyperplasia are usually
given trimethoprim-sulfamethoxazole for a period of:
A. 3 days C. 7-14 days
B. 5 days D. 14-24 days
103. Women with acute uncomplicated pyelonephritis are usually given trimethoprim-
sulfamethoxazole for a period of:
A. 3 days C. 7 days
B. 5 days D. 14 days
104.A 55 year old male develops urinary incontinence following a vehicular accident.
Which of the following is most likely to be responsible for the incontinence?
A. urinary tract infection C. acute glomerulonephritis
B. spinal cord injury D. acute pyelonephritis
105.A 60 year old male is suspected of having benign prostatic hyperplasia. Which of the
following would be the most common effect on micturition?
A. overflow incontinence C. increase in voiding frequency
B. loss of voluntary micturition D. burning sensation on micturition
106.Which of the following is expected to occur upon ascent to an altitude of 10,000 feet
of an unacclimatized male?
A. decreased auditory acuity C. personality changes
B. reflex irritability D. impaired judgment
107.Which of the following acute altitude diseases would best benefit from a Gamow bag?
A. acute mountain sickness C. high altitude pulmonary edema
B. high altitude cerebral edema D. high altitude retinal hemorrhage
108.A popping sensation in the ears may occur during ascent or descent of a pressurized
aircraft because:
A. cabin pressure is at an altitude of 5,000-8,000 feet
B. the air in a pressurized aircraft cabin is extremely dry
C. there is mismatch between vestibular and visual cues
D. vestibular dysfunction can lead to auditory manifestations
109.The most serious hazard in long-term missions in aerospace is:
A. muscle atrophy C. space adaptation syndrome
B. decrease in RBC count D. decalcification of bone
110.While underwater, a deep sea diver was alarmed when his companion started to
behave irrationally. This is most probably due to:
A. nitrogen narcosis C. decompression sickness
B. oxygen toxicity D. high pressure nervous syndrome

Module 25 – Genetics – IO Number 3 (10 questions)

1. The fundamental unit of genetic information is:


A. DNA C. nucleotides
B. genes D. nucleosides
2. The genetic code contained in DNA is basically determined by the
____________ of nucleotides.
A. shape C. sequence
B. number D. all of the above
3. One of the following statements is NOT TRUE regarding RNA:
A. The sugar moiety in RNA is ribose.
B. Instead of thymine, RNA contains uracil.
C. Purine nucleotides does not necessarily equal the number of pyrimidine
nucleotides.
D. RNA exists as a double helix like DNA.
4. The messenger RNA serves as a messenger between these two
organelles of the cell:
A. cell membrane and nucleus C. nucleus and ribosomes
B. nucleus and cytosol D. cell membrane and mitochondria
5. The type of bonds which hold A to T and C to G in DNA is:
A. sulfide C. hydroxyl
B. hydrogen D. sulfhydryl
6.    This  substance  can  hydrolyze  RNA  to  2’,  3’  cyclic  diesters  of  its
mononucleotides:
A. alkali C. water
B. acid D. hydrogen
7. Transfer RNA contains approximately this number of nucleotides:
A. 20 C. 50
B. 35 D. 75
8. The primary function of transfer RNA is to translate the information
contained in this:
A. ribosomal RNA C. nucleus
B. DNA D. messenger RNA
9. The process of translation begins from this end of the m RNA:
A. the anticodon arm C.  the  5’  end
B.  the  D  arm                                                                                  D.  the  3’  end
10. Translation of the mRNA takes place in the _________:
A. ribosome C. nucleus
B. mitochondria D. cytosol

Module 25 – Genetics – IO Number 3 (10 questions)

1. The fundamental unit of genetic information is:


A. DNA C. nucleotides
B. genes D. nucleosides
2. The genetic code contained in DNA is basically determined by the
____________ of nucleotides.
A. shape C. sequence
B. number D. all of the above
3. One of the following statements is NOT TRUE regarding RNA:
A. The sugar moiety in RNA is ribose.
B. Instead of thymine, RNA contains uracil.
C. Purine nucleotides does not necessarily equal the number of pyrimidine
nucleotides.
D. RNA exists as a double helix like DNA.
4. The messenger RNA serves as a messenger between these two
organelles of the cell:
A. cell membrane and nucleus C. nucleus and ribosomes
B. nucleus and cytosol D. cell membrane and mitochondria
5. The type of bonds which hold A to T and C to G in DNA is:
A. sulfide C. hydroxyl
B. hydrogen D. sulfhydryl
6.    This  substance  can  hydrolyze  RNA  to  2’,  3’  cyclic  diesters  of  its
mononucleotides:
A. alkali C. water
B. acid D. hydrogen
7. Transfer RNA contains approximately this number of nucleotides:
A. 20 C. 50
B. 35 D. 75
8. The primary function of transfer RNA is to translate the information
contained in this:
A. ribosomal RNA C. nucleus
B. DNA D. messenger RNA
9. The process of translation begins from this end of the m RNA:
A.  the  anticodon  arm                                                        C.  the  5’  end
B. the D  arm                                                                                  D.  the  3’  end
10. Translation of the mRNA takes place in the _________:
A. ribosome C. nucleus
B. mitochondria D. cytosol
Module 1 Cell

1. Selective permeability of the cell membrane is made possible by the presence of: R
A. Carrier proteins C. voltage-gated channels
B. Aqueous channels D. glycoproteins
2. Without this property the cell membrane cannot be restimulated once it is excited. R
A. core conductor C. resistor
B. condenser D. rectifier
3. Which of the following is true of voltage-gated channels? R
A. made up of transmembrane proteins
B. allows the free entry of small water soluble molecules
C. change in membrane voltage always results in their opening
D. opening of these channels always leads to formation of action potentials
4. Which of these channels are open at all times? R
A. voltage-gated channels C. mechanically gated channels
B. lligand-gated channels D. aqueous channels
5. Gap junctions allow the direct transmission of electrical activity between adjacent cells
because they: U
A. are present in heart muscles
B. have voltage-gated channels
C. they actually fuse the surfaces of adjacent cells
D. have channels that directly link the cytoplasms of adjacent cells
6. The presence of tight junctions joining epithelial cells restrict the passage of
substances through the region of the tight junctions because: U
A. a dense material connects the adjacent cell membranes together
B. there is actual fusion of the adjacent cell membranes
C. numerous actin filaments connect the adjacent cell membranes
D. The junction encircles the cell
7. Two compartments (A and B) are separated by a semipermeable membrane. If 100
ml of 3% NaCl was poured into cpt A and 500 ml of the same solution was poured into
cpt B, which of the following would occur? P
A. net movement of water from cpt A to cpt B
B. net movement of water from cpt B to cpt A
C. no net movement of water across the membrane
D. no movement of water across the membrane
8. Substance A is small, nonpolar and moves across the cell membrane by simple
diffusion. Its rate of transport into the cell may be increased by increasing the: U
A. number of carrier proteins
B. negativity inside the cell
C. surface area of the cell membrane
D. concentration of substance A outside the cell
9. During a physiology experiment, the first year medical students exposed the red cells
to 1.5 % NaCl solution. Which of the following most likely happened to the red cells
after a few minutes of exposure? R
A. swollen C. hemolyzed
B. crenated D. no change in volume
10. Which of the following is true of the Na+/K+ pump? R
A. it is an extrinsic protein
B. maintains the integrity of the cell membrane
C. moves three Na+ into the cell and two K+ out of the cell
D. maintains the concentration gradients of Na+ and K+ across membrane
11. If the Na+/K+ pump is destroyed, which of these is most likely to occur? R
A. more K+ enters the cell
B. more Na+ leaves the cell
C. the inside of the cell becomes more negative
D. cell becomes swollen and may eventually burst
12. Substance Z is transported out of the cell without its having to pass through the
membrane components. This kind of transport is known as:
A. Phagocytosis C. pinocytosis
B. Exocytosis D. transcytosis
13. This process removes a portion of the cell membrane as it transports a substance
across the cell membrane.
A. Exocytosis C. carrier-mediated transport
B. endocytosis D. primary active transport
14. When the Na+/K+ pump is inhibited, which of these processes will also be inhibited?
A. Osmosis C. sodium-glucose cotransport
B. facilitated transport D. phagocytosis
15. Which of the following is a characteristic of facilitated transport but not of simple
diffusion? R
A. it requires the presence of ATP
B. it requires the presence of carriers
C. it transports substances into the cell
D. it is enhanced when concentration gradient is increased
16. Two compartments (A and B) are separated by a semipermeable membrane. If 100
ml of 1.5% NaCl was poured into cpt A and 500 ml of the same solution was poured
into cpt B, which of the following would occur? P
A. net movement of water from cpt A to cpt B
B. net movement of water from cpt B to cpt A
C. no net movement of water across the membrane
D. no movement of water across the membrane
17. Which of the following is a characteristic of primary but not of secondary active
transport? R
A. It is an active process
B. it directly makes use of ATP
C. it requires the presence of carriers
D. it moves substances into or out of the cell
1

1. The cell membrane can go back to its original state after it has been excited because
of this property:
A. acts as core conductor C. acts as a condenser
B. serves as a rectifier D. serves as a resistor
2. Which of the following is true of cell membrane proteins?
A. peripheral proteins can serve as carriers of substances to be transported
B. transmembrane proteins can serves as ion channels
C. intregral proteins may be removed without destroying the membrane
D. peripheral proteins are amphipathic
3. Which of these channels allow the entry of water-soluble molecules with diameter of
less than seven Angstroms?
A. aqueous channels C. ligand-gated channels
B. voltage-gated channels D. mechanically-gated channels
4. This type of membrane junction allows the passage of small molecules directly from
one cell to another via small channels connecting the cytoplasms of the two cells.
A. tight junction C. gap junction
B. zonula adherens D. desmosome
5. Which of the following is a characteristic of secondary but not of primary active
transport? R
A. It is an active process
B. it requires the presence of carriers
C. it moves substances into or out of the cell
D. an ion concentration gradient is its energy source
6. During a physiology experiment, the first year medical students exposed the red cells
to 0.9 % NaCl solution. Which of the following most likely happened to the red cells
after a few minutes of exposure? R
A. swollen C. hemolyzed
B. crenated D. no change in volume
7. Which of the following characteristics is true of both simple diffusion and facilitated
transport?
A. it requires the presence of ATP
B. it requires the presence of carriers
C. net transport of substances is against concentration gradient
D. it is enhanced when concentration gradient is increased
8. Which of the following is true of the Na+/K+ pump?
A. moves 3 K+ into the cell C. it is a form of cotransport
B. moves 3 Na+ out of the cel l D. directly requires ATP
9. This process removes a portion of the cell membrane as it transports a substance
across the cell membrane.
A. Exocytosis C. carrier-mediated transport
B. endocytosis D. primary active transport
10. When the Na+/K+ pump is inhibited, which of these processes will also be inhibited?
A. Osmosis C. sodium-glucose cotransport
B. facilitated transport D. phagocytosis
11. When ATP is absent, this transport process is inhibited.
A. Osmosis C. endocytosis
B. facilitated diffusion D. simple diffusion
12. Which of these processes transports solutes across the cell membrane?
A. Osmosis C. filtration
B. solvent drag D. bulk flow
13. Two compartments (A and B) are separated by a semipermeable membrane. If 100
ml of 1.5% NaCl was poured into cpt A and 500 ml of the same solution was poured
into cpt B, which of the following would occur? P
A. net movement of water from cpt A to cpt B
B. net movement of water from cpt B to cpt A
C. no net movement of water across the membrane
D. no movement of water across the membrane
14. Concentration of solute J is greater inside the cell than outside the cell.

ESSAY

1. Absence of ATP leads to inactivation of the sodium-glucose cotransport. Trace the


sequence of events that lead to such an effect. 1.5 pts

2. In one phrase/sentence, state why tight junction restricts the passage of substances
through the region of the tight junction. 0.5 pt
Quiz on Genetics

I.O.#4

1. The initial step involved in DNA replication:


A. formation of the replication fork
B. reconstitution of chromatin structure
C. identification of the origins of replication*
D. initiation of DNA synthesis
2. The term which refers to the site of unwinding where DNA synthesis
starts:
A. replication fork* C. Okazaki fragment
B. lagging strand D. origin recognition complex
3. The principal DNA polymerase enzyme responsible for DNA elongation:
A. I B. II C. III* D. beta
4. The enzyme responsible for unwinding of duplex DNA segments:
A. DNA topoisomerase C. ligase
B. helicase* D. polymerase gamma
5. Term which refers to a single-base mutation characterized by changes
From purine to either of the two pyrimidines:
A. transversion* C. missense mutation
B. transition D. nonsense codon
6. Replication of the DNA genome occurs at what specified time during the
life span of the cell:
A. S phase* C. M phase
B. G1 phase D. G2 phase
7. The type of DNA damage which results from ionizing radiation:
A. single-base alteration C. chain breaks*
B. two-base alteration D. cross-linkage
8. A characteristic of DNA replication:
A. unidirectional
B. proceeds from well-defined multiple origins in each chromosome
C. both DNA strands are replicated in succession
D. there must be a separation of the two DNA strands*
9. DNA replication process generates on completion what structure?
A. leading DNA strand C. retrograde DNA strand
B. replication bubbles* D. Okazaki fragments
10. The enzyme which initiates the synthesis of RNA primers:
A. DNA polymerase C. DNA primase*
B. helicase D. topoisomerase
11. A mismatch DNA repair is primarily due to:
A. chemotherapy
B. chemical damage to a single base
C. copying errors*
D. radiation damage to a DNA segment

ENDOCRINE BM3 08

1. This refers to the relatively stable physical and chemical composition of the internal
environment of the body resulting from the activities of regulatory systems.
A. Hemostasis C. homeotherm
B. Homeostasis D. homomorphy
This refers to an adaptive change in the functioning of an existing physiological sys
ENDOCRINE QUIZ
1. Which of the following is true of antidiuretic hormone (ADH)?
A. stored & released by same hypothalamic neurons that synthesized them
B. Travel down the nerve axons and released bound to neurophysin II
C. The released oxytocin enters the capillary plexus of superior hypophyseal artery
D. Are released into the median eminence
2. Which of these is true of the hypothalamic-releasing and –inhibiting hormones?
A. Synthesized by magnocellular neurons in the anterior hypothalamus
B. Travel down the nerve axons and discharged into the median eminence
C. stored within granules in terminal swellings called Herring bodies
D. pass through three capillary plexuses before reaching the target cells
3. Which of the following events immediately follows after the hypothalamic-releasing and
–inhibiting hormones are released from the axons of the parvicellular neurons into the
median eminence?
A. Hormones are stored within granules in the median eminence
B. Hormones enter the specific target cells in the adenohypohysis
C. Hormones enter capillary plexus of the superior hypohyseal artery
D. Hormones are transported down the long portal veins
4. Binding of thyroid stimulating hormone (TSH) with specific TSH receptors causes the
production of this substance as the second messenger.
A. Adenylyl cyclase C. protein kinase
B. cAMP D. calcium
5. Decreased plasma levels of cortisol enhances the synthesis of adrenocorticotropic
hormone (ACTH). This is an example of:
A. positive feedback C. short-loop negative feedback
B. long-loop negative feedback D. ultra short-loop negative feedback
6. Which of these hormones exert ultra short-loop negative feedback on
thyrotropin releasing hormone (TRH) synthesis?
A. Thyroxine C. TRH
B. Thyroid stimulating hormone (TSH) D. TRH and TSH
7. Every night Mr Mutley sleeps at 10:00 pm and wakes up at 6:00 am the following day.
Based  on  these  limited  data,  Mr  Mutley’s  plasma  level  of  adrenocorticotropic  hormone  
(ACTH) is expected to peak at about this time of the day.
A. 9:00pm C. 4:00am
B. 11:00pm D. 8:00am
8. Binding of Gonadotropin releasing hormone (GnRH) to specific receptors on
gonadotrophs results in formation of these two principal second messengers.
A. Calcium-calmodulin & phosphatidylinositol products
B. cAMP & adenylyl cyclase
C. cAMP & calcium-calmodulin
D. cAMP & phosphatidylinositol products
9. The first step in the formation of thyroid hormones is:
A. Movement of iodide from basal to apical membrane
B. Oxidation of iodide to iodine
C. Iodination of thyroglobulin
D. Iodide trapping
10. After iodide trapping, which of these steps in thyroid hormone synthesis immediately
follows?
A. Iodide moves from basal to apical surface of epithelial cells
B. Iodide is transported into lumen of follicle by pendrin transporter
C. Iodide is oxidized to iodine
D. Thyroglobulin is iodinated
11. The thyroid hormone that is most biologically active is:
A. Thyroxine (T4) C. reverse T3
B. Triiodothyronine (T3) D. diiodotyrosine (DIT)
12. As regards thyroid hormones, which of these is true?
A. T4 is the major secretory product of the thyroid gland
B. T3 and T4 are stored inside epithelial cells of follicles
C. Most of the T4 and T3 in the plasma are bound to albumin
D. T3 is converted to T4 before it binds to intracellular thyroid receptors
13. Thyroid hormones cause the increase of which of the following:
A. Heart rate, respiratory rate, glycolysis, & body weight
B. Heart rate, respiratory rate, & glycoslysis
C. Heart rate & respiratory rate
D. Heart rate
14. The main regulator of parathyroid gland activity is the plasma level of:
A. Parathyroid hormone (PTH) C. calcium
B. Calcitonin D. calcitriol
15. Increased PTH results in decreased:
A. Bone resorption
B. renal reabsorption of calcium
C. renal synthesis of 1,25-(OH)2-D
D. renal reabsorption of phosphate
16. As regards the hormone cortisol, which of the following sentences is correct?
A. It is synthesized primarily in the zona reticularis.
B. Most circulating cortisol is bound to albumin.
C. The final step in cortisol synthesis occurs in the endoplasmic reticulum.
D. It is released into the plasma as soon as it is formed.
17. Cortisol enhances which of the following?
A. insulin action on glucose metabolism C. glomerular filtration rate
B. inflammatory response to injury D. bone formation
18. Aldosterone secretion is increased in response to decreased plasma:
A. Volume, sodium, potassium, & ACTH
B. Volume, sodium, & potassium
C. Volume & sodium
D. Volume
19. Epinephrine and norepinephrine cause an increase in the following:
A. Glycogenolysis, gluconeogenesis, lipolysis, & glucose utilization
B. Glycogenolysis, gluconeogenesis, & lipolysis
C. Glycogenolysis & gluconeogenesis
D. Glycogenolysis
20. The aldosterone-indiuced increase in blood pressure is
21. The most important stimulant for insulin secretion is:
A. Glucose C. gastric inhibitory polypeptide
B. Fatty acids D. amino acids
22. The first rate-limiting step in insulin-action is:
A. The transport of insulin through the capillary wall
B. The flow of insulin in the circulating blood
C. Binding insulin with the plasma membrane receptor
D. Generation of second messenger
23. Insulin decreases plasma levels of:
A. Glucose, free fatty acids, glycerol, & amino acids
B. Glucose, free fatty acids, & glycerol
C. G;ucose & free fatty acids
D. Glucose

2. tem induced by prolonged exposure to an environmental stress without alteration in


the  person’s  genetic  endowment.
A. modulation C. accommodation
B. conformation D. acclimatization
3. Substances secreted by cells into the ECF and affecting neighboring cells of a
different type are called;
A. Paracrines C. cytokines
B. Autocrines D. endocrine hormones
4. The first step in the action of any intercellular chemical messenger is to:
E. alter the membrane permeability of the target cell
F. bind to its specific receptor
G. signal the activation of cAMP
H. trigger the activation of protein kinase
5. As regards upregulation of messenger receptors, which of these is true?
A. takes place in the presence of a high concentration of messenger substance
B. makes  the  target  cell  more  sensitive  to  the  messenger’s  effect
C. involves the sequestration of the receptor molecules to the inside of the cell
D. is brought about by inactivation of receptor molecules.
6. Norepinephrine and acetylcholine bind with:
A. G-protein linked hormone receptors
B. Enzyme linked hormone receptors
C. Ion-channel linked hormone receptors
D. Nucleotide linked hormone receptors
7. This is one of the general characteristic of a receptor.
A. acts as a signal C. non-specifically binds to a ligand
B. may be a protein or a lipoprotein D. is almost always an ion channel
8. “Down  regulation”  refers  to  the  decrease  in  the:
A. synthesis of hormones by the endocrine cells
B. effects of the hormones on the target tissues
C. sensitivity of the endocrine organ to stimulation
D. number of receptors associated with increased hormone level
9. This is one of the general characteristics of hormones.
A. act as first messengers
B. Circulate in the lymphatic system
C. Circulate at very high concentrations
D. binds directly to DNA to stimulate transcription
10. This is one of the general characteristic of a receptor.
A. acts as a signal C. non-specifically binds to a ligand
B. may be a protein or a lipoprotein D. is almost always an ion channel
11. Which of these hormones exert a negative feedback on the synthesis of corticotrophin-
releasing hormone?
A. Insulin C. cortisol
B. Aldosterone D. epinephrine
12. This substance is synthesized by neuroendocrine cells in the hypothalamus but
released by the posterior pituitary.
A. Vasopressin C. growth hormone
B. dopamine D. thyrotropin-releasing hormone
13. This group of hormones are secreted as soon as they are formed.
A. Steroids C. thyroid hormones
B. Proteins D. catecholamines
14. A decrease in thyroid stimulating hormone stimulates the hypothalamus to secrete
thyrotropin-releasing hormone. This is an example of;
A. positive feedback C. short loop negative feedback
B. feedforward regulation D. long loop negative feedback
15. Estrogen stimulates the anterior pituitary to secrete luteinizing hormone (LH), which in
turn stimulates the production of more estrogen by the ovaries and which in turn
causes more secretion of LH. This is an example of:
A. positive feedback C. short loop negative feedback
B. feedforward regulation D. long loop negative feedback
16. Which of these is true of hormonal control mechanisms?
A. make use of hormones as second messengers
B. are directly stimulated by changes in external environment
C. make use of blood, lymph and axons to carry their messages
D. are related to and functionally interdependent with the nervous system
17. A hormone binds to a cell membrane receptor allowing the coupling of the receptor to
a Gs protein. Which of the following immediately follows?
A. simulation of adenylyl cyclase
B. conversion of ATP to cAMP
C. activation of protein kinase
D. activation of a cascade of enzymes
18. Steroid hormones bind with receptor proteins located:
A. on the cell membrane surface C. in the cell nucleus
B. in the cytosol D. on the nuclear envelope
19. Binding of this hormone to its receptor in the cell nucleus can cause transcription by
specific genes in the nucleus.
A. Vasopressin C. angiotensin II
B. Thyroxine D. thyroid-releasing hormone
20. Which of these second messengers causes the release of calcium from the
endoplasmic reticulum?
A. calcium C. diacylglycerol
B. cyclic AMP D. inositol triphosphate
21. Milk let-down is a function of this hormone.
A. Vasopressin C. follicle stimulating hormone
B. Oxytocin D. prolactin

ENDOCRINE QUIZ
24. Which of the following is true of Oxytocin?
E. stored & released by same hypothalamic neurons that synthesized them
F. Travel down the nerve axons and released bound to neurophysin I
G. The released oxytocin enters the capillary plexus of superior hypophyseal artery
H. Are released into the median eminence
25. Which of these is true of the hypothalamic-releasing and –inhibiting hormones?
E. Synthesized by magnocellular neurons in the anterior hypothalamus
F. Travel down the nerve axons and discharged into the median eminence
G. stored within granules in terminal swellings called Herring bodies
H. pass through three capillary plexuses before reaching the target cells
26. Which of the following events immediately follows after the hypothalamic-releasing and
–inhibiting hormones are released from the axons of the parvicellular neurons into the
median eminence?
E. Hormones are stored within granules in the median eminence
F. Hormones enter the specific target cells in the adenohypohysis
G. Hormones enter capillary plexus of the superior hypohyseal artery
H. Hormones are transported down the long portal veins

ENDOCRINE QUESTIONS IO5

1. One of the following is a general characteristic of a receptor:


a. acts as a signal
b. may be a protein or a lipoprotein
c. non-specifically binds to a ligand
d. is almost always an ion channel
ANS: B

2. Receptors linked to G proteins are characterized by:


a. They are the largest family of ligands
b. They couple cell-surface receptors to downstream effectors
c. Have intracellular N terminus
d. Hydrophilic domain at the N terminus
ANS: B (extracellular N terminus & hydrophilic C terminus)

3. Paracrine hormones are:


a. Released by cells and act on the same cells that released them
b. Released by cells and act on neighboring cells
c. Released by organs involved in excitation of distant organs
d. Released by nerves
ANS: B

4. For paracrine hormonal signals to be delivered to their target organs, their diffusion must be
limited. This is achieved through any of the following EXCEPT:
a. Rapid exocytosis of the chemical signal
b. Destruction by cellular enzymes
c. Immobilization by the extracellular matrix
d. Exemplified by events at the neuromuscular junction
ANS: A (ENDOCYTOSIS)

5. Recognition of a signal by its receptor involves the same noncovalent interactions that
characterize substrate-enzyme interactions. They include:
a. Ionic bonds between groups of the same charge
b. Electrostatic interaction like the van der waals interaction
c. Hydrophobic interactions between polar groups
d. Disulfide interactions
ANS: B (ionic bonds bet opposite charges; hydrophobic bonds bet nonpolar groups; no
disulfide interactions)

6. With regards to second messengers, the following are true EXCEPT:


a. Rapidly produced by the cell
b. Broken down by apoptosis
c. Provides opportunities to amplify a signal
d. Exemplified by cyclic AMP
ANS: B

7. Ligand-gated ion channels are examples of plasma membrane receptors that


a. bind with lipid-soluble messengers
b. are integral membrane proteins
c. opens & closes as controlled by the receptor itself
d. act via metabolic pathways, hence, are called ionotropic receptors
ANS: B
8. General characteristics of hormones include the following EXCEPT:
a. They are chemicals secreted from endocrine tissues
b. Located on the surface, in the cytosol or in the nucleus of specific cells
c. Carried through the blood
d. Circulate at very high concentrations
ANS: D (circulate at very low concentrations 10(-9) to 10(-12) M)

9. Physiologic regulation of receptors include the following EXCEPT:


a. decrease the affinity of the receptors to their messengers
b. down regulation
c. exocytosis of receptors
d. endocytosis of ligands
ANS: D

Exam on endocrine glands: 11/29/07

_____1. The most rostral region of the hypothalamus:


C. Anterior hypothalamus
D. Preoptic
E. Tuberal (B)
F. Mammillary
_____2. Which of the following is not true of the Pituitary gland?
E. Its blood supply is derived from the external carotid artery
F. Consists of two glands
G. Lies in the sella turcica of the sphenoid bone (A)
H. Weighs about 0.5 g.
_____3. A muscular band that connects the pyramidal lobe of the thyroid gland
to the hyoid bone:
E. Prevertebral layer of deep fascia
F. Isthmus (D)
G. Pretracheal layer of deep fascia
H. Levator glandulae thyroidea
_____4. Which of the following thyroid veins is a tributary of the brachiocephalic vein?
C. Highest
D. Superior (D)
E. Middle
F. Inferior
_____5. Which of the following is not true of the pancreas?
A. It is both an exocrine and endocrine glands
B. An intraperitoneal organ (B)
C. Its main ducts open into the 2nd part of the duodenum
D. Suppled by the splenic artery
_____6. Which of the following describe the right adrenal gland?
E. Crescentic in shape
F. Lies behind the pancreas (D)
G. Supplied by a single artery
H. A retroperitoneal organ
_____7. Which of the following is true of the parathyroid glands?
E. Are intimately related to the posterior border of the thyroid glands
F. Usually two in number ( A)
G. Measure about 6 cm long
H. The superior gland lie close to the superior poles of the thyroid gland
_____8. Which of the following structures is medially related to the thyroid gland?
E. Recurrent laryngeal nerve
F. Common carotid artery
G. Vagus nerve (A)
H. Internal jugular vein

_____9. Endocrine gland that secretes growth hormone:


C. Adenohypophysis
D. Hypothalamus
E. Posterior pituitary (A)
F. Thyroid
_____10. Organ that secretes erythropoieten:
C. Liver
D. Heart
E. Kidney (C)
F. Spleen

POTENTIALS

1. Which of the following is true of acetylcholine?


A. it is synthesized in the body of the motor neuron
B. its choline component is taken up by the neuron from the ECF
C. it is released from the axon terminal as soon as it is formed
D. it is degraded by acetylcholinesterase located in the presynaptic membrane
2. The small depolarization at the motor end plate resulting from spontaneous quantal
release of acetylcholine from a motor neuron is called:
A. An excitatory postsynaptic potential (EPSP)
B. An inhibitory postsynaptic potential (IPSP)
C. Miniature end plate potential (MEPP)
D. End plate potential (EPP)
3. To be able to produce an action potential in a skeletal muscle, the EPP must be a
depolarization of the motor end plate of ______ mV.
A. 15 C. 40
B. 25 D. 50
4. Which of the following is true of EPP?
A. It is a graded response
B. It occurs spontaneously
C. it is conducted along the entire muscle membrane
D. it must be summated to produce an action potential
5. After acetycholine binds to its receptor in the postjunctional membrane, which of these
events immediately follows?
A. Degradation of acetylcholine
B. Generation of an action potential
C. Depolarization of the postjunctional membrane
D. opening of ligand-gated Na+ and K+ channels
6. Which of the following can initiate an action potential at the muscle membrane
adjacent to the end plate?
A. MEPP C. EPSP
B. EPP D. IPSP

ESSAY

1. In one or two sentences, explain briefly why an action potential cannot occur if the n
gates would open as fast as the m gates would open. 2 pts
EXTRA QUESTIONS

Genetics:

1. The immediate precursor of deoxyguanosine monophosphate is:


A. GMP C. GTP
B. GDP D. IDP
2. Atom number 6 of purine ring is derived from:

C. copying errors
11. The term which refers to the site of unwinding where DNA synthesis
starts:
A. replication fork C. Okazaki fragment
B. lagging strand D. origin recognition complex
12.The activity of DNA dependent RNA polymerase is mainly influenced by:
F. linking with the promoter on the template stand
G. catalazing the coupling of first bases
H. transcribing down into transcription unit
I. release of the RNA from the template
13.Splicing of exons sequences occurs in this part of the cell:
A. Mitochondria C. Cytoplasm
B. Nucleus D. Golgi complex
14. In translation, which of these steps/stages follows elongation?
A. activation of amino acid C. folding and processing
B. termination and release D. initiation of the polypeptide chain
15. Which of these amino acids represents the start of translation?
A. Glycine C. methionine
B. Tyrosine D. serine

Endocrine

2. Which of the following associations between hormone and location of its primary
receptor is correct?
A. thyroid hormone – cell nucleus
B. cortisol – cell membrane
C. epinephrine – cell nucleus
D. insulin – cell cytoplasm
3. Example of second intracellular messenger
A. guanosine triphosphate
B. cAMP dependent protein kinase
C. diacyglycerol
D. phospholipase C
4. Low plasma levels of thyroid hormone stimulates the anterior pituitary gland to secrete
more thyroid stimulating hormone. This is hormonal control is known as:
A. positive feedback C. short-loop negative feedback
B. ultrashort negative feedback D. long-loop negative feedback
3. One of the following is NOT a principle involved in homeostatic control
systems.
A. inputs = outputs = homeostasis
B. Regulated variables will have a range of normal values.
C. Normal levels may be re-set.
D. Every variable is always maintained constant.
4. The high hemoglobin levels of people who live in Baguio is an example of
A. Adaptation C. Biologic rhythm
B. Acclimatization D. Aging
5. When confronted by a stressor, a well-adjusted individual will most probably respond
by:
A. ignoring or avoiding the stressor
B. introducing humor into the situation

C. involving several coping strategies


D. developing a support system
6. Which of the following is not true of the pancreas?
E. It is both an exocrine and endocrine glands
F. An intraperitoneal organ
G. Its main ducts open into the 2nd part of the duodenum
H. Suppled by the splenic artery

Final Exams

Module 30

1. When an agonist muscle is stimulated, the action of the antagonist muscle is inhibited.
This is based on which of the following? C
A. Bell-Magendie law C. reciprocal innervation
B. autogenic inhibition D. central delay
2. The crossed extensor response is part of which of the following reflexes? B
A. stretch reflex C. inverse stretch reflex
B. withdrawal reflex D. monosynaptic reflex
3. Receptors found in the carotid sinus are examples of: D
A. nociceptors C. exteroceptors
B. proprioceptors D. interoceptors
4. Position sense is detected by:
A. mechanoreceptors C. interoceptors
B. proprioceptors D. nociceptors

Module 31

1. Muscle spindles serve as receptors of which of the following postural reflexes?


A. neck righting reflex * C. tonic neck reflex
B. body on head righting reflex D. negative supporting reaction
2. A biologic rhythm that occurs every week is called: C
A. circadian B. circatrigintan C. circaseptan D. ultradian
3. Which of the following components of the internal clock can bring about phase shifts?
D
A. suprachiasmatic nucleus C. retina
B. intergeniculate leaflet D. pineal gland
4. Which of the following hormones will have peak levels at night?
A. cortisol B. ACTH C. prolactin D. FSH
5. Which of the following waves in the ECG characterizes the awake state? B
A. alpha B. beta C. theta D. delta
6. The awake state is made possible by activity of which of the following? D
A. serotoninergic nuclei of pontine raphe
B. gigantocellular tegmental field
C. nucleus ceruleus
D. ascending reticuloactivating system
7. Irresistible daytime sleepiness associated with increased appetite for food and sex is
known as: C
A. narcolepsy C. Kleine-Levin syndrome
B. periodic hypersomnia D. neutral state syndrome
Module 33

1. Which of the following aspects of emotion involves mental processes? C


A. cognition B. affect C. conation D. physical change
2. Emotional responses are most likely to be exaggerated in which of the following
animals? B
A. spinal animal C. decerebrate animal
B. decorticate animal D. decerebellate animal
3. Antigravity reflexes are most likely to be exaggerated in which of the following
animals? C
A. spinal animal C. decerebrate animal
B. decorticate animal D. decerebellate animal

4. Initiative and memory is retained in which of the following animals? D


A. spinal animal C. decerebrate animal
B. decorticate animal D. decerebellate animal
5. Processing of language which determines whether a sound is a word or not is believed
to take place at the: B
A. inferior gyrus of frontal lobe C. middle temporal gyrus
B. left temporal lobe D. superior temporal gyrus
6. When a person makes use of a descriptive phrase to substitute for a word which he
cannot recall, he might have: C
A. verbal aphasia C. nominal aphasia
B. syntactical aphasia D. semantic aphasia
7. The end product of motivated behavior, whether satisfactory or not is: D
A. learning B. memory C. speech D. emotion

Module 34

1. The following basic principles of sensation are applicable to receptors, EXCEPT:


A. law of appropriate stimulus C. adaptation
B.  Mueller’s  doctrine    * D. contrast
2. The most important factor that influences reaction to pain is the: C
A. personality of the sufferer C. significance of the pain
B. age of the sufferer D. nature of the pain
3. In examination of a peripheral neuropathy, which of the following involves examination
of small unmyelinated fibers? D
A. light touch C. vibration sense
B. two point discrimination D. temperature perception

Module 35

1. Which of the following layers of the retina is most important in preventing degradation
of the formed image by absorbing any stray light and internal reflections? C
A. internal limiting membrane C. pigment epithelium
B. external limiting membrane D. photoreceptor layer
2. The following are true as regards rods and cones, EXCEPT: D
A. the fovea centralis does not contain blue cones
B. the peripheral retina contains rods exclusively
C. both rods and cones contain light-sensitive pigments
D. rods resolve fine image detail
3. The basic of color television is:
A. flicker fusion frequency C. monocular parallax
B. color fusion D. binocular parallax
4. An individual confuses brown and orange, but distinguishes purple and yellow. He is
most probably a: B
A. protanope B. deuteranope C. tritanope D. monochromat
5. Integration of disparate images of an object as seen by both eyes is called: D
A. flicker fusion frequency C. monocular parallax
B. color fusion D. binocular parallax

Module 36

1. According to this theory of aging, with advancing age, cells die and are not replaced:
B
A. biologic clock theory C. wear and tear theory
B. apoptosis theory D. rate of living theory

2. The following are changes associated with aging, EXCEPT: C


A. increase in exposure to carcinogens
B. increase in bronchial secretions
C. increase in parietal cell mass
D. decrease in muscle strength

Unusual Environments

1. The most common manifestation of type I decompression sickness is: B


A. skin rash C. hearing loss
B. limb pain D. chest pain
2. At an alveolar P O2 of 30 mm Hg, one is likely to experience: D
A. impaired judgment C. personality changes
B. mild tremors D. unconsciousness
3. Which of the following is considered as a life-threatening emergency? B
A. high altitude pulmonary edema
B. high altitude cerebral edema
C. high altitude retinal hemorrhage
D. acute mountain sickness
4. The following are expected to occur in aerospace, EXCEPT:
A. decalcification of bone C. headward shift of fluid
B. muscle atrophy D. increase in red cell mass

Exercise Physiology

1. An increase in the efficiency of the cardiovascular system and skeletal muscles as a


result of exercise is called: A
A. adaptation B. endurance C. conditioning D. fitness
2. Which of the following sports is most likely to involve the anaerobic system as its
principal energy source? B
A. 100 meter dash C. 200 meter swim
B. 100 meter swim D. 1 mile run
3. The following are specific benefits of exercise, EXCEPT: D
A. heart beat becomes stronger C. reduction of chronic fatigue
B. increased endurance D. shallower breathing
4. One of the following exercises is designed to improve circulation:
A. knee lift B. sprinter C. skipping rope D. push-up

Learning and Memory

1. Entrance of material into short term memory occurs in which of the following phases of
learning? B
A. apprehension phase C. recall phase
B. acquisition phase D. retention phase
2. The ability of an individual to discriminate facts is part of: A
A. apprehension phase C. recall phase
B. acquisition phase D. retention phase
3. Photographic memory is also known as:
A. short-term memory C. eidetic memory
B. long-term memory D. iconic memory
4. When an individual who recalls a past experience includes present data, the condition
is called: B
A. paramnesia C. anterograde amnesia
B. confabulation D. retrograde amnesia

Module 27

1. Which of the following embryonic structures will differentiate into the penis? B
A. urogenital sinus C. genital swelling
B. genital tubercle D. genital fold
2. Maturation of spermatids into spermatozoa takes place during which of the following
phases of spermatogenesis? D
A. spermeation C. spermatocytogenesis
B. meiosis D. spermiogenesis
3. During the ___ phase of spermiogenesis, the acrosomal vesicle flattens and spreads
over the anterior half of the condensing nucleus. B
A. Golgi phase C. acrosome phase
B. cap phase D. maturation phase
4. Semi-spastic contraction of facial muscles occurs during which of the following phases
of the male sexual response cycle? B
A. excitement phase C. orgasmic phase
B. plateau phase D. resolution phase
5. Which of the following hormones is mainly responsible for male-pattern balding?
A. testosterone C. estradiol
B. dihydrotestosterone * D. androstenedione
6. Hypogonadism associated with anosmia and unilateral renal agenesis is called:
A. mosaicism C.  Klinefelter’s  syndrome
B. XYY D. Kallmann syndrome

Module 28

1. Development of the breasts in females at age 8 in females is called: C


A. menarche B. pubarche C. thelarche D. puberty
2. Basal cells of the glands of the stratum basalis begin to proliferate during which of the
following phases of the endometrial cycle? B
A. pre-menstrual phase C. proliferative phase
B. menstrual phase D. secretory phase
3. Which of the following cells is responsible for conversion of androgen to estradiol and
secretion of progesterone? D
A. theca interna cells C. theca lutein cells
B. granulosa cells D. granulosa lutein cells
4. In a 28 day cycle, progressive increase in estrogen is seen during which of the
following phases? B
A. menstrual phase C. ovulation phase
B. follicular phase D. secretory phase
5. A change in color of the labia minora from bright red to burgundy occurs at which of
the following phases of the female sexual response cycle? B
A. excitement phase C. orgasmic phase
B. plateau phase D. resolution phase
6. Estrogen dominance may be characterized by the following, EXCEPT: A
A. hyperglycemia C. depression
B. water retention D. decreased sex drive

Module 29

1. Which of the following hormones is secreted as early as 9 days after fertilization?


A. human chorionic gonadotropin * C. estriol
B. human placental lactogen D. progesterone
2. Which of the following is a presumptive evidence of pregnancy?
A.  Goodell’s  sign C. mastodynia
B.  Hegar’s  sign D. ballottment
3. A 24 year old female consulted an obstetrician because of amenorrhea. Her last
normal menstrual period was Jan. 20, 2006. Assuming she has a normal pregnancy,
her expected date of confinement is:
A. Oct. 27, 2007 C. Sept. 27, 2007
B. Nov. 27, 2007 D. Dec. 27, 2007
4. In preparing mothers for breastfeeding, which of the following is recommended if the
nipples are flat or inverted? B
A. expose the nipples to air C. use well-fitted support bras
B. perform Hoffman maneuver D. rub nipples briskly with a towel
5. Which of the following methods of fertility control takes into consideration the character
of cervical mucus as indicative of the fertile period? C
A. post-coital douche C.  Billing’s  method
B. calendar rhythm D. temperature rhythm

XAVIER UNIVERSITY – JOSE P RIZAL COLLEGE OF MEDICINE


Basic Biomedical Sciences
Final Examination
PHYSIOLOGY
NAME ____________________________ March 2009 SCORE ____________
MULTIPLE CHOICE: write the letter of the BEST answer on the answer sheet. Use capital
letters in blue/black ink. NO EASURES/SUPERIMPOSITIONS ALLOWED!

51. These cells are normally found in circulating blood.


A. neutrophils, lymphocytes, monocytes, plasma cells
B. neutrophils, lymphocytes and monocytes
C. neutrophils and lymphocytes
D. neutrophils only
52. When this factor is deficient, both primary and secondary hemostasis are deficient:
A. fibrinogen C. thrombogen
B. prothrombin D. thrombospondin
53. If Sam has factor VII deficiency, which of the following tests would be prolonged?
A. Bleeding time (BT)
B. Activated Partial Thromboplastin time (APTT)
C. Prothrombin time (PT)
D. Thrombin time(TT)
54. The discharge rate of pacemaker cells may be increased by:
A. Decreasing the slope of diastolic depolarization
B. Decreasing threshold potential
C. Increasing initial resting potential
D. Increasing sympathelic stimulation
55. ardiac systole?
A.
56. Applying  Starling’s  law  of  the  heart,  increasing  the  _______  increases  the  force  of  
myocardial contraction, and thus the stroke volume.
A. End systolic volume C. preload stress
B. aortic blood pressure D. heart rate
57. Ms Mitra is suspected of having diseased papillary muscles of the left ventricle. If this
is true, which of the following would result?
A. systolic murmur C. decreased end diastolic volume of left ventricle
B. aortic regurgitation D. increased stroke volume of left ventricle
58. Blood flow to a particular organ increases when this is increased.
A. diameter of arterioles C. blood viscosity
B. number of open capillaries D. mean arterial pressure
59. Mean arterial pressure (MAP) tend to increase when this/these is/are increased.
A. heart rate, blood volume, hematocrit, and vasoconstriction
B. heart rate, blood volume and hematocrit
C. heart rate and blood volume
D. heart rate only
60. In the presence of right ventricular failure, which of the following will be decreased?
A. peripheral venous pressure C. venous return
B. central venous pressure D. lymph flow
61. The principal cause of the edema in varicose veins is the increase in:
A. mean arterial pressure C. central venous pressure
B. peripheral venous pressure D. capillary permeability
62. Entry of food into the stomach reflexly increases activity of the terminal ileum resulting
in ejection of chyme into the cecum. This is known as the:
A. ileogastric reflex C. gastrocolic reflex
B. gastroileal reflex D. enterogastric reflex
63. Which of these salivary glands secrete amylase?
A. Parotid C. sublingual
B. Submandibular D. small buccal glands
64. Which of these motor activities is present in the esophagus down to the colon?
A. Peristalsis C. haustral shuttling
B. Segmentation D. mass movement
65. Respiration is inhibited during this/these phase/s of swallowing. R
A. oral phase C. esophageal phase
B. pharyngeal phase D. pharyngeal and esophageal phases
66. Which of the following is the immediately preceding step to secretion of interleukin 1
by the antigen-presenting cell?
A. phagocytosis of foreign antigen by the macrophage
B. secretion of interleukin 2 by T helper cell
C. clonal prliferation of T helper cells
D. epitope is presented to T helper cell
67. Failure of the immune system to be activated as a result of exposure to too little
antigen is known as:
A. low zone tolerance C. prozone phenomenon
B. high zone tolerance D. postzone phenomenon
68. Blood vessels arranged in series will exhibit which of the following?
A. continuity principle C.  Poiseuille’s  law
B. Bernouilli principle D.  Laplace’s  law
69. Which of the following is the major factor that influences venous return flow?
A. mean circulatory filling pressure C. gravity
B. vessel compliance and distensibility D. venomotor tone
70. Which of the following muscles is involved only during forced expiration?
A. diaphragm C. external intercostals
B. levatores costarum D. internal intercostals
71. Which of the following is most likely to increase alveolar dead air space?
A. extension of the neck C. upright position
B. bronchodilators D. snorkel breathing
72. Which of the following may occur as a result of ascent to high altitudes?
A. apnea C. hyperventilation
B. hyperpnea D. dyspnea
73. Turbulent flow is most likely to occur in the:
A. bronchi C. bronchioles
B. alveoli D. alveolar ducts
74. Which of the following will most likely have lower carbon dioxide?
A. lung bases C. right lung
B. lung apices D. left lung
75. Which of the following best describes the carbon dioxide dissociation curve?
A. it is a hyperbolic curve showing the relation between carbon dioxide and
bicarbonate
B. it is an S-shaped curve showing the relation between carbon dioxide and oxygen
C. it is a linear curve that reflects reactions of carbon dioxide with blood buffers
D. it is a bell-shaped curve that reflects reactions of carbon dioxide with water
76. Substance X is present in the urine. Which of the following is the best explanation for
the presence of substance X in the urine? Substance X is:
A. filtered and reabsorbed C. filtered and secreted
B. filtered and excreted D. secreted and degraded
77. The first desire to void is experienced when bladder volume reaches:
A.100-150 ml C. 250-400 ml
B. 150-250 ml D. 400-600 ml
78. The greater bulk of sodium, chloride and water is reabsorbed at the:
A. proximal tubule C. loop of Henle
B. distal tubule D. collecting duct
79. Which of the following will most likely happen in the absence of ADH?
A. principal cells of the collecting duct become more permeable to water
B. absorption by the vasa recta ensures excretion of a hyperosmolar urine
C. water going out of the collecting duct is absorbed by the vasa recta
D. fluid in the collecting duct becomes dilute

Module 26 - #s 2, 3 & 4 ----- 7 Qs

1. One of the following statements is NOT true regarding homeostasis:


A. It refers to the dynamic steady state of the extracellular fluid.
B. It maintains physical and/or chemical parameters of the body relatively
constant.
C. Feedback is the only type of regulation which exists in the human
homeostatic system.
D. Homeostatic control minimizes changes in the internal environment when there
are changes in the external environment.
2. This type of regulation in homeostatic systems anticipates changes in the external
environment causing a faster response to the change:
A. negative feedback C. positive feedback
B. feedforward regulation D. anticipated feedback
3. One of the following is NOT a principle involved in homeostatic control
systems.
E. inputs = outputs = homeostasis
F. Regulated variables will have a range of normal values.
G. Normal levels may be re-set.
H. Every variable is always maintained relatively constant.
4. 4. An example of inputs and outputs maintaining the stability of an internal environment
is:
A. normal heart rate C. normal blood pressure
B. normal body temperature D. normal sleep patterns
5. This term refers to the rhythmic changes which occur in body functions.
A. biologic rhythm C. acclimatization
B. apoptosis D. none of the above
6. Acclimatization is most associated with one of the following:
A. adaptation C. biologic rhythm
B. apoptosis D. aging
7. This programmed cell response plays a major role in growth and
development of organisms by removing undesirable cells:
A. aging C. acclimatization
B. apoptosis D. biological rhythm

BB NERVOUS TISSUE QUIZ

1. The main determinant of the resting membrane potential is:


A. K+ efflux C. Na+ efflux
B. K+ influx D. Na+ influx
2. After an action potential, these gates should be open for the membrane to be excitable
again. U
A. m gates C. h gates
B. n gates D. m, n, and h gates
3. In an experiment, the Na+ and K+ channels of a motor neuron were opened at the
same time. Which of these most likely developed? U
A. an action potential C. a graded potential
B. a threshold potential D. an end plate potential
4. Which of the following is true of the nervous system? R
A. it is composed primarily of glial cells
B. the brain and spinal cord are surrounded by three meninges
C. the central nervous system consists of the brain and cranial nerves
D. the peripheral nervous system consists of the spinal cord and spinal nerves
5. Neurotransmitter action may be terminated by: R
A. activity of specific enzymes
B. binding with presynaptic receptors
C. uptake by the postsynaptic neuron
D. increasing the number of receptors in the postsynaptic neuron
6. Which of the following is true of the resting membrane potential? U
A. it is mainly due to sodium influx
B. it is maintained by the Na+/K+ pump
C. at RMP the Na+/K+ pump is inhibited
D. at RMP there is net movement of K+ out of the cell
7. Which of the following applies to both graded and action potentials? R
A. can be summated
B. can occur spontaneously
C. can be conducted by local current flow
D. can be a depolarization or a hyperpolarization
8. During the upstroke phase of an action potential, which of these gates are open? R
A. m and n gates C. n and h gates
B. m and h gates D. m, n, and h gates
9. The rectifier property of the cell membrane is primarily due to:
A. Na+ influx C. K+ influx
B. Na+ efflux D. K+ efflux
10. The cell membrane is absolutely refractory during which phase/s of the action
potential? R
A. upstroke only C. late downstroke
B. early downstroke D. upstroke and early downstroke
11. Which of these gates are open during the repolarization phase of the action potential?
U
A. m and h gates C. h and n gates
B. m and n gates D. m, n, and h gates
12. This is true of the relative refractory period. R
A. membrane is back to its resting potential
B. coincides with the entire downstroke phase
C. during this period most of the K+ channels are closed
D. membrane my be stimulated by stronger than normal stimulus
13. Which of the following is true of myelinated axons? R
A. action potential (AP) conduction is slower than in unmyelinated fibers
B. APs jump from one internode to the next internode
C. APs occur only at the nodes of Ranvier
D. AP conduction is decremental
14. Which of these neurons release epinephrine as their transmitters? R
A. preganglionic parasympathetic neurons
B. preganglionic sympathetic nerurons
C. postganglionic sympathetaic neurons
D. presynaptic motor neuron
15. Which of these events immediately precedes the production of inhibitory postsynaptic
potential (IPSP)? R
A. depolarization of the terminal knob of presynaptic neuron
B. binding of transmitters with receptors in postsynaptic neuron
C. diffusion of transmitter towards postsynaptic membrane
D. local K+ efflux or Cl- influx
16. Which of these potentials can be summated? R
A. action potential C. end plate potential
B. threshold potential D. excitatory postsynaptic potential
17. Which of these substances block the synthesis of neurotransmitters? R
A. botulinum toxin C. hemicholinium
B. organophosphate D. succinylcholine
18. To produce an IPSP, the release of neurotransmitters from the presynaptic neuron is
immediately followed by which of these events?
A.

19. Four action potentials reached the terminal knob of a presynaptic neuron at intervals of
10 secs. Which of the following is most likely to occur in the postsynaptic neuron? U
A. a summated EPSP C. an action potential
B. four separate EPSPs D. an IPSP
20. Which of these types of synaptic inhibition is mediated mainly by GABA? R
A. Presynaptic C. indirect
B. Postsynaptic D. direct
21. A motor unit is composed of a motor neuron and: R
A. all the muscle fibers it innervates
B. the motor end plate
C. all its terminal branches
D. its neurotransmitter
22. Which of the following can initiate an action potential in the muscle membrane? R
A. summated EPSPs
B. end plate potential
C. summated miniature end plate potentials
D. an action potential in the motor end plate
23. Which of the following electrolytes plays a major role in the release mechanisms of the
neurotransmitter? R
A. Sodium C. calcium
B. Potassium D. chloride

Essay
1. If the m, h, and n gates open at the same time, will an action potential occur?

1.1 Encircle your answer: 1 pt

YES / NO

1.2 Explain your answer in 1.1 above. 2 pts

BB NERVOUS TISSUE QUIZ

24. Reversal of the transmembrane potential is primarily due to:


A. Na+ influx C. K+ influx
B. Na+ efflux D. K+ efflux
25. When the cell membrane is at its resting potential, the membrane: R
A. is not excitable
B. is not using ATP
C. is positive inside and negative outside
D. allows passive fluxes of k+ and Na+
26. Which of the following is a characteristic of graded potential but not of an action
potential? R
A. Its amplitude is related to the magnitude of the stimulus
B. it is a change in the resting membrane potential
C. it is conducted along the cell membrane
D. it is initiated by a stimulus
27. A stimulus was applied to a nerve axon. A depolarization resulted but it was
conducted only a short distance. The most likely explanation is: U
A. the stimulus was suprathreshold
B. the axon was unmyelinated
C. the axon was at its refractory period
D. the depolarization did not reach threshold potential
28. The undershoot phase of an action potential is due to: R
A. slow closure of the K+ channels
B. inactivation of the Na+ channels
C. closure of the h gates
D. opening of the m gates
29. The absolute refractory period is primarily due to the closure of the: R
A. h gates C. n gates
B. m gates D. h, m, and n gates
30. Two nerve axons (A and B) were stimulated simultaneously by the same amount of
electrical current and action potentials were conducted in the two nerves. However,
the rate of conduction in A was faster than in B. Which of the following is the most
likely explanation? P or U
A. A is myelinated and B is not
B. A has a longer length than B
C. A has a smaller diameter than B
D. A has a higher threshold than B
31. If the membrane potential is equal to the K+ equilibrium potential, this means that: U
A. the membrane is permeable to K+ only
B. concentration gradient for K+ is lost
C. there is no net movement of K+ across the membrane
D. electrical force acting on K+ is greater than the concentration force
32. A synapse is best defined as:
A. the point of fusion between two neurons
B. a specialized junction between two neurons
C. the point of direct transmission of action potentials between two neurons
D. junction between two neurons whereby channels connect the two neurons
33. A presynaptic neuron was stimulated by an adequate stimulus, but the EPSP
produced at the postsynaptic membrane was of lesser amplitude than expected.
Which of the following is the most logical conclusion? U
A. transmitter release is less than usual
B. postsynaptic membrane was desensitized
C. a neuromodulator may have been released also
D. the postsynaptic neuron is at its absolute refractory period

34. Which of the following occurs to inhibit the postsynaptic membrane?


A. Opening of Na+ channels
B. Decreased conductance of Cl- ions out of the neuron
C. increased conductance of K+ out of the neuron
D. Decreased number of inhibitory membrane receptors
35. To produce an excitatory post synaptic potential, which of these steps immediately
follows the depolarization of the terminal knob?
A. Ca ++ bind with synaptic vesicles
B. Release of transmitter into the synaptic cleft
C. Diffusion of Ca++ into the terminal knob
D. Diffusion of transmitter toward postsynaptic membrane
36. Summation  is  “temporal”  when  the  increase  in  the  magnitude of the EPSP is:
A. enough to reach threshold and an action potential develops
B. enough to result in facilitation of postsynaptic membrane
C. due to simultaneous arrival of two separate excitatory inputs
D. due to arrival of two or more excitatory inputs in rapid succession
37. Which of the following is true of electrical synapses? R
A. more commonly found in humans
B. signal transmission is bidirectional
C. synaptic cleft is wider than that found in chemical synapses
D. speed of transmission is slower than that in chemical synapses
38. Which of the following is true of excitatory postsynaptic potential (EPSP)? R
A. it is propagated decrementally
B. it usually lasts for about 15 seconds
C. it is a depolarizing graded potential
D. it can occur only at the body of postsynaptic neuron
39. Which of the following factors can cause a decrease in the rate of synaptic
transmission? R
A. Alkalosis C. coffee
B. Hypoxia D. tea
40. Which of these substances competes with the transmitters for receptor sites? R
A. botulinum toxin C. hemicholinium
B. organophosphate D. succinylcholine
41. Four action potentials reached the terminal knob of a presynaptic neuron at intervals of
10 secs. Which of the following is most likely to occur in the postsynaptic neuron? U
A. a summated EPSP C. an action potential
B. four separate EPSPs D. an IPSP
42. Presynaptic inhibition is mediated mainly by; R
A. Glycine C. aspartic acid
B. glutamic acid D. gamma-amino butyric acid
43. Acetylcholine may be released spontaneously from the motor neuron axon terminal in
small quantities, and this results in the production of an: R
A. EPSP C. end plate potential
B. IPSP D. miniature end plate potential
44. The potential that is produced at the muscle membrane which lies directly under the
axon terminal of a neuron is termed: R
A. action potential C. EPSP
B. end plate potential D. IPSP
45. Which of these events immediately follows after the release of acetylcholine (Ach) into
the neuromuscular junction? R
A. Ca++ diffuses into terminal knob
B. Ach binds with receptors at motor end plate
C. Ach diffuses across the synaptic cleft
D. Opening of voltage-gated sodium channels
46. Which of these is considered as a neuromodulator? R
A. Acetylcholine C. dopamine
B. Norepinephrine D. angiotensin II
47. Which of the following relates to the motor end plate? R
A. acetylcholine receptors in sarcolemma with junctional folds
B. synaptic cleft between axon terminal and sarcolemma
C. synaptic vesicles and mitochondria in axon terminal
D. myelinated expanded axon terminal

Essay

2. If the m, h, and n gates open at the same time, will an action potential occur?

1.3 Encircle your answer: 1 pt

YES / NO

1.4 Explain your answer in 1.1 above. 2 pts

Module 1 Cell

18. Selective permeability of the cell membrane is made possible by the presence of: R
A. Carrier proteins C. voltage-gated channels
B. Aqueous channels D. glycoproteins
19. Without this property the cell membrane cannot be restimulated once it is excited. R
A. core conductor C. resistor
B. condenser D. rectifier
20. Which of the following is true of voltage-gated channels? R
A. made up of transmembrane proteins
B. allows the free entry of small water soluble molecules
C. change in membrane voltage always results in their opening
D. opening of these channels always leads to formation of action potentials
21. Which of these channels are open at all times? R
A. voltage-gated channels C. mechanically gated channels
B. lligand-gated channels D. aqueous channels
22. Gap junctions allow the direct transmission of electrical activity between adjacent cells
because they: U
A. are present in heart muscles
B. have voltage-gated channels
C. they actually fuse the surfaces of adjacent cells
D. have channels that directly link the cytoplasms of adjacent cells
23. The presence of tight junctions joining epithelial cells restrict the passage of
substances through the region of the tight junctions because: U
A. a dense material connects the adjacent cell membranes together
B. there is actual fusion of the adjacent cell membranes
C. numerous actin filaments connect the adjacent cell membranes
D. The junction encircles the cell
24. Two compartments (A and B) are separated by a semipermeable membrane. If 100
ml of 3% NaCl was poured into cpt A and 500 ml of the same solution was poured into
cpt B, which of the following would occur? P
A. net movement of water from cpt A to cpt B
B. net movement of water from cpt B to cpt A
C. no net movement of water across the membrane
D. no movement of water across the membrane
25. Substance A is small, nonpolar and moves across the cell membrane by simple
diffusion. Its rate of transport into the cell may be increased by increasing the: U
A. number of carrier proteins
B. negativity inside the cell
C. surface area of the cell membrane
D. concentration of substance A outside the cell
26. During a physiology experiment, the first year medical students exposed the red cells
to 1.5 % NaCl solution. Which of the following most likely happened to the red cells
after a few minutes of exposure? R
A. swollen C. hemolyzed
B. crenated D. no change in volume
27. Which of the following is true of the Na+/K+ pump? R
A. it is an extrinsic protein
B. maintains the integrity of the cell membrane
C. moves three Na+ into the cell and two K+ out of the cell
D. maintains the concentration gradients of Na+ and K+ across membrane
28. If the Na+/K+ pump is destroyed, which of these is most likely to occur? R
A. more K+ enters the cell
B. more Na+ leaves the cell
C. the inside of the cell becomes more negative
D. cell becomes swollen and may eventually burst
29. Substance Z is transported out of the cell without its having to pass through the
membrane components. This kind of transport is known as:
A. Phagocytosis C. pinocytosis
B. Exocytosis D. transcytosis
30. This process removes a portion of the cell membrane as it transports a substance
across the cell membrane.
A. Exocytosis C. carrier-mediated transport
B. endocytosis D. primary active transport
31. Which of the following is a characteristic of facilitated transport but not of simple
diffusion? R
A. it requires the presence of ATP
B. it requires the presence of carriers
C. it transports substances into the cell
D. it is enhanced when concentration gradient is increased
32. Which of the following is a characteristic of primary but not of secondary active
transport? R
A. It is an active process
B. it directly makes use of ATP
C. it requires the presence of carriers
D. it moves substances into or out of the cell

ESSAY
1. If ATP is not made available, will the sodium-glucose cotransport still function?
1.1 Answer yes or no. Encircle your answer: 1 pt

YES / NO

1.2 Explain your answer in 1.1 above. 2 pts

XAVIER UNIVERSITY – DR JOSE P RIZAL SCHOOL OF MEDICINE


BASIC BIOMEDICAL SCIENCES
SECOND BIMONTHLY - LONG QUIZ 5

NAME: __________________________ September 12, 2011

1. Which of the following is mainly responsible for phagocytic defense along the major
bronchi?
A. macrophages C. lymph nodes
B. histiocytes D. lymphatic vessels
2. Which of the following mechanisms will come into play when a foreign agent
measuring 4 um enters the respiratory tract?
A. Removal by vibrissae
B. Removed by mucociliary escalator
C. Phagocytosis by alveolar macrophage
D. Suspended as aerosol and removed during exhalation
3. Which of the following substances is formed and released by pulmonary cells for local
use?
A. Angiotensin I B. Endoperoxide C. Surfactant D. Norepinephrine
4. Which of the following muscles functions only during forced inspiration?
A. diaphragm C. external intercostals
B. sternocleidomastoid D. internal intercostals
5. Which of the following muscles functions only during forced expiration?
A. Parasternal intercartilagenous C. rectus abdominis
B. Scalene D. levatores costarum
6. During quiet inspiration, the increase in thoracic volume is primarily due to an increase
in the:
A. superoinferior diameter C. transverse diameter
B. anteroposterior diameter D. anteroposterior and lateral diameters
7. Which of the following is directly responsible for entrance of air into the alveoli during
inspiration?
A. An increase in thoracic volume
B. An increase in alveolar transmural pressure
C. Intrathoracic pressure becomes more negative
D. A fall in alveolar pressure below atmospheric pressure
8. Which of the following immediately precedes expansion of pulmonary alveoli?
A. Increase in thoracic volume
B. Increase in intrathoracic negative pressure
C. Increase in alveolar transmural pressure gradient
D. Decrease in alveolar pressure
9. Which of the following immediately follows alveolar elastic recoil?
A. Intrathoracic pressure becomes less negative
B. Alveolar pressure becomes greater than atmospheric pressure
C. Alveolar transmural pressure gradient decreases
D. Air flows out of the alveoli
10. Which of the following is mainly responsible for air flow out of the lungs during quiet
expiration?
A. A decrease in thoracic volume
B. Elastic recoil of the alveoli
C. A decrease in intrathoracic pressure
D. Contraction of expiratory muscles
11. A subject inhaled maximally and then exhaled forcefully into the mouthpiece of a
spirometer. The resultant reading measures:
A. inspiratory reserve volume C. vital capacity
B. expiratory reserve volume D. functional residual capacity
12. Which of the following lung capacities is used as a measure of physical fitness?
A. vital capacity C. functional residual capacity
B. inspiratory capacity D. total lung capacity
13. A subject inhales quietly and then exhales exhales quietly into the mouthpiece of a wet
spirometer.. The amount of air that is exhaled is called:
A. inspiratory capacity C. tidal volume
B. expiratory reserve volume D. vital capacity
14. A subject inhaled quietly then exhaled maximally into the mouthpiece of a wet
spirometer. The amount of air that remains in his lungs is called:
A. expiratory reserve volume C. functional residual capacity
B. tidal volume D. residual volume
15. Which of the following methods of measurement of lung volumes and capacities is the
only method than can measure trapped gases?
A. wet spirometry C. helium dilution technique
B. body plethysmography D. nitrogen washout method
16. Stimulation of receptors in the upper airways and tracheobronchial tree can produce:
A. sneezing C. bronchoconstriction
B. rise in BP D. bradycardia
17. Which of the following is characterized by an increase in intrathoracic pressure above
atmospheric pressure?
A. quiet inspiration C. forced inspiration
B. quiet expiration D. forced expiration
18. Which of the following will most likely happen if gas exchange between the alveolus
and pulmonary capillary is impaired? An increase in:
A. anatomic dead air space C. compliance
B. physiologic dead air space D. elastance
19. Which of the following will increase anatomic dead air space?
A. increasing age C. flexion of the neck
B. bronchoconstriction D. snorkel breathing
20. Which of the following will decrease alveolar dead air space?
A. reduced blood flow to alveoli C. obstruction of pulmonary vessels
B. pulmonary embolism D. recumbent position
21. Which of the following is mainly responsible for adjusting surface tension in alveoli in
response to changes in lung volume?
A. compliance C. surfactant
B. elastance D. law of Laplace
22. Which of the following is considered as the most sensitive indicator of diffuse
obstructive bronchopulmonary disease?
A. maximal voluntary ventilation C. forced expiratory volume
B. forced expiratory vital capacity D. maximal breathing capacity
23. Which of the following formulae is correct?
A. Vital capacity = inspiratory reserve volume + expiratory reserve volume
B. Dead air space = resting tidal volume + residual volume
C. Inspiratory reserve volume = vital capacity – resting tidal volume
D. Functional residual capacity = expiratory reserve volume + residual volume

ESSAY

1. What procedure must a student follow if he wants to determine his functional residual
capacity? (Limit your answer to three short sentences.) 2 points

XAVIER UNIVERSITY – DR JOSE P RIZAL SCHOOL OF MEDICINE


BASIC BIOMEDICAL SCIENCES
SECOND BIMONTHLY - SHORT QUIZ 1

NAME: __________________________ September 15, 2011

Completion – Write the best answer.


_________________________ 1. The term given to rhythmic waxing and waning
of
respiratory efforts.
_________________________ 2. The gas law that describes the relationship between
gas
volume and absolute temperature.
_________________________ 3. The reason why the oxyhemoglobin dissociation curve
is
sigmoid shaped.
_________________________ 4. It is responsible for stabilizing the deoxy configuration
of
hemoglobin.
_________________________ 5. The chemical reaction of carbon dioxide with water and
with
blood buffers is responsible for this shape of the
carbon dioxide dissociation curve.
_________________________ 6. It results from inactivation of the inspiratory
cutoff
mechanism.
_________________________ 7. They serve as the principal initiators of phrenic
nerve
activity.
_________________________ 8. The reflex initiated by negative pressure in the
upper
airways.
_________________________ 9. The reflex initiated by immersion of the face in water.

_________________________ 10. The receptors stimulated by pulmonary embolism.

Modified true or false – Write TRUE if the statement is correct. If the statement is false,
write the word/s that would best make it true, in place of the underlined word/s. No erasures.

_________________________ 1. Normal exercise can give rise to hyperventilation.

_________________________ 2. Avogadro’s   law describes the relationship between


gas
volume and gas pressure.
_________________________ 3. Movement of gas as a result of differences in total
pressure
is called bulk flow.
_________________________ 4. According to Dalton’s  law, the greater the molecular weight
of a gas, the slower the rate of diffusion.
_________________________ 5. An increase in blood temperature strengthens the
salt
bridges of the hemoglobin molecule.
_________________________ 6. The relationship between carbon dioxide
and
oxyhemoglobin dissociation is called the Bohr effect.
_________________________ 7. Breathing is automatically generated by neurons located in
the brain stem.
_________________________ 8. The primary motor area is responsible for modification
of
respiration during speech.
_________________________ 9. The nucleus parabrachialis medialis functions to modulate
the activity of the apneustic center.
_________________________ 10. Stimulation of receptors in the nasal mucosa can activate
the sneeze reflex.

Module 14 BBS Short Quiz (Completion) – Write the best answer.

_________________________ 1. The term given to rhythmic waxing and waning


of
respiratory efforts.
_________________________ 2. The gas law that describes the relationship between
gas
volume and absolute temperature.
_________________________ 3. The reason why the oxyhemoglobin dissociation curve
is
sigmoid shaped.
_________________________ 4. It is responsible for stabilizing the deoxy configuration
of
hemoglobin.
_________________________ 5. The chemical reaction of carbon dioxide with water and
with
blood buffers is responsible for this shape of the
carbon dioxide dissociation curve.
_________________________ 6. It results from inactivation of the inspiratory
cutoff
mechanism.
_________________________ 7. They serve as the principal initiators of phrenic
nerve
activity.
_________________________ 8. The reflex initiated by negative pressure in the
upper
airways.
_________________________ 9. The reflex initiated by immersion of the face in water.

_________________________ 10. The receptors stimulated by pulmonary embolism.

Answer key:

1. Cheyne-Stokes respiration
2. Charles’  law
3. It is a plot of 4 reactions / heme-heme interaction / positive cooperativity
4. 2,3 diphosphoglycerate (2,3 DPG)
5. Linear
6. Apneusis
7. Dorsal inspiratory group of neurons of NTS
8. Pharyngeal dilator reflex
9. Diving reflex
10. J receptors in pulmonary vessels

Modified true or false – Write TRUE if the statement is correct. If the statement is false,
write the word/s that would best make it true, in place of the underlined word/s. No erasures.

_________________________ 1. Normal exercise can give rise to hyperventilation.

_________________________ 2. Avogadro’s   law describes the relationship between


gas
volume and gas pressure.
_________________________ 3. Movement of gas as a result of differences in total
pressure
is called bulk flow.
_________________________ 4. According to Dalton’s  law, the greater the molecular weight
of a gas, the slower the rate of diffusion.
_________________________ 5. An increase in blood temperature strengthens the
salt
bridges of the hemoglobin molecule.
_________________________ 6. The relationship between carbon dioxide
and
oxyhemoglobin dissociation is called the Bohr effect.
_________________________ 7. Breathing is automatically generated by neurons located in
the brain stem.
_________________________ 8. The primary motor area is responsible for modification
of
respiration during speech.
_________________________ 9. The nucleus parabrachialis medialis functions to modulate
the activity of the apneustic center.
_________________________ 10. Stimulation of receptors in the nasal mucosa can activate
the sneeze reflex.

Answer key:

1. Hyperpnea
2. Boyle’s  law
3. True
4. Graham’s  law
5. Hydrogen ions
6. True
7. True
8. Broca’s  area
9. Pneumotaxic center
10. True
11. 40. What is the first and reversible step in the catabolism of amino acid?
12. A transfer of ammonia
13. B removal of alpha amino nitrogen by transamination
14. C oxidative deamination of glutamate
15. D reactions of the urea cycle
16.
17. 41. What are the two enzymes that catalyze the transfer of amino group to pyruvate or alpha-
ketoglutarate?
18. A alanine aminotransferase and glutamate aminotransferase
19. B pyridoxal phosphate and alanine aminotransferase
20. C L-glutamate dehydrogenase and glutamate aminotransferase
21. D glutaminase and pyridoxal phosphate
22. 42.Which of the following are amino acids whose carbon skeletons are degraded to acetyl-CoA?
23. A arginine and histidine
24. B leucine and lysine
25. C serine, glycine and alanine
26. D threonine and lysine
27. 43.What is the only amino acid that undergoes oxidative deamination at an appreciable rate?
28. A glycine
29. B phenylalanine
30. C threonine
31. D L-glutamate
32. 44What are the two oxaloacetate forming amino acids?
33. A glutamine and glutamate
34. B arginine and praline
35. C cyteine and tyrosine
36. D aspartate and asparagine
37.
38.
39.
40.
41.
42.

Muscle contraction- quiz


1. In skeletal muscle contraction, which of the following events immediately follows the
binding of Ca++ with troponin C?
A. release of more calcium from the sarcoplasmic reticulum (SR)
B. transport of calcium by the calcium pump back to the SR
C. movement of tropomyosin away from actin binding sites
D. binding of myosin heads to actin binding sites
2. Which of the following is immediately responsible for the opening of calcium channels
in the sarcoplasmic reticulum (SR)?
A. an action potential in the T tubule
B. an action potential in the SR membrane
C. hydrolysis of ATP in the SR membrane
D. calcium influx from the extracellular fluid
3. Calcium plays a significant role in the following events, EXCEPT:
A. muscle contraction C. bone formation
B. blood coagualtion D. nerve action potential propagation

4. When an AP reaches the T-tubule, which of the following immediately follows?


A. stimulation of the calcium pump in the sarcoplamic reticulum (SR)
B. opening of calcium channels in the SR
C. binding of calcium to troponin-C
D. Calcium entry into the cell
5. In the relaxed state of a skeletal muscle fiber, which of the following is true?
A. binding sites in actin are not covered by tropomyosin
B. myosin heads are attached to actin
C. cytosolic calcium level is low
D. calcium is bound to troponin-C
6. During muscular contraction, which of the following would shorten?
A. actin filaments C. sarcomere and myosin filaments
B. sarcomere D. sarcomere and actin filaments

Muscle contraction- quiz


1. In skeletal muscle contraction, which of the following events immediately follows the
binding of Ca++ with troponin C?
A. release of more calcium from the sarcoplasmic reticulum (SR)
B. transport of calcium by the calcium pump back to the SR
C. movement of tropomyosin away from actin binding sites
D. binding of myosin heads to actin binding sites
2. Which of the following is immediately responsible for the opening of calcium channels
in the sarcoplasmic reticulum (SR)?
A. an action potential in the T tubule
B. an action potential in the SR membrane
C. hydrolysis of ATP in the SR membrane
D. calcium influx from the extracellular fluid
3. Calcium plays a significant role in the following events, EXCEPT:
A. muscle contraction C. bone formation
B. blood coagualtion D. nerve action potential propagation

4. When an AP reaches the T-tubule, which of the following immediately follows?


A. stimulation of the calcium pump in the sarcoplamic reticulum (SR)
B. opening of calcium channels in the SR
C. binding of calcium to troponin-C
D. Calcium entry into the cell
5. In the relaxed state of a skeletal muscle fiber, which of the following is true?
A. binding sites in actin are not covered by tropomyosin
B. myosin heads are attached to actin
C. cytosolic calcium level is low
D. calcium is bound to troponin-C
6. During muscular contraction, which of the following would shorten?
A. actin filaments C. sarcomere and myosin filaments
B. sarcomere D. sarcomere and actin filaments

MODULE 26 – ENDOCRINE SYSTEM

1.1 Identify the endocrine organs and their parts, their respective tissues and cellular
contents, if given descriptions or microscopic slides.
1.2 Relate an endocrine cell to its functions.

11. Which of the following associations between each layer of the adrenal gland and
cellular content is least likely true?
A. zona glomerulosa – closely packed columnar cells
B. zona fasciculata – lipofuscin laden polyhedral cells
C. zona reticularis – anastomosing irregularly shaped-cells
D. medulla – polyhedral cells supported by reticular tissue
12. Which of the following hormones secreted by the adrenal cortex play a role in
glycogenesis?
A. Mineralocorticoids C. Glucocorticoids
B. Androgens D. Catecholamines
13. Which of the following is true of the islets of Langerhans?
A. encapsulated by reticular fibers
B. staining divides the islet cells into three types
C. equally distributed throughout the entire pancreas
D. tight junctions allows communication between cells
14. This islet of Langerhans cell type secretes hormones which inhibit secretion of other
islet cell hormones:
A. A B. B C. C D. D
15. Which of the following best describes the parafollicular cells?
A. arrange in anastomosing cords
B. abundant rough endoplasmic reticulum
C. has cytoplasmic granules containing hormones
D. stain more intensely than thyroid follicular cells
16. Which of the following is true of the parathyroid gland?
A. Septated C. reticular tissue capsule
B. Oxyphil cells secrete PTH D. irregular cell arrangement
17. Which of the following is true of the adenohyphosis?
A. funnel-shaped pars intermedia
B. pars distalis contains 3 cell types
C. pars tuberalis cells secrete FSH & LH
D. cord-like arrangement of the pars distalis
18. An endocrine organ covered by pia mater:
A. pineal gland C. thyroid gland
B. pituitary gland D. adrenal medulla

MODULE 25 – GENETICS
IO # 5

2. Which of the following events least likely happens in RNA synthesis?


A. utilization of ribonucleotides
B. involvement of a primer
C. absence of proofreading
D. copying of only a portion of the genome
3. The activity of DNA dependent RNA polymerase is mainly influenced by:
A. linking with the promoter on the template stand
B. catalazing the coupling of first bases
C. transcribing down into transcription unit
D. release of the RNA from the template
4. A product released during the polymerization reaction in RNA synthesis:
A. Water C. Pyrophosphates
B. CO2 D. Oxygen
5. The length of DNA unwinding per polymerase molecule in the elongation process in
terms of base pairs:
A. 20 B. 25 C. 30 D. 35
6. The activity of this enzyme prevents the formation of superhelical complexes:
A. Hydroxylase C. Epimerase
B. Transferase D. Topoisomerase
7. Which of the following is least likely true of the TATA box?
A. about 30 base pairs upstream from the transcription start site
B. its consensus sequence is generally TATAAA
C. attached by binding proteins
D. needs additional cis elements at all times
8. Which of the following best describes the CAAT box?
A. involves DNA sequences
B. binds Sp1 protein via DNA binding domains
C. requires strict spacing of trans elements
D. decreases the rate of transcription
9. Splicing of exons sequences occurs in this part of the cell:
A. Mitochondria C. Cytoplasm
B. Nucleus D. Golgi complex
10. The second methylation process of mRNA occurs in the:
A. Mitochondria C. Cytoplasm
B. Nucleus D. Golgi complex
11. The main function poly(A) tail is to:
A. efficiently start translation
B. shield  the  3’  end  of  mRNA  from  an  exonuclease
C. protect  the  5’  end  from  an  exonuclease
D. initiation site for splicing
12. Which of the following is true of the termination process of transcription?
A. dependent on DNA cis elements
B. modulated by cis-acting protein factors
C. signaled by RNA splicing of exons
D. formation  of  5’  to  3’  tail

BB NERVOUS TISSUE QUIZ

48. The main determinant of the resting membrane potential is:


A. K+ efflux C. Na+ efflux
B. K+ influx D. Na+ influx
49. After an action potential, these gates should be open for the membrane to be excitable
again. U
A. m gates C. h gates
B. n gates D. m, n, and h gates
50. In an experiment, the Na+ and K+ channels of a motor neuron were opened at the
same time. Which of these most likely developed? U
A. an action potential C. a graded potential
B. a threshold potential D. an end plate potential
51. Which of the following is true of the nervous system? R
A. it is composed primarily of glial cells
B. the brain and spinal cord are surrounded by three meninges
C. the central nervous system consists of the brain and cranial nerves
D. the peripheral nervous system consists of the spinal cord and spinal nerves
52. Neurotransmitter action may be terminated by: R
A. activity of specific enzymes
B. binding with presynaptic receptors
C. uptake by the postsynaptic neuron
D. increasing the number of receptors in the postsynaptic neuron
53. Which of the following is true of the resting membrane potential? U
A. it is mainly due to sodium influx
B. it is maintained by the Na+/K+ pump
C. at RMP the Na+/K+ pump is inhibited
D. at RMP there is net movement of K+ out of the cell
54. Which of the following applies to both graded and action potentials? R
A. can be summated
B. can occur spontaneously
C. can be conducted by local current flow
D. can be a depolarization or a hyperpolarization
55. During the upstroke phase of an action potential, which of these gates are open? R
A. m and n gates C. n and h gates
B. m and h gates D. m, n, and h gates
56. The rectifier property of the cell membrane is primarily due to:
A. Na+ influx C. K+ influx
B. Na+ efflux D. K+ efflux
57. The cell membrane is absolutely refractory during which phase/s of the action
potential? R
A. upstroke only C. late downstroke
B. early downstroke D. upstroke and early downstroke
58. Which of these gates are open during the repolarization phase of the action potential?
U
A. m and h gates C. h and n gates
B. m and n gates D. m, n, and h gates
59. This is true of the relative refractory period. R
A. membrane is back to its resting potential
B. coincides with the entire downstroke phase
C. during this period most of the K+ channels are closed
D. membrane my be stimulated by stronger than normal stimulus
60. Which of the following is true of myelinated axons? R
A. action potential (AP) conduction is slower than in unmyelinated fibers
B. APs jump from one internode to the next internode
C. APs occur only at the nodes of Ranvier
D. AP conduction is decremental
61. Which of these neurons release epinephrine as their transmitters? R
A. preganglionic parasympathetic neurons
B. preganglionic sympathetic nerurons
C. postganglionic sympathetaic neurons
D. presynaptic motor neuron
62. Which of these events immediately precedes the production of inhibitory postsynaptic
potential (IPSP)? R
A. depolarization of the terminal knob of presynaptic neuron
B. binding of transmitters with receptors in postsynaptic neuron
C. diffusion of transmitter towards postsynaptic membrane
D. local K+ efflux or Cl- influx
63. Which of these potentials can be summated? R
A. action potential C. end plate potential
B. threshold potential D. excitatory postsynaptic potential
64. Which of these substances block the synthesis of neurotransmitters? R
A. botulinum toxin C. hemicholinium
B. organophosphate D. succinylcholine
65. To produce an IPSP, the release of neurotransmitters from the presynaptic neuron is
immediately followed by which of these events?
A.

66. Four action potentials reached the terminal knob of a presynaptic neuron at intervals of
10 secs. Which of the following is most likely to occur in the postsynaptic neuron? U
A. a summated EPSP C. an action potential
B. four separate EPSPs D. an IPSP
67. Which of these types of synaptic inhibition is mediated mainly by GABA? R
A. Presynaptic C. indirect
B. Postsynaptic D. direct
68. A motor unit is composed of a motor neuron and: R
A. all the muscle fibers it innervates
B. the motor end plate
C. all its terminal branches
D. its neurotransmitter
69. Which of the following can initiate an action potential in the muscle membrane? R
A. summated EPSPs
B. end plate potential
C. summated miniature end plate potentials
D. an action potential in the motor end plate
70. Which of the following electrolytes plays a major role in the release mechanisms of the
neurotransmitter? R
A. Sodium C. calcium
B. Potassium D. chloride

Essay

3. If the m, h, and n gates open at the same time, will an action potential occur?
1.5 Encircle your answer: 1 pt

YES / NO

1.6 Explain your answer in 1.1 above. 2 pts

SPECIAL EXAM
FOURTH BIMONTHLY PERIOD

NAME___________________________

1. Describe the muscles of the orbit to include the following: 10 pts


1.1 origin
1.2 insertion
1.3 action
1.4 nerve supply
2. Describe the middle ear (tympanic cavity) to include the following: 10 pts.
2.1 contents
2.2 wall and parts found on each wall

SPECIAL EXAM
FOURTH BIMONTHLY PERIOD

NAME___________________________

1. Describe the muscles of the orbit to include the following: 10 pts


1.1 origin
1.2 insertion
1.3 action
1.4 nerve supply
2. Describe the middle ear (tympanic cavity) to include the following: 10 pts.
2.1 contents
2.2 wall and parts found on each wall

SPECIAL EXAM
FOURTH BIMONTHLY PERIOD

NAME___________________________

1. Explain how vibration of the basilar membrane lead to release of transmitters


from auditory hair cells. 5pts.

2. Jane turns her head to the left while she is in upright posture.
2.1 What vestibular organ will be stimulated by her action? 1 pt.
2.2 Explain how such head turning can lead to stimulation of the hair cells of
that vestibular organ. 4 pts.
SPECIAL EXAM
FOURTH BIMONTHLY PERIOD

NAME___________________________

1. Explain how vibration of the basilar membrane lead to release of transmitters


from auditory hair cells. 5pts.

2. Jane turns her head to the left while she is in upright posture.
2.1 What vestibular organ will be stimulated by her action? 1 pt.
2.2 Explain how such head turning can lead to stimulation of the hair cells of
that vestibular organ. 4 pts.

NERVOUS TISSUE QUIZ

Essay

1. If the m, h, and n gates open at the same time, will an action potential occur?
1.1 Encircle your answer: 1 pt

YES / NO

1.2 Explain your answer in 1.1 above. 2 pts

MCQ

1. When the cell membrane is at its resting potential, the membrane is: R
A. not excitable
B. not using ATP
C. positive inside and negative outside
D. more permeable to potassium than to sodium
2. Which of the following is true of the resting membrane potential (RMP)? U
A. it is mainly due to sodium influx
B. it is maintained by the Na+/K+ pump
C. at RMP the Na+/K+ pump is inhibited
D. at RMP there is net movement of K+ out of the cell
3. Which of the following is a characteristic of graded potential but not of an action
potential? R
A. Its amplitude is related to the magnitude of the stimulus
B. it is a change in the resting membrane potential
C. it is conducted along the cell membrane
D. it is initiated by a stimulus
4. Which of the following applies to both graded and action potentials? R
A. can be summated
B. can occur spontaneously
C. can be conducted by local current flow
D. can be a depolarization or a hyperpolarization
5. A stimulus was applied to a nerve axon. A depolarization resulted but it was
conducted only a short distance. The most likely explanation is: U
A. the stimulus was suprathreshold
B. the axon was unmyelinated
C. the axon was at its refractory period
D. the depolarization did not reach threshold potential
6. During the upstroke phase of an action potential, which of these gates are open? R
A. m and n gates C. n and h gates
B. m and h gates D. m, n, and h gates
7. Membrane rectification is primarily due to:
A. Na+ influx C. K+ influx
B. Na+ efflux D. K+ efflux
8. The undershoot phase of an action potential is due to: R
A. slow closure of the K+ channels
B. inactivation of the Na+ channels
C. closure of the h gates
D. opening of the m gates
9. The cell membrane is absolutely refractory during which phase/s of the action
potential? R
A. upstroke only C. late downstroke
B. early downstroke D. upstroke and early downstroke

10. Which of these gates are open during the repolarization phase of the action potential?
U
A. m and h gates C. h and n gates
B. m and n gates D. m, n, and h gates
11. The absolute refractory period.is primarily due to the closure of the: R
A. h gates C. n gates
B. m gates D. h, m, and n gates
12. This is true of the relative refractory period. R
A. membrane is back to its resting potential
B. coincides with the entire downstroke phase
C. during this period most of the K+ channels are closed
D. membrane my be stimulated by stronger than normal stimulus
13. Two nerve axons (A and B) were stimulated simultaneously by the same amount of
electrical current and action potentials were conducted in the two nerves. However,
the rate of conduction in A was faster than in B. Which of the following is the most
likely explanation? P or U
A. A is unmyelinated while B is myelinated
B. A has a larger diameter than B
C. A has a higher threshold than B
D. A has a longer length than B
14. Which of the following is true of myelinated axons? R
A. action potential (AP) conduction is slower than in unmyelinated fibers
B. APs jump from one internode to the next internode
C. APs occur only at the nodes of Ranvier
D. AP conduction is decremental
15. Which of the following is true of electrical synapses? R
A. no synaptic delay
B. more commonly found in humans
C. synaptic cleft is wider than that found in chemical synapses
D. speed of transmission is slower than that in chemical synapses
16. Which of these neurons release epinephrine as their transmitters? R
A. preganglionic parasympathetic neurons
B. preganglionic sympathetic nerurons
C. postganglionic sympathetaic neurons
D. presynaptic motor neuron
17. When an action potential arrives at the terminal knob of presynaptic neuron, which of
these events immediately follows? R
A. diffusion of Ca++ into the terminal knob
B. depolarization of the terminal knob
C. release of transmitters into the synaptic cleft
D. binding of transmitters with the receptors
18. Which of these events immediately precedes the production of inhibitory postsynaptic
potential (IPSP)? R
A. depolarization of the terminal knob of presynaptic neuron
B. binding of transmitters with receptors in postsynaptic neuron
C. diffusion of transmitter towards postsynaptic membrane
D. local K+ efflux or Cl- influx
19. Which of the following is true of excitatory postsynaptic potential (EPSP)? R
A. it is a depolarizing graded potential
B. it causes a voltage change of at least 15 mV
C. it usually lasts for about 15 seconds
D. it can occur only at the body of postsynaptic neuron
20. Which of these potentials can be summated? R
A. action potential C. inhibitory postsynaptic potential
B. threshold potential D. excitatory postsynaptic potential
21. Which of the following factors can cause a decrease in the rate of synaptic
transmission? R
A. Alkalosis C. coffee
B. Hypoxia D. tea
22. Which of these substances block the synthesis of neurotransmitters? R
A. botulinum toxin C. hemicholinium
B. organophosphate D. succinylcholine
23. Which of these substances competes with the transmitters for receptor sites? R
A. botulinum toxin C. hemicholinium
B. organophosphate D. succinylcholine
23. Four action potentials reached the terminal knob of a presynaptic neuron at intervals
of 10 secs. Which of the following is most likely to occur in the postsynaptic neuron?
U
A. a summated EPSP C. an action potential
B. four separate EPSPs D. an IPSP
24. To produce an action potential in a skeletal muscle the voltage change in the muscle
membrane should be at least: R
A. 15 mV C. 50 mV
B. 30 mV D. 60 mV
25. Presynaptic inhibition is mediated mainly by;
A. Glycine C. aspartic acid
B. glutamic acid D. gamma-amino butyric acid
26. A motor unit is composed of a motor neuron and: R
A. all the muscle fibers it innervates
B. the motor end plate
C. all its terminal branches
D. its neurotransmitter
27. The potential that is produced at the muscle membrane which lies directly
under the axon terminal of a neuron is termed an:
A. action potential C. EPSP
B. end plate potential D. IPSP
28. Which of the following can initiate an action potential in the muscle membrane?
A. summated EPSPs
B. end plate potential
C. summated miniature end plate potentials
D. an action potential in the motor end plate

BB NERVOUS TISSUE QUIZ


71. Reversal of the transmembrane potential is primarily due ot:
A. Na+ influx C. K+ influx
B. Na+ efflux D. K+ efflux
72. After an action potential, these gates should be open for the membrane to be excitable
again.
A. m gates C. h gates
B. n gates D. m, n, and h gates
73. If the membrane potential is equal to the K+ equilibrium potential, this means that: U
A. the membrane is permeable to K+ only
B. concentration gradient for K+ is lost
C. there is no net movement of K+ across the membrane
D. electrical force acting on K+ is greater than the concentration force
74. Which of the following is true of the nervous system? R
A. it is composed primarily of glial cells
B. the brain and spinal cord are surrounded by three meninges
C. the central nervous system consists of the brain and cranial nerves
D. the peripheral nervous system consists of the spinal cord and spinal nerves
75. A synapse is best defined as:
A. the point of fusion between two neurons
B. an anatomically specialized junction between two neurons
C. the point of direct transmission of action potentials between two neurons
D. junction between two neurons whereby channels connect the two neurons
76. A presynaptic neuron was stimulated by an adequate stimulus, but the response of the
postsynaptic neuron was less than expected. Which of the following is the most
logical conclusion? U
A. transmitter release is less than usual
B. a neuromodulator may have been released also
C. the postsynaptic neuron is at its absolute refractory period
D. transmitter causes increased permeability of postsynaptic neuron to K+
77. Neurotransmitter action may be terminated by:
A. activity of specific enzymes
B. binding with presynaptic receptors
C. uptake by the postsynaptic neuron
D. increasing the number of receptors in the postsynaptic neuron
78. Which of the following occurs to inhibit the postsynaptic membrane?
A. Opening of Na+ channels
B. Decreased conductance of Cl- ions out of the neuron
C. increased conductance of K+ out of the neuron
D. Decreased number of inhibitory membrane receptors
79. To produce an excitatory post synaptic potential (EPSP), which of these steps
immediately follows the depolarization of the terminal knob?
A. Ca ++ bind with synaptic vesicles
B. Release of transmitter into the synaptic cleft
C. Diffusion of Ca++ into the terminal knob
D. Diffusion of transmitter toward postsynaptic membrane
80. Which of the following associations between transmitters and main modes of action is
correct?
A. dopamine – excitation C. glutamate - inhibition
B. glycine – excitation D. serotonin – inhibition
81. This is true of neuropeptides.
A. they are slowly acting
B. effect is of short duration
C. less potent than the small-molecule transmitters
D. synthesized in the cytosol of presynaptic terminals

82. Summation  is  “temporal”  when  the  increase  in  the  magnitude  of  the  EPSP  is:
A. enough to reach threshold and an action potential develops
B. enough to result in facilitation of postsynaptic membrane
C. due to simultaneous arrival of two separate excitatory inputs
D. due to arrival of two or more excitatory inputs in rapid succession
83. When the cell membrane is at its resting potential, the membrane is: R
A. not excitable
B. not using ATP
C. positive inside and negative outside
D. more permeable to potassium than to sodium
84. Which of the following is true of the resting membrane potential (RMP)? U
A. it is mainly due to sodium influx
B. it is maintained by the Na+/K+ pump
C. at RMP the Na+/K+ pump is inhibited
D. at RMP there is net movement of K+ out of the cell
85. Which of the following is a characteristic of graded potential but not of an action
potential? R
A. Its amplitude is related to the magnitude of the stimulus
B. it is a change in the resting membrane potential
C. it is conducted along the cell membrane
D. it is initiated by a stimulus
86. If the membrane potential is equal to the Na+ equilibrium potential, this means that:
A. Na+ influx is equal to Na+ efflux
B. Membrane is permeable to Na+ only
C. Na+ influx is along concentration gradient
D. No Na+ concentration gradient exists across the membrane
87. Which of the following applies to both graded and action potentials? R
A. can be summated
B. can occur spontaneously
C. can be conducted by local current flow
D. can be a depolarization or a hyperpolarization
88. A stimulus was applied to a nerve axon. A depolarization resulted but it was
conducted only a short distance. The most likely explanation is: U
A. the stimulus was suprathreshold
B. the axon was unmyelinated
C. the axon was at its refractory period
D. the depolarization did not reach threshold potential
89. During the upstroke phase of an action potential, which of these gates are open? R
A. m and n gates C. n and h gates
B. m and h gates D. m, n, and h gates
90. The rectifier property of the cell membrane is primarily due to:
A. Na+ influx C. K+ influx
B. Na+ efflux D. K+ efflux
91. The undershoot phase of an action potential is due to: R
A. slow closure of the K+ channels
B. inactivation of the Na+ channels
C. closure of the h gates
D. opening of the m gates
92. The cell membrane is absolutely refractory during which phase/s of the action
potential? R
A. upstroke only C. late downstroke
B. early downstroke D. upstroke and early downstroke
93. Which of these gates are open during the repolarization phase of the action potential?
U
A. m and h gates C. h and n gates
B. m and n gates D. m, n, and h gates
94. The absolute refractory period.is primarily due to the closure of the: R
A. h gates C. n gates
B. m gates D. h, m, and n gates
95. This is true of the relative refractory period. R
A. membrane is back to its resting potential
B. coincides with the entire downstroke phase
C. during this period most of the K+ channels are closed
D. membrane my be stimulated by stronger than normal stimulus
96. Two nerve axons (A and B) were stimulated simultaneously by the same amount of
electrical current and action potentials were conducted in the two nerves. However,
the rate of conduction in A was faster than in B. Which of the following is the most
likely explanation? P or U
A. A is unmyelinated while B is myelinated
B. A has a larger diameter than B
C. A has a higher threshold than B
D. A has a longer length than B
97. Which of the following is true of myelinated axons? R
A. action potential (AP) conduction is slower than in unmyelinated fibers
B. APs jump from one internode to the next internode
C. APs occur only at the nodes of Ranvier
D. AP conduction is decremental
98. Which of the following is true of electrical synapses? R
A. more commonly found in humans
B. signal transmission is bidirectional
C. synaptic cleft is wider than that found in chemical synapses
D. speed of transmission is slower than that in chemical synapses
99. Which of these neurons release epinephrine as their transmitters? R
A. preganglionic parasympathetic neurons
B. preganglionic sympathetic nerurons
C. postganglionic sympathetaic neurons
D. presynaptic motor neuron
100. When an action potential arrives at the terminal knob of presynaptic neuron,
which of these events immediately follows? R
A. diffusion of Ca++ into the terminal knob
B. depolarization of the terminal knob
C. release of transmitters into the synaptic cleft
D. binding of transmitters with the receptors
101. Which of these events immediately precedes the production of inhibitory
postsynaptic potential (IPSP)? R
A. depolarization of the terminal knob of presynaptic neuron
B. binding of transmitters with receptors in postsynaptic neuron
C. diffusion of transmitter towards postsynaptic membrane
D. local K+ efflux or Cl- influx
102. Which of the following is true of excitatory postsynaptic potential (EPSP)? R
A. it is propagated decrementally
B. it usually lasts for about 15 seconds
C. it is a depolarizing graded potential
D. it can occur only at the body of postsynaptic neuron
103. Which of these potentials can be summated? R
A. action potential C. inhibitory postsynaptic potential
B. threshold potential D. excitatory postsynaptic potential
104. Which of the following factors can cause a decrease in the rate of synaptic
transmission? R
A. Alkalosis C. coffee
B. Hypoxia D. tea
105. Which of these substances block the synthesis of neurotransmitters? R
A. botulinum toxin C. hemicholinium
B. organophosphate D. succinylcholine
106. Which of these substances competes with the transmitters for receptor sites? R
A. botulinum toxin C. hemicholinium
B. organophosphate D. succinylcholine
107. Four action potentials reached the terminal knob of a presynaptic neuron at
intervals of 10 secs. Which of the following is most likely to occur in the postsynaptic
neuron? U
A. a summated EPSP C. an action potential
B. four separate EPSPs D. an IPSP
108. To produce an action potential in a skeletal muscle the voltage change in the
muscle membrane should be at least: R
A. 15 mV C. 50 mV
B. 30 mV D. 60 mV
109. Presynaptic inhibition is mediated mainly by; R
A. Glycine C. aspartic acid
B. glutamic acid D. gamma-amino butyric acid
110. Which of these types of synaptic inhibition is mediated mainly by GABA? R
A. Presynaptic C. indirect
B. Postsynaptic D. direct
111. Acetylcholine may be released spontaneously from the motor neuron axon
terminal in small quantities, and this results in the production of an: R
A. EPSP C. end plate potential
B. IPSP D. miniature end plate potential
112. A motor unit is composed of a motor neuron and: R
A. all the muscle fibers it innervates
B. the motor end plate
C. all its terminal branches
D. its neurotransmitter
113. The potential that is produced at the muscle membrane which lies directly
under the axon terminal of a neuron is termed an: R
A. action potential C. EPSP
B. end plate potential D. IPSP
114. Which of the following can initiate an action potential in the muscle membrane?
R
A. summated EPSPs
B. end plate potential
C. summated miniature end plate potentials
D. an action potential in the motor end plate
115. Which of these events immediately follows after the release of acetylcholine
(Ach) into the neuromuscular junction? R
A. Ca++ diffuses into terminal knob
B. Ach binds with receptors at motor end plate
C. Ach diffuses across the synaptic cleft
D. Opening of voltage-gated sodium channels
116. Which of the following electrolytes plays a major role in the release
mechanisms of the neurotransmitter? R
A. Sodium C. calcium
B. Potassium D. chloride
117. Which of these is considered as a neuromodulator? R
A. Acetylcholine C. dopamine
B. Norepinephrine D. angiotensin II
118. Which of the following relates to the motor end plate? R
A. acetylcholine receptors in sarcolemma with junctional folds
B. synaptic cleft between axon terminal and sarcolemma
C. synaptic vesicles and mitochondria in axon terminal
D. myelinated expanded axon terminal
Essay

4. If the m, h, and n gates open at the same time, will an action potential occur?

1.7 Encircle your answer: 1 pt

YES / NO

1.8 Explain your answer in 1.1 above. 2 pts

ORAL EXAM – PHYSIOLOGY

THE GASTROINTESTINAL SYSTEM


1. Describe/define gastrocolic reflex.

Answer: The presence of food in the stomach triggers a reflex that results in
increased colonic activity.

2. What is the most frequent type of movement (motor activity) in the small intestine?

Answer: segmentation (contraction)

2.1 Describe this motor activity.

Answer:
A set of ring-like contractions appear at fairly regular intervals along the gut,
then disappear and are replaced by another set of ring-like contractions in the
segments between the previous contractions.

ORAL EXAM – PHYSIOLOGY

THE ENDOCRINE SYSTEM


1. Discuss how a low plasma cortisol level is brought back towards normal via the
negative-feedback mechanism. Mention specific hormones, cells and organs involved.

Answer:
Decreased plasma cortisol
l
stimulates the hypothalamus
(parvicellular cells)
l
increased secretion of corticotropin releasing hormone (CRH)
l
CRH stimulates anterior pituitary gland*
(corticotropes)
l
increased secretion of adrenocorticotropic hormone (ACTH)
l
stimulate the adrenal cortex
(zona fasciculate and z. reticularis)
l
increased secretion of cortisol back towards normal

* Decreased cortisol also directly stimulates the anterior pituitary to


increase secretion of ACTH

ORAL EXAM – PHYSIOLOGY

THE CARDIOVASCULAR SYSTEM


1. Trace the sequence of excitation of the heart from the normal pacemaker until
depoplarization of the ventricles.

Sinoatrial node (SAN)


l
spreads radially to the atria
l
atrioventricular node (AVN) or AV junction
l
Bundle of His or AV bundle
l
right and left bundle branches
l
terminal purkinje network
l
Ventricular muscles

 Spread of excitation to the left atrium and to the AVN is enhanced via the
internodal and interatrial tracts.

2. What is the effect of the following conditions on arterial blood pressure. Answer
INCREASE or DECREASE.

Answers:
2.1 Low blood volume – decrease
2.2 High hematocrit - increase
2.3 diminished venous return - decrease
2.4 Generalized arteriolar vasoconstriction- increase
PHYSIOLOGY
1. Enumerate the segment/s of the gastrointestinal tract that exhibit the migrating
myoelectric complex (MMC). 2 pts. Right minus wrong.

Ans. Stomach
Duodenum, jejunum, ileum
(if student answers small intestine, let him specify the segments)

2. In correct sequence enumerate the different phases of swallowing. 1.5 pts

Ans. Oral or voluntary


Pharyngeal
Esophageal

3. Where is the integrating center for the defecation reflex? 0.5 pt

Ans. sacral spinal cord


4. What is the most frequent type of movement of the small lintestine? 0.5 pt

Ans. segmentation
5. In which part of the stomach does the major mixing activity occur? 0.5 pt

Ans. antrum

PHYSIOLOGY
1. Enumerate the segment/s of the gastrointestinal tract that exhibit the migrating
myoelectric complex (MMC). 2 pts. Right minus wrong.

Ans. Stomach
Duodenum, jejunum, ileum
(if student answers small intestine, let him specify the segments)

2. In correct sequence enumerate the different phases of swallowing. 1.5 pts

Ans. Oral or voluntary


Pharyngeal
Esophageal

3. Where is the integrating center for the defecation reflex? 0.5 pt

Ans. sacral spinal cord


4. What is the most frequent type of movement of the small lintestine? 0.5 pt

Ans. segmentation
5. In which part of the stomach does the major mixing activity occur? 0.5 pt

Ans. antrum
PHYSIOLOGY PRE-TEST
Encircle the letter of the BEST answer.
NAME ____________________________________
1. Which of the following is a characteristic of aqueous/leak channels in the cell
membrane?
A. Open all the time
B. made up of extrinsic proteins
C. open wide when electrically stimulated
D. allow passage of large molecules when needed by the cell
2. After a cell membrane was stimulated, it soon went back to its original unstimulated
state. This is because it has this property.
A. Core conductor C. rectifier
B. Condenser D. resistor
3. When the membrane is at its resting membrane potential (RMP):
A. The membrane is excitable
B. The Na+/K+ pump is inhibited
C. The membrane is more permeable to Na+ than to K+
D. There is net movement of Na+ and K+ along their concentration gradients
4. Which of the following tissues exhibits the longest duration of action potential?
A. Motor neuron C. ventricular muscle
B. Skeletal muscle D. smooth muscle of the small intestine
5. As regards action potentials in GIT smooth muscles, which of these is true?
A. Appear on the peaks of all slow waves
B. Only one AP can occur in each slow wave
C. Magnitude of AP determines strength of muscle contraction
D. Occur  when  peak  of  slow  wave  exceeds  the  cell’s  threshold
6. Which of these is true of the upstroke phase of the AP in neurons?
A. Membrane polarity remains negative
B. Begins at the start of stimulation
C. Due to rapid and large K+ influx
D. Membrane is resistant to stimulation
7. Which of the following can lead to an action potential in a skeletal muscle membrane?
A. End plate potential C. excitatory postsynaptic potential
B. Miniature end plate potential D. inhibitory postsynaptic potential
8. The spontaneous depolarization of the resting membrane to threshold in nodal cells (P
cells) is termed:
A. End plate potential C. threshold potential
B. Pacemaker potential D. graded potential
9. According to Berne & Levy, the pacemaker potential is due to these ionic currents:
A. Inward  “funny”  current,  inward  Ca++  current  &  outward  K+  current
B. Inward  “funny”  current,  inward  Ca++  current,  &  inward  K+  current
C. Inward  “funny”  current,  inward  K+  current,  &  outward  Ca++  current
D. Inward  “funny”  current,  outward  Ca++  current, & outward K+ current
10. The SA node is the normal pacemaker of the heart because:
A. It has the highest number of pacemaker cells
B. Its pacemaker cells are centrally located
C. It has the highest rate of spontaneous impulse formation
D. It is located at the uppermost position of the conduction system
11. In which of these conditions will systolic murmur likely to occur?
A. Tricuspid stenosis & pulmonary insufficiency
B. Mitral insufficiency & aortic insufficiency
C. Tricuspid stenosis & pulmonary stenosis
D. Mitral insufficiency & aortic stenosis
12. Systolic pressure refers to the maximum or highest pressure in the aorta (and other
large arteries) which is reached at the end of this phase of the cardiac cycle.
A. Isovolumetric contraction phase C. reduced ejection phasse
B. Rapid ejection phase D. isovolumetric relaxation phase
13. This is true of the first heart sound
A. Due to the closure of the semilunar valves
B. Clinically marks the onset of diastole
C. Can be heard at all the clinical valve areas
D. Occurs at the onset of isovolumetric relaxation phase of the cardiac cycle
14. Applying  Starling’s  law  of  the  heart,  the  force  of  myocardial  contraction  is  increased  
when the ________ is increased.
A. Stroke volume C. preload stress
B. Residual volume D. afterload stress
15. Most of the circulating blood is present in the:
A. Aorta C. veins, venules, venous sinuses
B. Large arteries D. arterioles & capillaries
16. Central venous pressure (CVP) refers to the pressure in the:
A. Right atrium C. right and left atrium
B. Left atrium D. superior vena cava
17. As a result of right ventricular failure, which of these will increase?
A. Blood volume C. CVP
B. Venous return D. CVP & peripheral venous pressure
18. The most important and the main form of transport across the capillary wall is:
A. Simple diffusion C. secondary active transport
B. Facilitated transport D. filtration/absorption
19. Which of these conditions enhance capillary filtration?
A. Hyperproteinemia C. decreased blood pressure
B. Arteriolar constriction D. varicose veins
20. Which of these conditions will most likely lead to hypotension?
A. High hematocrit D. sympathetic stimulation
B. Arteriolar dilatation D. hypervolemia
__________________________1.
__________________________2. The sinoatrial node (SA node) is composed primarily of
P cells.

__________________________3. The electrocardiogram is a graphic recording of the


electrical activity of the heart as recorded from the
surface of the heart.
__________________________4. The P wave of the elelctrocardiogram represents atrial
depolarization.
__________________________5. The amount of blood ejected by the left ventricle during
systole is called systolic volume.
__________________________6. The amount of blood ejected by the left ventricle is
greater than the amount of blood ejected by the right
ventricle in one minute.
__________________________7. Atrial contraction occurs at the end of diastole.

__________________________8. The first heart sound is due to closure of the semilunar


valves.

__________________________9. Atria and ventricles are relaxed during the middle part of
cardiac diastole.
__________________________10. Aortic pressure is greater than left ventricular pressure
during diastole.
__________________________11. Ejection of blood into the pulmonary arteries occurs
during systole.

II. MULTIPLE CHOICE: Write the letter of the BEST answer on the blank space provided
before each
item. Use capital letters. No erasures/superimpositions allowed

______ 1. The SA node is the normal pacemaker of the heart because:


A. it is located in the right atrium
B. its cells exhibit pacemaker potential
C. it is composed of P cells and transitional cells
D. it has the highest rate of spontaneous generation of action potentials

______ 2. Which of the following structures has the fastest rate of impulse conduction?
A. SA node C. Purkinje network
B. AV node D. ventricular muscles
______ 3. When the SA node is sick, which of these structures takes its place as the
pacemaker of the heart?
A. AV node C. bundle of His
B. Internodal tract D. bundle branches

______ 4. Purkinje cells are the main components of which of these structures?
A. SA node C. bundle of His
B. AV node D. atria and ventricles

______ 5. Which of these events occur during cardiac systole?


A. Atria contract
B. First heart sound (S 1) is produced
C. Ventricles are filling with blood from the atria
D. atrial pressure is greater than ventricular pressure

______ 6. Which of these events occur during cardiac diastole?


A. atrioventricular valves are closed
B. blood flows from atria to ventricles
C. ventricular pressure is greater than atrial pressure
D. ventricular pressure is greater than aortic pressure

______ 7. Depolarization of the AV node is immediately followed by depolarization of the:


A. SA node C. internodal pathways

B. bundle of His D. terminal Purkinje network

______ 8. Which of the following is true of the second heart sound (S 2)


A. it is due to the closure of the atrioventricular valves
B. clinically, it marks the onset of diastole
C. it is dull in quality
D. it is low-pitched

______ 9. Cardiac diastole is the period from:


A. S 1 to the next S 2 C. S 2 to the next S 1
B. S 1 to the next S 1 D. S 2 to the next S 2

______ 10. One cardiac cycle is the period from the closure of:
A. the atrioventricular (AV) valves to the opening of the semilunar valves
B. the semilunar valves to the opening of the AV valves
C. the semilunar valves to the closure of the AV valves
D. the AV valves to the next closure of the AV valves

NAME ________________________________
1. Give the three main criteria for a good research problem. 1.5 pts

2. What type of hypothesis is preferred in testing suspected risk and prognostic factors?
0.5 pt

Encircle your answer: Positive or Null

3. Given the following problems, write on the blank space before each item: 0.5 pt each
GP – if the problem is general
SP – if the problem is specific

_______ What is the percentage of the elderly population of Cagayan de Oro City who
has serum albumin deficiency?
_______ Is Safeguard liquid soap as effective as Povidone-iodine as antiseptic?

_______ What is the extent of caring behaviors of nurses as perceived by clients of


the JPRSM Hospital from January 2009 to December 2010?
4. Given the following hypotheses, write on the blank space before each item:0.5 pt each
OT – if the hypothesis is one-tailed
TT – if the hypothesis is two-tailed
N – if an association does not exist

_______ There is a direct association between cervical cancer and poverty.

_______ Clotrimazole cream and terbinafine cream are equal in efficacy in the
treatment of tinea pedis.
_______ There is an association between the medical condition and malnutrition in the
elderly population.
Questions on Stress

1. Which of the following is the first to be activated when a stressor confronts an


individual?
A. sympathetic nervous system C. pituitary gland
B. hypothalamus D. adrenal gland
2. Which of the following, when activated, will tend to modulate the responses to stress?
A. sympathetic nervous system C. hypothalamus
B. immune system D. endocrine system
3. Which of the following can function as a psychological stressor?
A. problem with faculty C. peer pressure
B. threat of terrorism D. traffic
4. Which if the following is most probably taking place if, as a result of a stressor, a
normal  female  suddenly  behaves  like  a  “superwoman”?
A. shock phase C. stage of resistance
B. countershock phase D. stage of exhaustion
5. Which   of   the   following   is   mainly   involed   in   preparing   an   individual   for   “fight   or   flight”  
when subjected to a stressor?
A. sympathetic nervous system C. hypothalamus
B. immune system D. endocrine system
6. Which of the following is an immediate effect of stress?
A. increased mental activity C. increased blood glucose
B. increased heart rate D. decreased level of immunity
7. Which of the following is attributable to the effects of cortisol only?
A. bronchodilation C. hyperglycemia
B. vasoconstriction D. increased muscle strength
8. Which of the following is considered as a negative effect of stress?
A. decrease in coagulation time C. decrease in lymphocytes
B. increase in red cells D. increase in blood glucose
9. Which of the following, when activated, will indicate that the effects of stress are still
positive?
A. ACTH axis B. thyroid axis C. pituitary axis D. discrete autacoids
10. Which of the following can be considered as a personal coping skill?
A.  learn  how  to  say  “no” C. develop relations with co-workers
B. develop hobbies D. respond rather than react to a stressor
11. When confronted by a stressor, a well-adjusted individual will most probably respond
by:
A. ignoring or avoiding the stressor
B. introducing humor into the situation

C. involving several coping strategies


D. developing a support system
12. Which of the following corresponds to a conflict resolution skill?
A. respond, rather than react, to a stressful situation
B. organize thoughts, then respond soon to a stressful situation
C. practice peaceful disagreement if the situation is stressful
D. nurture relationships with co-workers to minimize stress

Dr.  M.  Sison’s  questions  for  GENETICS

15. A  ribose  sugar  with  a  purine  base  attached  to  its  C1’  position  is  called  a:
A. nucleotide
B. nucleoside
C. puridine
D. ribonucleotide

16. If  the  ribose  in  a  nucleotide  has  a  hydrogen  atom  attached  to  its  C2’  instead  of    OH  the  
nucleotide is part of a strand of
A. DNA
B. mRNA
C. tRna
D. rRna

17. The naturally occurring nucleotide that is a component of the precursor of glycogen is
derived from
A. adenosine
B. guanosine
C. uridine
D. inosine

18. The backbone of a DNA strand is composed of


A. bases and phosphates
B. sugars and bases
C. phosphates and nucleotides
D. phosphates and sugars

19. A synthetic nucleotide derivative is able to prevent the formation of uric acid from
purine due to its resemblance to the base of
A. inosine
B. adenosine
C. cytidine
D. thymidine

20. Base pairs of DNA are stabilized by


A. Van  der  Waal’s  forces
B. Polar bonds
C. Ester linkages
D. Hydrogen bonds

DR.  M.  SISON’S  QUESTIONS  FOR  ENDOCRINE

5. The intercellular messengers secreted into the extracellular fluid and act on
neighboring cells of a different type are called:
A. endocrine agents
B. autocrine agents
C. paracrine agents
D. neuroendocrine agents

6. The first step in the action of any intercellular chemical messenger is:
A. alter the membrane permeability of the target cell
B. bind to its specific receptor
C. signal the activation of cAMP
D. trigger the activation of protein kinase

7. Upregulation of messenger receptors


A. takes place in the presence of a high concentration of messenger substance
B. makes  the  target  cell  more  sensitive  to  the  messenger’s  effect
C. involves the sequestration of the receptor molecules to the inside of the cell
D. is brought about by inactivation of receptor molecules.

8. G-proteins are named for their ability to bind with


A. guanosine nucleotides
B. gamma amino butyrate
C. STAT proteins
D. Glucagon receptor proteins

9. Steroid hormones and thyroid hormones bind with receptor proteins located
A. on the cell membrane surface
B. in the cytosol
C. in the cell nucleus
D. on the nuclear envelope

10. Norepinephrine and acetylcholine bind with


A. G-protein linked hormone receptors
B. Enzyme linked hormone receptors
C. Ion-channel linked hormone receptors
D. Nucleotide linked hormone receptors
11. Example of tyrosine-derived hormones
A. growth hormone
B. luteinizing hormone
C. adrenocorticotrophic hormone
D. thyroid hormone

12. When G-protein is activated, one of its trimeric components dissociates to bind with
other intracellular signaling proteins. This component is the
A. alpha component
B. beta component
C. gamma component
D. GTP component

13. All G-protein linked receptors


A. are proteins that penetrate through the cell membrane only once
B. have segments that penetrate the cell membrane seven times
C. alter the ion permeability of the cell membrane
D. function directly as enzymes

14. Example of second intracellular messenger


A. guanosine triphosphate
B. hormone response element
C. diacyglycerol
D. phospholipase C

15. There are no known hormones that exist as


A. polysaccharides
B. polypeptides
C. steroids
D. amino acids

Answer keys:

Genetics: 1B, 2A, 3C, 4D, 5A, 6D

Endocrine: 1C, 2B, 3B, 4A, 5B, 6C, 7D, 8A, 9B, 10C, 11A

SPECIAL EXAM – FIRST BIMONTHLY

NAME_____________________________

1. Give five (5) differences between a graded potential and an action potential. Tabulate
your answer. 2.5 pts.

2. Give five (5) characteristics of carrier-mediated transport and briefly explain each. 2.5
pts
3. Enumerate in sequence the events that lead to the production of an inhibitory
postsynaptic potential (IPSP) from the time an action potential arrives at the terminal
knob. 5 pts

4. Enumerate in sequence the events that would initiate muscular contraction. Start from
the time an action potential is initiated at the motor neuron until calcium binds with
troponin-C. 5 pts

You might also like